NINTH EDITION

LANGEQ&A™ PSYCHIATRY

Ivan Oransky, MD Clinical Assistant Professor of Medicine NewYork University School of Medicine New York, New York

Sean Blitzstein, MD Director, Psychiatry Clerkship Clinical Professor of Psychiatry University of Illinois at Chicago School of Medicine Chicago, Illinois

New York Chicago San Francisco Lisbon London Madrid Mexico City Milan New Delhi San Juan Seoul Singapore Sydney Toronto Copyright © 2007 by The McGraw-Hill Companies, Inc. All rights reserved. Manufactured in the United States of America. Except as permitted under the United States Copyright Act of 1976, no part of this publication may be reproduced or distributed in any form or by any means, or stored in a database or retrieval system, without the prior written permission of the publisher.

0-07-150947-X

The material in this eBook also appears in the print version of this title: 0-07-147567-2.

All trademarks are trademarks of their respective owners. Rather than put a trademark symbol after every occurrence of a trademarked name, we use names in an editorial fashion only, and to the benefit of the trademark owner, with no intention of infringement of the trademark. Where such designations appear in this book, they have been printed with initial caps.

McGraw-Hill eBooks are available at special quantity discounts to use as premiums and sales promotions, or for use in corporate training programs. For more information, please contact George Hoare, Special Sales, at [email protected] or (212) 904-4069.

TERMS OF USE

This is a copyrighted work and The McGraw-Hill Companies, Inc. (“McGraw-Hill”) and its licensors reserve all rights in and to the work. Use of this work is subject to these terms. Except as permitted under the Copyright Act of 1976 and the right to store and retrieve one copy of the work, you may not decom- pile, disassemble, reverse engineer, reproduce, modify, create derivative works based upon, transmit, distribute, disseminate, sell, publish or sublicense the work or any part of it without McGraw-Hill’s prior consent. You may use the work for your own noncommercial and personal use; any other use of the work is strictly prohibited. Your right to use the work may be terminated if you fail to comply with these terms.

THE WORK IS PROVIDED “AS IS.” McGRAW-HILL AND ITS LICENSORS MAKE NO GUARANTEES OR WARRANTIES AS TO THE ACCURA- CY, ADEQUACY OR COMPLETENESS OF OR RESULTS TO BE OBTAINED FROM USING THE WORK, INCLUDING ANY INFORMATION THAT CAN BE ACCESSED THROUGH THE WORK VIA HYPERLINK OR OTHERWISE, AND EXPRESSLY DISCLAIM ANY WARRANTY, EXPRESS OR IMPLIED, INCLUDING BUT NOT LIMITED TO IMPLIED WARRANTIES OF MERCHANTABILITY OR FITNESS FOR A PARTICULAR PURPOSE. McGraw-Hill and its licensors do not warrant or guarantee that the functions contained in the work will meet your requirements or that its operation will be uninterrupted or error free. Neither McGraw-Hill nor its licensors shall be liable to you or anyone else for any inaccuracy, error or omission, regardless of cause, in the work or for any damages resulting therefrom. McGraw-Hill has no responsibility for the content of any information accessed through the work. Under no circumstances shall McGraw-Hill and/or its licensors be liable for any indirect, incidental, special, punitive, consequential or similar damages that result from the use of or inability to use the work, even if any of them has been advised of the possibility of such damages. This limitation of liability shall apply to any claim or cause whatsoever whether such claim or cause arises in contract, tort or otherwise.

DOI: 10.1036/0071475672 Professional

Want to learn more? We hope you enjoy this McGraw-Hill eBook! If you’d like more information about this book, its author, or related books and websites, please click here. For more information about this title, click here

Contents

Preface ...... v

Acknowledgments ...... vii

1. Psychological Testing ...... 1 Questions ...... 1 Answers and Explanations ...... 7

2. Child and Adolescent Psychiatry ...... 13 Questions ...... 13 Answers and Explanations ...... 29

3. Adult Psychopathology ...... 41 Questions ...... 41 Answers and Explanations ...... 67

4. Somatic Treatment and Psychopharmacology ...... 85 Questions ...... 85 Answers and Explanations ...... 107

5. Psychological Treatment and Management ...... 123 Questions ...... 123 Answers and Explanations ...... 132

6. Legal and Ethical Issues in Psychiatry and Medicine ...... 141 Questions ...... 141 Answers and Explanations ...... 153

7. Differential Diagnosis and Management ...... 161 Questions ...... 161 Answers and Explanations ...... 179

8. Practice Test 1 ...... 195 Questions ...... 195 Answers and Explanations ...... 212

iii iv Contents

9. Practice Test 2 ...... 229 Questions ...... 229 Answers and Explanations ...... 248

Bibliography ...... 263

Index ...... 265 Preface

Welcome to Lange Q&A: Psychiatry, 9th edition. It is As much as possible, we have done our best to an exciting time to be studying or practicing psychi- replicate the format of the examination. That means atry as sophisticated imaging techniques herald that in this edition, all of the questions are clinical potential changes in how we think about diagnostic vignettes and there are no negatively phrased ques- criteria and the brain itself. Such methods aren’t yet tions (e.g., except, least likely). None of the vignettes ready for clinical use, and are certainly not part of the have more than two questions, and many have just fundamentals of psychiatry tested on the USMLE one. Still, because there are many times the number Step 2 examination, but they are worth noting of questions in this book as there are psychiatry nonetheless because future generations of medical questions on the USMLE Step 2, some sets of ques- students—users of future editions of this book—will tions based on a single vignette remain to give you a more than likely need to master such material. of how a single vignette can raise more than As psychiatry begins to change, so too does the one important (and testable) issue. format of the United States Medical Licensing There is material in this edition, as in previous Examination (USMLE) Step 2 examination. This edi- editions, that is unlikely to appear on the examina- tion of the book reflects those changes so that stu- tion. That’s because I recognize that some students dents can use it to prepare for the examination as use this book to review material during their psychi- effectively as possible. The subjects covered in the atry clerkships as well as in direct studying for the eighth and previous editions—from psychopatholo- examination. The answers to those “non-USMLE- gy to psychopharmacology and legal issues—are type” questions begin with a note that these are in a represented in nine chapters and 800 questions format unlikely to be found on that test. including two final chapters that are practice tests of A few words on how best to use this book. If you 100 questions each that simulate the real examina- are first picking it up within a few weeks of Step 2 and tion. The “definitions” chapter has been eliminated. don’t have time to review the entire book, I’d recom- Because so many students rely heavily or solely mend hitting Chapters 8 and 9, the practice examina- on books of questions such as this, answers include, tions. These best reflect what you’ll see on the exami- in addition to a discussion of the correct answer, a nation and will point out gaps in your knowledge. discussion of why the incorrect answers were wrong. If you have more time—say, a few months—pick That way, learning is not restricted to one term or dis- some questions from each chapter, other than cussion per question. All diagnostic criteria refer to Chapters 8 and 9 to test your knowledge of each sub- the American Psychiatric Association’s Diagnostic and ject, then concentrate on those chapters in which Statistical Manual of Mental Disorders, Fourth Edition, you’re weakest. Text Revision (DSM-IV-TR), the most recent edition. Finally, if you’re using this book during your A list of references is included for those students who clerkship, it’s probably best to work through all the wish to pursue further reading, but is not indexed chapters and save Chapters 8 and 9 for the end of directly to questions because many students will be your clerkship review. You’ll find basic questions using this book, as I did, at the last minute before that will help you master the material but that may their examinations. not appear on Step 2. Ivan Oransky, MD

v

Copyright © 2007 by The McGraw-Hill Companies, Inc. Click here for terms of use. This page intentionally left blank Acknowledgments

This book continues to reflect the hard work and dedication of a large group of contributors to past editions of the predecessor book, Appleton & Lange Review of Psychiatry. This list reflects the titles and affiliations of those contributors at the time of publication of earlier editions.

R. Andrew Chambers, MD Julie E. Peters, MD Resident Resident Department of Psychiatry Department of Psychiatry Yale University School of Medicine Yale University School of Medicine New Haven, Connecticut New Haven, Connecticut

Joseph R. Check, MD Greer Richardson, MD Assistant Clinical Professor of Psychiatry Resident Yale University Department of Psychiatry New Haven, Connecticut Yale University School of Medicine New Haven, Connecticut Vladimir Coric, MD Chief of Inpatient Services William Roman, MD Clinical Neuroscience Research Unit Resident Assistant Clinical Professor of Psychiatry Department of Psychiatry Yale University Yale University School of Medicine New Haven, Connecticut New Haven, Connecticut

Frank G. Fortunati, MD, JD Louis Sanfilippo, MD Resident Assistant Clinical Professor of Psychiatry Department of Psychiatry Yale University Yale University School of Medicine New Haven, Connecticut New Haven, Connecticut Raziya Sunderji, MD Brian Greenlee, MD Resident Resident Department of Psychiatry Department of Psychiatry Yale University University of Kentucky New Haven, Connecticut Lexington, Kentucky

vii

Copyright © 2007 by The McGraw-Hill Companies, Inc. Click here for terms of use. viii Acknowledgments

Blake Taggart, MD Chung-Che Charles Wang, MD Resident Resident Department of Psychiatry Department of Psychiatry Yale University School of Medicine Yale University New Haven, Connecticut New Haven, Connecticut

Elizabeth Walter, MD Resident Department of Psychiatry Yale University School of Medicine New Haven, Connecticut Acknowledgments ix

Thanks to medical students Adam Beall, Alvin Kwok, and Mark McDonnell, who made helpful suggestions on how to improve from the eighth edition. Thanks also to the extremely patient Marsha Loeb, our editor at McGraw-Hill. And of course, thanks to the patients and professors who taught me psychiatry. Ivan Oransky, MD This page intentionally left blank CHAPTER 1 Psychological Testing Questions

DIRECTIONS (Questions 1 through 27): Each of Questions 3 and 4 the numbered items in this section is followed by answers. Select the ONE lettered answer that is You admit an 83-year-old widowed White woman for BEST in each case. further evaluation because she is no longer able to care for herself at home. She has lost 30 lb in the past year, has poor hygiene, and admits to increasing Questions 1 and 2 forgetfulness. A 34-year-old woman suffering from severe depres- sion and chronic renal failure is a candidate for elec- 3. A commonly administered screening test to troconvulsive therapy (ECT). evaluate for dementia is which of the following? (A) Geriatric Rating Scale 1. Which test listed below can be used to monitor (B) Glasgow Coma Scale the patient’s depressive symptoms with the greatest reliability and validity? (C) Folstein Mini-Mental Status Examination (MMSE) (A) Thematic Apperception Test (TAT) (D) Mental Status Examination (MSE) (B) Rorschach Test (E) Blessed Rating Scale (C) Halstead-Reitan Neuropsychological Battery 4. Another psychiatric diagnosis that may mimic (D) Draw-a-Person Test dementia in this patient is which of the (E) Beck Depression Inventory following? (A) panic disorder 2. The most useful test to assess for (B) depression impairments in the setting of ECT is which of the following? (C) posttraumatic stress disorder (D) generalized anxiety disorder (A) Brown-Peterson Task (E) obsessive-compulsive disorder (B) Beck Depression Inventory (C) State-Trait Anxiety Inventory (D) Bulimia Test—Revised (E) Eating Disorder Inventory-2 (EDI-2)

1

Copyright © 2007 by The McGraw-Hill Companies, Inc. Click here for terms of use. 2 1: Psychological Testing

Questions 5 and 6 7. The authors of the initial article regarding the screening test suggested that their test was A 28-year-old single African American man with a useful because of which of the following? long history of schizophrenia and prominent disorganization asks you if there are psychological (A) internal reliability tests that would demonstrate which part of his brain (B) test-retest reliability “isn’t working.” You explain that no test can indicate (C) content validity exactly which part of his brain is different from those (D) predictive validity without schizophrenia and suggest a certain test to (E) discriminative validity assess his ability to organize and correctly process information. 8. The authors of the second paper faulted the usefulness of this screening tool because of its 5. The neuropsychological test you recommend lack of which of the following? is which of the following? (A) internal reliability (A) MMSE (B) test-retest reliability (B) Wisconsin Card Sorting Test (WCST) (C) content validity (C) Bender Gestalt Test (D) predictive validity (D) Luria-Nebraska Neuropsychological Battery (E) discriminative validity (E) Draw-a-Person Test Questions 9 and 10

6. This test assesses which of the following? A patient is administered a test that consists of view- (A) executive functions of the brain ing a set of 10 inkblots sequentially. The responses to the inkblots are noted by the examiner in relationship (B) attentional state to the content of the perception, the area of the blot (C) the ability to draw that forms the basis of the response, and the aspects (D) orientation of the area that are used to form the response. (E) memory 9. The test being administered is which of the Questions 7 and 8 following? While reading a medical journal, you find an article (A) TAT about a new screening test that predicts a patient’s (B) Rorschach Test risk for suicide. The authors have shown that high (C) Sentence Completion Test scores on their screening test correlate with an (D) Draw-a-Person Test increased risk for completed suicide. Nine months (E) Minnesota Multiphasic Personality later, another group repeats the study and demon- Inventory (MMPI) strates that male patients consistently scored higher on the screening test than female patients. 10. This type of projective test is best classified as Furthermore, the authors of the second article state which of the following? that when gender is taken into account, the previ- ously reported correlation between the screening test (A) self-expression and increase in suicide rate vanishes. The authors of (B) constructions the second study conclude that this limits the use- (C) completions fulness of the screening test to predict a patient’s risk (D) associations of suicide. (E) choice of ordering Questions: 5–16 3

11. An 89-year-old married African American Questions 14 and 15 woman is admitted to the medical service with a diagnosis of failure to thrive. The team fol- A 20-year-old college student is referred for testing to lowing her has asked you to assess her. In their evaluate poor academic performance. He reports that notes, they observe that the patient is not ori- he has always “struggled” to pass his classes despite ented to time. To evaluate orientation, you per- studying for many hours. He attends all of his lec- form which of the following? tures and is able to pay , yet he does not seem to be able to adequately learn the material. (A) Spatial Orientation Memory Test While he is very frustrated, he denies significant (B) Fargo Map Test depression. (C) Stroop Test (D) WCST 14. Which of the following tests would be most appropriate to determine this patient’s problem? (E) Temporal Orientation Test (A) MMPI-2 Questions 12 and 13 (B) Draw-a-Person Test A 25-year-old single African American woman who (C) Wechsler Adult Intelligence carries the diagnosis of dependent personality dis- Scale—Revised (WAIS-R) order is referred for psychological testing. Your first (D) Wechsler Intelligence Scale for Children consideration is whether to perform a set of projec- (WISC) tive or objective tests. (E) Wechsler Memory Test (WMT)

12. You decide to perform projective tests because 15. The IQ is calculated by which of the following of which of the following reasons? formulas? (A) They provide numerical scores. (A) performance IQ/verbal IQ × 100 (B) They ask specific questions with (B) mental age/full-scale IQ × 100 itemized responses. (C) chronological age/performance IQ × 100 (C) They are unstructured and allow for a (D) mental age/chronological age × 100 variety of responses from the examinee. (E) actual IQ/theoretic IQ × 100 (D) Statistical analysis is easily undertaken on the data. Questions 16 and 17 (E) You want to determine how the patient feels about you. A 72-year-old man is suspected to have suffered a stroke in his right parietal region. 13. After performing a battery of projective tests, you chose to continue testing with the MMPI-2. 16. The test most likely to show abnormalities is You chose this test because of which of the fol- which of the following? lowing reasons? (A) WMT (A) It is the most widely used test in the (B) WCST evaluation of personality structure. (C) MMPI-2 (B) It can be scored by nonprofessionals. (D) Rey-Osterrieth Test (C) It is a specific type of projective test. (E) Rorschach Test (D) The patient is single. (E) It is designed to evaluate attitudes about test taking at the time the test is administered. 4 1: Psychological Testing

17. This test assesses which of the following? breast cancer and the available treatment options. Which of the following defense mech- (A) visual nonverbal memory anisms is she using? (B) verbal memory (C) tactile memory (A) sublimation (D) long-term procedural memory (B) dissociation (E) long-term implicit memory (C) intellectualization (D) rationalization 18. A 70-year-old man with multiple medical prob- (E) self-observation lems is suspected to have had a stroke, affect- ing his ability to speak. The test most likely to 22. A 35-year-old woman presents with episodic adequately describe the nature of the difficulty anxiety and complains of the occasional feeling is which of the following? that she has heard or perceived things prior to actually hearing them. She expresses her con- (A) Stroop Test cern that she is “going crazy.” You assure her (B) Boston Diagnostic Aphasia Examination that this can occur in anxiety disorders. What is (C) Folstein MMSE this phenomenon called? (D) Bender Gestalt Test (A) déjà vu (E) Sentence Completion Test (B) jamais vu 19. A compulsive gambler tells her psychiatrist (C) déjà entendu that once she begins to play the slot machines, (D) folie à deux she cannot stop, particularly if she wins a few (E) la belle indifférence times. Which of the following reinforcement schedules best explains the phenomena? 23. After being severely reprimanded by his employer, a man goes home and is extremely (A) continuous nasty to his wife. What is his behavior an exam- (B) fixed-interval ple of? (C) fixed-ratio (A) sublimation (D) variable-interval (B) dissociation (E) variable-ratio (C) displacement 20. When an examiner asks a patient to count back- (D) rationalization ward by 7, starting at 100 (referred to as serial (E) conversion sevens), what is principally being tested? 24. A psychiatrist discovers that she is frustrated (A) recent memory and easily angered with one of her patients for (B) remote memory no obvious reason. While talking to a colleague, (C) concentration she admits that the patient reminds her of her (D) fund of knowledge abusive father. Which of the following best (E) mathematics skills describes the clinician’s reaction? (A) transference 21. A 39-year-old woman presents to the outpa- (B) countertransference tient mental health clinic at the request of her oncologist 3 weeks after being diagnosed with (C) reaction formation metastatic breast cancer. The patient denies (D) displacement strong feelings in relation to the diagnosis, but (E) projection talks a great deal about the epidemiology of Questions: 17–31 5

25. A 36-year-old woman was placed on alprazo- (D) Random Letter Test lam (Xanax) 3 years ago for panic disorder. (E) simple calculations After watching a news report on television, she became frightened about addiction. She DIRECTIONS (Questions 28 through 35): Each abruptly stopped the medication and during group of items in this section consists of lettered the next 3 days experienced increased anxiety headings followed by a set of numbered words or attacks, but claims that she is now doing better. phrases. For each numbered word or phrase, select She denies any tremor, sweating, increased the ONE lettered heading that is most closely asso- heart rate, or uneasiness except in relation to ciated with it. Each lettered heading may be the panic attacks. Which of the following best selected once, more than once, or not at all. describes this phenomenon? (A) recurrence Questions 28 through 35 (B) rebound Match the clinical presentation with the appropriate (C) withdrawal neuropsychological test. (D) dystonic reaction (A) MMPI-2 (E) akathisia (B) WISC (C) Rey-Osterrieth Test 26. A 37-year-old patient presents to your office for the first time for long-term psychotherapy. (D) Beck Depression Inventory You decide it would be helpful to compile a (E) Blessed Rating Scale personality inventory. You place a set of 10 (F) WAIS-R inkblots in front of the patient and note the (G) Boston Diagnostic Aphasia Examination responses in terms of the content of the per- (H) Rorschach Test ception and the use of the various areas of the (I) Folstein MMSE inkblot. To score and interpret this test using a data-based system, one would rely on which of (J) Bender Gestalt Test the following? (K) WCST (L) Wada Test (A) Exner Comprehensive System (B) Five-Factor Model 28. A 40-year-old woman who scores a 26/30 on (C) Eysenck Personality Questionnaire the Folstein MMSE gave many answers of, “I (D) California Personality Inventory don’t know, I’m too tired to answer.” You want (E) MMPI-2 to assess for the possibility of depression.

27. You are asked to evaluate a 68-year-old man on 29. A 65-year-old man has difficulty organizing, the inpatient medicine service for increasing sequencing, and planning activities but no confusion. The patient was admitted 2 days impairment in memory. earlier for pneumonia. After performing a mental status evaluation, you suspect delirium. 30. The family of an 80-year-old man with mild The patient dropped out of school in the sev- dementia has asked you to evaluate his ability enth grade. The best test to assess this patient’s to continue to live in his current environment. ability to maintain and focus attention is which You would like to ask his family and friends of the following? their assessment of how he has been doing.

(A) serial sevens 31. A 16-year-old boy with a family history of (B) counting by two’s to 20 mental retardation presents with long-standing (C) serial threes poor school performance and aggressive behavior toward peers. 6 1: Psychological Testing

32. A 37-year-old man has a history of avoiding 34. A 45-year-old woman has a long history of social situations, no close friends, and a pref- unstable relationships, self-injurious behavior, erence for being alone. He has been described and affective instability. She does not meet by others as unemotional and detached. You criteria for an Axis I disorder. would like to evaluate this patient with a pro- jective test. 35. You are asked to preoperatively evaluate hemi- spheric dominance in an 18-year-old left- 33. You would like to quickly assess for dementia handed woman with a history of seizure in a 75-year-old woman admitted to the emer- disorder who is about to undergo surgery to gency department for failure to thrive. remove a seizure focus in the left hemisphere. Answers and Explanations

1. (E) The Beck Depression Inventory is a widely Overall scores range from 3 to 15, with lower used test that allows clinicians to follow the scores reflecting more severely impaired con- severity of previously diagnosed depression. sciousness. The Geriatric Rating Scale is a rating The TAT, Rorschach Test, and Draw-a-Person scale for nonprofessional staff to evaluate Test are all types of projective testing. Projective patients’ abilities to perform their activities of tests, although useful clinical tools, often suffer daily living and interact with others. It may be low reliability and validity. Projective tests most helpful in evaluation of the moderately require a person skilled at this type of evalua- to severely demented individual. The Blessed tion and often do not have rigorous empirical Rating Scale is a tool that typically asks a data and group comparison. The evaluation of patient’s friends or relatives to assess the ability depression in relation to the Halstead-Reitan of the patient to function in his or her current Test is limited by the fact that many depressed environment. An MSE is the formal psychiatric patients fail to show deficits on such classic examination that includes appearance, assess- neuropsychological batteries. In addition, these ment of mood and affect, presence of , tests, even when demonstrating deficits in cog- and evaluation of insight and judgment. It is not nitive domains such as attention and learning, a substitute for the Folstein MMSE. still have very limited usefulness in evaluat- ing the severity of the depression. 4. (B) When evaluating the total score for demen- tia, it is critical to consider that depression (also 2. (A) The Brown-Peterson Task is a test specifically known as pseudodementia) may produce designed to evaluate short-term memory, a similar scores. The other choices would not capacity that can be affected during ECT. The decrease the Folstein MMSE score. Bulimia Test—Revised and the EDI-2 are both useful for the evaluation of bulimia and eating 5. (B) In the WCST, examinees are asked to sort disorders, respectively. The State-Trait Anxiety cards depicting various pictures and symbols Inventory and the Beck Depression Inventory according to a variety of different criteria that are used to evaluate anxiety disorders and change over time without the subject know- depression, respectively. ing. The WCST assesses a person’s ability to switch sets, reason abstractly, and solve prob- 3. (C) The Folstein MMSE is a frequently used lems. These capacities are also known as exec- screening assessment for dementia. It is a utive functions and are thought to be localized 30-point scale with deductions for incorrect in the frontal lobes. Current research regarding answers. The Glasgow Coma Scale is an easy-to- the psychopathology of schizophrenia suggests perform instrument that evaluates level of con- that there are abnormalities in the frontal lobes, sciousness. There are three general categories specifically in the dorsolateral prefrontal cortex, that the examiner tests: eye opening, verbal that are reflected in poor performance on the response, and best motor response. Each cate- WCST. People with schizophrenia perform gory receives a number for patient response. more poorly on the WCST than people without

7 8 1: Psychological Testing

schizophrenia; however, individuals with 9. (B) The Rorschach Test was developed in 1921 damage to their frontal lobes from a variety of by Hermann Rorschach. It consists of showing causes also show executive function deficits. a subject a set of 10 inkblot stimuli in a sequen- The Draw-a-Person Test requires the examinee to tial manner while noting (1) the responses in draw a person. It was initially devised to test relationship to the content of the perception, (2) intelligence in children, but is now used pri- the area of the blot that forms the basis of the marily as a screening test for brain damage. response, and (3) the aspects of the area that The Luria-Nebraska Neuropsychological Battery are used to form the response. The TAT, is a comprehensive set of neuropsychological Sentence Completion Test, and Draw-a-Person tests used to assess specific cortical areas and Test are similar to the Rorschach in that they are aids in assessment of hemispheric dominance. also projective tests. The TAT was developed in The MMSE is a commonly used scale to assess the 1930s by Henry Murray and consists of 20 the possibility of dementia. The Bender Gestalt cards featuring ambiguous situations. The sub- Test involves copying figures, which helps ject is asked to develop a narrative based on the determine if organic brain disease is present. scene. In the Sentence Completion Test, the answer is evaluated for its relationship with 6. (A) See answer 5. the stimulus and for content, among other criteria. The Draw-a-Person Test is evaluated in 7. (D) Because the screening test seemed to indi- relationship to detail, size of various parts in cate which patients are at a higher risk for com- relationship to one another, and the size of pleted suicide, it has utility based on its the overall picture. The MMPI-2 consists of predictive validity. Content validity is a test’s abil- true/false questions designed to assess per- ity to cover the conceptual domain that the test sonality characteristics; it is the most widely intends to measure. Conceptual domain is usu- used and highly standardized test of personal- ally established by a large group of experts ity structure. through a wide review of the literature. It is unclear in this case as to whether suicide or 10. (D) In 1961, Lindzey proposed a method of depression, critical aspects of this test’s con- classifying projective tests based on the type of ceptual domain, were adequately addressed. activity. The Rorschach is classified in the cat- Discriminative validity is the measure of a test’s egory of associations. Another test in this cate- ability to differentiate between issues that are gory is the Word Association Test. The theoretically unrelated. Reliability is a test’s abil- construction category requires the subject to ity to provide reproducible results. Test-retest construct content based on a stimulus such as reliability measures a test’s reproducibility over a story in the TAT. The self-expression category a short period of time in a person whose state consists of tests such as the Draw-a-Person is assumed not to have fluctuated. Internal reli- Test in which the subject produces a response ability evaluates whether the questions within without a stimulus. Tests such as the Sentence the test are measuring the same thing. Completion Test fall into the category of com- Reliability may be a problem with this test, but pletions, in which a person completes an it is not a concern in determining the test’s use- uncompleted stimulus. The last category is the fulness. choice of ordering in which patients place objects in a rank order with respect to preference. 8. (E) In this case, the authors of the second study show that the test fails to discriminate suicide 11. (E) The Temporal Orientation Test asks the risk when gender is removed. Knowing that patient to identify the appropriate day, month, men are at an increased risk for successful sui- day of the week, and current time. Deviation cide attempts may decrease the test’s useful- from the correct response is differentially ness. See answer 7 for definitions of other scored in each category. Total score and the choices. data incorrectly identified separate the patients Answers: 6–17 9

into two groups: patients with brain damage Together with the clinical history, this test pro- and patients without brain damage. The vides valuable information about personality Temporal Orientation Test is also sensitive to structure. Its accuracy does not depend on the cognitive abnormalities in dementing illnesses. patient’s marital status, and the results do not The Spatial Orientation Memory Test evaluates reflect how the examinee feels about test the ability to immediately recall the orienta- taking. tion of figures and is used to evaluate immedi- ate memory. The Stroop Test has a number of 14. (C) The Wechsler Adult Intelligence Scale was different formats, but the general concept is initially published in 1955 and has undergone that it takes longer to correctly identify a color a series of revisions to reach the current scale, than to read words and longer yet to correctly the WAIS-R. The test is composed of 11 differ- identify a word (e.g., name of a color) when ent subtests—6 verbal and 5 performance— that word is in a color different from that word. that allow for the calculation of the full-scale This test seems to be primarily an assessment IQ, performance IQ, and verbal IQ. The WAIS- of ability to concentrate. The Fargo Map Test R has high reliability, which means that in assesses recent and remote spatial memory and normal subjects retesting does not lead to sig- visuospatial orientation by using maps of the nificantly different evaluations. This means that United States and different regions within it. it can be used to follow people over time. The Patients are asked to identify certain areas. MMPI-2 is a test used for the assessment of Education level and age influence the score on personality structure, and the Draw-a-Person this test. The WCST is used to evaluate execu- Test is used to evaluate for organic brain dis- tive functioning of the brain. ease. The WMT was designed to evaluate a variety of aspects of memory function in adults. 12. (C) Objective tests usually involve questions None are useful in the determination of IQ. with lists of possible responses. These tests provide numerical scores on which statistical 15. (D) The IQ is calculated by dividing the mental analyses are easily performed. Examples age by the chronological age and multiplying include the United States Medical Licensing this by 100. Examinations. Conversely, projective tests usu- ally involve stimuli to which there are a variety 16. (D) The Rey-Osterrieth Test is a complex figure of responses without a single correct answer. that the patient is asked to copy while looking Many of these tests, including the Rorschach at the figure. The figure is then taken away and Test, the TAT, the Sentence Completion Test, the patient is asked to draw the picture from and the Draw-a-Person Test, require specific immediate memory. The patient is again asked training in giving the test and interpreting the to draw the figure 5 minutes and 30 minutes results. Projective tests do not necessarily tell after the figure has been removed. The tasks the interviewer how the patient feels about assess visual nonverbal memory. People with him or her. right parietal lesions usually show abnormali- ties in copying the figure correctly by neglect- 13. (A) The MMPI and now the MMPI-2 are objec- ing the items in the left visual field. Conversely, tive tests that have been used for over 50 years a patient with a right temporal lobectomy may in the assessment of personality structure. They have no difficulty in copying the figure, but consist of more than 500 statements, which are show marked abnormalities in drawing the condensed into 10 clinical scales. Additionally, item from memory. The WMT, WCST, MMPI-2, questions are asked at the time of examination and the Rorschach Test would not be as useful to evaluate attitudes when taking the test. These in assessing a right parietal lesion as the Rey- questions help to provide information about Osterrieth figure. the validity of the examination. Interpretation of the results requires an experienced evaluator. 17. (A) See answer 16. 10 1: Psychological Testing

18. (B) The Boston Diagnostic Aphasia Examination is them into socially acceptable behaviors. Like a comprehensive set of tests given by a skilled sublimation, rationalization is a defense against interviewer to evaluate aphasic disorders and undesired motivations, but in this case, the to help define further interventions to improve motivations are concealed by elaborate and speech. The Stroop Test aids in the evaluation reassuring explanations that avoid the actual of concentration. The Folstein MMSE is used underlying motives. Dissociation is a defense for rapid assessment of dementia and delir- mechanism that deals with stressors with a ium. The Bender Gestalt Test is a constructional breakdown of the usual integration of memory, test for evaluation of brain damage and has behavior, and perception. Self-observation is a some ability to differentiate the location of the defense mechanism involving the reflection of lesion. The Sentence Completion Test is a pro- one’s own and behavior with appro- jective test used to describe personality struc- priate responses. ture. 22. (C) Déjà entendu is the feeling that one is hear- 19. (E) Behavior is most difficult to extinguish ing something one has heard before. It is usu- when reinforced on a partial rather than con- ally associated with anxiety states or fatigue. tinuous schedule. Gambling on a slot machine Déjà vu is a similar experience, but refers to the is even more likely to continue because it is sensation that something has been seen before. reinforced on a variable-ratio schedule, that is, Jamais vu is the opposite of déjà vu in that it money is given based on a random (but refers to something that should be familiar unknown) number of times. but seems quite unfamiliar. Folie à deux is a shared delusion aroused in one person by 20. (C) Concentration refers to the ability to sustain the influence of another. La belle indifférence is focus on a cognitive task. Performing serial the indifference shown toward a deficit or sevens and spelling world backward are tests of loss of function classically seen in a conver- concentration. Although a certain facility with sion disorder. the remaining choices is necessary to perform each task (no cognitive function is tested in 23. (C) The man is naturally angry, anxious, and absolute isolation), the serial sevens test pro- sensitive at being reprimanded by his vides a window on a patient’s concentration. employer. He has found it difficult to express Remote memory involves the recall of events his feelings toward the disturbing person, the long past, for example, information from a employer. Instead of suppressing or repress- patient’s childhood. Recent memory is recall of ing the anger, or sublimating his tension in events occurring in the last several minutes. more forceful work, he displaces his anger onto Fund of knowledge is a test of information the a safer target, his wife. This is an example of patient readily has available to him or her; displacement. knowledge of current events is often used to assess this. The MSE often contains tests of 24. (B) Transference, in strict terms, is the patient’s mathematics skills, but testing mathematics reexperiencing of past experiences in the skills is not the purpose of the serial sevens setting of psychoanalytic psychotherapy. test. Any test of cognitive function must take Countertransference is the analyst’s (or thera- into account the patient’s cultural, educational, pist’s) response to this. These terms have come and social background. to mean the transferring of emotions and feel- ings that one has from one’s past to the other 21. (C) Intellectualization is the utilization of person; in the case of transference, the feelings abstract thinking to deal with or cover internal are experienced in the patient and relate to how or external stressors; in this case, the unaccept- he or she feels about the therapist. In the case able feelings of having cancer. Sublimation is a of countertransference, the feelings are experi- defense mechanism employed to deal with enced in the analyst or therapist and reflect unacceptable feelings or desires by channeling how he or she feels about the patient. Reaction Answers: 18–34 11

formation, displacement, and projection are all by raising a finger to indicate when he or she defense mechanisms used by the ego to keep hears the key letter. potentially anxiety-provoking feelings out of awareness. Reaction formation is the formation 28. (D) The Beck Depression Inventory is a 21-item of thoughts that are opposite to the anxiety- test with 3 responses per item that is an easily provoking feelings. Displacement is the trans- used screening tool to evaluate for depression. ferring of a feeling toward an object that is less Because it would be unusual for an individual threatening, as in the family pet or one’s spouse so young to have dementia and because many or children. Projection is the false attribution of of the answers reflect a lack of interest, the Beck one’s own unacceptable feelings to another. Depression Inventory, in conjunction with the Folstein MMSE, may help distinguish depres- 25. (B) Rebound is a return of symptoms that are sion from dementia. brief and transient and is frequently associ- ated with the abrupt discontinuation of ben- 29. (K) The WCST assesses executive functions of zodiazepines. Recurrence is the long-term the brain such as organizational abilities, mental return of the original symptoms. Withdrawal flexibility, and the ability to abstract and reason. is characterized by a specific set of signs and These capacities are believed to be located in the symptoms specific to a particular substance. frontal lobes. Damage to the frontal lobes can These are not necessarily similar to the original lead to abnormalities on this test. symptoms that were being treated by the med- ication. Akathisia is the subjective sensation of 30. (E) The Blessed Rating Scale is a tool that asks motor and mental restlessness. A dystonic reac- friends or family of the patient to assess the tion is an increase in muscle rigidity and spas- ability of the patient to function in his or her ticity that is usually associated with the use of usual environment. neuroleptics. 31. (F) An assessment of IQ is indicated to rule out 26. (A) The test that is being described here is the mental retardation and help better understand Rorschach Test. One data-based system used to the individual’s level of intellectual level of score and interpret the Rorschach Test is the functioning. IQ assessment will also aid the Exner Comprehensive System. It is limited in its school in developing an individualized study validity and requires highly trained examiners. program for this student. Because the patient is All of the other choices are types of personality older than 15, the WAIS-R would be used. assessment not associated with the Rorschach Test. The Eysenck Personality Questionnaire con- 32. (H) The Rorschach Test is a projective test that structs questions designed to assess aspects of may be used to assess personality structure. personality predicted to exist by theoretical con- The patient has characteristics of schizoid per- structs. The MMPI-2 is the most widely used sonality disorder, which may be better eluci- and highly standardized test of personality. The dated with further skilled assessment in the California Personality Inventory is similar to the context of his clinical history. MMPI but is used in counseling situations, rather than with pathologic populations. 33. (I) The Folstein MMSE is a quick, easily admin- istered test that allows for immediate assess- 27. (D) In this instance, the Random Letter Test, ment of dementia. Scores of less than 24 are which relies on concentration, cooperation, and suggestive of a dementia. the ability to hear, is the test of choice. The other tests rely not only on attention but also on 34. (A) The MMPI-2 is an objective test consisting calculation abilities and educational level. The of several hundred true/false questions used to Random Letter Test consists of telling a patient assess an individual’s personality. It is the most a letter and then in a monotone listing a widely used and highly standardized test of random string of letters. The patient responds personality. 12 1: Psychological Testing

35. (L) The Wada Test is used to evaluate hemi- The Rey-Osterreith figure is sensitive to deficits spheric language dominance prior to surgical in copying and lack of attention to detail in amelioration of seizure focus. Whereas most people with right-sided parietal lobe lesions. right-handed individuals show left hemi- The appropriate test to evaluate IQ would be spheric dominance for language, left-handed the WISC. The Boston Diagnostic Aphasia individuals may be either right or left domi- Examination is a series of tests given by an nant. The test consists of injecting sodium experienced clinician to evaluate and make amytal into the carotid artery and observing treatment recommendations for individuals the transient effects on speech. Injection into the with aphasia. The Bender Gestalt Test involves left carotid artery anesthetizes the left side of copying figures, which helps to determine if the brain, and those with left hemispheric lan- organic brain disease is present. guage dominance show interrupted speech. CHAPTER 2 Child and Adolescent Psychiatry Questions

DIRECTIONS (Questions 1 through 73): Each of (C) jumping from buildings the numbered items in this section is followed by (D) hanging answers. Select the ONE lettered answer that is (E) firearms BEST in each case. Questions 3 and 4 Questions 1 and 2 A mother brings her 7-year-old son to you because she A 17-year-old boy with a history of major depressive is worried about him. She tells you that he sits up in disorder (MDD) comes to your office for a routine in the middle of the night and screams. She says he is visit. When you walk into the examining room, you inconsolable but eventually falls back to . She starts notice that the boy is sitting slumped forward with crying as she tells you she cannot sleep after these his head bent down. He does not make eye contact episodes and is exhausted at work during the day. and says nothing. You suspect that he is having a recurrence of depressive symptoms and are con- 3. Which of the following initial responses is most cerned about his risk for suicide. appropriate?

1. Which of the following statements regarding (A) “I can see this is very upsetting for you.” depression and suicide in adolescents is true? (B) “I know you’re upset, but this problem will get better.” (A) Rates of suicidal behavior are similar in (C) “I can see you’re upset, but you’re really adolescent girls and boys. overreacting.” (B) More girls than boys commit suicide. (D) “Tell me why you can’t fall back to sleep (C) Suicide is a considerable risk in after your son does.” depressed adolescents and should be (E) “Are you worried that this is your fault?” specifically addressed during an interview. 4. During what stage of sleep do these episodes (D) A prior suicide attempt does not most likely occur? increase an adolescent’s risk of a subsequent one. (A) stage 1 (E) The adolescent suicide rate has (B) stage 2 remained stable over the past few (C) stages 3–4 decades. (D) rapid eye movement (REM) stage (E) any stage 2. Which of the following is the most common method by which adolescents commit suicide? (A) drug overdose (B) stabbing or cutting

13

Copyright © 2007 by The McGraw-Hill Companies, Inc. Click here for terms of use. 14 2: Child and Adolescent Psychiatry

Questions 5 and 6 7. Which of the following Diagnostic and Statistical Manual of Mental Disorders, Fourth Edition, Text A 17-year-old girl with a history of asthma presents Revision (DSM-IV-TR) diagnoses is most for a physical examination prior to entering college. appropriate? You note that she appears sad. Upon further ques- tioning, you learn that she has felt depressed for the (A) disorder of written expression past 6 months since breaking up with her boyfriend (B) expressive language disorder of 2 years. She says she feels tired all the time and (C) phonological disorder comes home from school every day, lies on the couch, (D) stuttering and sleeps or watches television. She reports that she (E) developmental coordination disorder quit the senior celebration committee, no longer “hangs out” with her friends, cannot imagine things 8. Which of the following would be most helpful will improve, and is considering not going to college. in confirming the diagnosis? In addition, she has been binge eating and abusing laxatives. Her physical and laboratory examinations (A) Intelligence quotient (IQ) test are normal. You diagnose her with MDD, single (B) Children’s Assessment Scale (CAS) episode and bulimia nervosa, purging type. (C) Diagnostic Interview for Children and Adolescents (DICA) 5. Which of the following is the most appropriate (D) Diagnostic Interview Schedule for pharmacologic agent to use to treat this girl’s Children (DISC) MDD? (E) Schedule for Affective Disorders and (A) carbamazepine (Tegretol) Schizophrenia for School-Age Children (B) imipramine (Tofranil) (K-SADS) (C) lithium (D) olanzapine (Zyprexa) Questions 9 and 10 (E) sertraline (Zoloft) A 6-month-old girl is brought in by her grandmother for a routine visit. The physical examination is 6. Which of the following types of psychother- normal except that the baby has not gained weight as apy is most effective for the treatment of expected. As of earlier visits, the baby had been gain- bulimia nervosa in adolescents? ing weight appropriately. The grandmother reports (A) cognitive-behavioral therapy that the baby likes to eat and is generally happy and playful. On further questioning, the grandmother (B) family therapy describes chaotic mealtimes at home and says that the (C) psychoanalytic psychotherapy baby drools more than usual. (D) psychodynamic psychotherapy (E) group therapy 9. Which of the following DSM-IV-TR eating disorders is the most likely diagnosis? Questions 7 and 8 (A) anorexia nervosa A 6-year-old boy is brought to the clinic by his par- (B) bulimia nervosa ents at the request of the boy’s teachers. The teachers (C) pica report that he is quiet in class. When he does talk, he (D) rumination disorder frequently makes errors with verb tense. His parents (E) feeding disorder of infancy or early recall that his speech was delayed. On examination, childhood the boy is friendly and cooperative. His speech is clear, but he uses simple sentences with a limited vocabulary. Otherwise, he has a normal physical and laboratory examinations are normal. Questions: 5–15 15

10. Which of the following tests would be most (A) schizophrenia helpful in making the diagnosis? (B) panic disorder (A) serum lead level (C) autistic disorder (B) calculation of the percentage of expected (D) moderate mental retardation weight that the child’s weight is (E) substance-induced psychotic disorder (C) 24-hour video monitoring (D) complete blood count (CBC) 14. Which of the following is the most common psychiatric emergency in the child and adoles- (E) esophageal pH measurement cent population? Questions 11 and 12 (A) psychotic behavior (B) assaultive behavior An 8-year-old boy is referred to you by a school nurse because he has been complaining of stomachaches (C) suicidal behavior every morning in school. On interviewing the boy’s (D) homicidal behavior mother, you learn that he does not like to go to (E) sexually inappropriate behavior school, insists on coming home immediately after school each day, and sleeps in his parents’ bed at Questions 15 and 16 night. The mother denies other complaints. A 15-year-old girl with a history of separation anxi- 11. Which of the following is the most likely ety disorder presents complaining of distress over diagnosis? her habit of biting her fingernails. Since starting high school 6 months earlier, the girl says that her nail- (A) social phobia biting has been “out of control” and that she is (B) posttraumatic stress disorder (PTSD) extremely embarrassed about how her fingers look. (C) separation anxiety disorder All her fingers bleed every day, and they throb con- (D) reactive attachment disorder of early stantly. The girl reports that high school is stressful childhood and that her grades are suffering because her nail- biting interferes with her ability to do her homework. (E) specific phobia She denies obsessional thoughts or compulsions. Physical and laboratory examinations are normal, 12. This boy’s stomachaches are an example of with the exception of the girl’s hands. Her fingers which of the following? show signs of extreme nail-biting and there are abra- (A) coprolalia sions on the back of her right hand. (B) anhedonia (C) reaction formation 15. Which of the following is true of the girl’s aca- demic difficulties related to her nail-biting? (D) coprophagia (E) somatization (A) They are part of the diagnostic criteria for her Axis I diagnosis and therefore do Questions 13 and 14 not need to be coded elsewhere. (B) They should be coded on Axis II. A 5-year-old boy is brought to the pediatric emer- (C) They should be coded on Axis III. gency department because he is frantically trying to run away from lions that he says are chasing him. He (D) They should be coded on Axis IV. says that when he sees the lions he has to jump out (E) They should be coded on Axis V. of their way or else they will attack him.

13. Which of the following disorders is the most likely diagnosis? 16 2: Child and Adolescent Psychiatry

16. Which of the following responses is most previously initiated treatment with intranasal desmo- appropriate to use to determine the etiology of pressin (DDAVP). You complete a full physical and the abrasions on the dorsum of the girl’s right laboratory examination. hand? 19. During this visit, you would be most likely to (A) “Tell me about the scratches on your identify which of the following signs and hand.” symptoms as an adverse effect of DDAVP? (B) “Do you scratch yourself in addition to biting your nails?” (A) sedation (C) “How did the scratches happen?” (B) hypotension (D) “I see you have scratches on your hand. (C) liver toxicity Do you have a cat?” (D) tremor (E) “Sometimes I see girls who make (E) headache themselves throw up. Have you ever done that?” 20. Which of the following laboratory abnormali- ties are you most likely to identify in association Questions 17 and 18 with DDAVP? An 8-year-old boy with a history of MDD treated (A) hyponatremia with fluoxetine (Prozac) is brought to the pediatric (B) hypocalcemia emergency department after running into the street (C) hypokalemia in front of a car on the way home from school. The (D) hypomagnesemia boy says that he wants to be dead and refuses to say (E) hypophosphatemia more. The boy was not hit by the car, and physical and laboratory examinations are normal. Questions 21 and 22

17. Which of the following symptoms of MDD is A 9-year-old boy with a history of panic disorder seen more commonly in children than in ado- treated with cognitive-behavioral therapy is brought lescents? to your office by his mother because he has been irri- (A) table and depressed. On physical examination, the boy appears depressed but otherwise normal. (B) hopelessness Laboratory examination is normal. After a thorough (C) psychomotor agitation history and mental status examination, you diagnose (D) weight change the boy with MDD and decide to initiate treatment (E) drug abuse with fluoxetine.

18. Which of the following is the most common 21. What is the approximate comorbidity of child- method by which children attempt suicide? hood anxiety disorders and MDD? (A) substance ingestion (A) 10% (B) stabbing or cutting (B) 25% (C) jumping from buildings (C) 33% (D) hanging (D) 50% (E) firearms (E) 75%

Questions 19 and 20 22. You inform the boy and his mother of possible adverse effects of fluoxetine. Which of the fol- An 11-year-old boy with enuresis presents to the lowing is the most common adverse effect of clinic for routine follow-up. His bed-wetting has not fluoxetine? responded to behavioral interventions, so you had Questions: 16–27 17

(A) hypotension she had a normal head circumference at birth, at (B) nausea 6 months, and at 12 months, and had seemed to be (C) liver toxicity developing normally. On physical examination, you note that the rate of head growth has slowed. (D) weight gain (E) sedation 25. Which of the following pervasive develop- mental disorders is the most appropriate Questions 23 and 24 diagnosis? A 3-year-old girl with moderate mental retardation is (A) autistic disorder brought to the clinic by her mother because she has (B) Rett disorder been complaining that her arm hurts. On physical (C) childhood disintegrative disorder examination, the girl has several bruises on her thorax and a tender right forearm. On laboratory examina- (D) Asperger disorder tion, you discover a normocytic, normochromic (E) pervasive developmental disorder not anemia and basophilic stippling. On x-ray, you find otherwise specified signs of multiple fractures of different ages and increased density at the metaphyseal plate of the 26. Which of the following statements best describes growing long bones. this girl’s disorder? (A) It is seen exclusively in girls. 23. Which of the following is the most appropriate (B) There is never an Axis II diagnosis. treatment? (C) There are no clear characteristic (A) edetate calcium disodium (CaEDTA) chromosomal abnormalities associated (B) flumazenil (Romazicon) with the disorder. (C) naloxone (Narcan) (D) There are familial clusters of the (D) deferoxamine (Desferal) disorder. (E) acetylcysteine (Mucomyst) (E) It is the most common of the pervasive developmental disorders. 24. Which of the following is the most likely cause of this girl’s bruises? Questions 27 and 28 (A) thrombocytopenia A 12-year-old boy is referred by the court for evalu- (B) developmental coordination disorder ation. He skips school, stays out late at night, and ver- (C) malnutrition bally abuses his parents. He has run away from home on three separate occasions prompting his parents to (D) physical abuse call the police. (E) hepatic failure 27. This adolescent is most likely to have an addi- Questions 25 and 26 tional Axis I diagnosis from which of the fol- A 24-month-old girl is brought to the clinic by her lowing DSM-IV-TR diagnostic categories? mother for a routine visit. The mother tells you that (A) substance-related disorders the girl has not spoken her first clear word yet, at (B) schizophrenia and other psychotic times seems not to understand what people say to disorders her, and does not play with her 3-year-old brother. (C) anxiety disorders The mother also tells you that her daughter seems clumsy and has started to make odd repetitive move- (D) eating disorders ments with her hands. According to the girl’s chart, (E) somatoform disorders 18 2: Child and Adolescent Psychiatry

28. Which of the following personality disorders is test is negative but you are concerned about the this boy most likely to develop? possibility of developmental delays due to the mother’s cocaine use. You learn that this girl (A) antisocial personality disorder has a 2-year-old brother who is infected with (B) paranoid personality disorder HIV. Which of the following is the best place for (C) schizoid personality disorder HIV-infected children under the age of 3 who (D) schizotypal personality disorder cannot be cared for by their family? (E) avoidant personality disorder (A) a hospital (B) a group home Questions 29 and 30 (C) a foster home A 10-year-old girl who has recently been diagnosed (D) a nursing home with diabetes mellitus Type I is referred to you by her (E) a hospice pediatrician for an evaluation. You notice that she seems sad. After interviewing the girl and her par- Questions 32 and 33 ents, you determine that she meets DSM-IV-TR cri- teria for adjustment disorder with depressed mood. A 9-year-old boy is referred to you for evaluation after increasingly disruptive behavior in school. The 29. Which of the following criteria for the diagno- teachers report that at any time, without warning, the sis of adjustment disorder distinguishes it from boy will make a disruptive sound or shout out in MDD? class. They describe him as polite and neat but rest- less and jumpy. (A) Symptoms develop following an identifiable stressor. 32. Which of the following is the most likely (B) Symptoms develop within 3 months of diagnosis? the onset of the stressor. (C) Symptoms do not persist for more than (A) oppositional defiant disorder 6 months following termination of the (B) conduct disorder stressor. (C) separation anxiety disorder (D) Symptoms cause marked distress or (D) panic disorder significant impairment in functioning. (E) Tourette disorder (E) Symptoms do not represent bereavement. 33. Which of the following medications is most 30. Approximately what percentage of children who appropriate? are diagnosed with diabetes mellitus Type I (A) bupropion (Wellbutrin) develop adjustment disorder following their medical diagnosis? (B) paroxetine (Paxil) (C) venlafaxine (Effexor) (A) 1% (D) haloperidol (Haldol) (B) 5% (E) clonidine (Catapres) (C) 10% (D) 33% Questions 34 and 35 (E) 75% A 7-year-old boy with leukemia is referred to you 31. A 9-month-old girl is brought to the clinic by because of concerns for his mood. His parents report her foster mother for a routine examination. that he fluctuates between appearing depressed and The girl’s biological mother was infected with acting angry. At times he plays quietly in his room, human immunodeficiency virus (HIV), was but at other times he displays angry outbursts, often enrolled in a methadone maintenance program, hitting his 4-year-old brother. and was using cocaine. The baby girl’s HIV Questions: 28–39 19

34. Which of the following methods would be the 37. Which of the following is true regarding the most useful way to interview this child? DSM-IV-TR diagnosis of Tourette disorder? (A) Sit quietly and allow the child to talk. (A) Motor and vocal tics must occur (B) Encourage the child to talk about concurrently. whatever is on his mind. (B) Onset must be before the age of 21. (C) Provide toys and allow the child to play. (C) Tics must be present every day (D) Use the DISC. consecutively for at least 1 year. (E) Ask the child specific questions about (D) Vocal tics must improve production of how he feels about his disease. words. (E) Multiple motor tics must occur. 35. Which of the following is true for children with a medical illness and a psychiatric illness? Questions 38 and 39

(A) The psychiatric illness does not need to After you complete a routine physical examination of be treated until the medical illness is a healthy 11-year-old girl, the girl’s mother asks to resolved. talk with you in private. She tells you that her daugh- (B) The psychiatric illness should not be ter was given an IQ test in school and that she scored treated pharmacologically in the setting a 68. The mother is extremely upset and wonders if of a medical illness. it is her fault that her daughter is “dumb.” She says (C) The psychiatric illness should be treated she lies awake at night worrying and wonders why by the same physician who is treating her daughter would have done so poorly on the test. the medical illness. (D) The psychiatric illness should be treated 38. Which of the following is the most common aggressively in the setting of a medical predisposing factor for mental retardation? illness. (A) heredity (E) Psychotherapy is unlikely to be effective (B) pregnancy and perinatal problems in the setting of a medical illness. (C) early alterations in embryonic development Questions 36 and 37 (D) general medical conditions acquired An 8-year-old boy presents to your office for a routine during infancy and childhood visit. One month earlier, you diagnosed him with (E) environmental influences and mental Tourette disorder and prescribed medication. He and disorders other than mental retardation his parents report that the medication has been helpful. 39. According to DSM-IV-TR, mental retardation 36. Which of the following is the most common is coded on which of the following axes? initial symptom of Tourette disorder? (A) Axis I (A) eye tics such as blinking (B) Axis II (B) facial tics such as grimacing or licking (C) Axis III (C) vocal tics, such as throat clearing or (D) Axis IV grunting (E) Axis V (D) whole-body tics, such as body rocking or pelvic thrusting (E) self-abusive tics such as hitting 20 2: Child and Adolescent Psychiatry

Questions 40 and 41 (D) Asperger disorder (E) schizophrenia An 8-year-old boy with a family history of tic disorders is referred to you for an evaluation of behavioral dif- 43. Which of the following percentages approxi- ficulties in school. His teachers report that he is unable mates those children and adolescents with to sit still, constantly fidgets, and is unable to complete learning disorders who drop out of school? class work because he is so easily distracted. The boy’s mother reports that he has always had a lot of energy. (A) 5% She says that preparing to leave for school in the morn- (B) 10% ing is extremely difficult because of her son’s disor- (C) 25% ganization and forgetfulness. Otherwise, she has no (D) 40% complaints. She denies that her son produces any repetitive movements or sounds. (E) 66%

40. Which of the following is the most likely Questions 44 and 45 diagnosis? An 8-year-old child is referred to you for an evalua- (A) dementia tion of bed-wetting. Several behavioral interventions (B) conduct disorder have been attempted, including eliminating fluid intake in the evening, scheduled awakenings at night (C) oppositional defiant disorder to use the bathroom, and a urine alarm (a bell and (D) attention deficit hyperactivity disorder pad). These techniques have been unsuccessful, and (ADHD) the child continues to urinate in bed every night. (E) disruptive behavior disorder not otherwise specified 44. A child with enuresis is most likely to be which of the following? 41. Which of the following classes of medications is most likely to unmask an underlying pre- (A) an otherwise normal boy disposition to developing tics? (B) an otherwise normal girl (C) a child with mental retardation (A) stimulants (D) a child with autistic disorder (B) selective serotonin reuptake inhibitors (SSRIs) (E) a child with obsessive-compulsive disorder (OCD) (C) monoamine oxidase inhibitors (MAOIs)

(D) D2 antagonists 45. Which of the following medications is most (E) benzodiazepines appropriate for you to prescribe to treat the enuresis? Questions 42 and 43 (A) methylphenidate (Ritalin) A 6-year-old boy is referred to you by his school to (B) paroxetine evaluate his difficulty with reading. It is suspected (C) trazodone (Desyrel) that he suffers from a reading disorder, but a subtle (D) imipramine impairment in vision or hearing must be ruled out as a causative or contributing factor. (E) benztropine (Cogentin)

42. Learning disorders are most commonly asso- Questions 46 and 47 ciated with which of the following disorders? A 10-year-old boy is referred to you because his pedi- (A) ADHD atrician suspects that he may have ADHD. After a (B) bipolar disorder thorough history, physical examination, and labora- tory investigation you make the diagnosis of ADHD. (C) Tourette disorder Questions: 40–51 21

After discussing the adverse effects of medications, 49. At 22 months of age, this girl should be able to you prescribe methylphenidate to be taken in the do which of the following? morning and at lunch on school days. (A) copy a circle 46. Which of the following is a common adverse (B) copy a square effect of methylphenidate? (C) identify her left hand (D) tell her age and gender (A) tremor (E) ride a tricycle (B) hypotension (C) weight gain Questions 50 and 51 (D) (E) liver toxicity A 4-year-old boy is referred to you for evaluation because he has not started walking. He appears healthy and well-cared for. He readily comes with 47. Stimulants such as methylphenidate effectively you to the examining room. In your office, he notices treat symptoms of ADHD in what percentage a windup toy and immediately becomes engrossed of patients? in winding it up, watching it move around until it (A) 10% winds down, and winding it up again. After about (B) 25% 10 minutes, you attempt to take the toy from him (C) 33% and he becomes extremely upset, making an insis- tent, piercing cry. (D) 50% (E) 70% 50. Which of the following is the most likely diagnosis? Questions 48 and 49 (A) oppositional defiant disorder During a routine office visit, the mother of a 22-month- (B) conduct disorder old girl tells you that she is concerned about her (C) separation anxiety disorder daughter’s behavior. Since the birth of her son 4 months earlier, the mother states that her daughter (D) autistic disorder is more irritable and angry. The mother is especially (E) selective mutism concerned that her daughter is aggressive and violent toward the baby. 51. Which of the following qualities is associated with a more favorable prognosis for children 48. Which of the following statements is the most similar to this boy? appropriate response to this mother? (A) easily toilet trained (A) “If you simply ignore your daughter’s (B) organized in play behavior, it will pass.” (C) interested in mechanical toys (B) “Your daughter’s behavior is normal for (D) able to converse her age.” (E) able to dance to a beat (C) “Every day, schedule some time for you and your daughter to spend together without the baby.” (D) “Explain to your daughter that the baby needs more attention that she does right now.” (E) “Your daughter must be precocious since the terrible twos are starting early.” 22 2: Child and Adolescent Psychiatry

Questions 52 and 53 (C) “This behavior suggests that you’re not spending enough time with your son.” A 17-year-old girl comes to your office for a routine (D) “It sounds as though your son is overly visit. She states that she feels fine and offers no com- attached to you.” plaints. On physical examination, you find that her weight is 92 lb and her height is 5 ft 5 in. One year (E) “This behavior is normal at your son’s earlier, her weight had been 126 lb, and her height age and will pass with time.” was 5 ft 5 in. After further discussion, you learn that this girl is terrified of gaining weight and thinks 55. Which of the following statements is true that she looks fat and needs to lose more weight. regarding attachment by an infant to a care- She reports that she has not menstruated in the past taker? 6 months. (A) Attachment is prevented by mental illness in the caretaker. 52. Which of the following laboratory abnormali- (B) Attachment requires consistent ties are you most likely to discover? interaction with the caretaker. (A) hypercholesterolemia (C) Attachment is prevented by physical (B) increased thyroid-stimulating hormone abuse from the caretaker. (TSH) (D) Attachment usually does not occur if (C) hypocarotenemia the infant is blind. (D) leukocytosis (E) Attachment requires contact beginning (E) hyperkalemia in the perinatal period.

53. Which of the following findings is most con- Questions 56 and 57 sistent with this girl’s disorder? A 14-year-old boy is referred to you due to extreme (A) sinus tachycardia difficulties in school. He has been held back a grade (B) increased resting energy expenditure and is still not passing. During the course of your evaluation, you learn that the boy hears voices telling (C) elevated serum estrogen levels in girls him that he is stupid and to leave the classroom. (D) elevated serum testosterone levels in Afraid to disobey, he goes to the bathroom frequently. boys He has difficulty falling asleep at night because the (E) increased ventricular-brain ratio on voices keep him awake. In addition, you learn that brain imaging the boy believes others can read his thoughts. Physical and laboratory examinations are normal. Questions 54 and 55 56. You suspect that the boy may be suffering from An 8-month-old boy is brought to the clinic by his schizophrenia. Which of the following sub- mother. She complains that her son has been experi- types of schizophrenia is most probable? encing screaming and crying fits when she leaves him with a babysitter. She says that in the past he did (A) paranoid type not object to being left with a babysitter and asks (B) disorganized type you why he becomes so upset now and what she can (C) catatonic type do about it. (D) undifferentiated type 54. Which of the following statements is the most (E) residual type appropriate response? 57. Which of the following is the worst prog- (A) “This behavior is characteristic of nostic feature of schizophrenia with onset in autistic children.” childhood? (B) “It is possible that your son has separation anxiety disorder.” Questions: 52–63 23

(A) affective symptoms (C) 5 years (B) acute onset (D) 7 years (C) onset before the age of 10 (E) 12 years (D) good premorbid adjustment (E) well-differentiated symptoms 61. Which of the following defense mechanisms is the boy employing when he starts sucking Questions 58 and 59 his thumb? (A) A 9-year-old girl with a family history of bipolar dis- order is referred to you by her school because of dis- (B) acting out ruptive behavior in class that has been worsening (C) denial over the past 3 months. Her teachers report that her (D) regression energy level is high. (E) blocking

58. Which of the following would be most helpful Questions 62 and 63 in distinguishing between mania and ADHD in children? A 12-year-old boy with Tourette disorder comes to your office for a routine visit. Two weeks earlier, you (A) pressured speech had prescribed clonidine for his illness. The boy (B) distractibility reports that his tics have subsided slightly since start- (C) motoric overactivity ing the clonidine, but complains about the medicine. (D) low self-esteem (E) impulsivity 62. This boy is likely experiencing which of the following as an adverse effect of clonidine? 59. Which of the following disorders is more (A) sedation common among girls than among boys? (B) light-headedness due to hypotension (A) ADHD (C) dry mouth (B) conduct disorder (D) tremor (C) Rett disorder (E) nausea (D) Tourette disorder (E) Asperger disorder 63. If this boy’s 7-year-old brother were to have ADHD, which of the following classes of med- Questions 60 and 61 ications would be the most appropriate choice for him? A 6-year-old boy is brought to your office by his (A) a stimulant such as methylphenidate mother for evaluation of an upper respiratory infec- tion. The mother mentions that her son has started (B) a tricyclic antidepressant (TCA) such as sucking on his thumb again. In addition, she men- desipramine (Norpramin) tions that the boy’s grandmother died recently and (C) an SSRI such as fluoxetine asks how children view death. (D) an MAOI such as phenelzine (Nardil) (E) a neuroleptic such as haloperidol 60. At what age can a child normally appreciate (Haldol) that death is irreversible? (A) 2 years (B) 3 years 24 2: Child and Adolescent Psychiatry

64. A 6-year-old boy is brought to the emergency shaking, gasping for air, and feeling like she is department by his mother, who reports that he going to die. These feelings intensify for a few was playing on some steps in front of the house minutes and resolve spontaneously. These when he slipped and fell. She tells you that she episodes have occurred at various times, in is concerned that he might have broken his various situations, and the girl is worried that arm. An x-ray of the boy’s arm shows a fracture she is going crazy. Which of the following is the of the ulna, as well as signs of several old frac- most appropriate treatment? tures of varying ages. Which of the following is (A) valproate (Depakote) the most appropriate action? (B) risperidone (A) Refer the boy to an orthopedist for (C) clozapine (Clozaril) further evaluation. (D) sertraline (B) Tell the mother that you notice that the (E) carbamazepine boy has had multiple broken bones and recommend that she limit the boy’s 67. A 5-year-old girl with asthma is seen by her sports activities. pediatrician for a routine visit. After returning (C) Set the current broken bone in a cast home from the clinic, the girl asks her friend to and recommend that the boy see his “play doctor.” This behavior is a manifestation pediatrician for follow-up care. of which of the following phenomena? (D) Recommend calcium supplements and a multiple vitamin daily. (A) reaction formation (E) Tell the boy that you notice that he has (B) identification had multiple broken bones and ask him (C) displacement how each of these fractures happened. (D) rationalization (E) dissociation 65. The mother of one of your 4-year-old patients calls and asks you for advice. She says that her 68. A 4-year-old boy whose mother had rubella son has been stealing toys from other children during her pregnancy is brought to the clinic at school and bringing them home with him. for a routine visit. The physical and laboratory Which of the following suggestions is most examinations are normal. The highest risk of appropriate? physical and/or mental defects in the fetus as (A) The mother should return the toys a result of maternal rubella infection occurs herself and buy her son similar toys. during which month of the pregnancy? (B) The mother should tell her son that if he (A) first stops stealing, she will give him a (B) third reward each week. (C) fifth (C) The mother should point out to her son (D) seventh that he is stealing and punish him for it. (E) ninth (D) The mother should help her son return the toys. 69. A 10-year-old girl with a history of asthma is (E) The mother should ignore the entire brought to the clinic after a recent increase in situation because her son’s behavior is asthma symptoms. During the visit, you learn part of a developmental phase that will that she is being beaten by her mother’s pass. boyfriend. Under which of the following cir- cumstances, does the law require mandatory 66. A 14-year-old girl presents to her pediatrician reporting of suspected child abuse by a physi- complaining that she has been freaking out. cian who evaluates a child? The girl describes episodes of palpitations, Questions: 64–73 25

(A) in all cases 72. A frustrated mother brings her 14-year-old son (B) only when the physician believes it is in to a child psychiatrist after he is expelled from a child’s best interests three high schools in 1 year. She reports the (C) only when consent of a parent or boy has tried twice to set his school on fire, has guardian is obtained slashed school bus tires, and has broken into the principal’s office to steal athletic trophies. In (D) only in cases in which the child shows addition, he has been suspended numerous behavioral manifestations of abuse times for getting into fights with other stu- (E) only when the physician has examined dents. She shudders and tearfully relates that all children in the family she recently caught him singeing one of the family cats with a cigarette butt. The diagnosis 70. A 4-year-old boy is referred to you because he of the above disorder is required for which of will not speak in preschool. Over the course of the following disorders? about 2 months, he gradually stops talking. Initially his mother reports that he objected to (A) schizotypal going to preschool, but no longer complains. (B) borderline She states that at times, her son is quiet and (C) obsessive-compulsive stays in his room, but that she has not noticed (D) antisocial a significant change in his speech or behavior. (E) histrionic Which of the following is the most likely diagnosis? 73. A 15-year-old girl is brought into the pediatri- (A) selective mutism cian’s office by her mother, who is concerned (B) MDD about her lack of apparent appetite. The patient (C) separation anxiety disorder is wearing baggy clothes, but her weight is in the 70th percentile for her height. She admits (D) social phobia that she “does not eat much,” but she claims to (E) dysthymic disorder have a voracious appetite. She is active in the school musical productions, and she is very 71. A 2-year-old boy is referred to you for eval- worried about “getting fat” and not getting the uation due to the suspicion that the child is lead part. Her past medical history is unre- the victim of physical abuse secondary to markable, and although she started menstru- Munchausen syndrome by proxy. Which of the ating at age 13, she has not had her period for following family members is most frequently at least 5 months. Which of the following lab- the perpetrator who fabricates the presenting oratory abnormalities would be most likely illness? found in this patient? (A) brother (A) decreased corticotropin-releasing (B) sister hormone (CRH) (C) father (B) hypercholesterolemia (D) mother (C) hyperglycemia (E) uncle (D) hyperthyroidism (E) leukocytosis 26 2: Child and Adolescent Psychiatry

DIRECTIONS (Questions 74 through 83): Each set Questions 76 through 79 of items in this section consists of a list of lettered (A) Rett disorder headings followed by several numbered words or phrases. For each numbered word or phrase, select (B) bulimia nervosa the ONE lettered option that is most closely associ- (C) stereotypic movement disorder ated with it. Each lettered option may be selected (D) social phobia once, more than once, or not at all. (E) separation anxiety (F) pica 74. A 13-year-old girl is seen by her psychiatrist (G) enuresis 1 year after an automobile accident. She demon- (H) conduct disorder strates intact language ability and complex motor skills. She has no abnormalities in the (I) panic disorder perception of stimuli that are identifiable, but (J) Asperger disorder she has lost the ability to read since the accident. What deficit is she demonstrating? 76. A 7-year-old boy with Toxocara and visceral larva migrans (A) anomia (B) aphasia 77. A 17-year-old girl with hypokalemia (C) alexia (D) apraxia 78. A 6-year-old boy with rectal bleeding and (E) agnosia anemia

75. Britney is a 10-year-old girl without significant 79. A 15-year-old boy with a urine toxicology medical history brought to the pediatrician by screen positive for cocaine her father for evaluation. Over the past school year, she has been having increasing difficulties Questions 80 through 84 going to sleep. Although she has “always had (A) OCD rituals,” they have extended in com- (B) pica plexity and length. Most of her time in the (C) hypochondriasis evening is now spent going around the house numerous times, locking and unlocking the (D) Tourette disorder doors and windows. While she knows the (E) panic disorder chances of a burglary are slim, she is extremely (F) bulimia nervosa anxious about her safety, and she “can’t stop” (G) anorexia nervosa the urges to perform these behaviors. As a (H) Asperger disorder result, she only obtains 5 hours of sleep, and (I) pyromania she has been falling asleep in class with dimin- ishing grades. Infection with which of the fol- (J) trichotillomania lowing organisms might have contributed to her current illness? 80. A 14-year-old girl with episodes of palpitations, chest pain, shortness of breath, and diaphore- (A) herpes simplex virus sis who has a normal physical and laboratory (B) HIV examination (C) prion proteins (D) streptococcus 81. A 10-year-old girl with physical findings that include hypotension, bradycardia, and lanugo (E) treponema hair Questions: 74–90 27

82. An 8-year-old boy with erythematous, chapped (C) conduct disorder hands, and an otherwise normal physical and (D) oppositional defiant disorder laboratory examination (E) childhood schizophrenia

83. A 13-year-old girl with a bald patch on the back DIRECTIONS (Questions 86 through 95): Each set of her head and an otherwise normal physical of items in this section consists of a list of lettered and laboratory examination headings followed by several numbered words or phrases. For each numbered word or phrase, select DIRECTIONS (Questions 84 and 85): Each of the the ONE lettered option that is most closely asso- numbered items in this section is followed by ciated with it. Each lettered option may be selected answers. Select the ONE lettered answer that is once, more than once, or not at all. BEST in each case. Questions 86 through 89 Questions 84 and 85 (A) autism 84. An 8-year-old boy is brought to your office by (B) selective mutism his anxious mother who says that he cannot (C) childhood schizophrenia read at grade level. She says he was doing well until last year in second grade when he began (D) Rett disorder falling behind. He has become somewhat of a (E) Asperger disorder behavioral problem in school since his trou- (F) ADHD bles with reading began. His mother insists that he is “at least as smart as every other kid 86. A 6-year-old girl whose parents are going in that school” but says she would be relieved through a divorce will not speak during school. to hear that he has a learning disorder “so he can get the help he needs.” You read with the 87. A 6-year-old girl who had reached normal early child and confirm that he is having difficulty. development milestones now does not speak in You should do which of the following? school or at home. (A) Tell the mother to schedule some 88. A 6-year-old boy who performs well in school sessions with a reading specialist. but does not seem to talk when other children (B) Ask the child if he likes to read. approach him and generally avoids other (C) Test the child’s vision. children. (D) Prescribe methylphenidate for ADHD. (E) Reassure the mother that her son is 89. A 6-year-old boy who is having difficulty in doing fine. school and avoids interactions with his class- mates and others, who notice him making odd 85. As the school psychologist, you are asked to see repetitive movements with his hands. a fourth grader who has been consistently acting out in class. He often lies about things he has Questions 90 through 95 done in class, such as trying to cheat on tests. (A) mild mental retardation You learn that at home he sprays water on the (B) moderate mental retardation family dog and sometimes barks at him. After he does so, he is seen cackling to himself. The most (C) severe mental retardation likely diagnosis is which of the following? (D) profound mental retardation (E) average intelligence (A) autistic disorder (B) Tourette disorder 28 2: Child and Adolescent Psychiatry

90. A 14-year-old boy with an IQ of 34 93. A 14-year-old boy with an IQ of 52

91. A 14-year-old boy with an IQ of 96 94. A 14-year-old boy with an IQ of 24

92. A 14-year-old boy with an IQ of 68 95. A 14-year-old boy with an IQ of 56 Answers and Explanations

1. (C) Suicide is a considerable risk in depressed other than her son’s episodes and her daytime adolescents, and should be specifically addressed sleepiness. Specifically directed statements, during an interview with a patient who appears such as “Tell me why you can’t fall back to depressed or agitated or has a history of a suicide sleep after your son does,” or leading ques- attempt. Surveys of adolescents in the general tions, such as “Are you worried that this is community have estimated that approximately your fault?” are less likely to be helpful in gain- 9% of adolescents have attempted suicide at least ing this information than an open-ended state- once in their lives. Among adolescents with ment or question. Usually, any statement known psychiatric disorders, the rate is much containing “but” is inappropriate as the initial higher. More girls than boys demonstrate suici- response to a person who is describing a prob- dal behavior, but nearly five times more teenage lem to you. Statements such as “I know you’re boys successfully commit suicide than do upset, but this problem will get better” or “I can teenage girls, because boys more frequently use see you’re upset, but you’re really overreact- guns and violent methods to attempt suicide. ing” may indicate that you do not take the History of a prior suicide attempt is an important problem seriously or that you are minimizing risk factor for suicide in all age groups; the major- how the person feels. Such an approach may ity of those who successfully complete suicide prevent you from obtaining further important have attempted suicide in the past. The adoles- information. cent suicide rate has increased substantially during the past few decades. 4. (C) Episodes of sleep terror, as well as of sleep- walking, occur during deep sleep (stages 3–4). 2. (E) The use of firearms is the most common Sleep terror does not occur during stage 1 or 2 method by which adolescents die by suicide. of sleep or during REM sleep. Drug overdose is the most common method of suicide attempts, but accounts for fewer com- 5. (E) An SSRI, such as sertraline, is the first-line pleted suicides. Stabbing, cutting, jumping agent for MDD in children and adolescents. from buildings, and hanging are other methods TCAs such as imipramine cause more adverse frequently employed. effects than SSRIs typically do, and an over- dose is much more likely to be lethal. Mood sta- 3. (A) A simple, nonjudgmental statement, such bilizers such as carbamazapine and lithium are as “I can see this is very upsetting for you,” used for bipolar disorder and as adjuncts to conveys to the mother that you are concerned the treatment of MDD refractory to antide- about her and that, at least on some level, you pressant medications alone. Antipsychotics understand her and sympathize with her suf- such as olanzapine are usually reserved for use fering. This form of sentence also acts as an as adjuncts when psychosis develops. open-ended question that is most likely to elicit the most important information most efficiently. 6. (A) Bulimia nervosa is most effectively treated The mother may be most troubled by something with cognitive-behavioral therapy. Family

29 30 2: Child and Adolescent Psychiatry

therapy, psychoanalytic psychotherapy, and overweight. Similarly, she is not suffering from group therapy may be used as adjuncts to cog- bulimia nervosa because she does not display nitive-behavioral therapy, but are not as effec- episodes of binge eating with compensatory tive in changing the behaviors associated with behavior, such as self-induced vomiting or bulimia nervosa. misuse of laxatives. A diagnosis of pica would require that the baby persistently eat nonnu- 7. (B) This boy most likely suffers from expressive tritive substances such as dirt, plaster, hair, language disorder. Typical symptoms include bugs, or pebbles. A diagnosis of feeding disor- having a markedly limited vocabulary, making der of infancy or early childhood is not appro- errors in tense, and having difficulty recalling priate because the baby’s disturbance is better words or producing sentences with develop- accounted for by a diagnosis of rumination mentally appropriate length or complexity. In disorder. addition, the boy’s difficulties are interfering with academic functioning. There is no indica- 10. (E) To confirm the diagnosis of rumination dis- tion that this boy has difficulty with written order, an associated gastrointestinal or other expression, so a diagnosis of disorder of written general medical condition, such as esophageal expression is inappropriate. This boy does not reflux, must be ruled out. Esophageal pH meas- suffer from a phonological disorder because his urement indicates if acidic gastric contents speech is clear and he has not failed to develop move into the esophagus during periods when expected speech sounds. Similarly, he is not suf- the infant is not eating or actively regurgitating, fering from stuttering, a DSM-IV-TR communi- as occurs in esophageal reflux. Measurement of cations disorder diagnosis, because there is no the serum lead level is appropriate when pica disturbance in the fluency and time patterning is suspected, because old paints may contain of his speech. There is no indication that he has lead, which can cause significant neurologic difficulty with motor coordination, so a diag- problems when ingested. Calculating the per- nosis of developmental coordination disorder centage of expected weight represented by the would be inappropriate. infant’s current weight would be required for a diagnosis of anorexia nervosa (85% or less) but 8. (A) An IQ test would be most helpful in con- is not a criterion for rumination disorder. firming the diagnosis of expressive language Twenty-four-hour video monitoring would disorder because a diagnosis of mental retar- greatly aid in assessing the baby’s behavior dation must be ruled out. The CAS, DICA, and the context in which the disturbance DISC, and K-SADS are semistructured inter- occurs, but would not rule out a gastrointesti- views that aid primarily in the assessment of nal condition as the cause of this problem. mood disorders, anxiety disorders, disruptive Results of a CBC would not be helpful in deter- behavior disorders, and eating disorders. mining the diagnosis.

9. (D) The most likely diagnosis is rumination 11. (C) This boy’s behavior and symptoms are disorder, in which there is repeated regurgita- most consistent with separation anxiety disor- tion and rechewing of food for a period of at der, characterized by developmentally inap- least 1 month following a period of normal propriate and excessive anxiety concerning functioning. Rumination disorder is often seen separation from the home or from those to in infants who have a variety of caretakers in an whom the individual is attached. Consistent unstable environment. The drooling exhibited with this diagnosis, the boy does not like to go by this baby is most likely the milk or food that to school, comes home immediately after school, has been regurgitated. Lack of appropriate sleeps in his parents’ bed at night, and has weight gain is typical when the baby does not repeated physical symptoms when at school. reswallow the food. This baby is not suffering The boy does not suffer from social phobia from anorexia nervosa because she likes to disorder because he does not have a marked eat and does not display fears of becoming and persistent fear of a social or performance Answers: 7–17 31

situation with exposure causing intense anxi- environmental problems that may affect the ety, expressed as a tantrum or a panic attack. diagnosis, treatment, and prognosis of her Axis The boy is not suffering from PTSD because I disorder (or are a significantly distressing there is no evidence of a traumatic event that is effect of her Axis I disorder). The fact that the persistently reexperienced and has caused girl’s behavior is interfering with normal activ- symptoms of increased arousal and avoidance ities (e.g., homework) is part of the diagnostic of associated stimuli. For a diagnosis of reactive criteria for her Axis I diagnosis, but the stress of attachment disorder of infancy or early child- falling grades requires coding on Axis IV. hood, the boy would need to have suffered Mental retardation and personality disorders markedly disturbed social relatedness, in most are coded on Axis II. Medical (physical) dis- contexts, beginning before the age of 5. Finally, orders are coded on Axis III. The Global a diagnosis of specific phobia would require Assessment of Functioning (GAF), the clini- the display of marked and persistent fear cued cian’s judgment of the individual’s overall level by the presence of anticipation of a specific of functioning, is coded on Axis V. object or situation, expressed as a tantrum or panic attack. 16. (A) A direct but open-ended statement, such as “Tell me about the scratches on your hand,” is 12. (E) Somatization is a defense mechanism in most likely to be helpful in this situation. It is which emotional concerns are manifested as not a secret that the abrasions are there, but physical symptoms. Coprolalia is the repetitive the patient has not mentioned them. Usually, a speaking of obscene words, seen in severe cases patient appreciates a direct question rather than of Tourette disorder. Anhedonia is the lack of wonder if you ignored or did not notice some- interest or pleasure in activities that were once thing obvious. Questions such as “Do you enjoyable, seen in MDD. Reaction formation is a scratch yourself in addition to biting your defense mechanism in which an unacceptable nails?” or “Do you have a cat?” are not open- impulse is transformed into its opposite. ended; they require “yes” or “no” answers and Coprophagia is the eating of feces. are unlikely to yield new information. “How did the scratches happen?” may sound accusa- 13. (E) Substance-induced psychotic disorder is the tory; patients are more likely to offer information most common cause of florid visual hallucina- if you seem nonjudgmental. Signs of bulimia tions in a child. The child may have ingested nervosa include erosion of tooth enamel caused prescription drugs, illicit drugs, over-the- by acidic vomitus and abrasions on the dorsum counter medications, or a household agent. of the hand due to scraping by the upper teeth Schizophrenia rarely presents in young chil- as the individual pushes the hand to the back of dren. When it does, it is much more likely to the throat to induce vomiting. If this girl does present with auditory than with not volunteer information that confirms to you visual hallucinations. Hallucinations do not that she is not inducing vomiting, you should occur in panic disorder, autistic disorder, or use a reassuring statement such as “Sometimes mental retardation. I see girls who make themselves throw up. Have you ever done that?” 14. (C) Suicidal behavior is the most common psy- chiatric emergency in children and adolescents. 17. (C) Psychomotor agitation is more commonly Psychotic behavior, particularly assaultive seen in children with MDD compared to ado- behavior, and homicidal behavior are less lescents with MDD. Children with MDD may common emergencies. Sexually inappropriate appear more anxious and irritable than sad and behavior alone is rarely an emergency. depressed. Hypersomnia, hopelessness, weight change, and drug abuse are more commonly 15. (D) This girl’s academic difficulties related to seen in adolescents with MDD compared to her nail-biting should be coded on Axis IV children with MDD. along with any other psychological and 32 2: Child and Adolescent Psychiatry

18. (A) Substance ingestion is the most common by the presence of multiple fractures of differ- method children use when attempting suicide. ent ages. Thrombocytopenia and malnutrition Other methods frequently used by children are rare in the United States. In developmental include stabbing, cutting, jumping from build- coordination disorder, there is a delay in ings, hanging, running in front of vehicles, and achieving major milestones and clumsiness, gas inhalation. Firearms are used less fre- but there is not typically a pattern of repeated quently than substance ingestion but are more injury. Hepatic failure is extremely rare in a often lethal. It is important to note that some 3-year-old child. suicide attempts may be mistaken for acci- dents, so it is important to directly ask chil- 25. (B) This girl’s history and presentation are con- dren if they intended to hurt or kill themselves. sistent with Rett disorder, which is characterized by normal prenatal and perinatal development, 19. (E) Headaches and nausea are common adverse normal head circumference at birth, and normal effects associated with the synthetic antidiuretic psychomotor development through the first hormone DDAVP. Sedation, hypotension, liver 5 months of life. Between the ages of 5 and toxicity, and tremor are not associated with 48 months, there is deceleration of head growth, desmopressin. loss of hand skills with development of stereo- typed hand movements such as hand wring- 20. (A) DDAVP is effective in treating enuresis ing, loss of social interaction (which may because it suppresses urine production for 7–10 improve later), appearance of poorly coordi- hours following administration by an intranasal nated gait or trunk movements, and severely spray. It lacks the pressor effects of an antidiuretic impaired expressive and receptive language hormone, but can cause a mild hyponatremia. development with severe psychomotor retar- Hypocalcemia, hypokalemia, hypomagnesemia, dation. Autistic disorder and pervasive devel- and hypophosphatemia are not side effects of the opmental disorder, not otherwise specified, drug. are not appropriate diagnoses because this girl’s presentation is better accounted for by 21. (D) The approximate comorbidity of childhood Rett disorder. Children with childhood disin- anxiety disorders (overanxious disorder, sepa- tegrative disorder do not develop normally ration anxiety disorder, panic disorder) and for at least 2 years after birth. Children with MDD is 50%. The approximate comorbidity of Asperger disorder do not show delay in lan- childhood anxiety disorders and ADHD is 30%. guage development (e.g., single words are used by the age of 2). 22. (B) The most common adverse effects of fluox- etine include gastrointestinal symptoms, insom- 26. (A) Rett disorder is the only DSM-IV-TR disor- nia, agitation, and headaches. In general, der that is seen only in girls. Children with hypotension, liver toxicity, weight gain, and Rett disorder commonly suffer from mental sedation are not side effects associated with flu- retardation, which is coded on Axis II. Although oxetine. there are no clear characteristic laboratory find- ings associated with the disorder, there is a 23. (A) CaEDTA is a chelating agent used to treat genetic test available. There are no known lead toxicity. Flumazenil is used to reverse the familial clusters of the disorder. Autistic disor- sedating effects of excess benzodiazepines, and der is the most common of the pervasive devel- naloxone is used to reverse the effects of excess opmental disorders. opiates. Deferoxamine is a chelating agent used to treat iron toxicity. Acetylcysteine is used to 27. (A) This boy is most likely to have a substance- treat acetaminophen toxicity. related disorder in addition to conduct disorder. Children and adolescents with conduct disorder 24. (D) The girl’s bruises are most likely the result often have a substance-related disorder and/or of physical abuse, which is strongly suggested ADHD. Schizophrenia and other psychotic Answers: 18–35 33

disorders, anxiety disorders, eating disorders, as purposefully disruptive behavior. Of note, and somatoform disorders are less frequently there is frequent comorbidity with ADHD. This associated with conduct disorder. boy does not suffer from oppositional defiant disorder or conduct disorder because he is 28. (A) Children and adolescents who are diag- polite and neat and does not display hostile, nosed with conduct disorder are at increased destructive, or angry behaviors. There is no risk for antisocial personality disorder as evidence that this boy experiences distress and adults. Antisocial personality disorder is not worry when separated from an important diagnosed until after the age of 18, and one of attachment figure as in separation anxiety dis- the criteria is evidence of conduct disorder order. The outbursts are not typical of panic prior to the age of 15. Earlier onset of conduct disorder, in which there are discrete panic disorder is associated with an increased risk of attacks, periods of intense fear, or discomfort developing antisocial personality disorder. with physical manifestations such as palpita- Conduct disorder is not as closely associated tions and subjective difficulty breathing. with paranoid personality disorder, schizoid personality disorder, schizotypal personality 33. (E) Clonidine has become the first-line treat- disorder, or avoidant personality disorder. ment for Tourette disorder. It has a limited side effect profile and helps control symptoms of 29. (C) By definition, symptoms of adjustment dis- the two frequently associated comorbid disor- order do not last longer than 6 months after a ders: ADHD and OCD. TCAs have been shown stressor or the termination of its consequences. to be effective in the treatment of Tourette dis- If depressive symptoms persist, MDD may be order, but other antidepressants such as bupro- diagnosed. In both adjustment disorder and pion, paroxetine, and venlafaxine are not MDD, symptoms may develop following a known to be effective. High-potency antipsy- stressor, may develop within 3 months of the chotics such as haloperidol and pimozide were onset of a stressor, must cause marked distress traditionally the first-line agents for Tourette or significant impairment in functioning, and disorder. The newer atypical antipsychotics, must not represent bereavement. such as risperidone, are not known to be effec- tive in treating the disorder. 30. (D) Following a diagnosis of diabetes mellitus Type I, approximately 33% of children develop 34. (C) A play interview in which a child is pro- symptoms of adjustment disorder. vided with toys and allowed to play is an effec- tive and efficient way to interview a child. Even 31. (C) A foster home is the optimal place for with encouragement, a child is unlikely to pro- young children with HIV. In a foster home, a duce much spontaneous talk in the uncom- child has the psychological benefit of belonging fortable setting of sitting with a stranger to a family and is more likely to receive ade- evaluating him or her. Likewise, asking spe- quate emotional support than in a group set- cific questions is not as helpful as observing ting. In addition, the risk of opportunistic the child’s play. Semistructured interviews are infections is higher in group settings, such as useful tools in combination with play inter- hospitals, group homes, nursing homes, and views. The DISC is unreliable in 6–9-year-old hospices, than in private home environments. children.

32. (E) Tourette disorder is the most likely diagno- 35. (D) A psychiatric illness in a child or adolescent sis because the boy’s outbursts are consistent with a medical illness should be treated aggres- with vocal tics, and the report of “restless and sively using the type of treatment most effec- jumpy” behavior is consistent with misinter- tive for the specific psychiatric illness. Effective pretation of motor tics. Tourette disorder most treatment of a mental illness may positively commonly develops in grade school-age boys, affect a medical illness if improvement of psy- and the involuntary tics may be misinterpreted chiatric symptoms enables the child to more 34 2: Child and Adolescent Psychiatry

fully participate in treatment of the medical ill- 39. (B) Mental retardation is coded on Axis II. All ness. In addition, emotional state is related to psychiatric disorders except mental retardation immune response. Appropriate pharmacologic and personality disorders are coded on Axis I. treatment should be avoided only if there are Medical (physical) disorders are coded on specific contraindications based on the med- Axis III. Psychosocial and environmental prob- ical treatment the child is receiving. There is no lems of clinical significance are coded on Axis IV. need for the psychiatric illness to be treated by The GAF, the clinician’s judgment of the indi- the same physician who is treating the medical vidual’s overall level of functioning, is coded illness. In general, the effectiveness of psy- on Axis V. chotherapy is related to the specific mental ill- ness, not to the existence of, or lack of, a 40. (D) This boy’s history is typical of ADHD, com- medical illness. bined type. He is fidgety and distractible in school as well as at home, and this is interfering 36. (A) Eye tics such as blinking and eye rolling are with his ability to function. Forgetfulness due to the most common initial symptoms in Tourette distractibility is commonly seen in ADHD and disorder. Facial tics such as grimacing or lick- is not a sign of dementia. This boy does not dis- ing movements and vocal tics such as throat play excessive aggression, destruction of prop- clearing or grunting are the next most common erty, deceitfulness, theft, or serious violations of initial symptoms. Whole-body tics, such as body rules, as seen in conduct disorder. His behaviors rocking or pelvic thrusting, and self-abusive tics, are not negativistic, hostile, or defiant, so he such as hitting, may develop later. does not suffer from oppositional defiant dis- order. Disruptive behavior disorder not other- 37. (E) In Tourette disorder, multiple motor tics wise specified is a diagnosis reserved for cases and one or more vocal tics must be present at in which there are conduct or oppositional defi- sometime during the illness. The motor and ant behaviors that do not meet the full criteria vocal tics do not need to occur concurrently. for conduct disorder or oppositional defiant Onset is before the age of 18. The average age disorder but cause significant impairment. of onset is 7. Tics must be present nearly every day or intermittently for at least 1 year, with no 41. (A) Stimulant medications have been associ- tic-free period longer than 3 consecutive ated with an increased risk of developing tics. months. Vocal tics include various sounds such In general, if a child suffers from tics or has a as grunts, yelps, coughs, and throat clearing, as family history of tics, stimulant medications well as words. should be avoided and a TCA should be used

to treat ADHD if necessary. SSRIs, MAOIs, D2 38. (C) Early alterations in embryonic develop- antagonists, and benzodiazepines are not ment, such as chromosomal changes (e.g., known to exacerbate tics. Down syndrome) and prenatal damage due to toxins (e.g., maternal alcohol consumption), 42. (A) Learning disorders are commonly associ- are the most common predisposing factors for ated with ADHD. Children with conduct dis- mental retardation, accounting for approxi- order, oppositional defiant disorder, MDD, and mately 30% of cases. Heredity accounts for dysthymic disorder are also at increased risk about 5%. Pregnancy and perinatal problems for learning disorders. Bipolar disorder, Tourette account for approximately 10%. General med- disorder, Asperger disorder, and schizophrenia ical conditions acquired during infancy and are not commonly associated with learning childhood account for approximately 5%. disorders. Environmental influences and mental disor- ders, other than mental retardation, account 43. (D) Approximately 5% of students in public for approximately 15–20%. There is no clear schools are identified as suffering from a learn- etiology in 30–40% of cases. ing disorder. Nearly 40% of those students drop out of school. Answers: 36–54 35

44. (A) Most commonly, enuresis occurs in other- social interactions and attachments, restricted wise normal boys. Enuresis is more common in interests and behaviors, and impaired com- boys than in girls. There is no increased preva- munication. He does not object to going with a lence of enuresis in children with mental retar- stranger to the examining room, does not inter- dation, autistic disorder, or OCD. act with the physician, becomes preoccupied with a toy to an abnormal degree, and screams 45. (D) Imipramine, a TCA, is the standard agent when the toy is taken away. He is not primarily used in the pharmacologic treatment of enure- negativistic and hostile as in oppositional defi- sis. Methylphenidate is commonly used in the ant disorder. He is not aggressive, destructive, treatment of attention deficit disorder, paroxe- deceitful, or in violation of rules, as in conduct tine is an SSRI, trazodone is an antidepressant disorder. He does not become upset when sep- most often used for insomnia, and benztropine arated from his mother, as in separation anxi- is used to prevent extrapyramidal symptoms ety disorder. caused by neuroleptic use. 51. (D) Children with autistic disorder have more 46. (D) The most common adverse effects of favorable prognoses if they are able to converse methylphenidate and other stimulant medica- meaningfully rather than unable to interact tions include insomnia, decreased appetite, positively with others. They are often easily weight loss, dysphoria, and irritability. Tremor, toilet trained, often enjoy mechanical toys, are hypotension, weight gain, and liver toxicity are rigidly organized in their play, and may not common. become preoccupied with the rhythm or beat of music. These latter characteristics are not asso- 47. (E) Stimulant medications, such as methyl- ciated with a good prognosis. phenidate, are effective in diminishing symp- toms of ADHD in approximately 70% of 52. (A) Hypercholesterolemia is common in patients. anorexia nervosa. Other findings associated with the starvation state are mild normo- 48. (C) Since the birth of the baby, the parents have cytic normochromic anemia and leukopenia. probably spent a lot of time tending to the new Hypercarotenemia, causing yellowing of the skin, baby’s needs and paying less attention to their may be seen if many carrots are eaten in an daughter. The girl may feel rejected, jealous, attempt to satisfy the appetite with a low-calorie and angry. The most helpful approach is for food. TSH is not typically altered. If vomiting the mother to spend time alone with her is induced, hypokalemia, hypochloremia, and daughter every day, giving her undivided metabolic alkalosis may be seen. attention, and showing her that she is still loved and wanted. The girl is unlikely to be 53. (E) During anorexia nervosa, the body is in a reassured by a single discussion about how the state of starvation and, as a result, most bodily baby needs more attention than she does. Being functions are suppressed. The resting energy consistently irritable, angry, and violent is not expenditure is decreased, there is a sinus brady- normal behavior at 20 months or at 24 months. cardia, serum estrogen levels are low in girls, The girl’s behavior should not be ignored. and serum testosterone levels are low in boys. Brain imaging may show an increase in 49. (A) A developmentally normal child can copy ventricular-brain ratios secondary to starvation. a circle at the age of 2. She can tell her age and gender and ride a tricycle at age 3. She can 54. (E) Stranger anxiety occurs as part of normal copy a square and can identify her left hand at child development and is evidence of the age 5. development of a strong bond of attachment. It does not suggest that a parent is inattentive; it 50. (D) This boy’s behavior is characteristic of usually appears by 7–8 months and generally autistic disorder. He suffers from impaired resolves with time. Typically, stranger anxiety 36 2: Child and Adolescent Psychiatry

is stronger toward completely unknown per- be able to fantasize, and have a solid sense of sons than toward those who are more familiar. his or her own individuality. Usually, this level Autistic children lack this developmental of maturity is achieved between the ages of 6 marker. Because stranger anxiety at this age is and 10. developmentally appropriate, this boy cannot be diagnosed with separation anxiety disorder 61. (D) This boy is displaying regression, a defense and is not overly attached to his mother. mechanism in which there is an attempt to return to an earlier developmental phase to 55. (B) Bonding is believed to begin in the imme- avoid the tension and conflict at the present diate postdelivery period. However, the slow- level of development (e.g., distress over the growing attachment between an infant and his grandmother’s death). Repression is a defense or her caretakers occurs with ongoing interac- mechanism in which an or feeling is tion over time and does not require contact in expelled or withheld from . the perinatal period. Attachment occurs in the Acting out is a defense mechanism in which an face of many obstacles, including sensory unconscious wish or impulse is expressed deficit, maltreatment, and physical disabilities through action to avoid an accompanying on both sides. affect. Denial is a defense mechanism in which the awareness of a painful aspect of reality is 56. (A) Schizophrenia, paranoid type, is the most avoided by negating sensory data. Blocking is a likely diagnosis because of the frequency of the defense mechanism in which a thought, auditory hallucinations without prominent dis- impulse, or effect is transiently inhibited, caus- organization or inappropriate affect, as seen in ing tension or distress. disorganized type, or motor abnormalities, as seen in catatonic type. Criteria for paranoid type are 62. (A) Sedation is a frequent adverse effect of met, so this is not an undifferentiated type. clonidine upon initiating treatment. With con- Hallucinations are not present in residual type. tinued treatment, the sedation usually sub- sides. Dry mouth is a less common adverse 57. (C) Very early onset of schizophrenia, before effect. Hypotension is a rare adverse affect. In the age of 10, is rare and associated with a poor general, tremor and nausea are not seen. outcome. A better outcome is associated with more affective symptoms, acute onset, older 63. (A) Stimulants are the most commonly used age at onset, good premorbid functioning, well- medication in the treatment of ADHD, and differentiated symptoms, and lack of a family although they can unmask tic disorders, history of schizophrenia. ADHD treatment guidelines state that stimu- lants can still be used with tics. A recent study 58. (D) Children with ADHD often suffer from low found that stimulants did not worsen tics in self-esteem, while children with mania are those with tic disorders. Although TCAs are more likely to be euphoric. Pressured speech, as effective as stimulants, there have been distractibility, motoric overactivity, and impul- reports of sudden death using the medications, sivity are seen in children with both disorders. and they are not used. SSRIs may be used to augment treatment in some cases, but not as a 59. (C) Rett disorder occurs exclusively in girls. first-line treatment. MAOIs and neuroleptics ADHD, conduct disorder, Tourette disorder, are not used for ADHD. and Asperger disorder are seen more com- monly in boys. 64. (E) The finding of multiple fractures especially when they are of different ages is a red flag for 60. (D) Typically, children are 7 or 8 years old when physical abuse. Even though the boy may be they begin to understand the irreversibility of scared to report what happened for fear of pun- death. To understand that death is irreversible, ishment, it is important to try to talk with him a child must be aware of the continuity of time, alone and find out as much as possible. It is not Answers: 55–73 37

appropriate to refer the boy to an orthopedist defects decreases inversely with the duration of or his pediatrician prior to investigating the pregnancy at the time of disease. possibility of physical abuse. Recommending limited sports or extra vitamins does not 69. (A) In all jurisdictions in the United States, cli- address the question of whether the boy is safe nicians are required to report all cases of sus- at home. pected child abuse. The law does not leave the issue to the clinician’s discretion. 65. (D) The most appropriate advice is that the mother should help her son return the toys. 70. (A) This boy’s history is most consistent with The boy is in a developmental period when his selective mutism. He does not speak at school conscience is being established, and the parental but continues to talk at home. Consistent fail- model is important. The parent can show the ure to speak in a specific social situation despite child how to act responsibly by helping him speaking in other situations is not characteris- return the toys himself. At the age of 4, a child tic of MDD, separation anxiety disorder, social cannot adequately grasp the abstract concept of phobia, or dysthymic disorder. stealing, so punishment is not helpful. His sense of time is limited, so he cannot integrate 71. (D) The mother is most commonly the perpe- the idea of a reward after a week. trator of intentionally producing physical or psychological symptoms in her child in order 66. (D) SSRIs such as sertraline are effective in the to assume the sick role by proxy. The victim is treatment of panic disorder. Mood stabilizers, usually a preschool child, and an Axis I diag- such as valproate and carbamazepine, and nosis of physical abuse of a child may be antipsychotics, such as risperidone and cloza- appropriate. pine, are not indicated for the treatment of panic disorder. 72. (D) DSM-IV-TR criteria for antisocial personal- ity disorder requires evidence of a conduct dis- 67. (B) Identification is the process of adopting other order before the age of 15. No other personality people’s characteristics. Identification with a disorder requires a previous diagnosis of con- parent is important in personality formation. duct disorder. In addition, there is no evidence This girl’s behavior may occur as an attempt to of a reduced capacity for close relationships imitate the doctor because she admires her, or or eccentric behavior, making a diagnosis of it may represent an effort to cope with anxiety schizotypal personality disorder unlikely. about the doctor because she fears her. Reaction Recurrent suicidal behavior or an identity dis- formation is a defense mechanism in which an turbance is a common finding in borderline per- unacceptable impulse is transformed into its sonality disorder, and there is no evidence of opposite. Displacement is a defense mechanism excessive attention-seeking behavior, thus in which emotions are shifted from one idea or making histrionic personality disorder unlikely. object to another that resembles the original but evokes less distress. Rationalization is a 73. (B) This patient meets the criteria for anorexia defense mechanism in which rational expla- nervosa, a serious and potentially life- nations are offered in an attempt to justify threatening eating disorder, characterized by unacceptable attitudes, beliefs, or behaviors. intense fear of gaining weight, distortion of body Dissociation is a defense mechanism in which a image, and weight <85% of normal. The pur- person’s character or sense of identity is tem- poseful starvation can lead to numerous physi- porarily but drastically modified in order to ological abnormalities, including amenorrhea avoid emotional distress. and anemia. It can also lead to elevated choles- terol, increased secretion of corticotropin-releas- 68. (A) When the mother is infected with rubella ing hormone, hypoglycemia, hypothyroidism, during the first month of the pregnancy, the and leukopenia. risk of congenital defects is 50%. The risk of 38 2: Child and Adolescent Psychiatry

74. (C) Alexia is the inability to read. Anomia is the 5 months of life. Between the ages of 5 months specific inability to name objects even though and 48 months, there is deceleration of head the object is recognizable and can be described growth, loss of hand skills with development by the patient. Aphasia is more global and is an of stereotyped hand movements such as hand abnormality in either the expression or the wringing, loss of social interaction (which comprehension of language. Agnosia is the may improve later), appearance of poorly inability to recognize objects, but as in anomia, coordinated gait or trunk movements, and the object can be perceived by the patient’s severely impaired expressive and receptive intact . Apraxia is an inability to perform language development with severe psy- learned motor skills despite normal strength chomotor retardation. To date, Rett disorder and coordination. has been observed only in girls. Patients suf- fering from social phobia (D) demonstrate a 75. (D) This patient is suffering from OCD. marked and persistent fear of a social or per- Although all of the other listed organisms may formance situation with exposure causing cause psychiatric symptoms, prior infection intense anxiety expressed as a tantrum or a with Group-A beta-hemolytic Streptococcus has panic attack. Separation anxiety disorder (E) been associated with several psychiatric ill- is characterized by developmentally inappro- nesses, such as OCD and Tourette disorder. priate and excessive anxiety concerning sepa- ration from the home or from those to whom 76. (F) When children with pica eat soil or feces, the individual is attached. Enuresis (G) refers eggs of Toxocara canis or cati may be ingested to bedwetting. In panic disorder (I), there are causing visceral larva migrans. The other dis- discrete panic attacks, periods of intense fear orders are not associated with behavior likely or discomfort with physical manifestations to include ingestion of parasite eggs. such as palpitations, and subjective difficulty breathing. Asperger disorder (J) is a particular 77. (B) Bulimia nervosa is seen in 1–3% of adoles- type of autism most commonly seen in boys. cent girls and young women. In the purging type of disorder, repeated vomiting can pro- 80. (E) Palpitations, chest pain, shortness of breath, duce fluid and electrolyte abnormalities, most and diaphoresis commonly occur during panic frequently hypokalemia, hyponatremia, and attacks. These symptoms do not occur rou- hypochloremia. Metabolic alkalosis may occur tinely in any of the other disorders. as well. The other disorders are not associated with hypokalemia. 81. (G) Hypotension, bradycardia, and lanugo hair are classic sequelae of starvation seen in 78. (C) One of the presentations of stereotypic anorexia nervosa. These signs are not associ- movement disorder is self-inflicted bodily ated with any of the other disorders. injury that is severe enough to require medical attention. An individual’s repetitive motor 82. (A) Chapped hands and other dermatologic behavior may cause chronic rectal bleeding problems are often present in OCD due to severe enough to lead to anemia. The other dis- excessive washing with water or caustic clean- orders are not associated with rectal bleeding. ing agents. Chapped hands are not specifically associated with any of the other disorders. 79. (H) Adolescents with conduct disorder often abuse substances including alcohol, marijuana, 83. (J) Trichotillomania usually occurs around ages and cocaine. The other disorders are not as 5–8 or age 13. The repeated pulling out of hair often associated with substance abuse. results in decreased or complete loss of hair in Rett disorder (A) is characterized by nor- a specific area. The scalp, eyebrows, and eye- mal prenatal and perinatal development, nor- lashes are the sites most commonly involved. mal head circumference at birth, and normal Patches of baldness are not associated with any psychomotor development through the first of the other disorders. Pica (B) is the eating of Answers: 74–95 39

nonnutritional substances such as dirt or paint. animals, destruction of property, deceitfulness, Hypochondriasis (C) is the preoccupation that and serious violations of rules. There is nothing one has a serious disease based on misinter- to suggest an autistic spectrum disorder, par- pretation of symptoms, despite appropriate ticularly because it would probably have been medical evaluation and reassurance. Eye tics diagnosed much earlier; his actions seem will- such as blinking and eye rolling are the most ful and not repetitive or stereotyped. Similarly, common initial symptoms in Tourette disorder although one manifestation of Tourette syn- (D). Facial tics such as grimacing or licking drome might be a bark-like noise, it would not movements and vocal tics such as throat clear- occur only at will and be directed only at the ing or grunting are the next most common ini- dog. Oppositional defiant disorder requires tial symptoms. Whole-body tics, such as body 6 months of negativistic, hostile, and defiant rocking or pelvic thrusting, and self-abusive behavior directed mostly at authority figures. tics, such as hitting, may develop later. Bulimia Finally, there is no evidence of the psychosis, nervosa, seen in 1–3% of adolescent girls and such as hallucinations, that would characterize young women, is a purging type of eating dis- childhood schizophrenia. order. Asperger disorder (H) is a pervasive developmental disorder characterized by 86–89. (86–B, 87–D, 88–E, 89–A) The purpose of impairments in social interactions and the these questions is to differentiate some of the development of stereotyped or repetitive pat- autistic spectrum disorders by symptom and terns of behaviors without significant delay in epidemiology. Selective mutism is rare, and language skills or cognitive function. Asperger children who suffer from it are only mute in occurs more commonly in males and is com- certain situations (e.g., school). It often involves monly referred to as a high-functioning autism. A a stressful life event such as parents’ divorce. DSM-IV-TR diagnosis of pyromania (I) requires The 6-year-old girl in question 85 who had several criteria, including deliberate and pur- reached normal early development milestones poseful fire setting and tension or affective but now does not speak in school or at home is arousal before the act, which is not better most likely to have Rett disorder (D), which is accounted for by conduct disorder, a manic also rare and seen only in girls. The boy in ques- episode, or antisocial personality disorder. tion 86 who performs well in school but avoids social interactions is most likely to have 84. (C) Although it is possible that this child has a Asperger disorder, because those with autism, learning disorder such as dyslexia that is keep- as in question 87, are likely to have other symp- ing his reading skills from progressing, it is toms such as repetitive movements and to have important to rule out a vision problem that difficulty in school. There is no evidence in any could be easily fixed with glasses and that of the cases for the psychosis that would sug- would in fact impair other parts of his life. The gest childhood schizophrenia. child may need a reading specialist, or even have ADHD, but all the interventions for these 90–95. (90–C, 91–E, 92–A, 93–B, 94–D, 95–A) Although conditions will be for naught if you miss a these are not “board-style” questions, it is worth simple vision problem. reminding yourself of the definitions of the severity of mental retardation: mild (IQ of 55–70), 85. (C) This child is fulfilling the DSM-TR criteria moderate (IQ of 40–54), severe (IQ of 25–39), and for conduct disorder, namely, three of the four profound (IQ below 25). following acts: aggression toward people and This page intentionally left blank CHAPTER 3 Adult Psychopathology Questions

DIRECTIONS (Questions 1 through 116): Each of (D) 100% the numbered items in this section is followed by (E) the same incidence as in the general answers. Select the ONE lettered answer that is population BEST in each case. Questions 3 and 4

Questions 1 and 2 A patient reports to you that for the past week or A 22-year-old single White man is referred to you two he has had the belief that his intestines and his for a 1-year history of strange behavior character- heart have been removed. When asked about his lack ized by talking to the television, accusing local police of getting out in the world, he responds “What of bugging his room, and carrying on conversations world? There is no world!” with himself. His mother also describes a 3- to 4-year history of progressive withdrawal from social activ- 3. This aspect of the patient’s illness would best ities. The patient dropped out of college in his final be referred to as which of the following? year and since then has been living in his room at (A) schizoaffective disorder home. Attempts to hold a job as a busboy at a local (B) Capgras syndrome restaurant and as a night janitor have abruptly ended (C) folie à deux after disputes with the employers. (D) Cotard syndrome 1. How prevalent is this patient’s illness in the (E) major depression general population? 4. If this patient consistently reported to you the (A) 0.1% belief that his mother and father have been (B) 0.5% replaced by “cyborg alien robots” that look (C) 1% identical to his parents, this would be most (D) 3% indicative of which of the following? (E) 5% (A) delusional disorder (B) Capgras syndrome 2. The monozygotic (identical) twin concordance (C) folie à deux for this patient’s illness is which of the following? (D) Cotard syndrome (A) 10% (E) simple schizophrenia (simple (B) 25% deteriorative disorder) (C) 50%

41

Copyright © 2007 by The McGraw-Hill Companies, Inc. Click here for terms of use. 42 3: Adult Psychopathology

5. A 32-year-old woman 6 days postpartum is (A) low intelligence brought into the emergency room at the (B) head trauma encouragement of her husband. She is a poor (C) progressive social withdrawal historian, not believing that anything is wrong. (D) a neglectful mother He states that his wife has no past psychiatric history, although she does have an unknown (E) physical or sexual abuse family history of “some sort of mental illness.” He is very worried as over the last several days 7. A 42-year-old woman presents to a therapist his wife has not been sleeping even while the with a history since early adolescence of dra- newborn is. He has noticed her walking around matic mood swings, quickly becoming deeply their apartment in the middle of the night depressed for hours to days, usually in weeping while talking to no one in particular. response to separation from a loved one. She She has begun ignoring their baby, but he also admits to “rage attacks,” where she will brought her in today because earlier she vol- break items, scream, or scratch herself superfi- unteered that their child “is the Antichrist and cially on her arms. She claims to drink in must be destroyed.” Which of the following is “binges,” up to 1–2 pints of hard liquor at a her most likely diagnosis? time. She has had over 30 sexual partners, many times without using contraception. (A) bipolar disorder Which of the following defense mechanisms is (B) delusional disorder most likely employed by this patient? (C) major depressive disorder (MDD) with (A) altruism psychotic features (B) intellectualization (D) schizoaffective disorder (C) splitting (E) schizophrenia (D) sublimation 6. An 18-year-old woman, without a history of (E) undoing psychiatric illness, has recently started her freshman year of college. The woman is brought 8. In performing a Mental Status Examination to the psychiatric emergency department after (MSE) on a 26-year-old male graduate student, being found on the roof of her dormitory you make the following notes: The patient is dressed only in her underwear despite below poorly oriented to time and place, is dressed in freezing temperatures. The campus police who pink bathroom slippers, and is wearing only a brought her in report that she appeared to be trench coat. He reports auditory hallucinations dancing in a disorganized and agitated fashion, and fears that his classmates are conspiring to speaking angrily to someone that was not embarrass him in front of his teachers. If, in apparent to them, and accused the policemen the course of this patient’s chronic mental ill- of being “Satan’s horsemen” and “adulterers ness, he becomes jobless, homeless, and desti- from the Court of King Herod.” On psychiatric tute, the history would exemplify which examination, she reports to you that “Lucifer is concept? telling me I’m an angel of death. He told me to (A) stress-diathesis model flap my wings and soar to my death.” A care- (B) downward drift hypothesis ful history is obtained. Her roommates report (C) sometimes intolerable side effects of that for the past 5 weeks the patient has been antipsychotic drugs delusional, mostly with religious or persecu- (D) socioeconomic burden of schizophrenia tory content, and her thoughts seem disorgan- ized. In talking with this patient’s family, you (E) Bleuler’s classical model of would be most likely to gather a history of schizophrenia which of the following? Questions: 5–13 43

9. A 36-year-old patient you have seen twice is 11. Which of the following factors would influ- brought to your walk-in department by family ence you to recommend electroconvulsive members. On previous occasions, he presented therapy (ECT)? with delusions, hallucinations, and prominent (A) severe, melancholic depression and negative symptoms. During the visit, he history of poor response to medications exhibits motoric, waxy flexibility and resist- ance to all instructions. He methodically (B) severe, adverse reactions to all selective repeats words and phrases and appears to serotonin reuptake inhibitors (SSRIs), crudely mimic your movements. The most tricyclic antidepressants (TCAs), and likely diagnosis is which of the following? continued suicidal ideation (C) known history of psychotic symptoms (A) elective mutism in the context of her depressive (B) malingering symptoms (C) schizophrenia, disorganized type (D) known history of bipolar illness (D) schizophrenia, catatonic type (E) poor compliance with medications (E) drug-induced psychosis 12. The most important diagnosis to rule out in 10. A 49-year-old bank teller without a psychiatric this patient before administering an antide- history is referred to your office for the first pressant medication would be which of the time by her internist for an evaluation. For the following? past 2 months, she has been increasingly con- (A) panic disorder vinced that a well-known pop music star is in love with her and that they have had an ongo- (B) bipolar disorder ing affair. She is well-groomed, and there is no (C) obsessive-compulsive disorder (OCD) evidence of thought disorder or hallucinations. (D) drug-induced depressive disorder Her husband reveals that she has been func- (E) malingering tioning well at work and in other social rela- tionships. Which of the following is the most Questions 13 and 14 likely diagnosis? A 34-year-old White man complains of feeling blue at (A) delusional disorder his first visit to you. An MSE reveals a disheveled (B) acute reactive psychosis appearance, a depressed mood, psychomotor retar- (C) prodromal schizophrenia dation, and suicidal thoughts. Thought processes are (D) paranoid personality disorder significant for thought blocking and some slowing. (E) schizophreniform disorder Deficits with remote and short-term memory are noted. Judgment and insight are also impaired. Your Questions 11 and 12 diagnosis is a major depressive episode.

A 46-year-old divorced, middle-class, African 13. The type of sleep disturbance you would most American woman is admitted to your inpatient psy- expect to see in this patient is which of the chiatric ward after a suicide attempt by overdosing following? on acetaminophen (Tylenol). She has a history of (A) sleeping too deeply (difficulty being multiple psychiatric hospitalizations with similar awakened) presentations. She has been unable to work because of her depression and has lost interest in activities (B) early morning awakening she once enjoyed. Additionally, the patient reports a (C) increased rapid eye movement (REM) 2-week history of decreased sleep, difficulty concen- stage latency trating, low energy, hopelessness, and a decreased (D) decreased response to sedative drugs appetite. (E) sleeping too lightly (awakened too easily) 44 3: Adult Psychopathology

14. You would expect the biochemical/hormonal The patient returns for a follow-up visit 4 weeks after profile of this patient to be significant for which you initiate treatment with divalproex sodium. He of the following? presents looking irritable and demonstrates pres- sured speech and grandiose delusions. He reports (A) increased catecholamine activity that he is the lost son of a famed millionaire. He (B) increased cortisol secretion exhibits symptoms of depression characterized by (C) increased sex hormones decreased sleep, poor appetite and concentration, (D) increased immune functions and thoughts of suicide. He admits to drinking alco- (E) decreased monoamine oxidase (MAO) hol every day since his last visit. His wife reports activity that his moods seem to change with the seasons.

Questions 15 and 16 16. In addition to starting valproic acid, the other pharmacologic management change indicated A 52-year-old man with a history of major depression at the prior visit would have been to which of is treated with paroxetine (Paxil). He has been your the following? patient for 2 years and has recently missed his last two appointments reporting to you that “I’ve been (A) Check the patient’s serum paroxetine spectacular!” You have documented two calls from level. his wife since his last visit. She initially reported that (B) Select a more effective antidepressant. her husband has been “very different.” You learn (C) Stop the antidepressant. that he has spent more money on frivolous items (D) Select a medication as needed for than he ever has. In addition, when she went to the insomnia. bank to withdraw money for groceries, she was told (E) Start lithium. that the account was overdrawn. Recently, his wife called again to make an appointment for her hus- Questions 17 and 18 band. She reports that one moment her husband is giddy and without notice he quickly becomes agi- 17. A patient places a call after hours to the practice tated and angry. During the interview, the patient you are covering. You do not have access to the questions your credentials and accuses you of being patient’s chart but learn that the history suggests more loyal to his wife than to him. Most of the inter- the patient experiences dysthymia followed by view is spent interrupting the patient as you try to episodes of hypomania. In making a decision decipher his rapid speech. about the patient’s medication, you quickly rec- ognize the diagnosis. Which of the following 15. Which of the following statements is true? choices best categorizes the patient’s diagnosis? (A) Patients with bipolar disorder usually (A) bipolar II disorder have a more favorable course than (B) bipolar I disorder depressive patients. (C) cyclothymia (B) A lower percentage of patients with (D) bipolar III disorder bipolar disorder eventually get (E) double depression treatment as compared to patients with unipolar depression. 18. On a subsequent visit, the patient brings his (C) Bipolar disorder has a stronger genetic brother to your office. His brother asks what his link than depression. risk of developing a mood disorder is. What do (D) More women are diagnosed with you answer? bipolar disorder than with major depression. (A) the same as anyone in the general population (E) Bipolar disorder is more common than major depression in the northern (B) 25% hemisphere. Questions: 14–23 45

(C) 10% patient’s type of hallucinations in the context of (D) 1% which of the following? (E) the answer is unknown (A) delirium (B) delusional disorders Questions 19 and 20 (C) schizophrenia A 26-year-old White woman presents to the psychi- (D) conversion disorder atric emergency department in an acutely distressed, (E) brief psychotic disorder nervous state. The emergency department staff are unable to calm her down or gain an adequate history Questions 22 and 23 from the patient. She complains of terrible anxiety. She is slightly diaphoretic, tachycardic, and the A 62-year-old woman presents to the nursing home pupils are mildly dilated. She is on no medications. where you work as a consulting psychiatrist. She has had a history of bilateral temporal lobectomy for an 19. The cause of anxiety important to rule out in intractable seizure disorder. After a few weeks at the this patient is which of the following? new facility, the staff report the following observa- tions about the patient in addition to her short-term (A) cocaine induced memory difficulties: She is extremely docile and dis- (B) hypochondriasis plays very little emotion. She has a large appetite (C) serotonin syndrome and compulsively puts both food and nonfood items (D) specific phobia in her mouth. She also displays sexual disinhibition, (E) lysergic acid diethylamide (LSD) often walking out of her room without her pants on. induced 22. This patient’s clinical condition is most consis- 20. Abdominal pain in this patient might cause tent with which of the following? you to investigate which of the following? (A) Pick disease (A) blood alcohol level (B) Klüver-Bucy syndrome (B) acetaminophen level (C) Arnold-Chiari syndrome (C) urinary porphobilinogen (D) punchdrunk syndrome (D) urine toxicology screen (E) Möbius syndrome (E) serum catecholamine metabolites 23. The loss of what temporal lobe structure is 21. A 46-year-old man is admitted to the emer- most closely associated with this condition? gency department acutely diaphoretic, tachy- (A) cardic, hypertensive, tremulous, and agitated. (B) insula He refuses to give a urine sample for toxicology (C) superior temporal gyri studies. He is apparently hallucinating, judging from his insistence that he be allowed to (D) amygdala “squash those bugs on the wall” (there are (E) inferior horn of the lateral ventricle none). One would expect to most likely see this 46 3: Adult Psychopathology

Questions 24 and 25 (C) chlordiazepoxide (Librium) (D) phenelzine (Nardil) A 22-year-old woman is referred by her gynecolo- gist for the treatment of a possible eating disorder. (E) divalproex sodium The gynecologist informs you that the patient pre- sented with a chief complaint of amenorrhea that Questions 27 and 28 had been present for 4 months. In addition, the A 34-year-old woman complains of a 3-month history patient is at 83% of her expected body weight. Upon of “feeling down” that has steadily worsened. After evaluation, the patient describes a repeated pattern of losing her job as a sales representative 1 month ago, bingeing on large meals followed by forced vomiting. she has been living with her parents and has not She also uses laxatives every night before she goes to looked for work. The patient reports that she is bed “to prevent constipation.” The patient reports unmotivated to do anything even the things she used that she had been running 3 miles every day, but in to enjoy. She says that “nothing really matters...I don’t the last few weeks she has become too tired to run. matter.” She has been sleeping 10–14 hours a night and has no appetite. More than once, she has pon- 24. The most accurate diagnosis for this patient is dered suicide as a possible escape route. which of the following? (A) exercise-induced amenorrhea 27. Given this patient’s diagnosis, what is the like- (B) OCD lihood that she would fail to suppress her cor- tisol levels in a dexamethasone suppression (C) psychogenic gastritis test? (D) anorexia nervosa (E) bulimia nervosa (A) 10% (B) 30% 25. Laboratory findings associated with severe (C) 50% cases of this disorder include which of the (D) 70% following? (E) 90% (A) decreased serum cortisol levels (B) blunted thyroid-stimulating hormone 28. What is the likelihood that she would have a (TSH) response to thyrotropin-releasing blunted response of TSH to an administration hormone (TRH) of TRH? (C) polycythemia (A) 10% (D) low serum growth hormone (B) 30% (E) high serum blood urea nitrogen (BUN) (C) 50% (D) 70% 26. A 23-year-old college student has experienced (E) 90% frequent episodes of feeling utter doom for the past 3 months. During these episodes, he also 29. A 37-year-old woman, who works the night experiences tremulousness, sweating, dizzi- shift at a local grocery store taking inventory, ness, and tingling in his extremities. He reports reports that her childhood and college years experiencing these episodes at least once a were uneventful but happy. She spends most of week and is now becoming fearful of attending her time alone when she is not at work. She classes lest he have an episode. The medication does not venture out of her house and her of choice for this condition is which of the social contacts are limited to work-related following? interactions with coworkers. She is an avid (A) alprazolam (Xanax) plant lover and she spends most of her free (B) fluoxetine (Prozac) time taking care of her indoor plants. She Questions: 24–33 47

reports that she is quite content with her life. 32. A 42-year-old business executive presents for The most accurate diagnosis for this patient is his first contact with a mental health provider. which of the following? He reports that for the last 4 months he has been feeling depressed. His low energy level (A) agoraphobia and poor motivation are affecting his job per- (B) avoidant personality disorder formance and the CEO of his company advised (C) schizoid personality disorder him to “take a couple of weeks off.” The (D) schizotypal personality disorder patient reports that he started feeling down (E) autistic disorder when his wife discovered that he was involved in his third extramarital affair. Since then he 30. An 18-year-old, pregnant, human immunode- has moved into a small apartment by himself. ficiency virus (HIV)-positive White woman He is sleeping almost 12 hours every night, has presents for the treatment of heroin addiction. a poor appetite, and is experiencing financial She reports using heroin for the last 8 months difficulty due to indiscriminate purchases. He with substantial efforts to quit for the last laments the loss of his former self. He reports 4 months. She is now homeless and has recently that he used to need only 4–5 hours of sleep been arrested for shoplifting. Which of the fol- and once was able to “party all night and work lowing is important to discuss with the patient all day.” This case best illustrates an Axis I diag- when considering methadone maintenance? nosis of which of the following? (A) Methadone may cause bone decay. (A) bipolar I disorder (B) Methadone substantially reduces libido. (B) bipolar II disorder (C) Methadone worsens dependence on (C) MDD heroin. (D) narcissistic personality disorder (D) Effective treatment is likely to require (E) impulse control disorder not otherwise more than 60 mg/day of methadone. specified (E) Methadone may cause Cushing syndrome. 33. A 52-year-old woman who has been treated with medication for 3 years for a chronic mood dis- 31. Following the birth of her first child, a 29-year- order reports that although she feels well, she old woman is diagnosed with postpartum psy- wonders if her medication is causing side effects. chosis. Which of the following would be the She complains of dry mouth, trouble urinating, most important first step in the management of and occasional dizziness when she gets out of this patient? bed. Which of the following medications is she most likely being prescribed? (A) Admit her to the hospital. (B) Begin a mood stabilizer. (A) fluoxetine (C) Begin an antidepressant. (B) imipramine (Tofranil) (D) Begin an atypical antipsychotic. (C) phenelzine (E) Begin ECT. (D) lithium (E) divalproex sodium 48 3: Adult Psychopathology

Questions 34 and 35 (A) the patient’s inability to stop drinking despite knowledge of its harmful effects A 72-year-old man is brought in by his wife to your and his desire to quit geriatric psychiatry clinic. The patient’s wife is con- (B) the high quantity of alcohol consumed cerned about his progressive confusion over the last on a regular basis year. She is particularly distressed that he repeatedly asks the same questions throughout the day. She also (C) the patient’s history of drunk driving notes that her husband has become increasingly (D) the fact that the patient cannot sleep if unsteady on his feet and needs to use a walker when he does not drink they go out. She wonders if these symptoms may be (E) a medical complication due to drinking related to the meningitis he suffered from 3 years ago. 37. On laboratory examination, this patient is likely 34. For you to make a diagnosis of normal pressure to have which of the following? hydrocephalus (NPH), further investigation in (A) increased alanine aminotransferase this case would have to reveal which of the (ALT) and aspartate aminotransferase following? (AST), with increased ALT/AST ratio (A) elevated opening pressure upon lumbar (B) decreased uric acid puncture (C) increased gamma-glutamyl (B) a history of incontinence transpeptidase (GGT) (C) oculomotor difficulties (D) microcytic anemia (D) frontal release signs (E) decreased serum triglycerides (E) perseveration 38. A 26-year-old computer programmer without 35. Neuroimaging with noncontrast computed a past psychiatric history has been married for tomography (CT) in this case of NPH is most 4 years. His wife is expecting their first child. likely to reveal which of the following? She reports that 3 months ago the patient became preoccupied with the idea that she (A) dilated lateral ventricles became pregnant by another man. During this (B) normal ventricles time, he began missing work and isolated him- (C) frontal sulcal widening self in his . His affect has progressively (D) focal subcortical hypointensities become more blunted. Recently, he believes (E) cerebellar atrophy that his wife is carrying a child conceived by “extraterrestrial forces.” He urged her to have Questions 36 and 37 an abortion and she refused. The patient denies any history of substance abuse and his recent A 48-year-old man has been drinking up to 6 beers medical evaluation was within normal limits. per night during the week and up to 12 beers a night Which of the following is the most appropriate on the weekend. A year ago, he had his driver’s diagnosis? license suspended for drunk driving. His marriage is failing. Last month, he was diagnosed with a gastric (A) brief psychotic disorder ulcer as a result of alcohol consumption. He admits (B) delusional disorder to an alcohol problem and has tried to stop on (C) psychosis not otherwise specified numerous occasions. He finds that he experiences (D) schizophreniform disorder insomnia if he does not drink for more than 2 days. (E) schizophrenia

36. The feature of this case that suggests alcohol 39. A 27-year-old woman was involved in a major dependence rather than alcohol abuse is which automobile accident 2 weeks ago in which a of the following? friend sitting next to her in the passenger Questions: 34–43 49

seat was killed. Since the accident, she has is a follow-up visit for a single, attractive 31-year- experienced recurrent thoughts of the acci- old woman who is finishing her antibiotic regi- dent and has recurrent each night. men for treatment of pneumonia. The best Lately, she is reluctant to drive and has psychodynamic term for this doctor’s response to chosen to ride a bus to work. When she does his patient is which of the following? ride in the car as a passenger, she feels nerv- (A) countertransference ous and is overly concerned about the safety of her companion’s driving. The most appro- (B) priate diagnosis for this patient is which of (C) identification the following? (D) projection (A) acute stress disorder (E) transference (B) adjustment disorder Questions 42 and 43 (C) posttraumatic stress disorder (PTSD) (D) generalized anxiety disorder (GAD) A 31-year-old woman was admitted to a psychiatric (E) MDD hospital after a failed suicide attempt by medication ingestion. Recently, she broke up with her boyfriend Questions 40 and 41 whom she met in a bar 4 months ago. She is noted for episodes of mood lability, marked by feelings of An 18-year-old woman diagnosed with schizophre- depression, and anger directed toward the psychiatric nia presents to the psychiatric emergency depart- resident who completed the rotation 5 days after her ment after a suicide attempt by carbon monoxide admission. When the resident left, she reported that she poisoning. Medical evaluation was noncontributory was having urges to cut her wrists. As the patient was and a urine toxicology screen was negative. The nearing her discharge date, she reported that “all the patient reports that she sat in her mother’s car with staff hates me except for Dr Johnson.” Dr Johnson, a a glove stuck into the exhaust pipe and eventually fell medical student, had a recent difference of opinion asleep awaiting death. She relates that she believes with the nursing staff regarding the patient’s discharge. that she was personally responsible for the Holocaust and that she should therefore die in a similar way. 42. The single most appropriate diagnosis for this The patient’s mother reports that she has been refus- patient is which of the following? ing to take haloperidol (Haldol) for the past 3 weeks. (A) psychotic disorder not otherwise specified 40. The lifetime risk of death by suicide in patients (B) MDD with schizophrenia is closest to which of the (C) histrionic personality disorder following? (D) cyclothymic disorder (A) 1% (E) borderline personality disorder (B) 5% 43. The outpatient treatment of choice for patients (C) 10% with the diagnosis illustrated in this case is (D) 30% which of the following? (E) 50% (A) antidepressant medications 41. A 27-year-old internal medicine resident is work- (B) benzodiazepine medications ing in an outpatient clinic. He prefers the inpa- (C) mood stabilizer medications tient service and is unhappy most of the time he (D) neuroleptics is there. Today, however, he is looking forward to (E) psychotherapy his clinical work because one of his appointments 50 3: Adult Psychopathology

Questions 44 and 45 47. The first-line pharmacotherapy for this condi- tion is which of the following? A 34-year-old woman presents for the treatment of her severe, medication-refractory, major depression. She (A) lorazepam (Ativan) is referred to you by her outpatient psychiatrist (B) benztropine (Cogentin) because of your expertise in ECT. After reviewing her (C) nortriptyline (Pamelor) past psychiatric history and interviewing the patient, (D) haloperidol you conclude that she should indeed undergo ECT. (E) fluvoxamine (Luvox) 44. In discussing the effects of ECT with the 48. A 72-year-old woman presents to the emer- patient, you should relate that the most likely gency department from a nursing home. On side effect may be which of the following? physical examination, she appears malnour- (A) ished and dehydrated. The medical service ini- (B) anesthesia-related respiratory tiates intravenous fluid replacement. On MSE, complications she is alert and oriented to person only. She (C) fractures from convulsions reports that the president is Lyndon B. Johnson. During the interview, the patient is easily dis- (D) abnormal cardiac arrhythmias tracted. She often forgets things shortly after (E) psychosis she learns them. One hour later, her MSE has improved, reporting the correct day, time, and 45. You tell the patient to expect the best possible place. However, she cannot remember why she outcome, she is likely to require how many is there. When asked about the current presi- treatments? dent, she cannot remember his name but (A) 2 describes his appearance. She is able to per- (B) 4 form serial sevens slowly but accurately with- out distraction. Based on the information in (C) 10 this case, the most appropriate diagnosis for this (D) 20 patient’s presenting symptoms is which of the (E) 40 following?

Questions 46 and 47 (A) amnestic disorder not otherwise specified A 38-year-old woman presents to your clinic telling (B) cognitive disorder not otherwise you that she has had disturbing, recurrent thoughts specified about harming her 7-month-old infant. She imagines (C) delirium using a knife to stab her child. Since having these distressing thoughts, she has removed all sharp (D) dementia not otherwise specified objects from her kitchen. Because of this, she has not (E) major depression been able to prepare meals at home and has chosen to buy fast food or take-out food for the family meals. Questions 49 and 50 She has not shared these thoughts with her husband. You are a research psychiatrist conducting a double- blind, placebo-controlled trial of a new antidepres- 46. The most accurate diagnosis for this condition sant. You have enrolled 200 patients in the study, all is which of the following? of whom meet the criteria for uncomplicated major (A) impulse control disorder not otherwise depression. You plan to randomize 100 patients to a specified placebo medication and the other 100 patients to the (B) OCD experimental antidepressant. (C) obsessive-compulsive personality disorder (D) psychosis not otherwise specified (E) schizotypal personality disorder Questions: 44–53 51

49. Of the 100 patients taking the placebo, approx- (A) catatonic type imately how many patients would be expected (B) disorganized type to improve after 4–6 weeks? (C) paranoid type (A) 5 (D) residual type (B) 10 (E) undifferentiated (C) 30 (D) 50 52. Recent epidemiologic trends concerning this type of schizophrenia are notable for which of (E) 70 the following? 50. Of the 100 patients taking the experimental (A) increased prevalence compared to antidepressant (assuming this drug is as effi- 35 years ago cacious as standard antidepressants), approxi- (B) about the same prevalence compared to mately how many patients would be expected 35 years ago to improve after 4–6 weeks? (C) its being the most common subtype of (A) 10 schizophrenia (B) 30 (D) decreased prevalence compared to (C) 50 35 years ago (D) 70 (E) its having an association with history of LSD use (E) 90 53. A 32-year-old single successful Wall Street exec- Questions 51 and 52 utive tells you that on weekends he likes to A 24-year-old man is brought to the psychiatric emer- visit a dominatrix. His regular, paid appoint- gency department by his parents. They report that he ment with this person is described as humili- has been living with them since he dropped out of ating and somewhat painful but also very college a year ago. In the last few months, he has spent sexually arousing. He describes his life as oth- an increasing amount of time by himself in his room. erwise normal. The best way to describe this Rather than look for employment, he is absorbed in case by Diagnostic and Statistical Manual of doing crossword puzzles. The parents also report that Mental Disorders, Fourth Edition, Text Revision in the last 2 months, their son’s behavior has become (DSM-IV-TR) diagnostic criteria is which of the increasingly bizarre. His sleep schedule has become following? disorganized and he has started wearing his clothes (A) fetishism inside out. They note that he makes strange gestures (B) frotteurism while at the dinner table. In addition, he has begun smiling for very long periods of time although there (C) no diagnosis is nothing to laugh at. At times, the patient does not (D) sexual masochism respond to questions asked of him, and at other (E) sexual sadism times, he simply repeats the questions.

51. The type of schizophrenia that best describes the presentation is which of the following? 52 3: Adult Psychopathology

Questions 54 and 55 56. As you begin to discuss the possible diagnosis and treatment options with the patient, she tells A 28-year-old woman presents complaining of falling you that she is 6 months pregnant. Which of the asleep during the day. She says this problem has been following psychiatric medications is consid- ongoing for 3 months. It has been interfering with her ered safest in pregnancy? work as a telephone operator because she falls asleep two or three times a day while speaking with cus- (A) lithium tomers. At times, she finds herself falling asleep at her (B) divalproex sodium desk and she is awakened when her head hits the (C) haloperidol computer console in front of her. Oddly enough, she (D) chlorpromazine reports, this can happen when she becomes particu- (E) fluoxetine larly stressed out as she is managing many calls. The patient also reports that this disturbance has not 57. The patient’s husband tells you that he does not improved even though she is sure she sleeps 8 hours believe the patient needs medication. He is a each night. spiritual man and wants to take her to a healer from his community. What can you tell him 54. The most appropriate diagnosis for this case is about the prognosis of this disorder? which of the following? (A) The majority of people spontaneously (A) circadian rhythm recover. (B) not otherwise specified (B) If she does not take medication, the (C) patient will suffer a chronic (D) disorder deteriorating course. (E) primary hypersomnia (C) Prognostic factors may be taken into account, but the actual course of the 55. The most useful pharmacotherapy for this illness for this patient cannot be predicted. condition is which of the following? (D) If the patient does not take medication, (A) lorazepam at bedtime she will require permanent commitment (B) bupropion (Wellbutrin) in a psychiatric facility. (C) phenelzine (E) There is no risk to the medication you’ve prescribed. (D) methylphenidate (Ritalin) (E) fluoxetine 58. A 28-year-old woman presents for her annual gynecology appointment. She complains that Questions 56 and 57 in the week before her period, she often expe- A 27-year-old woman is brought into the psychiatric riences marked anger and irritability and emergency department by her distraught husband argues more with her boyfriend. She also because she has been acting strangely. She reports reports diminished energy and concentration that for about 1 year the neighborhood children have and sleeping more than is usual for her. These been watching and following her sometimes even symptoms, in addition to breast tenderness and breaking into her home. She adds that for the last 6 headaches, always remit in the week after her months she has been hearing a conversation between menses is finished. The most likely diagnosis is two voices typically when she is alone. Her husband which of the following? says that she has been increasingly withdrawn and (A) MDD rarely talks anymore with her family and friends. (B) normal female behavior She now spends most of the day in their bedroom. He (C) premenstrual dysphoric disorder reports that he often hears her talk to herself and has (PMDD) seen her checking the telephone for “bugs.” (D) GAD (E) dysthymic disorder Questions: 54–63 53

59. Mr P is a 37-year-old accountant who presents 62. A 38-year-old male veteran from the Gulf War to the primary care clinic with complaints of presents to the mental health clinic at the insomnia. Upon further questioning, he admits urging of his family members. While stationed that he has felt “blue” for 6 weeks since getting in the Middle East, he had witnessed a friend passed over for promotion. Since that time, he killed during an explosion. Since that time, he has had poor sleep often awakening early in has had chronic insomnia with ongoing night- the morning. He also has had a decreased mares of the event as well as occasional flash- appetite with a 10–15-lb weight loss, poor backs. He describes always being “on edge,” energy, guilt over “not being good enough,” avoiding crowds, and becoming easily startled and he has been distracted at work. For the with loud noises. He admits to regular alcohol past 3 days, he has had thoughts of “ending my use, especially when his symptoms are worse, life.” What is this patient’s most likely risk of up to 12 beers at a time. Which of the following completed suicide? is the most appropriate treatment to begin for this patient? (A) 0–10% (B) 10–20% (A) atypical antipsychotic (C) 20–30% (B) benzodiazepine (D) 30–40% (C) lithium (E) 40–50% (D) serotonin-specific reuptake inhibitor (E) valproic acid Questions 60 and 61 63. A 40-year-old married woman is referred by A 36-year-old man is brought to the emergency her internist to a psychologist for further treat- department in respiratory arrest. On examination, ment. She presents an 8-month history of recur- he is unresponsive, and the medical student rotating rent bouts of “terror,” associated with chest through the emergency department observes pin- pain, tachypnea, tremors, flushing, nausea, and point pupils and antecubital track marks. There is fears that “I’m going to die.” These episodes suspicion that the patient’s condition may be the last for approximately 15 minutes and do not result of a drug overdose. have a particular stressor. As a result, she has had increasing difficulty traveling far from her 60. The patient most likely has overdosed on home due to concerns over having further which of the following drugs? attacks. Despite adequate treatment with ser- (A) cocaine traline, she remains symptomatic and in sig- (B) phencyclidine (PCP) nificant distress. Which of the following psychotherapies would be the most appropri- (C) heroin ate for her condition? (D) alcohol (E) inhalants (A) cognitive-behavioral therapy (CBT) (B) eye movement desensitization and 61. Which of the following would reverse the reprocessing effects of the suspected drug? (C) insight-oriented therapy (A) acetylcysteine (D) interpersonal psychotherapy (B) naloxone (E) supportive psychotherapy (C) deferoxamine (D) methylene blue (E) methadone 54 3: Adult Psychopathology

64. You are treating a 48-year-old housewife on an 65. The most likely diagnosis for this patient is inpatient medical unit for exacerbation of which of the following? asthma. On the day that you inform her of her (A) adjustment disorder pending discharge, she complains of feeling feverish and achy and has a nonproductive (B) bipolar disorder cough. The nursing staff reports that she has a (C) MDD sudden fever of 103°F. You treat the fever with (D) PTSD acetaminophen and perform a physical exam- (E) schizophrenia ination, order chest x-rays, draw blood, and order a urinalysis. While you are awaiting these 66. The mean age of onset for this disorder is clos- results, the nurse informs you that she wit- est to which of the following ages? nessed the patient dipping her thermometer into a hot cup of tea before another nurse went (A) 10 back into the patient’s room to read the ther- (B) 20 mometer. The most likely diagnosis to account (C) 30 for this woman’s behavior is which of the (D) 40 following? (E) 50 (A) conversion disorder 67. A 68-year-old man has a history of a left middle (B) factitious disorder cerebral artery stroke. Which of the following (C) hypochondriasis psychiatric disturbances is most common fol- (D) malingering lowing such a neurologic event? (E) somatoform disorder (A) anxiety Questions 65 and 66 (B) OCD (C) depression A 32-year-old man is brought to the psychiatric emer- (D) mania gency department by the police after having been arrested for public nudity. Laboratory tests show a (E) panic symptoms negative drug screen and alcohol levels. On MSE, you find that the patient cannot sit down and is only 68. A 25-year-old man presents to your office partly cooperative. He interrupts the interview sev- asking for help stopping smoking. He has tried eral times demanding to be allowed to contact his quitting cold turkey and using nicotine replace- lawyer “because my rights given to me by God and ment therapy but neither has worked. He ordained by the Jeffersonians and Washingtonians wants to know about . Which of the and Lincolnians have been infringed...and you, sir, following best describes this technique? are committing illegalities of the highest order.” His (A) a state in which critical judgment is speech is pressured. suspended and a person experiences Examination of the patient’s psychiatric records alterations in perception, memory, or indicate that he has a history of a major depressive mood in response to suggestions episode treated successfully with an SSRI for 1 year (B) an individual psychotherapy directed at when he was age 23. The patient’s parents arrive in changing behavior the emergency department and tell you that the (C) a group psychotherapy directed at patient is a college graduate and a computer pro- changing maladaptive thoughts grammer. Lately, his job has been rather stressful as (D) a procedure that directly accesses he has been overlooked for promotion, and his girl- repressed friend left him 1 month ago. He is also in a great deal of credit card debt. (E) an altered brain state that can be mapped by electroencephalogram (EEG) Questions: 64–73 55

69. A 48-year-old man presents with a chief com- team has consulted a psychiatrist to help with plaint of impotence that has troubled him for the evaluation and management of her condi- the last year. Based on epidemiology of known tion. On examination, she is somnolent at times, causes of this disorder in this patient’s age fluctuating with alert. She is not cooperative, group, the chance that his problem is due to a hostile, and clearly hallucinating. Her insight psychological cause rather than a medical and memory are poor. The primary team wishes cause is closest to which of the following? to know if she is “delirious or demented.” Which of the following signs/symptoms in this (A) 10% patient is the most specific for delirium? (B) 30% (C) 50% (A) aggressivity (D) 70% (B) fluctuating consciousness (E) 90% (C) poor memory (D) psychosis 70. Postmortem studies and cerebrospinal fluid (E) uncooperativeness (CSF) sampling from living patients have revealed a correlation between aggression, 73. A 45-year-old married woman, formerly impulsivity, and suicide and a decrease in the employed as an accountant, had onset of severe metabolites of which of the following neuro- anhedonia, amotivation, anergia, and flattened transmitters compared to control subjects? affect when she was 40 years old. Initially, her psychiatrist thought she might be suffering (A) dopamine from major depression because of her promi- (B) gamma-aminobutyric acid (GABA) nent family history of depression on her (C) glutamate father’s side. At age 41, she had the sudden (D) norepinephrine onset of auditory hallucinations and a fixed (E) serotonin false belief that the government was harassing her. Her psychiatrist then diagnosed her with 71. A 38-year-old woman diagnosed with multiple schizophrenia and placed her on neuroleptics. sclerosis of moderate severity has had symp- After starting these medications, her auditory toms of depression and memory loss increasing hallucinations disappeared and she no longer over the last year. On MSE, you notice blunted believed in her delusions. However, she con- affect and decreased speed of mental process- tinued to have amotivation, anhedonia, and ing. A magnetic resonance imaging (MRI) flattened affect, and was unable to return to examination is likely to reveal which of the her employment as an accountant. Which following? prognostic indicator suggests a poorer prog- nosis in this case? (A) normal brain (B) global cerebral atrophy (A) the onset of symptoms relatively late in the patient’s life (C) ventricular enlargement (B) the fact that the patient is married (D) multiple plaques of frontal white matter (C) a positive family history of depression (E) rare periventricular plaques (D) the sudden onset of psychotic 72. The patient is an 80-year-old widowed woman symptoms admitted to the hospital under the medical serv- (E) the predominance of negative ice who is “confused.” Her primary medicine symptoms 56 3: Adult Psychopathology

Questions 74 and 75 77. A 23-year-old single White medical student comes to your office complaining of difficulty A 34-year-old woman with schizophrenia has been sleeping. During the past year, he has been stable on her psychiatric medications for 3 years and worrying excessively about a number of things, has become pregnant for the first time. Her family including his studies, his relationship with his psychiatric history is not known because she was parents, and that his girlfriend of 2 years is adopted. The father of the unborn child does not going to break up with him, despite being have schizophrenia. happy with their relationship. He admits that he has difficulty controlling the time he spends 74. What percentage most closely approximates worrying. He notes feeling irritable at times the likelihood that this patient’s child will have and has been experiencing muscle tension. schizophrenia? Because of these symptoms, he has trouble (A) 1% with schoolwork and his grades have suffered. (B) 5% He does not endorse a depressed mood and asks, “Can you help me with my problems?” (C) 10% You diagnose him with GAD, and prescribe a (D) 20% combination of psychotherapy and benzodi- (E) 30% azepines. Three months later, the patient comes back to your office reporting that his mood has 75. Suppose adoption records indicate that the been down in the dumps, and he feels like he patient has an identical twin that was adopted may never feel better. Recently, he has been into another family. Which percentage most thinking that life was not worth living any- closely represents the likelihood that her twin more and has passive thoughts of suicide. sister has schizophrenia? When asked about specific suicidal plans, he (A) 10% reports recent thoughts of taking an overdose (B) 30% of pills. The most appropriate next step would be which of the following? (C) 50% (D) 70% (A) Tell the patient he is going to get better (E) 90% and schedule weekly outpatient visits. (B) Prescribe lorazepam for his excessive 76. You are a research psychiatrist who is studying worries. signs and symptoms associated with certain (C) Refer the patient to the psychiatric psychiatric disorders that are not mentioned emergency department. in the DSM-IV-TR for those disorders. Your (D) Prescribe an antidepressant and instruct spreadsheet contains a category of patients him to follow up with you in 1 month. who have sensory gating deficits, short-term (E) Call his parents to pick him up and memory difficulties, and abnormalities in arrange for a family meeting. smooth-pursuit eye movements. The most fre- quent DSM-IV-TR diagnosis you are likely to 78. The patient is a 36-year-old married White find in this category is which of the following? female who presents to the emergency room (A) attention deficit hyperactivity disorder with a 2-month history of depression, terminal (ADHD) insomnia, fatigue, decreased appetite, anhedo- (B) major depression nia, and excessive guilt. When pressed, she admits to suicidal ideation for the past week, (C) OCD with thoughts of “taking all of my medicines.” (D) PTSD After further questioning, she states that “I would (E) schizophrenia Questions: 74–83 57

never do it” as she is a devout Catholic who 81. A patient in the emergency department tells attends church regularly. Which of the follow- the physician that over the past 3 weeks his ing characteristics increases this patient’s risk of television has been talking directly to him. suicide? From what specific type of delusion is this patient suffering? (A) age (B) gender (A) thought insertion (C) marital status (B) somatic (D) race (C) thought broadcasting (E) religion (D) paranoid (E) idea of reference Questions 79 and 80 82. A 45-year-old patient tells her doctor that after A 29-year-old married White woman is brought into hearing that her husband died, she could not the psychiatric emergency department by her hus- remember leaving her office and going home. band who reports that despite feeling depressed In every other respect, her memory is intact. 1 month ago, she now has been acting bizarre for the Which of the following types of amnesia is this past week. On initial interview, the patient states, “I an example of? feel superbly supreme, and you have no idea what an amazing person I am! I am a direct descendant of (A) continuous amnesia Queen Elizabeth!” The patient is talking so fast you (B) retrograde amnesia cannot interrupt her. Her husband reports that the (C) localized amnesia patient has not slept in over a week and that she has (D) generalized amnesia recently been adjusting her own medication(s) since (E) selective amnesia her last appointment over a month ago. In addition, her husband reports that this past week alone she has 83. Mr V is a 26-year-old single male brought into bought a new car, a diamond tennis bracelet, and the emergency room for acute paranoia. He her third designer handbag. admits to smoking “pot laced with something,” and soon after, he quickly became fearful that 79. The most likely diagnosis is which of the fol- other individuals were going to harm him. He lowing? was unable to be calmed down by his friends (A) a mood disorder and he developed palpitations and started (B) a psychotic disorder screaming that he was going to die. He claims (C) a personality disorder to only smoke cannabis on weekends and he denies any prior psychiatric history. Although (D) an anxiety disorder he was adopted, he was informed his birth (E) a factitious disorder mother had been schizophrenic and he him- self is now afraid of developing it. What are his 80. Which of the following medications was most chances of developing schizophrenia? likely to be the one adjusted by the patient? (A) 0–10% (A) buspirone (B) 10–20% (B) ranitidine (Zantac) (C) 20–30% (C) divalproex sodium (D) 30–40% (D) propranolol (E) 40–50% (E) zolpidem (Ambien) 58 3: Adult Psychopathology

84. When asked about his level of education, a 48- (D) hallucinations year-old man with a history of schizophrenia (E) catalepsy describes his high school grounds, friends he had at the time, clubs he joined, and his high 88. A 22-year-old man is brought to the emergency school graduation. He concludes by saying, department 2 weeks after a motor vehicle acci- “And that was the end of my schooling.” dent in which he suffered significant head Which of the following does this answer trauma. His parents are concerned because, demonstrate? although the patient recognizes them on the (A) loosening of associations telephone and responds appropriately, when he sees them face-to-face, he believes them to be (B) circumstantiality imposters who have replaced his real parents. (C) tangentiality He agrees that these people look like his real (D) pressured speech parents but is convinced that they are indeed (E) perseveration imposters. What delusion is this known as? (A) cacodemonomania 85. In her psychiatrist’s office, a patient suddenly lowers herself to the floor, begins flailing about (B) doppelganger wildly, then flings a garbage pail against the (C) folie à deux wall, and runs out of the office. Immediately (D) Capgras syndrome afterward, she returns. She is alert and ori- (E) delusion of reference ented, yet does not remember the incident. What is the most likely diagnosis? 89. A 50-year-oId woman is admitted to the hos- (A) temporal lobe epilepsy pital after complaining that she is dead. She believes that her flesh is rotting and that she is (B) jacksonian seizure able to smell the rancid odor. What delusion is (C) tonic-clonic seizure this known as? (D) complex partial seizure (A) Cotard syndrome (E) pseudoseizure (B) Capgras syndrome 86. In a 29-year-old man being treated for depres- (C) Fregoli syndrome sion, which of the following statements would (D) cacodemonomania most likely characterize his sleep patterns? (E) koro (A) His sleep patterns are unaffected. 90. A 36-year-old man presents to the emergency (B) He has less sleep latency. department after being found without cloth- (C) He has few if any episodes of early ing in the street. He has multiple excoriations morning awakening. all over his body and states that bugs are crawl- (D) He experiences less REM latency. ing all over him. His toxicology screen is posi- (E) His REM sleep tends to be distributed to tive for cocaine. What is this particular tactile the latter half of the night. known as? (A) hypnagogia 87. A 26-year-old man with narcolepsy explains that he has episodes of brief paralysis without any (B) hypnopompia loss of consciousness or other deficits. These (C) haptia attacks are usually precipitated by laughter or (D) formication anger. What is this phenomenon known as? (E) (A) hypersomnia 91. A 21-year-old man is noted to be restless and (B) constantly moving. He states that he feels as if (C) hypnagogic hallucinations Questions: 84–97 59

he has to be moving all the time and is uncom- (A) clang associations fortable if he sits still. Which of the following is (B) flight of this sensation known as? (C) hypermnesia (A) akathisia (D) logorrhea (B) akinesia (E) confabulation (C) tardive dyskinesia (D) dystonia Questions 95 and 96 (E) rabbit syndrome A 66-year-old college professor is being seen for a reg- ular checkup. On examination, the patient is noted to 92. A 21-year-old woman diagnosed with panic dis- demonstrate flat speech with no melodic intonation. order comes to the outpatient mental health In addition, although demonstrating proper word clinic with increased frequency of panic attacks choice and grammar, the patient’s answers are exten- and complains of feeling as if her surrounding sive with unnecessary detail but do reach a point. environment is unreal and strange. As a result of the increasing frequency of her attacks and this 95. When describing the lack of emotional com- new symptom, she has been unwilling to leave ponent of this patient’s speech, which of the fol- her apartment for several weeks. How is the lowing terms should be used? feeling of a surreal environment best described? (A) aprosody (A) depersonalization (B) stuttering (B) derealization (C) scanning (C) hypermnesia (D) aphasia (D) dereism (E) dysphagia (E) paresthesia 96. To describe the patient’s thought process in Questions 93 and 94 relation to the verbose, highly detailed answer, which of the following terms should be used? A 60-year-old man with alcoholism is brought to the emergency department by his family after they notice (A) flight of ideas a decline in memory. On evaluation, the patient’s (B) tangential remote memory is intact as verified by the family, but (C) circumstantial his recent recall is severely impaired. The patient (D) loosening of associations provides verbose but erroneous answers in response to questions testing recent recall. (E) derailment

93. This patient’s memory distortion is most likely 97. An 86-year-old woman in the intensive care which form of memory disorder? unit awakes at night and mistakes her intra- venous (IV) pole for a family member coming (A) anterograde amnesia for a visit. She calls the nurses to ask them to (B) retrograde amnesia have the visitor leave until morning. Which (C) dissociative amnesia term best describes the perceptual problem? (D) prosopagnosia (A) illusion (E) astereognosis (B) gustatory hallucination (C) micropsia 94. Which of the following best describes the (D) macropsia patient’s answers in response to recent memory testing? (E) palinopsia 60 3: Adult Psychopathology

98. A 24-year-old graduate student in philosophy (D) paralinguistic components of speech is referred by his student health center for a (E) prosopagnosia psychiatric evaluation. Although he claims to have had similar but attenuated symptoms 101. A 36-year-old woman complains of depression. since childhood, since beginning his thesis, She reports a decrease in sleep, weight, and he describes an acute worsening of fears that libido. She also complains of recent constipa- he will contract HIV. While he understands tion. The changes in the patient’s sleep, weight, the modes and risks of contraction and prac- libido, and bowel habits are all which of the tices safe sex, he is unable to “get rid of these following? thoughts.” As a result, he feels compelled to wash his hands many times per day, even to the (A) neurovegetative signs point of their becoming raw and bleeding. (B) word salad Despite his insight that his concerns are irra- (C) conversion tional, he is not able to stop the behaviors. A (D) catatonia positron emission tomography (PET) scan of (E) neologisms this patient’s brain would most likely demon- strate increased activity in which of the fol- 102. A 35-year-old man complains that his partner lowing structures? enjoys sexual activity only when inflicting pain (A) amygdala on him. This disturbs and frustrates him. This (B) caudate nucleus type of paraphilia exhibited by his partner is known as which of the following? (C) cerebellum (D) hippocampus (A) sexual masochism (E) parietal lobes (B) sexual sadism (C) exhibitionism Questions 99 and 100 (D) frotteurism While interviewing a 78-year-old woman, you dis- (E) transvestic fetishism cover that, although she is able to recognize objects, she fails to provide an accurate name for many of 103. A 34-year-old man reveals to his doctor that them. When the patient is shown a pen, the response he derives sexual satisfaction from rubbing up given is “that thing that you write with,” and when against women he doesn’t know while on shown a watch she replies “the time teller.” crowded subway trains. Which of the para- philias is this an example of? 99. The patient’s answers are examples of which of (A) exhibitionism the following? (B) fetishism (A) clang association (C) pedophilia (B) circumlocution (D) voyeurism (C) neologism (E) frotteurism (D) confabulation (E) apraxia 104. An anxious 23-year-old Asian male university student presents to student health services 100. Which of the following best describes this par- claiming that his penis is shrinking into his ticular type of language disorder? abdomen. Despite reassurances from the staff and the physician, he remains convinced of this (A) alexia belief. This patient is likely demonstrating a spe- (B) apraxia cific delusion known as which of the following? (C) anomia Questions: 98–109 61

(A) koro (A) formication (B) kuru (B) trichotillomania (C) taijin kyofusho (C) ideas of reference (D) zar (D) reaction formation (E) Capgras syndrome (E) hallucinations

105. A 21-year-old man was arrested after being 108. During a shift in the psychiatric emergency found outside of a window masturbating. He room, you are asked to see a 75-year-old man admitted to having been watching the young whose family is concerned that he is unable to girl inside over a period of several months. live by himself. You suspect dementia in the This patient demonstrates which paraphilia? patient who smiles and is pleasant but does not know who the president is or where he (A) transsexualism lives. If, while you are performing an MSE, the (B) hermaphroditism patient stands uncomfortably close to you as (C) voyeurism you talk to him, this would most likely be an (D) exhibitionism example of which of the following? (E) transvestic fetishism (A) formication (B) flight of ideas 106. A 23-year-old single White man is seen in the psychiatric emergency room after he was found (C) circumstantiality running naked through the streets and telling (D) ideas of reference police that he was Jesus Christ. You are able to (E) loss of ego boundaries elicit a history of paranoid ideation from the patient. His parents report a history of pro- 109. You are seeing a 26-year-old single White man gressive withdrawal from social activities. who began to experience auditory hallucina- Based on this patient’s likely diagnosis, in tions and delusions in college. Since then, he has which brain region is he most likely to have been hospitalized a number of times, and has abnormal findings on MRI? not been able to hold a job. His effect is bizarre. What are the negative symptoms of the condi- (A) hippocampus tion this patient most likely suffers from? (B) occipital lobe (C) parietal lobe (A) include affective lability (D) cingulate gyrus (B) are more prominent than positive symptoms in the prodromal phase of (E) basal ganglia the illness 107. You are seeing a 28-year-old unemployed man (C) do not include autistic features with a history of auditory hallucinations and (D) usually become milder over the course recurrent fears that his parents are trying to of the illness kill him. He does not know what year it is and (E) are easily distinguished from depressive is dressed poorly. He makes poor eye contact illness with you as you interview him. His fears about his parents represent what phenomenon? 62 3: Adult Psychopathology

Questions 110 and 111 man is more confused and does not seem to know who you are. There is no history of head You are asked to see a 32-year-old man who was trauma. The patient is unsteady on his feet, diagnosed with major depression 2 weeks ago and having brought a walker and, his wife notes, prescribed fluoxetine (Prozac). Once you enter the embarrassed that he has become incontinent. room and introduce yourself, you cannot get a ques- Neuroimaging with noncontrast CT in this case tion in as the man speaks rapidly about how terrific is most likely to reveal which of the following? a doctor you are and how wonderful his life is. He tells you about the three cars he recently purchased. (A) dilated lateral ventricles When you try to interrupt him, he says angrily, (B) normal ventricles “You’re just like my skinflint wife.” (C) frontal sulcal widening (D) focal subcortical hypointensities 110. This patient’s affect is best described as which (E) cerebellar atrophy of the following? (A) expansive and irritable 114. You are asked to give a psychiatric consultation (B) guarded and suspicious on a 28-year-old woman with systemic lupus (C) labile and dysphoric erythematosus who was admitted to the med- ical service. After you see her, one of your med- (D) euphoric and bizarre ical colleagues tells you that she will no longer (E) euthymic speak to any of them because she hates all of them and insists on seeing you because you 111. His speech is best described as which of the are the best doctor in the hospital. The psycho- following? dynamic term best used to describe the (A) uninterruptible patient’s conflict is which of the following? (B) pressured (A) acting out (C) hypermotoric (B) externalization (D) agitated (C) regression (E) tangential (D) splitting (E) sublimation 112. A 23-year-old White graduate student comes to the psychiatric emergency room complaining 115. A 26-year-old woman who recently gave birth of great anxiety. She has never been seen by a to twins presents complaining of falling asleep psychiatrist before and is taking no medica- without warning. Sometimes, she only awak- tions. Her vital signs are taken, and are notable ens when one of the twins begins crying loudly. for a heart rate of 100 beats/min. She is also She often finds that she has collapsed ungrace- slightly diaphoretic and has mildly dilated fully on the floor. She notes that the episodes pupils. Given her present state, one would tend to occur when she is trying to juggle sev- expect which area of this patient’s brain to be eral tasks at once and feeling overwhelmed. hyperactive at this time? The term used to describe the phenomenon of (A) locus ceruleus sudden loss of muscle tone associated with (B) hippocampus stressful situations is which of the following? (C) thalamus (A) cataplexy (D) amygdala (B) catalepsy (E) basal ganglia (C) catatonia (D) sleep jerks 113. One of your regular patients, a 75-year-old man, (E) primary atonia presents for a regular checkup with his wife. You notice that since his last visit a year ago, the Questions: 110–119 63

116. A 42-year-old woman is admitted to a medical 117. A 46-year-old married White woman is referred unit after a car accident that rendered her to you by a neurologist. She has had an exten- unconscious. A neighbor reports that she was sive neurologic workup, which was entirely taking an antidepressant but does not know negative, for the sudden onset of right arm which one. The patient regains consciousness paralysis 1 month ago. She says her husband after 3 days and corroborates that she was on was initially worried she had a stroke. When an antidepressant but says she cannot remem- describing the onset of the paralysis, she relates ber which one. She is started on paroxetine that she was just about to punish her 15-year- (Paxil) for her depression. However, 2 days old daughter who has been involved in heavy after this medication is started, she develops drug use and increasingly difficult to discipline tachycardia, diaphoresis, and myoclonic jerks. lately. In the central nervous system (CNS), the neu- rotransmitter associated with the above reac- 118. A 33-year-old divorced White woman is admit- tion is mostly synthesized in which of the ted to the inpatient psychiatric ward for appar- following? ently attempting suicide by cutting her wrists. When asked why she did it, she describes that (A) nucleus accumbens “the physical pain makes my emotional pain go (B) locus ceruleus away, for a little while, anyway.” She vacillates (C) raphe nucleus about whether this was a suicide attempt. She (D) caudate nucleus relates multiple complaints about various (E) substantia nigra levels of strife with roommates and family members. She is known from previous admis- DIRECTIONS (Questions 117 through 123): Each sions to the ward for labeling certain staff as her group of items in this section consists of lettered favorites and others as people “I hate.” On headings followed by a set of numbered words or MSE, you note significant emotional lability. phrases. For each numbered word or phrase, select Her father committed suicide when she was the ONE lettered heading that is most closely asso- 9 years old. ciated with it. Each lettered heading may be select- ed once, more than once, or not at all. 119. A 35-year-old man who has spent much of the past decade in jail or on parole for various mis- Questions 117 through 123 demeanors and felonies initially presents to your substance abuse unit under a false name. Choose the diagnosis that best fits the scenario given. He gives you his history in extensive detail, (A) borderline personality disorder but it nonetheless seems implausible. You talk to his sister, who tells you, “He’s the biggest (B) somatization disorder con artist there is—don’t believe a thing he (C) conversion disorder says!” She then relates that the patient stole his (D) antisocial personality disorder mother’s car last weekend. She believes he sold (E) schizoid personality disorder it for drugs, which he’s done in the past, she (F) schizotypal personality disorder says. (G) factitious disorder (Munchausen syndrome) (H) hypochondriasis (I) agoraphobia 64 3: Adult Psychopathology

120. A 36-year-old woman is referred to you by her encephalopathy including a sixth nerve palsy. gynecologist, who reports a long history of After appropriate treatment and admission to sexual and other nonspecific complaints that the hospital, the patient’s sixth nerve palsy have apparently shown no evidence of verifi- almost completely resolves. However, he has able disease. The gynecologist further relates nystagmus and an improved but wide-based that she believes this patient has a long history gait. He accurately describes his wife, who died of doctor shopping. On interview, the patient 10 years ago, and can name the last five presi- demands your attention to multiple physical dents. Immediate memory and attention are complaints: her back, belly, and chest pain; her normal. Each morning, he is alert and engages dyspareunia and excessive menstrual bleed- in conversation with the medical team, ing; and her constipation and lactose intoler- although careful examination reveals he has ance. She points out additionally that “nobody’s no memory of them from the day before. How been able to figure out why I can’t feel anything can these symptoms be explained? on the back side of my arm.” She has had mul- (A) alcoholic dementia tiple surgeries for abdominal complaints in the past, all with no significant findings or findings (B) Korsakoff syndrome inconsistent with her complaints. She relates (C) stroke that “I’ve always been sickly, most of my life.” (D) delirium (E) Alzheimer disease 121. A 41-year-old woman claims she is “certain I have breast cancer.” Workup by her internist, 125. You are seeing a 45-year-old man with no psy- including physical examination and mammo- chiatric history who tells you that his parents gram, is negative. are conspiring to destroy his privacy. He says they have planted a bug in his refrigerator to 122. A 26-year-old woman is admitted for sepsis. Her see what he eats. He is seeing you on his lunch roommate reports that a few days before admis- break from his job of 5 years, and appears well- sion she found the patient with a needle and dressed. He denies illicit drug use, and says syringe full of what the patient admitted was that he has a number of close friends who have toilet water. The patient’s physical examination tried to convince him his refrigerator is not reveals intravenous needle marks not made by bugged. What is the most likely diagnosis? any hospital staff. Records reveal she was admit- ted a year ago for gastrointestinal distress and (A) schizophrenia later admitted drinking drain cleaner. (B) brief psychotic disorder (C) delusional disorder 123. A 44-year-old stockbroker has discovered over (D) paranoid personality disorder the last few months that he experiences intense (E) MDD with psychotic features anxiety while taking the subway to work. He relates that “I can’t go to the supermarket or 126. You are asked by the surgical service at your any large store anymore, especially when hospital to see a 34-year-old woman well they’re crowded. I’m afraid my heart’s going to known to them from previous admissions with jump out of my chest!” a number of vague complaints that have no obvious physical cause: abdominal pain, DIRECTIONS (Questions 124 through 126): Each headache, and lower back pain. She appears of the numbered items in this section is followed tearful and says that although she does not by answers. Select the ONE lettered answer that is think she has anything serious tells you she BEST in each case. just wants to figure it all out so she can get home. What is the most likely psychiatric 124. You are seeing a 66-year-old man who was diagnosis? recently treated with thiamine for Wernicke Questions: 120–137 65

(A) somatization disorder (D) Alzheimer disease (B) conversion disorder (E) Hamilton (C) hypochondriasis (F) schizophrenia (D) malingering (G) glucocorticoids (E) factitious disorder (H) shared psychosis (I) PCP Questions 127 and 128 (J) Korsakoff syndrome A 42-year-old woman who lives alone is brought into the emergency room unconscious after an apparent 129. Most common diagnosis among psychiatric suicide attempt. She has left a note for her on-again, hospitalized patients in the United States off-again boyfriend and has taken a bottle of sleeping pills and drunk a bottle of wine. Her neighbor does 130. Folie à deux not know her psychiatric history but says she has been admitted to the hospital for previous suicide 131. Often causes psychotic and/or depressive attempts. symptoms

127. Which of the following disorders is most 132. Amyloid plaques closely associated with suicide? 133. Most dangerous withdrawal syndrome (A) alcohol dependence (B) schizophrenia 134. Depression rating scale (C) mood disorder (D) multiple sclerosis 135. Dementia praecox (E) cancer 136. Confabulation 128. Which of the following predictive factors is Questions 137 through 140 most associated with suicide risk? (A) depression Use the list below to identify the proper term for each question. (B) unemployment (C) age 45 and older (A) lability (D) male gender (B) irritability (E) divorce (C) dyscalculia (D) verbigeration DIRECTIONS (Questions 129 through 136): Each (E) dysarthria of the numbered items or incomplete statements (F) gustation in this section is followed by answers or by com- (G) glossolalia pletions of the statement. Select the ONE lettered answer or completion that is BEST in each case. (H) hypochondriasis

137. A 24-year-old man whom you have been seeing Questions 129 through 136 for 2 years in psychotherapy tells you that he (A) heroin was startled to learn that he could speak (B) Rorschach Spanish, which he never could before. (C) alprazolam 66 3: Adult Psychopathology

138. A 32-year-old man with schizophrenia repeats 141. I don’t even need to make money. Money the words “wong, wong, wong,” followed by makes the world go ’round, ’round the mul- “raizon, raizon, raizon.” berry bush the monkey chased the weasel. Weasels make good politicians you know. 139. A 79-year-old woman who has suffered a stroke is frustrated by her difficulty to form words. 142. People are in need wherever you go, and I’ve been to a lot of places. It is amazing how much 140. A 23-year-old man is alarmed that he cannot traveling I have actually done. I do suffer a bad chocolate. What sensation has he lost? case of wanderlust.

Questions 141 through 145 143. Medical school in pools with spools of thread till you’re dead, Fred. Five responses to the question “Why did you go to medical school?” are given. Match the response with 144. Religions are the most important of all thought. the appropriate disorder of thought process or com- Why do you bleed your school colors? I just ponent of thought process disorder. wanted some publicity for my movie. (A) circumstantiality (B) tangentiality 145. Well, medical school was actually something my parents...I am so worried about the state of (C) flight of ideas the world today. (D) loosening of associations (E) clanging (F) derailment Answers and Explanations

1. (C) The patient’s 3- to 4-year history of bizarre schizoaffective disorder, and schizophrenia, are behavior, delusions, and decline in social func- rarer. tioning strongly suggest that his illness is schiz- ophrenia. Its prevalence is 1%. 6. (C) Progressive social withdrawal is commonly seen as part of the prodrome of schizophrenia; 2. (C) In twin studies, schizophrenia’s monozy- at this point the patient’s diagnosis must be gotic concordance is 50% suggesting that there schizophreniform disorder because her symp- is a strong genetic component to the illness. toms have lasted longer than 1 month but less than 6 months. All other choices—low intelli- 3. (D) Nihilistic delusion content is classic for gence, head trauma, a neglectful mother, and Cotard syndrome, a psychotic/delusional abuse—have not been proven in any conclusive theme seen potentially in multiple psychotic way to be significantly linked to the disease, illnesses. The belief that people have been although early theories held that a history of replaced by imposters is the hallmark of one or more was a predisposing factor. Capgras syndrome. Folie à deux is a shared delusion aroused in one person by the influ- 7. (C) The patient meets the criteria for borderline ence of another. Although nihilism and nega- personality disorder characterized by rapid tivism can be observed in patients with MDD, mood swings, efforts to avoid abandonment, the predominance of psychosis in this patient chronic feelings of emptiness, intense anger makes that diagnosis unlikely. Similarly, this outbursts, impulsivity, fluctuations between patient does not meet the criteria for delusional idealization and devaluation, and recurrent disorder, which is characterized by nonbizarre self-mutilation or suicidality. Persons with this delusions. personality disorder commonly employ prim- itive defense mechanisms, such as denial, pro- 4. (B) The belief that people have been replaced jective identification, and splitting. Splitting is by imposters is the hallmark of Capgras syn- dividing up individuals into “all good” or “all drome. Simple schizophrenia is a disorder con- bad” categories. Intellectualization and undo- sisting entirely of negative symptoms with no ing are considered neurotic defenses, while positive symptoms. See the answer to question altruism and sublimation are mature defenses. 3 for definitions of the other choices. 8. (B) The downward drift hypothesis points out 5. (A) This case demonstrates an episode of post- chronic mental illness’s tendency to cause its partum psychosis characterized by depression, sufferers to move downward through the social mood lability, delusions, and hallucinations. strata. This downward movement has to do Although there can be different etiologies, most with the effects of the illness itself, not the side cases are a result of bipolar disorder. A less fre- effects of medications or the stress-diathesis quent underlying etiology is MDD. Primary model, which addresses an entirely different issue psychotic illnesses, such as delusional disorder, of susceptibility to illness. The socioeconomic

67 68 3: Adult Psychopathology

cost of schizophrenia is the result of drift, not depression, for which the treatment is removal its cause. Bleuler is known for his four As— of the inciting agent. association, affect, autism, and ambivalence— used to describe schizophrenia. 13. (B) Although many sleep disturbances have been described in depression (e.g., the other 9. (D) Clearly, on previous occasions, this patient’s choices, among others), early morning awak- presentation has strongly suggested schizo- ening has been most consistently linked with phrenia and now he is presenting with classic major depression. catatonic features: negativism, hypomotorism, and echolalia with echopraxia. The diagnosis of 14. (B) Increased cortisol in depression is one of the schizophrenia makes elective mutism very earliest observations in biological psychiatry, unlikely. There is no evidence of secondary and is well borne out in subsequent studies. gain, making a diagnosis of malingering simi- Catecholamines are decreased in depression as larly unlikely. Drug-induced psychosis is unlikely are sex hormones and immune function. Levels to result in the classical symptoms of catatonic of MAOs are unknown. schizophrenia. 15. (C) Bipolar I disorder clearly has a stronger 10. (A) This patient is likely suffering from a delu- genetic link than major depression and proba- sional disorder. This woman’s age, lack of prior bly has the strongest genetic link of all the psychiatric illness, and high level of social and major psychiatric illnesses. Bipolar II disorder occupational functioning are not consistent with has a strong genetic link but not as strong as a schizophrenia-spectrum diagnosis. Although bipolar I disorder. Bipolar illness usually carries she is delusional, her delusion is not bizarre a poorer prognosis than unipolar depression. A (i.e., it could possibly happen) and there is no higher percentage of patients with bipolar dis- associated hallucinations or disorganization. ease are eventually treated compared with patients with unipolar depression. Bipolar dis- 11. (A) Depression severity, history of poor order has no gender predilection and has not response to many medications (many clinicians been shown to be any more common in broad believe that all medication options should be geographic areas. exhausted), and the need for quick antidepres- sant action are really the only universally 16. (C) Antidepressants can precipitate or exacer- accepted criteria that encourage the clinician to bate mania in patients with bipolar disorder use ECT in major depression. Poor compliance who are not infrequently first diagnosed as suf- would push a clinician toward ECT only if all fering from unipolar depression. SSRI levels other methods of encouraging compliance had have not been shown to correlate with efficacy. failed. The other options all describe diagnoses Because the paroxetine appears to have pre- that would likely respond to ECT but would cipitated a mania, the drug should have been not necessarily move the clinician to ECT over stopped. Insomnia would have likely resolved medications. itself with the discontinuation of the SSRI and resolution of the manic episode. Starting two 12. (B) It is always essential to rule out bipolar dis- mood stabilizers at once is not supported by order before administering antidepressants, clinical trial evidence. because all antidepressants can induce mania in susceptible patients. However, antidepres- 17. (C) Cyclothymia is cycling between hypomania sants are sometimes carefully administered to and dysthymia. Treatment is the same as in bipolar patients who are predominantly bipolar disorder. Bipolar III disorder does not depressed. All the other diagnoses mentioned exist. Double depression occurs when a major would likely respond favorably to antidepres- depressive episode is superimposed on dys- sants, with the exception of drug-induced thymic disorder. Answers: 9–26 69

18. (B) First-degree relatives of patients with bipo- hippocampus, although important to short- lar disorder have a 25% risk of any mood dis- term memory, is not directly involved with reg- order. ulating aggression drives, sexual behaviors, and fear responses. The insula, found deep 19. (A) Dangerous drug abuse should be ruled out within the central sulcus, is medial to the tem- first in this patient; cocaine is more acutely dan- poral lobe and generally not considered a part gerous than LSD. She is on no medications, so of it. Superior temporal gyri are more generally she would have no reason to develop serotonin involved with processing complex auditory syndrome, a rare, severe adverse reaction to information such as the understanding of lan- serotonergic drugs. Carcinoid tumor, another guage. The lateral ventricle is a CSF-containing cause of serotonin syndrome, is equally unlikely. space that has no direct neuropsychiatric func- Hypochondriasis and specific phobia are seldom tional role. acutely life threatening. 24. (D) This patient would meet the criteria for 20. (C) Porphyria is manifested not uncommonly bulimia except that she missed three consecu- by psychiatric (manic or psychotic appearing) tive periods, which automatically places her in symptoms. It rarely occurs without abdominal the category of anorexia. Her anorexia appears pain. Each of the other choices would have similar to bulimia, however, and she would psychiatric symptoms less directly connected likely be further qualified as having the binge to the abdominal pain. eating-purging type of anorexia. Exercise can induce amenorrhea but other clues in this case 21. (A) Any type of hallucinations can be seen in indicate a more complex etiology of the loss of delirium. In schizophrenia-spectrum illness, menses. OCD is often found comorbid with including brief psychotic disorder, one rarely sees eating disorders in women, and indeed many hallucinations other than auditory. Nonauditory of the food-associated behaviors in anorexia hallucinations, then, as a general rule, suggest may seem similar to OCD-related rituals. In delirium of some cause. Delusions are thoughts this case, all of the patient’s symptoms are (fixed false beliefs) and do not include sensory explained in the context of an eating disorder. phenomena like hallucinations. Conversion dis- Psychogenic gastritis is not a DSM-IV-TR order does not usually show hallucinations of diagnosis. any sort. 25. (E) As with many forms of starvation, the pre- 22. (B) Klüver-Bucy syndrome presents with docil- dominance of catabolic over anabolic processes ity, lack of fear response, anterograde amne- tends to elevate BUN. Cortisol levels tend to be sia, hyperphagia, and hypersexuality. Pick elevated in anorexia reflecting the stress state of disease is a form of dementia, often indistin- the body. In anorexia, TSH responsiveness to guishable from Alzheimer disease, in which TRH remains normal. Persons with anorexia the frontal and temporal lobes are atrophied. also tend to be anemic. Serum growth hormone Arnold-Chiari syndrome describes a condition is often elevated in anorexia. with hydrocephalus and cerebellar anatomic and functional abnormalities. Punchdrunk syn- 26. (B) Fluoxetine, an SSRI, is the treatment of drome describes an acquired movement disor- choice for panic disorder with or without ago- der associated with traumatic damage to the raphobia. Alprazolam, although commonly substantia nigra. Möbius syndrome is congen- used for panic disorder, is a benzodiazepine ital absence of the facial nerves and nuclei with that is effective in the short term but has con- resulting bilateral facial paralysis. siderable dependence liability. In addition, its short half-life may actually cause withdrawal 23. (D) Klüver-Bucy syndrome is most closely states that mimic panic. Chlordiazepoxide is a associated with severe damage to, or discon- long-acting benzodiazepine that is also effec- nectivity of, the amygdala bilaterally. The tive in panic disorder but also carries some 70 3: Adult Psychopathology

dependence liability. SSRIs have been shown in patient desires this route. There is no associa- trials to be as effective as benzodiazepines but tion of methadone treatment with Cushing syn- without the abuse potential. Phenelzine, a drome. monoamine oxide inhibitor (MAOI), has been shown to be effective in some studies in anxi- 31. (A) Cases of are consid- ety disorders but its side effect profile is ered to be a psychiatric emergency. Because of regarded as problematic. Divalproex sodium the immediate danger to the infant in this case, has not been demonstrated to be effective for the patient requires admission to the hospital. panic. While the other choices may certainly be appro- priate in her management, protecting the baby 27. (C) This is one of the most important biological is the first and most important matter. If a findings in affective disorder research. Studies patient refuses admission, certification to a show that in major depression, about half of all state hospital and/or involvement of Child patients do not have blunted cortisol levels to Protective Services will likely be necessary. an administration of dexamethasone the night before. This is thought to indicate abnormal 32. (B) This patient’s history is most consistent feedback control in the hypothalamic-pituitary- with a mood pattern defined by prolonged adrenal axis in major depression. Patients with periods of hypomania (symptoms of mania not psychotic depression are even more likely to severe enough to cause occupational dysfunc- have poor dexamethasone suppression. tion or psychiatric treatment) and now a major depressive episode. Hypomania with major 28. (B) About 30% of all patients with major depression defines bipolar II disorder. In bipo- depression do not show an increase of TSH lar I disorder, the mania is more severe causing with administration of TRH. notable occupational dysfunction and usually contact with psychiatrists. This patient is in 29. (C) Persons with schizoid personality are reclu- the midst of a major depressive episode but his sive and do not mind the lack of social interac- history of mania indicates a bipolar diagno- tion. Agoraphobia is tied to the fear of panic sis. This is an important distinction to make in symptoms in public. Such symptoms are not diagnosis because improper treatment with mentioned in this case. Persons with avoidant antidepressants can precipitate a manic episode. personality are shy and fearful of social rejec- Narcissistic personality disorder is an Axis II tion. However, their lack of socialization is dis- diagnosis. This patient does display impulsiv- tressing to them. Schizotypal persons can have ity but impulse control disorder, not otherwise schizoid features but they also have bizarre specified, can be diagnosed only after the thinking. Patients with autism have pro- exclusion of major mental illness such as bipo- nounced deficits in language, communication, lar disorder, which may have impulsive fea- and socialization, which are not prominently tures. reflected in this case of a woman who had uneventful but happy formative years. 33. (B) Dry mouth, dizziness (associated with hypotension), and urinary hesitancy are due 30. (D) Studies show that most addicts do not to anticholinergic and adrenergic blocking experience a sufficient decrease of their crav- effects of TCAs such as imipramine. Fluoxetine, ings at less than 60 mg/day during initial treat- an SSRI, is most often associated with gas- ment. The fear of bone decay and loss of libido trointestinal upset, sexual dysfunction, and agi- are popular misconceptions about the effects of tation. Phenelzine, an MAOI, can be associated methadone. Although an opiate, methadone with hypotension but is less likely to have anti- has a longer half-life than heroin (and thus a cholinergic effects. Lithium is most often asso- milder withdrawal syndrome) and is given in ciated with polyuria, polydipsia, tremor, and a treatment setting in which complete opiate mental confusion at higher doses. Divalproex detoxification is more likely to succeed if the sodium may cause gastrointestinal upset but is Answers: 27–42 71

otherwise commonly associated with sedation duration of the disturbance must be 1 month or and tremor at higher doses. less. This patient’s delusions are too bizarre to meet the criteria for delusional disorder. 34. (B) The classic triad of NPH is confusion, gait Psychosis, not otherwise specified, is reserved ataxia, and incontinence. Elevated opening for cases of psychosis in which the etiology pressure is not found in NPH (thus the “normal may be related to substance abuse, a medical pressure” part of the diagnosis). Oculomotor condition, or an endogenous psychotic illness difficulties are a part of the Wernicke-Korsakoff of unknown duration. syndrome. Frontal release signs and persever- ation are nonspecific findings common in 39. (A) This patient has symptoms consistent with demented patients. PTSD or acute stress disorder (i.e., traumatic event, intrusive memories of trauma, avoid- 35. (A) CT scan commonly reveals dilated ventri- ance of reminders, hypervigilance), but the cles thought to be the result of increased pres- fact that the trauma was only 3 weeks prior sure waves impinging within the ventricular means that she would be diagnosed with acute system. Normal ventricles would not be expected distress disorder. If her symptoms persist after in NPH patients. Frontal sulcal widening is found 4 weeks, then she would be diagnosed with in dementias that have underlying cerebral atro- PTSD. Adjustment disorder most often repre- phy. Hypointensities found in subcortical areas sents a change in mood, anxiety, or conduct are often indicative of lacunar strokes. Cerebellar that happens after a nontraumatic event. In atrophy is seen most often in congenital disorders GAD, there is no associated trauma. The patient and alcoholism. has no evidence of MDD.

36. (A) The patient’s inability to quit drinking 40. (C) The lifetime incidence of suicide in patients regardless of a desire to quit or knowledge of with schizophrenia is about 10%, compared its negative aspects best differentiates depend- with less than 1% in the general population. ence from abuse. Neither dependence nor abuse is determined based on quantity of alco- 41. (A) Feelings and attitudes originating from the hol consumed. The patient’s history of drunk treater, evoked by the patient, are called coun- driving and gastric ulcer are both consequences tertransference. Transference denotes feelings and of the abuse construct of DSM-IV-TR. The attitudes about the treater coming from the patient’s statement that he cannot sleep when patient. Empathy is the ability of the treater to he tries to quit drinking does imply physio- psychologically put himself in his patient’s logic dependence but the alcohol dependence shoes and thereby understand his patient’s criteria can be met with a “without physiologic thinking, feelings, or behavior. Identification is a dependence” qualifier. rather unconscious incorporation of someone else’s traits into one’s own manner—for exam- 37. (C) Patients with alcoholism often are found to ple, adolescents having hairstyles similar to have elevated GGT on routine laboratory admired rock stars. Projection occurs when one screening. They also may have an increased assigns emotions to another person in an ALT and AST but with an increased AST/ALT attempt to psychologically cover up the pres- ratio not increased ALT/AST. Patients with ence of those emotions within himself. For alcoholism also commonly have increased uric example, the doctor in the case may tell his acid levels, macrocytic anemia, and increased fellow resident that he believes that his female serum triglycerides. patient is attracted to him, when in actuality he is attracted to her. 38. (D) This patient would meet the criteria for schizophrenia except the duration of his illness 42. (E) This patient displays symptoms of fear of has been less than 6 months. Brief psychotic loss and abandonment, intense interpersonal disorder may present with psychosis but the relationships, recurrent suicidal behavior or 72 3: Adult Psychopathology

threats, affective instability, and difficulty con- effects versus memory impairment (which is trolling intense anger. Patients with borderline more likely with 20 or 40 treatments). When personality disorder have depressive symp- ECT is given for psychosis or mania, 20 or more toms that may become full-blown as a result of treatments may be necessary for a positive their social isolation, but this scenario does not response. mention full criteria for major depression. Patients with histrionic personality disorder, 46. (B) This patient has recurrent thoughts that are like those with borderline personality disor- distressing. They are conceived as a product der, may display excessive emotionality and of her mind and not as implanted messages, as attention seeking, but their core symptoms may happen in schizophrenia. She attempts to center around superficial seductiveness and ignore or suppress her symptoms by not theatricality. Patients with cyclothymia often explaining to her husband how she feels or by present as a diagnostic question with regard getting rid of all the knives in her kitchen. to the possibility of a borderline personality Impulse control disorder, not otherwise speci- diagnosis. This is because of their prominent fied, is reserved for the inaction of impulses mood instability and impulsivity, which hap- that are not as directly ego-dystonic (disturb- pens in both disorders. In this case, the other ing), as in this case, and that do not have an criteria for a borderline diagnosis resolves this obsessional nonenacted (thinking) component. dilemma. Psychotic disorder, not otherwise Obsessive-compulsive personality disorder is a specified, is reserved for more frank delusional pervasive character style marked by a pattern symptoms of unclear etiology than are evident of preoccupation with orderliness, perfection- with this patient who is paranoid that “all the ism, and mental and interpersonal control. staff hates me.” These persons are often described as control freaks by laypersons and are noted to be par- 43. (E) Psychotherapy and steady social support ticularly inflexible. Psychosis, not otherwise represent the best long-term method for the specified, is inappropriate because this patient management of borderline personality disor- has fair insight regarding her obsessional think- der. Dialectic behavioral therapy has gained ing. She does act on her thoughts and they are recent popularity as a form of psychotherapeu- disturbing to her. She is thus not psychotic. tic intervention for these patients. Neuroleptics, This patient does not display a thinking style mood stabilizers, and antidepressants have all that is marked by magical thinking. There is no been shown to have some efficacy in treating mention of a history of strangeness prior to the target symptoms in some borderline patients. birth of her child. Therefore, schizotypal per- Benzodiazepines may treat anxiety in these sonality disorder is not appropriate. patients but they have particular abuse liability in these impulsive patients. 47. (E) Fluvoxamine and other SSRIs (paroxetine, fluoxetine, sertraline) are known for their 44. (A) Amnesia is a common side effect of ECT antiobsessional effects as well as their antide- presenting in the form of short-term memory pressant and antianxiety effects. Lorazepam, a deficits. Anesthesia-induced respiratory dys- benzodiazepine, may decrease anxiety but has function, fractures, and arrhythmias are rare no direct antiobsessional effects. Benztropine is side effects. Psychosis can be improved rather an anticholinergic typically used to counter than worsened; indeed, ECT is regarded as neuroleptic-induced extrapyramidal symp- most indicated in psychotic depressions. toms. Nortriptyline and other TCAs have not proven efficacious for OCD. Haloperidol is a 45. (C) For catatonic conditions, two or four ECT typical high-potency neuroleptic that may be treatments may be effective. In major depres- useful in forms of OCD in which there is a psy- sion, 6–12 sessions are generally optimal for a chotic component. However, it does not ame- good risk-benefit ratio of positive treatment liorate purely obsessional thinking. Answers: 43–54 73

48. (C) This patient’s MSE is most consistent with tranquility so that I can get what’s coming to delirium in that it is remarkable for relatively me upside down.” Patients with paranoid rapidly fluctuating memory, orientation, and schizophrenia may be relatively high func- attention. Also, her apparent dehydration and tioning between episodes of psychosis. They malnourishment provide evidence for an may have relatively little thought disorder and underlying medical cause of her mental status may not have other bizarre manners. The diag- change. Amnestic disorder is not likely given nosis of residual schizophrenia is reserved for this patient’s shifts in attention and memory patients who have not had prominent positive fluctuations. Cognitive disorder, not otherwise symptoms (primarily thought disorder and specified, is reserved for cases exclusive of psychosis) for some time and present with pri- dementia and delirium such as cases of trau- marily chronic negative symptoms (anergia, matic brain injury. Although this patient would anhedonia, and flattened affect). A diagnosis of seem a likely person to have dementia because undifferentiated schizophrenia is reserved for she came from a nursing home, her fluctuating cases in which the clinician cannot distinguish mental status is indicative of a delirium. a subtype. Often, these cases present with mild Dementias are diagnosed in the context of a components of all the types. relatively stable set of deficits on an MSE, and the underlying causes are rarely associated 52. (D) Catatonic schizophrenia has always been with acute medical insults. Importantly, how- the least prevalent subtype of schizophrenia ever, the presence of dementia does predispose according to most studies, but has also become to delirium. Major depression can cause cog- increasingly rare in recent decades. It has been nitive deficits on MSEs that are reversible with suggested that changes in diagnostic criteria antidepressant treatment. These pseudodemen- may account for this finding. Additionally, tias, like dementia, most often present as a some forms of mental illness that were diag- more stable examination than is illustrated in nosed as catatonic schizophrenia may have this case. been cases of neurosyphilis, which with antibi- otics has become increasingly rare. It is possi- 49. (C) About one-third of all patients with depres- ble also that the advent of neuroleptic and sion respond to the surprisingly powerful other modern medications may have extin- placebo effect. However, this response is not as guished catatonic symptoms in schizophrenic well sustained as in patients placed on antide- populations. pressant medications. 53. (C) There is no evidence of occupational or 50. (D) Standard antidepressant therapies, includ- social dysfunction or marked distress caused ing SSRIs and TCAs, have a positive response by this patient’s activities. Virtually all DSM- in 65–75% of patients. IV-TR diagnoses are qualified by these crite- ria. If the patient did describe such concerns, 51. (A) Schizophrenia, catatonic subtype, is defined the diagnosis would most likely be sexual by a predominance of symptoms that involve masochism, in which patients are aroused by motor behaviors and nonverbal forms of com- psychologically or physically punishing acts munication such as facial expression. This cat- by another person (or fantasies of punishment). egory includes famous cases of waxy flexibility, In sexual sadism, the patient is aroused by echolalia (repeating another’s words inappro- giving such punishment (or fantasies of giving priately), and echopraxia (repeating another’s it) to others. Fetishism involves sexual arousal gestures inappropriately). Disorganized schiz- connected to nonliving objects. In frotteurism, ophrenia, traditionally known as hebephrenia, the patient is sexually aroused by touching or is notable for pronounced and severe thought rubbing up against a nonconsenting person. disorder that can be relatively refractory to neu- roleptic treatment. Such patients may present 54. (C) Narcolepsy is a disorder often affecting per- with a chief complaint as “I came to restore my sons in their teens or twenties. It is a disorder 74 3: Adult Psychopathology

of REM sleep mechanisms that also affects 57. (C) Positive prognostic factors include late onset dogs. Circadian rhythm sleep disorder is a of illness, obvious precipitating stressors, acute sleep disorder caused most often by , good premorbid social and work func- scheduling changes such as or night- tion, marriage, family history of mood symp- shift work. Dyssomnia, not otherwise speci- toms, and positive symptoms (hallucinations). fied, is reserved for sleep disturbances of Negative prognostic factors include early onset unknown cause or causes associated with envi- of illness, lack of precipitating stressors, insidi- ronmental disturbances such as those that pro- ous onset, poor premorbid social and work duce prolonged . Restless legs function, withdrawn/autistic behavior, posi- syndrome falls under this category. Nightmare tive family history of schizophrenia, poor social disorder is diagnosed in the absence of other supports, negative symptoms (disorganized predisposing mental illness (e.g., PTSD) in behavior, amotivation), neurologic symptoms, which the patient has repeated nightmares and history of perinatal trauma. Choice A is not causing significant distress. Primary hypersom- true; although some patients may recover (often nia is excessive daytime drowsiness or hours with the proper treatment), the majority do not spent sleeping at night that is not better spontaneously recover. Choices B and D may be accounted for by an identifiable environmental possible scenarios but cannot be conclusively or medical cause, substance abuse, or mental stated about the prognosis of this patient. There disorder (e.g., major depression). Additionally, are, of course, no medications without risks. it is not associated with specific stigmata of narcolepsy such as cataplexy or . 58. (C) This patient’s symptoms fit DSM-IV-TR cri- teria for PMDD. Her symptoms are not severe 55. (D) Methylphenidate has been shown to be enough nor of sufficient duration (2 or more useful in inhibiting onset of narcoleptic episodes, weeks) to diagnose MDD. The remittance of likely owing to the capacity for the drug to symptoms in the week after menses is typical of enhance CNS arousal mechanisms that inhibit PMDD. In adults, dysthymic disorder is char- REM-related mechanisms. Nighttime benzodi- acterized by subthreshold depressive symp- azepine regimens are not particularly effective toms, for more days than not, over a 2-year for narcolepsy. Antidepressant treatment with period. The patient does not describe promi- TCAs has been found to be useful in combina- nent anxiety symptoms making GAD unlikely. tion with methylphenidate in some patients This is not just normal female behavior. with narcolepsy. However, the efficacy of bupro- pion, MAOIs such as phenelzine, and SSRIs 59. (B) This patient is displaying symptoms con- such as fluoxetine has not been well-studied. sistent with MDD. Asking about suicidal ideation and assessing suicide risk is extremely 56. (E) None of the listed medications have been important in individuals with depressive dis- proven completely safe in pregnancy, but stud- orders, as the risk of completed suicide is ies of fluoxetine have led many psychiatrists to approximately 10–15%. conclude that its benefits outweigh its risks. Haloperidol and chlorpromazine are Class C 60. (C) Heroin overdose is most likely to have drugs by Food and Drug Administration (FDA) caused the clinical situation described. Cocaine rating, signifying that adverse fetal effects have use causes pupillary dilatation not constriction. been seen in animals, but there are no human PCP and alcohol intoxication may cause coma data available. Lithium is a Class D drug, mean- but both are associated with nystagmus rather ing that there is known human fetal risk, but the than pupillary size changes. Inhalants are also drug may be used in a life-threatening situation known to cause coma but are not classically or when clinical circumstances justify its use. associated with pupillary constriction. Divalproex sodium is Category X, indicating that there is proven fetal risk in humans such 61. (B) Naloxone is used to reverse the acute effects that the risk outweighs the benefit. of opiate overdose by blocking CNS opioid Answers: 55–67 75

receptors. Acetylcysteine is administered in acet- 64. (B) This case best depicts a case of a patient aminophen overdose and deferoxamine is used who is feigning or producing symptoms of an in iron overdose. Methylene blue is used to treat illness to gain gratification from assuming the methemoglobinemia. Methadone is used for the sick role. There is little reason to believe this act long-term maintenance of opiate addiction and could help her achieve financial or other mate- would only worsen heroin overdose. rial gain as in malingering. In conversion dis- order, patients are primarily preoccupied with 62. (D) This patient is suffering from symptoms of the idea that they have an illness that presents posttraumatic stress disorder (PTSD), an anx- with a constellation of neurologic symptoms iety disorder consisting of reexperiencing symp- that are usually not physiologic. In hypochon- toms (e.g., nightmares, flashbacks, intrusive driasis, a patient believes that he or she has thoughts), increased arousal (e.g., hyperstartle, some particular medical diagnosis. It is the hypervigilence, irritability), and avoidance/ worry about this illness that causes much of numbing. The best studied and most efficacious the distress. Patients with somatoform disorder medications are considered to be the SSRIs. tend to have clusters of subjective symptoms Antipsychotics, including the atypicals, should over time affecting multiple possible organ not be used as monotherapy, although may be symptoms or anatomic parts that do not usu- used in conjunction, especially if there are asso- ally correlate to any specific medical diagnosis. ciated psychotic symptoms. Benzodiazepines should be avoided in this patient population 65. (B) This patient meets criteria for a current not only because of the significant comorbidity manic episode with symptoms of psychomotor with substance addiction but also as they have agitation, pressured speech, grandiose think- not been found to be particularly effective. ing, flight of ideas, and history of excessive Mood stabilizers, such as lithium and valproic spending as evidenced by his credit card debt. acid, occasionally are used in addition to SSRIs His history of good premorbid functioning and in order to target the mood lability or aggres- a remitted major depression are also consistent sion sometimes seen in PTSD. with a diagnosis of bipolar disorder rather than schizophrenia. Adjustment disorders present 63. (A) This is an example of a case of panic disor- with an identifiable stressor creating a milder der with agoraphobia inadequately treated mood, anxiety, or behavioral disturbance than with appropriate medication namely sertraline. is present in this case. The patient’s presenta- Many mental illnesses including anxiety are tion of mania rules out major depression. In best treated with a combination of pharmacol- fact, although he may have future major ogy and psychotherapy. The best-studied psy- depressive episodes, he should never be diag- chotherapy for panic disorder is CBT. Eye nosed with MDD due to the current episode of movement desensitization and reprocessing mania. This patient’s history does not reveal an (EMDR) is a specific therapy that has been exposure to an acute severe psychological developed for and studied in the treatment of stressor and he does not present with symp- PTSD. Insight-oriented therapy is a long-term toms of reliving past traumas, hypervigilance, dynamic therapy. Although it may be useful and avoidance as in PTSD. in some cases of panic disorder, it has not been as validated as CBT. Interpersonal therapy 66. (C) Bipolar disorder usually presents in a range addresses relationships as a contributor of of ages from childhood to age 50, with a mean depression and is, therefore, used to treat indi- age of onset of 30. viduals with MDD. The purpose of supportive psychotherapy is to strengthen the patient’s 67. (C) Classically, infarcts of the left frontal hemi- defense mechanisms in order to return them to spheres (part of left middle cerebral artery dis- a previous level of functioning. It has not been tribution) present with depression whereas adequately studied for panic disorder. those of the right frontal hemisphere present 76 3: Adult Psychopathology

with euphoria, inappropriate indifference, or are associated with the most common form of mania. Obsessive-compulsive behaviors pres- cortical dementia, Alzheimer disease. ent occasionally after diffuse bilateral frontal injury. Panic and anxiety symptoms have not 72. (B) The patient appears to be suffering from been described as having any particular asso- delirium. Although aggressiveness, memory ciation with left middle cerebral artery strokes deficits, psychotic symptoms, and uncoopera- although such comorbid cases probably exist. tiveness may be seen in either delirium or dementia, a fluctuation in the level of con- 68. (A) Hypnosis is a state in which critical judg- sciousness (i.e., from alertness to ) ment is suspended and a person experiences is the hallmark for delirium. alterations in perception, memory, or mood in response to suggestions. Hypnosis is not psy- 73. (E) Generally speaking, patients with pro- chotherapy. It is better understood as a proce- nounced negative syndromes have greater dure that may be used as an adjunct to social and occupational difficulty, refractori- psychotherapy. Hypnosis should not be used if ness to medications, and more chronicity of ill- the therapist is not willing or able to treat the ness compared to patients whose diseases are patient with psychotherapy, because it seems to more episodic with acute psychotic exacerba- help some patients talk about events that are tions. Onset of symptoms late in life is a good otherwise too emotionally laden to be prognostic indicator for patients with schizo- expressed. The judgment of whether these phrenia, as is good premorbid functioning. thoughts are repressed memories is controver- Patients who are married also do better than sial. Hypnosis cannot be tracked or mapped those who are single, possibly reflecting the with an EEG. advantages of social support in the marriage. Marriage is more likely be an option for those 69. (E) About 90% of cases of impotence in the age patients who are less socially impaired on group 30–50 have a psychological etiology. account of having milder negative symptoms. After 50, the etiology becomes increasingly Patients with a positive family history of affec- medically related to causes such as medica- tive disorders, rather than schizophrenia, have tions, diabetes, hypertension, and alcoholism. a better prognosis in schizophrenia. Generally speaking, the more schizophrenia-like the clin- 70. (E) The serotonin metabolite designated ical picture and the family history, the worse 5-hydroxyindole acetic acid (5-HIAA) measured the prognosis, and the more affective-like, the in the CSF has been shown to be lower than better the prognosis. The sudden-onset symp- control groups in postmortem analysis of vic- toms in schizophrenia, as opposed to insidi- tims of suicide and patients with impulsivity ous onset, is a good prognostic indicator, and violence or aggression. No such associa- possibly because it reflects less chronicity of tions have been established with the other neu- symptoms and a greater chance of medical or rotransmitters listed. psychoactive agent-related psychosis.

71. (D) Depression is the most common presenting 74. (C) About 12% of children who have one parent psychiatric symptom in patients with multiple with schizophrenia suffer from schizophrenia; sclerosis. It may present during the course of the risk climbs to 40% if both parents have mild progressive cognitive decline that is con- schizophrenia. sistent with a subcortical dementia (slowing of mental processing, motor difficulties, blunted 75. (C) Of all identical twin siblings with schizo- or inappropriate affect, relative preservation of phrenia, about one-half of their twins also have language-dependent skills). Neuroimaging of schizophrenia. These data have served as these patients usually shows diffuse white important evidence that schizophrenia repre- matter plaques affecting the frontal lobes. sents the result of a combination of environ- Ventricular enlargement and cerebral atrophy mental stressors and genetic vulnerability. Answers: 68–82 77

About 12% of dizygotic twins of schizophrenic considering possible consequences. A psychotic patients also have schizophrenia. A nontwin disorder like schizophrenia cannot be diag- sibling of a person with schizophrenia has nosed because there is no indication of promi- about an 8% risk of having schizophrenia. nent psychotic symptoms (hallucinations, disorganized thought processes and speech) 76. (E) There are a number of stigmata associated present for at least 6 months. Personality and with schizophrenia that are not included in the anxiety disorder do not present with the classic diagnostic criteria. These include soft neuro- signs of mania. There is no indication that she logic signs such as short-term memory deficits, is making up her symptoms for some second- unstable smooth-pursuit eye movements, and ary gain. decreased ability to habituate to repeated sen- sory stimuli (sensory gating-abnormalities). In 80. (C) From the choices given, the patient was addition, patients with schizophrenia have dif- most likely taking either lithium or divalproex ficulty in conceptualizing complex visual com- sodium, which are both mood stabilizers and positions. Patients with major depression, both indicated for the treatment of bipolar dis- PTSD, OCD, and ADHD also have been shown order. Buspirone, an anxiolytic, and propra- to have short-term memory difficulties, but not nolol, a beta-blocker used occasionally for GAD the other stigmata mentioned. and panic disorder, are not likely to have been prescribed for this patient’s bipolar disorder. 77. (C) The most appropriate next step is to send Ranitidine, although sometimes confused by the patient to the psychiatric emergency patients for the benzodiazepine alprazolam, department. This patient is experiencing an is an antacid. Changing the dose of zolpidem, acute episode of major depression and because a sleeping aid, would not account for her of his suicidal ideation with thoughts of over- symptoms. dosing, he should be referred to the emergency department for further assessment while ensur- 81. (E) The specific delusion referred to here is an ing his safety. Telling the patient he will be fine, idea of reference. The patient interprets an prescribing medication, or contacting his par- event as relating to him, even though it clearly ents about transportation and a family meeting does not. Thought insertion is a delusion in does not adequately address his immediate which the patient believes thoughts from some safety. external entity are placed in his mind. Somatic delusions refer to a delusion about the patient’s 78. (D) The woman presents with a classic presen- body, for example, a common belief is that tation for MDD. Having a depressive disorder one’s body parts are diseased. Thought broad- significantly increases one’s risk for suicide. casting is the belief that one’s thoughts can be Many other factors also increase the risk for heard or somehow known by others without completed suicide such as White race, male direct communication. Paranoia often colors gender, older age, single/divorced, and being many delusions and involves the belief that Protestant or Jewish religion. other people, agencies, or forces mean harm or danger to the patient. Any of the above delu- 79. (A) The most likely diagnosis is bipolar disor- sions may include elements of paranoia. der classified in the DSM-IV-TR as a mood dis- order in which the individual experiences 82. (C) The choices in this question describe differ- symptoms of a depressive disorder and symp- ent patterns of amnesia. Localized amnesia refers toms of mania independently during the to memory loss surrounding a discrete period of course of the illness. The current episode exem- time, typically occurring after a traumatic event; plifies a manic episode with symptoms that in this case, the patient’s amnesia results from include inflated self-esteem and grandiosity, learning of her husband’s death. Selective amne- pressured speech, a decreased need for sleep, sia involves the inability to recall certain aspects and an impulsive shopping excursion without of an event, though other memories of the event 78 3: Adult Psychopathology

may be intact. Rarely, a patient may forget his 86. (D) Abnormal sleep patterns are common in or her entire preceding life (generalized amnesia) depression and may be the chief complaint of or forget all events following a trauma, except the depressed patient. Classically, increased for the immediate past (continuous amnesia). sleep latency (difficulty falling asleep), early Retrograde amnesia is any amnesia for events morning awakening, and increased wakeful- that come before a traumatic event. ness are seen. REM sleep tends to be redistrib- uted so that most of it occurs in the first half of 83. (B) Having one parent with schizophrenia con- the night. REM latency (the period of time fers an increased risk of developing schizo- between falling asleep and starting REM sleep) phrenia of approximately 12%. Having two decreases. parents with the illness further increases the risk to over 40%. Having a monozygotic twin 87. (B) Narcolepsy is a sleep disorder characterized with schizophrenia increases the risk to by the tetrad of hypersomnia (excessive daytime 40–50%. somnolence), cataplexy (transient loss of motor tone associated with strong emotions), sleep 84. (B) Circumstantiality involves a circuitous, over- paralysis (total or partial paralysis in sleep-wake inclusive answer that only eventually gets to transition), and hypnagogic hallucinations (vivid the point as the patient demonstrates in this -like hallucinations occurring in the example. Loosening of associations is a response wake-sleep transition). Hypnopompic hallucina- composed of a series of disconnected ideas. tions occur in the sleep-wake transition. Catalepsy Tangentiality occurs when the patient strays off is a state of immobility sometimes seen in cata- the point entirely never returning to the origi- tonic states. nal intention of the answer. Pressured speech is a continuous, usually rapid, uninterruptible 88. (D) Capgras syndrome is the delusional idea that stream of ideas that may make sense but is imposters have replaced once familiar persons. often hard to follow. This patient’s speech may This belief is held despite the acknowledgment be pressured but the question gives no clues to that the people appear exactly the same. this. Pointless repetition of a word, a phrase, or Doppelganger is the belief in the existence of an an idea is perseveration. Generally, the speech of identical counterpart. Cacodemonomania is the a schizophrenic patient may demonstrate any delusion of being poisoned by an evil spirit. of these characteristics; his diagnosis provides Folie à deux is a delusion shared by more than no clue to the answer. one person, and delusion of reference is the ascribing of personal meaning or messages that 85. (E) A pseudoseizure is a psychogenically induced are not intended. behavior that resembles epileptic activity. Although the syndrome may in some cases be 89. (A) Cotard syndrome is a delusion that nothing motivated by secondary gain (as in malinger- exists (nihilism) or the person herself does not ing), it most often occurs in reaction to stress or exist. The person may feel that her body has dis- in a setting of personality disorder, severe affec- integrated or that she is dead. Occasionally, the tive disorders, or conversion reactions. The belief may be so vivid that it can even be asso- other choices, which each represent true epilep- ciated with the sensation of a rancid or rotting tic seizures, rarely, if ever, include purposeful, odor, which is quite unpleasant for the patient. complex activity as is seen in this case. Often, It can be seen in schizophrenia or severe bipolar patients with pseudoseizures have a concur- disorder. It can also be early evidence of rent seizure disorder, and it is quite difficult to Alzheimer disease. Capgras syndrome and sort out the true seizures from the pseudo- Fregoli syndrome are both delusions but are seizures. Diagnosing pseudoseizure often opposite in content. Capgras syndrome is the requires demonstrating a normal EEG at the belief that an imposter has replaced a familiar time of the pseudoseizure. The prognosis for person, whereas Fregoli syndrome is the delusion pseudoseizures is quite poor. that a person is taking the form of a number of Answers: 83–96 79

people or creatures. Cacodemonomania is the of long-term alcoholism. Anterograde amnesia is delusion that one is poisoned by an evil spirit. the loss of immediate or short-term memory. Koro is a traumatic fear that the penis is shrink- Patients are unable to form new memories. ing into the body cavity. Retrograde amnesia is the loss of remote or pre- viously formed memories. Dissociative amnesia 90. (D) Formication is a particular type of tactile is the loss of memory without the loss of abil- hallucination in which one has the sensation of ity to form new memories. It is usually asso- bugs crawling on or under the skin. It can be ciated with emotional trauma and is not due seen in cocaine intoxication and in major alco- to drugs or a medical condition. Prosopagnosia hol withdrawal. Haptic hallucination is another is the inability to remember faces despite tactile hallucination in which one feels as if he being able to recognize that they are faces. is being touched, for example, by a phantom. Astereognosis is the inability to recognize an Hypnagogic and hypnopompic hallucinations are object by touch despite the tactile sensations not tactile hallucinations but are hallucinations being intact. experienced in the transition state from wake- fulness to sleep and from sleep to 94. (E) Confabulation is the fluent fabrication of fic- respectively. Synesthesia is a secondary sensa- titious responses in compensation of memory tion following an actual perception (e.g., the disturbance. Clang associations are the use of sensation of a color associated with taste). words based on sound and not with reference to Synesthesia usually is secondary to neurologic the meaning. Flight of ideas is another form of disease or hallucinogen use, most notably LSD. speech in which one shifts rapidly from one idea to another but the relationship between the 91. (A) All of the choices are possible side effects, themes can sometimes be followed. Hypermnesia either short term or long term, of neuroleptics. is the ability to recall detailed material that is not Akathisia is the subjective sensation of motor usually available to recall. Logorrhea is uncon- and mental restlessness. Akinesia is a dysfunc- trollable or excessive talking sometimes seen in tion of slowed or absent movement that can be manic episodes. associated with pseudoparkinsonism in the set- ting of neuroleptics. A dystonia, like akathisia 95. (A) All of these terms are used to describe and akinesia, can occur acutely and involve speech. Prosody describes the melody, rhythm, muscle rigidity and spasticity. Tardive dyskinesia or intonation of speech that carries its emo- is a late-onset neurologic effect of neuroleptics tional quality. The lack of this type of emotional and can include tongue and lip movements and variation is called aprosodic speech. Stuttering is choreic movement of the trunk or limbs. Rabbit the disturbance of the fluidity of speech as in syndrome is another late-onset side effect that repeating sounds or syllables or using broken involves fine, rhythmic movements of the lips. words. Scanning speech is irregular pauses between syllables, which also breaks the fluid- 92. (B) Both depersonalization and derealization can ity but does not cause the repeating of sounds be seen in anxiety disorders. Derealization is the or syllables. Aphasia is the inability to commu- sense that one’s surroundings are strange or nicate by speech or language. Dysphagia is dif- unreal, and depersonalization is the feeling that ficulty in swallowing. one’s identity is lost or the feeling of being unreal or strange. Dereism is simply mental activity not 96. (C) Circumstantial speech is the overuse of in accordance with reality. Hypermnesia is an detailed information providing extraneous abnormal recall of details. Paresthesia is an abnor- detail in a digressive manner to convey an idea. mal sensation such as tingling or prickling. Tangential speech quickly moves off topic but can be followed and the patient never arrives at 93. (A) This patient may be suffering from the point that is trying to be made. Flight of Wernicke-Korsakoff syndrome, a complication ideas is like tangential speech but the changes 80 3: Adult Psychopathology

in ideas or topics are more frequent and more Prosopagnosia refers to the inability to recognize abrupt; like tangential speech, the connections faces despite perception of all the components. are still recognizable. In loosening of associa- tions, the logical connections between ideas are 101. (A) Neurovegetative signs are physiologic aspects completely lost; although proper grammar and of depression such as changes in sleep, bowel words may be used, the speech is not logical or habits, and weight. Conversion symptoms are goal directed. Derailment refers to the abrupt physical manifestations of underlying uncon- interruption of an idea and then resuming after scious motivation or drives. Catatonia is a state a period of time (a few seconds) to a new topic. of stupor, negativism, and rigidity. A neolo- This is usually without the patient’s being gism is a novel word that is used by a patient, aware of the switch in material. and apraxia is the inability to perform previ- ously learned motor skills. Word salad is an 97. (A) An illusion is a misperception of an actual incoherent mix of words and phrases. stimulus, and a hallucination is the perception in the absence of a stimulus. Micropsia and macrop- 102. (B) Sexual sadism and sexual masochism are, sia are the misperceptions of objects being respectively, the derivation of sexual pleasure smaller or larger than they really are. Palinopsia from causing or receiving mental or physical is the persistence of the visual image after the abuse. Exhibitionism is exposure of the genitalia stimulus has been removed. in public to an unwilling participant and usu- ally occurs in men. Transvestic fetishism is 98. (B) This patient suffers from OCD, an anxiety arousal by cross-dressing. Frotteurism, the rub- disorder characterized by recurrent obsessions bing of genitals against another to achieve (e.g., fear of contracting HIV) and/or compul- arousal and orgasm, is usually seen in men and sions (e.g., frequent hand washing), which is usually performed in crowded places. cause significant distress or impairment in functioning. The neurophysiologic nature of 103. (E) Paraphilias encompass a wide variety of this illness can be demonstrated through neu- maladaptive sexual behaviors and fantasies. roimaging, which shows increased activity Frotteurism is the syndrome of recurrent, (metabolism) in the caudate nucleus, frontal intense sexual fantasies and behaviors involv- lobes, and cingulum. These differences are ing touching or rubbing against a nonconsent- actually reversed after adequate pharmacologic ing adult. Exhibitionism involves revealing or behavioral therapy. one’s genitals to unsuspecting strangers. Pedophilia is sexual fantasies and behaviors 99. (B) Circumlocution is the substitution of a word involving children. Voyeurism involves secretly or description for a word that cannot be watching someone involved in disrobing, recalled or spoken. Confabulation is the fluent nudity, or sexual behavior. Fetishism is a sexual fabrication of fictitious responses in compen- attraction to an inanimate object. sation of memory disturbance. Clang associa- tion is the use of words based on sound and not 104. (A) Koro, zar, and taijin kyofusho are all exam- with reference to the meaning. A neologism is a ples of culture-bound delusions. Koro is the novel word that is used by a patient; apraxia is worry that the penis is shrinking into the the inability to perform previously learned abdomen and is found in South and East Asia. motor skills. Zar is the delusional possession by a spirit, and taijin kyofusho is the belief that one’s body is 100. (C) Anomia is an inability to name objects out- offensive to others. Kuru is a slowly progressive side of an aphasia. Alexia is the inability to read. neurologic disease leading to death, thought to Apraxia is the inability to perform previously be caused by an agent similar to Creutzfeldt- learned motor skills. Paralinguistic components of Jakob disease. Capgras syndrome is the delusional speech refer to nonverbal communications such idea that imposters have replaced once famil- as facial expression and body movements. iar persons. Answers: 97–114 81

105. (C) Voyeurism is deriving sexual pleasure from 109. (B) Prodromal schizophrenia is normally com- watching another person or persons involved posed of increasing negative symptoms in the in the act of undressing or other sexually ori- absence of positive symptoms. Negative symp- ented activity. Transvestic fetishism involves toms do not include affective lability or become being dressed in clothing of the opposite sex milder over time nor are they easy to tell apart for sexual excitement. It is often present in from depressive symptoms. They include autis- heterosexual men and differs from transsexu- tic features. alism in that the person is usually comfort- able and content with his gender identity. It is, 110. (A) This is a typical presentation for mania, an however, possible to see these disorders con- episode that seems to have been at least par- comitantly. Exhibitionism is another para- tially induced by treatment with an antide- philia and involves exposing one’s genitalia to pressant (all antidepressants can induce or an unsuspecting audience. Hermaphroditism is exacerbate mania). This patient’s effect, like not a paraphilia but is a rare disorder in which that of many patients with mania, is expansive one possesses both male and female sexual and irritable. He is to a lesser extent guarded characteristics. and suspicious but this does not capture the full picture of his mania. He is labile but hardly 106. (A) More than half a dozen well-controlled dysphoric, and he is euphoric but not bizarre. studies of schizophrenics have shown decreases He is not euthymic. in the size of the hippocampus, parahippocam- pal gyrus, and amygdala. The other regions 111. (B) Although all of the other choices might mentioned in the question have not been con- explain part of his speech characterization, pres- clusively shown to be altered in schizophrenia sured is used to describe speech that is difficult by MRI or CT scans. to interrupt and at a fast rate.

107. (C) Ideas of reference is the delusion that others 112. (A) The locus ceruleus is the “alarm” center of are focusing on you in some positive or nega- the brain and is hyperactive in anxiety states. It tive way commonly seen in paranoid schizo- is the center of most of the norepinephrine- phrenia. For example, a patient might believe containing neurons in the brain. The hip- that the television is speaking directly to him. pocampus, part of the limbic system, and the Formication is the sensation that bugs are amygdala are involved in memory formation. crawling on one’s skin. Trichotillomania is com- The thalamus is the brain’s relay center. The pulsive hair pulling accompanied by signifi- basal ganglia coordinate motor activity. cant hair loss. Reaction formation, a defense mechanism, involves an unacceptable impulse 113. (A) The presence of the classic triad of confu- being expressed as its opposite. Hallucinations sion, gait ataxia, and incontinence suggests are sensory phenomena that should not be con- strongly that this patient is suffering from fused with delusions. NPH. CT scan commonly reveals dilated ven- tricles thought to be the result of increased 108. (E) Loss of ego boundaries is commonly mani- pressure waves impinging within the ventric- fested as inappropriate conversational distance. ular system. Normal ventricles would not be Formication is the sensation that bugs are expected in NPH patients. Frontal sulcal widen- crawling on one’s skin. Flight of ideas is the ing is found in dementias that have underlying flow of nonconnected speech and thoughts; cir- cerebral atrophy. Hypointensities found in sub- cumstantiality is speech and thoughts with cortical areas are often indicative of lacunar identifiable connections between thoughts, but strokes. Cerebellar atrophy is seen most often in which drift off the point and ramble, eventually congenital disorders and alcoholism. coming back to the point. Ideas of reference is the delusion that others are focusing on you in 114. (D) Splitting is the view that people around some positive or negative way. you are either all good or all bad. It is common 82 3: Adult Psychopathology

in patients with borderline personality disorder in the CNS is synthesized in the dorsal and although there is no other evidence of that in medial raphe nucleus of the brain stem. The this case. Acting out generally represents the substantia nigra synthesizes dopamine and the enactment of a behavior coming from an locus ceruleus synthesizes norepinephrine. impulse that had presented conflict to relieve the sense that the conflict exists at all. 117. (C) Conversion disorder is described as the Externalization, a generalized form of projec- often sudden onset of a singular neurologic tion, represents the tendency to believe in the deficit, often in the setting of stress. As in this existence of patterns of behavior in others that case, there can be (often subconscious) guilt is really true about oneself. Regression is a linkage to the development of the symptom. return to patterns of relating, thinking, or feel- ing that had come before one’s current devel- 118. (A) Characteristic features of borderline per- opmental stage. For example, many medical sonality disorder, as demonstrated in this students who return home act as if they are patient, include self-mutilative behavior (which teenagers with regard to their parents or other can temporarily bring relief of emotional angst), hometown friends. Sublimation is the channel- emotional volatility, and instability of relation- ing of drives or conflicts into goals that even- ships with others, including tendencies to view tually become gratifying. For example, some others as either all good or all bad (also referred people remember being afraid of blood and to as splitting). Borderline personality disorder hospitals and worked out these fears in medical often arises in patients who have histories of school training. abandonment and/or abuse at a young age.

115. (A) Cataplexy is the sudden loss of muscle tone 119. (D) Antisocial personality disorder describes associated with stressful situations, a special individuals with long histories of disregard to stigmata of narcolepsy and points to the con- the rights of others and dishonesty used in nection of this disorder with REM sleep mech- attempts to gain something for themselves. anisms (which inhibit movement and muscle Often comorbid with substance abuse, this dis- tone during REM sleep). Catalepsy and catato- order is extremely common in prison inmates nia are used to describe bizarre movement and (30–50%). Schizotypal personality disorder is posture behaviors found in catatonic schizo- marked by a pattern of difficulty with close phrenia and some psychotic depressions. Sleep relationships as well as cognitive and percep- jerks and primary atonia are meaningless terms. tual disturbances. Schizoid personality disor- der differs from schizotypal personal disorder 116. (C) This patient’s reaction to paroxetine sug- by the absence of cognitive and perceptual gests that she had been on an MAOI. In disturbances. patients already using an MAOI, blocking reuptake of catecholamines and indolamines 120. (B) Somatization disorder describes the condition with SSRIs can result in a potentially life- suffered by individuals who have a long history threatening drug interaction known as sero- of multiple somatic complaints (gastrointesti- tonin syndrome. Medications that block such nal, sexual, and neurologic) that, despite reuptake include the SSRIs, TCAs, buspirone, exhaustive medical workups, have either no and other antidepressants such as venlafaxine. identifiable cause or are out of proportion to Features of mild serotonin syndrome include the medical findings. These patients often tachycardia, flushing, fever, hypertension, describe themselves as having been sickly all ocular oscillations, and myoclonic jerks. Severe their lives and, presumably in their frustration serotonin syndrome may result in severe with doctors’ inability to find reasons for their hyperthermia, coma, autonomic instability, problems, chronically doctor shop. convulsions, and death; therefore, wait at least 14 days after discontinuing an MAOI before 121. (H) The hallmark of hypochondriasis is the starting a serotonergic agent. Most serotonin belief by a patient that he or she has a specific Answers: 115–129 83

medical illness. Patients with the disorder often features suffer from delusions or other psy- interpret somatic complaints as signifying def- chotic symptoms that are mood congruent (e.g., inite serious illness. “my organs are rotting”).

122. (G) Factitious disorder, Munchausen’s type, 126. (A) Somatization disorder is characterized by describes the condition suffered by patients multiple physical complaints that cannot be who intentionally produce signs and/or symp- explained by any objective physical findings. toms of medical illness. Commonly, patients As in this case, patients’ symptoms frequently are former health care workers or have such date back many years and have prompted mul- workers in their families. tiple doctor visits, even hospitalizations. Conversion disorder is a loss or deficit of motor 123. (I) Agoraphobia describes the condition suffered or sensory function that is inconsistent with by individuals who fear open, crowded spaces, any known physiologic basis, for example, particularly ones from which escape may be paralysis of both hands. Conversion disorders difficult. It is often comorbid with panic disor- typically follow an identifiable stressor. der as suggested by this patient’s symptom of Hypochondriasis is a fear of serious disease; it palpitations. results from a hypervigilance and misinterpre- tation of normal bodily sensations. Malingering 124. (B) This patient demonstrates a typical is the confabulation of symptoms with the con- response to thiamine, namely, near complete scious goal of personal gain, for example, using resolution of his sixth nerve palsy, persistent a doctor’s diagnosis to get out of work. nystagmus, and some improvement in his gait. Factitious disorder is the purposeful produc- However, as his acute delirium and apathy tion of signs or symptoms for the express pur- clear, an underlying inability to learn and retain pose of becoming a patient (and assuming the new information emerges. This is known as sick role). Korsakoff psychosis; the amnestic state is best described as anterograde amnesia. Korsakoff 127. (C) The psychiatric disorder that carries the psychosis is also caused by thiamine deficiency greatest risk for suicide in both men and women but is usually hidden by the acute delirium of is a mood disorder. Although up to 15% of all Wernicke encephalopathy. Immediate memory alcohol-dependent persons commit suicide and and attention are normal; this is not true in up to 10% of schizophrenic patients commit sui- delirium. Some degree of alcoholic dementia cide, the age-adjusted suicide rates for patients and Alzheimer disease may be present as well with mood disorders is estimated at 400 per but the clinical setting most strongly points to 100,000 men and 180 per 100,000 women. Korsakoff syndrome. Multiple sclerosis and cancer are distractors.

125. (C) The encapsulated, fixed delusion seen here 128. (C) Suicidal ideation is a serious manifestation is typical of delusional disorder. The delusion of myriad disease processes. High-risk charac- in this illness is most often of the persecutory teristics of suicidal persons have been studied sort, as in this case, but may well be somatic, extensively and predictive variables exist to jealous, grandiose, and so on. Although this assist the physician when called to assess sui- delusion is plausible, albeit unlikely, the delu- cidal ideation. Age 45 or greater is the most sions encountered in schizophrenia and brief predictive factor. Alcohol dependence, rage or psychotic disorder tend to be more bizarre and violence, prior suicidal behavior, and male impossible, such as believing that others are gender follow in that order. controlling one’s thoughts. Paranoid person- ality disorder does not present as a fixed, iso- 129. (F) Although this set of matching questions lated delusion, rather, there is a more pervasive is not in United States Medical Licensing paranoia permeating many aspects of the Examination (USMLE) Step 2 format, it tests patient’s life. Patients with MDD with psychotic concepts that are important both for ward 84 3: Adult Psychopathology

discussions and for understanding fundamen- (B) refers to being easily brought to anger. tal ideas. Schizophrenia is the most common ill- Dyscalculia (C) is difficulty with simple arith- ness in hospitalized psychiatric patients in the metic. Verbigeration (D) is repetitive and mean- United States. ingless talking. Dysarthria (E) is difficulty with articulation. Gustation (F) is taste sensation. 130. (H) Folie à deux is a shared delusion aroused in Glossolalia (G) is the ability to speak a new lan- one person by the influence of another. guage suddenly. Hypochondriasis (H) is the mis- interpretation of bodily symptoms leading one 131. (G) Glucocorticoids (e.g., prednisone) are a to erroneously believe he or she has a physical common iatrogenic cause of reversible psychi- illness. atric symptoms such as depression and psy- chosis. 141–145. [141 (C), 142 (B), 143 (E), 144 (D), 145 (F)] Thought process disorders can be identified by 132. (D) Beta-amyloid plaque buildup is seen in the nature of the response. A circumstantial Alzheimer dementia. thought (A) finally reaches a point but after much unnecessary detail and digressions. It 133. (C) Like alcohol, benzodiazepines have poten- can be seen in disorders such as OCD, but can tially extremely dangerous withdrawal syn- also be normal in some people. A tangential dromes, which have essentially identical thought (B) differs from a circumstantial thought clinical features (due to their near-identical in that it never comes to a point, but like cir- actions on GABA modulation in the brain). The cumstantial thought it remains logically con- danger of withdrawal is even greater with the nected and can be followed. Flight of ideas short-acting benzodiazepines (short-acting (C) represents a change of direction every sen- drugs in general cause more acute withdrawal) tence or two, again with identifiable connec- and alprazolam is one of the very shortest. tions or associations tying them together. It is Despite the profound discomfort of withdrawal often a symptom of manic behavior and reflects from heroin and other opiates, opiate with- underlying racing thoughts. Loosening of asso- drawal is not life-threatening. Withdrawal from ciations (D) is characterized by the lack of log- PCP is not well characterized. ical connections between ideas, but sentence structure remains intact. Another component of 134. (E) The Hamilton Depression Scale is a com- thought disorders, clanging or clang associations monly used measure of depressive symptoms. (E), is word choice based on sound and not The Rorschach Test is a projective test using meaning or semantics. inkblots, which measures perceptual accuracy, reality testing, and integration of affective and In question 87, in the first sentence, the intellectual functioning. thoughts can be connected but do not reach a point. In question 88, the thoughts are more 135. (F) Dementia praecox was an early term for goal directed than in question 87 and this sen- schizophrenia coined by Kraepelin. tence is more consistent with tangentiality. In question 89, clanging is easily identified by 136. (J) Korsakoff dementia (alcohol-induced per- word choice based on similar sounds rather sisting amnestic disorder) commonly evinces than semantics. Associations between ideas are confabulation. lost in question 90. Question 91 is a represen- tation of thought blocking with derailment. 137–140. [137 (G), 138 (D), 139 (E), 140 (F)] Lability The patient stops in one thought, pauses, and (A) refers to rapid fluctuations in affect, often then resumes with an unrelated thought. between states of elation and sadness. Irritability CHAPTER 4 Somatic Treatment and Psychopharmacology Questions

DIRECTIONS (Questions 1 through 74): Each of (D) rapid-cycling bipolar disorder the numbered items in this section is followed by (E) opisthotonos answers. Select the ONE lettered answer that is BEST in each case. Questions 3 and 4

Questions 1 and 2 A 24-year-old woman presents to her primary care physician for a regular checkup. During the examina- A 32-year-old man is admitted to a general hospital tion, her doctor notes a 15-pound weight gain. The after ingesting an unknown quantity of phenelzine patient complains that she always feels tired during (Nardil) in a suicide attempt. After gastric lavage the day despite sleeping from 8 PM to 9 AM daily. On and administration of a charcoal slurry, he is trans- further questioning, the physician learns that the ferred to the medical intensive care unit for moni- patient feels sad and empty, often thinks about death, toring. Twenty-four hours later, he begins to see cannot concentrate at work, and lacks the energy to horses running in the halls and pulls out his intra- care for her two children. However, her mood picks up venous (IV) lines. when her physician questions her about her children.

1. The most important treatment at this time 3. The most effective agent would be which of should include which of the following? the following? (A) chlorpromazine (Thorazine) (A) fluoxetine (Prozac) (B) bretylium (Bretylol) (B) nortriptyline (Pamelor) (C) lorazepam (Ativan) (C) phenelzine (D) phenytoin (Dilantin) (D) sertraline (Zoloft) (E) meperidine (Demerol) (E) trazodone (Desyrel)

2. Twelve days after his suicide attempt, he 4. Her physician initiates treatment with citalo- receives venlafaxine (Effexor) to treat his depres- pram (Celexa). Which of the following is con- sion. One hour after ingestion of the venlafaxine, sidered an adequate trial of this agent? he becomes tachycardic and diaphoretic and (A) 2 weeks develops myoclonic jerks. This condition is known as which of the following? (B) 6 weeks (C) 3 months (A) neuroleptic malignant syndrome (NMS) (D) 6 months (B) akathisia (E) 8 months (C) serotonin syndrome

85

Copyright © 2007 by The McGraw-Hill Companies, Inc. Click here for terms of use. 86 4: Somatic Treatment and Psychopharmacology

Questions 5 and 6 following: She has never attempted suicide before and has never seen a psychiatrist. She was engaged While on call in a general hospital, you are called at to be married 2 years ago but her fiancé was killed in 4:30 AM to evaluate a patient who wants to be dis- an auto accident. She has difficulty falling asleep at charged. Upon arriving on the floor, the nurse tells night and wakes up often during the night. She can you he just pulled out his third IV of the day and no longer concentrate on her work. She does not started to swing the IV pole in the air while yelling enjoy painting as she once did. She has lost 10 profanities. He tells a nurse’s aide he must capture pounds over the past 2 months without dieting. She the tiger that is loose in the parking lot. You quickly denies hallucinations of any type. There is no signif- look at his chart and see that he is a 55-year-old man icant family history of psychiatric illness. admitted 36 hours earlier for abdominal pain, nausea, and vomiting. His transaminases were elevated on 7. You decide to prescribe a selective serotonin admission, but his hepatitis profile is still pending. reuptake inhibitors (SSRI). After explaining the His orders are for acetaminophen (Tylenol) and indications and side effects, she asks why it Maalox as needed, normal saline at 125 cc/h, and will take several weeks to see a significant nothing by mouth (NPO) for upper endoscopy in the improvement in her depression. What is the morning. most likely mechanism for the delayed thera- peutic response? 5. The most appropriate initial medication at this time is which of the following? (A) downregulation of postsynaptic

5-hydroxytryptamine-2 (5-HT2) binding (A) oxazepam (Serax) sites (B) chlordiazepoxide (Librium) (B) upregulation of postsynaptic 5-HT2 (C) haloperidol (Haldol) binding sites (D) chlorpromazine (C) increased serotonin in the synaptic cleft (E) disulfiram (Antabuse) (D) increased mitochondrial serotonin (E) downregulation of dopamine receptors 6. You speak briefly with this patient and are able to settle him down. His temperature is 102.1°F, 8. After 5 weeks of the treatment prescribed, the pulse is 130 beats/min, and blood pressure is patient no longer has suicidal thoughts and is 220/120 mm Hg. The most appropriate able to function well at work. She has begun to medication and route should be which of the date again and is considering resuming a sexual following? relationship with an old boyfriend. The most (A) labetalol (Normodyne) IV common side effect of sexual function of the med- (B) haloperidol intramuscularly (IM) ications prescribed is which of the following? (C) haloperidol IV (A) anorgasmia (D) lorazepam IM (B) dyspareunia (E) lorazepam IV (C) decreased desire (D) vaginismus Questions 7 and 8 (E) female analog of premature ejaculation A 32-year-old single woman presents to the emer- gency department escorted by her family after she Questions 9 and 10 revealed to them a plan to take a full bottle of aceta- A 25-year-old man presents to the emergency depart- minophen and drink a bottle of wine in a suicide ment accompanied by his family who report that the attempt. She claims that she cannot handle work any patient has not left his bedroom in 3 weeks. They longer because her boss is trying to have her fired and report that he spends his day reading science fiction her coworkers are helping him find fault in her work. novels that they provide for him. He was once pre- She states they have even tapped her phone and are scribed perphenazine (Trilafon), which they tell you having her followed. Your detailed history reveals the Questions: 5–12 87 seemed to help him, but he refuses to take it. On (D) behavioral therapy examination, you find him to be malodorous and (E) hypnosis disheveled. He is guarded when you speak with him. He does not make eye contact and he avoids looking Questions 11 and 12 at the television in the waiting room because, he says, he wants to prevent intruders from controlling his A 33-year-old mentally retarded male resident of a thoughts. After completing the examination, you group home presents to the emergency department admit him to the inpatient psychiatric unit. escorted by staff members who believe he has become confused and disoriented over the past few 9. Because the patient did not take the per- days. He has been a resident there for several years phenazine, you decide to try fluphenazine without incident, but last week his thioridazine was (Prolixin). After 5 days, he is no longer afraid to changed to haloperidol because of concerns about look at the television but he spends much of the the long-term use of high doses of thioridazine. On day pacing the floor. He claims he paces the examination, you find him to be disoriented to place floor because he does not like sitting in his and time; he is diaphoretic with a temperature of room. His family informs you that he often 105.8°F, a heart rate of 130 beats/min, and a respira- walked the streets of his neighborhood when tory rate of 20/min. His extremities are stiff. Routine he was taking perphenazine. After reducing laboratories reveal a white blood cell (WBC) count of the fluphenazine to the lowest dose you feel he 15,000/mL; blood urea nitrogen (BUN) 75 mg/dl, needs, he still paces the halls. Next, you do creatinine 1.2 mg/dl, and creatinine phosphokinase µ which of the following? 2300 I /L.

(A) discontinue fluphenazine and try another 11. The most common serious complication this agent patient may experience is which of the following? (B) add propranolol (A) respiratory failure (C) add lorazepam (B) pulmonary embolism (D) add benztropine (Cogentin) (C) rhabdomyolysis (E) add diphenhydramine (Benadryl) (D) myocardial infarction (MI) 10. A 25-year-old White woman comes to your (E) diffuse intravascular coagulation office complaining that whenever she takes the elevator in her boyfriend’s building, she devel- 12. The long-term consequence of high-dose thior- ops heart palpitations and breaks out into a idazine use that probably caused his physician cold sweat. The thought of being in the subway to switch to haloperidol is which of the fol- terrifies her. You tell her that you are planning lowing? to cure her by exposing her to those things that (A) tardive dyskinesia (TD) scare her most. Flooding is an example of what (B) priapism form of psychotherapy? (C) retinal pigmentation (A) family therapy (D) hyperprolactinemia (B) couples therapy (E) agranulocytosis (C) psychodynamic psychotherapy 88 4: Somatic Treatment and Psychopharmacology

Questions 13 and 14 15. The minimum initial laboratory data that should be obtained before starting treatment A 45-year-old unemployed investment banker pre- are which of the following? sents to your office accompanied by his wife. He has been out of work for 1 month because he is afraid to (A) serum creatinine, BUN and electrolytes, take the train into work. He never had difficulty trav- thyroid studies, urinalysis eling until 2 months ago, when on a train during a (B) liver function tests, serum creatinine, family vacation, he suffered chest pain, shortness of BUN and electrolytes breath, nausea, and a fear of dying. These symptoms (C) complete blood count (CBC) with lasted about 15 minutes. A workup by a cardiologist differential was negative. However, these symptoms, also accom- (D) prothrombin and partial thromboplastin panied by intense fear and a feeling of being detached times from himself, have continued. He has had about two (E) serum ammonia attacks a week and also has fear just thinking about returning to work because he fears he will die on the 16. You decide to start lithium. After starting the train. He tells you that he could not have come to see treatment, the patient sprains his ankle while you without his wife’s help. exercising. His primary care doctor told him to take over-the-counter ibuprofen, which he 13. A reasonable first choice of pharmacotherapy started 2 days ago. His ankle still hurts, and he for this patient would be which of the following? plans to continue taking the medication. You (A) bupropion (Wellbutrin) advise him to which of the following? (B) trazodone (A) Increase the dose of ibuprofen. (C) propranolol (B) Decrease the dose of ibuprofen. (D) buspirone (BuSpar) (C) Continue ibuprofen at the prescribed (E) fluoxetine dose. (D) Switch to aspirin. 14. The most potent antianxiety agent is which of (E) Stop the medication you prescribed. the following? (A) alprazolam (Xanax) Questions 17 and 18 (B) buspirone A 9-year-old boy is brought to the physician because (C) clonazepam (Klonopin) his parents have received a note from his second- (D) chlordiazepoxide grade teacher complaining that he is disruptive in (E) diazepam (Valium) class. His teacher believes he could achieve better grades if he could sit still and pay attention. His par- Questions 15 and 16 ents report he has always been an active child but does have difficulty getting along with other chil- A 22-year-old single male college music student dren in the neighborhood. In the examination room, reluctantly presents to your office on the request of he cannot sit still and has difficulty completing a task his parents. His parents are worried because he has you have assigned. You decide to prescribe him not been eating, talks constantly, and never seems to methylphenidate (Ritalin). sleep. He says he has never felt better, he’s writing several songs a day, and needs only a few hours of 17. The most common side effect of this treatment sleep. He believes he will be the next John Lennon is which of the following? and wants to get back to writing but he cannot stay focused long enough to answer any of your ques- (A) difficulty falling asleep tions. He does not have any psychiatric history and (B) daytime drowsiness denies using any substances. Questions: 13–23 89

(C) slowed growth Generally, he has done well and has not required major (D) increase in appetite medication changes or hospitalizations. About 2 years (E) decrease in systolic blood pressure ago, you noticed some lip smacking and tongue pro- trusions that did not bother him; however, now he 18. Treatment with this medication has been also has odd, irregular movements of his arms that reported to exacerbate which of the following? make it difficult for him to eat.

(A) bedwetting 21. The syndrome that best describes these symp- (B) tics toms is which of the following? (C) poor impulse control (A) Meige syndrome (D) reading difficulties (B) anticholinergic toxicity (E) myopia (C) TD (D) Huntington disease Questions 19 and 20 (E) Sydenham chorea A 9-year-old boy is brought to your office by his mother because of his “compulsions,” which have 22. The only medication that may improve these worsened over the past year. She first noticed them symptoms is which of the following? last year when several times a day he would repeat- (A) benztropine edly blink his eyes and frown while clearing his throat. He continues to do this despite trying not to, (B) trihexyphenidyl (Artane) and in addition, he often sticks out his tongue and (C) olanzapine smells his shirt while speaking with classmates. (D) levodopa (Dopar) These “compulsions” have caused him to lose friends (E) clozapine (Clozaril) at school. 23. A 24-year-old married secretary has com- 19. Pharmacologic blockade of which of the fol- plained of dizziness, palpitations, and sweaty lowing receptors provides the greatest relief of palms for the past 10 months. Sometimes, she these symptoms? also has extreme muscle tension in her neck (A) norepinephrine that occurs both at work and at home. She has difficulty falling asleep and feels “edgy” most (B) serotonin of the day. She also has difficulty concentrating (C) dopamine at work. She has gone to a local emergency (D) alpha2 department three times because of the palpita- (E) gamma-aminobutyric acid (GABA) tions but nothing was found. Her family prac- titioner placed her on a hypoglycemic diet and 20. Which of the following agents is most effec- referred her to a neurologist who found no tive to treat this disorder? abnormalities. She tells you she has many wor- ries about her family but does not feel partic- (A) amitriptyline (Elavil) ularly depressed. Which of the following will (B) haloperidol provide the most immediate relief of her (C) clonidine (Catapres) symptoms? (D) alprazolam (A) alprazolam (E) paroxetine (Paxil) (B) fluoxetine Questions 21 and 22 (C) bupropion (D) buspirone You have been treating a 55-year-old man with schiz- (E) imipramine (Tofranil) ophrenia for 20 years with haloperidol and benztropine. 90 4: Somatic Treatment and Psychopharmacology

24. A 22-year-old woman is admitted to a psychi- on her figure, she continued to lose another atric unit after a serious suicide attempt. She 10 lb. Her eating habits are ritualized: She cuts has had many suicide attempts in the past with her food into small pieces and moves it around varying severity. Her arms are scarred by prior on her plate. She has an intense fear of being attempts at hurting herself. She had been a overweight. Which of the following statements good student until high school, when she took most accurately reflects pharmacotherapy for up with a “fast” crowd, began abusing alcohol this type of patient? and marijuana, and ran away from home sev- (A) Successful pharmacotherapy should eral times. She has had several intense, stormy include appetite stimulants such as relationships with men. Outpatient treatment cyproheptadine (Periactin). has mostly consisted of her complaints to her therapist about her family. She usually calls her (B) Pharmacotherapy generally has a therapist daily about different crises; however, limited role in the treatment of these her therapist was on vacation during her most patients. recent crisis. Which of the following classes of (C) High-dose chlorpromazine is useful in agents have been shown to be helpful for this treating these patients. patient? (D) Amitriptyline is useful in treating these patients. (A) tricyclic antidepressants (TCAs) and antipsychotics (E) Promotility agents such as cisapride (Propulsid) are useful in treating (B) SSRIs and antipsychotics patients who purge. (C) antipsychotics and benzodiazepines (D) anticonvulsants and stimulants 27. A 36-year-old man whom you have treated for (E) stimulants and SSRIs 8 months with paroxetine for his first episode of major depression has done very well. Because 25. A 36-year-old man is referred to you by his of the sexual function side effects, he decided to internist for evaluation of his eating disorder. stop taking it 3 days ago without first informing He stands 6 feet 2 inches tall and had weighed you. Two days after he stopped it, he became 270 lb until he lost 90 lb last year because his irritable and even had a crying spell. He had wife thought he was “fat.” Since losing weight, the flu 2 weeks ago but the body aches, chills, he is obsessed with becoming fat. He hates feel- and general lethargy have returned. Which of ing full after a meal. Sometimes, he binges on the following should you consider? junk food but always induces vomiting after- (A) Start bupropion because it lacks sexual ward. Now, he can no longer enjoy food and side effects. cannot stop dieting. Which of the following medications has shown success in treating such (B) Treat his flu symptomatically. patients? (C) Consider a mood stabilizer for his irritability. (A) fluoxetine (D) Restart the paroxetine and taper it off. (B) mirtazapine (Remeron) (E) Prescribe phenytoin for seizure (C) phenytoin prophylaxis. (D) protriptyline (Vivactil) (E) quetiapine (Seroquel) 28. A 30-year-old man presents to a general hospi- tal after ingesting 20 tablets of his prescribed 26. A 19-year-old woman is admitted to an inpa- carbamazepine (Tegretol) in a suicide attempt. tient psychiatric facility for her eating disor- He has no prior psychiatric history, but has a der. She weighs 82 lb and stands 5 feet 6 inches history of seizures. Your detailed history reveals tall. She began dieting in high school to lose an several months of depressed mood, weight unwanted 5 lb. Encouraged by compliments loss, anhedonia, sleeplessness, and difficulty Questions: 24–32 91

concentrating. Four days after admission, his (C) anorgasmia carbamazepine level has returned to normal (D) impotence and has been restarted to treat his seizure dis- (E) gynecomastia order. Which of the following medications would be most appropriate for this patient? Questions 31 and 32 (A) fluoxetine A 57-year-old man, whom you have treated for (B) nefazodone (Serzone) 20 years for bipolar disorder, is admitted to the gen- (C) paroxetine eral hospital with chest pain. (D) clomipramine (Anafranil) (E) amitriptyline 31. The hospital physician asks whether the lithium he is receiving can affect his electrocardiogram Questions 29 and 30 (ECG). You inform her that lithium has which of the following characteristics? A 65-year-old man presents to your psychiatric office on a referral from his primary care physician for eval- (A) has no effect on the ECG uation of depression. He feels depressed and hope- (B) can prolong the PR interval less and has lost his appetite. He no longer enjoys (C) can invert the T waves seeing his grandchildren. It takes him several hours (D) can prolong the QT interval to fall asleep at night and he stays asleep only about (E) can shorten the QT interval 3 hours. He occasionally does not buy food because he is concerned about going broke; however, his 32. The patient was found to have suffered from an family informs you that he is financially well off. He MI and underwent angioplasty. He was dis- stands 6 feet tall and weighs 155 lb, much less than charged from the hospital on propranolol, his usual 175 lb. Additionally, he feels guilty about losartan, topical nitroglycerin, hydrochloro- becoming a burden on his family. thiazide, and one baby aspirin per day, in addi- tion to his lithium. Prior to his MI, his lithium 29. An antidepressant medication that might be level was always 0.8 mmol/L on 900 mg/day useful to help him fall asleep at night is which of lithium carbonate. Now it is 1.3 mmol/L on of the following? the same dose. The most likely cause for this (A) diazepam change is which of the following? (B) diphenhydramine (A) decreased volume of distribution (C) trazodone post-MI (D) fluoxetine (B) aspirin (E) bupropion (C) hydrochlorothiazide (D) captopril 30. A rare but troubling sexual side effect of this (E) propranolol medication is which of the following? (A) priapism (B) premature ejaculation 92 4: Somatic Treatment and Psychopharmacology

Questions 33 and 34 20 mg/day for 1 month. Despite a reduction in the voices, he remains isolated from family and friends. A 47-year-old male novelist feared contamination A trial of quetiapine also fails to decrease his isolation. from anything he believed was dirty. In restaurants, he would use his own plastic utensils. Elsewhere, he 35. Which of the following agents should be tried would wear gloves or use paper towels to avoid next? touching “dirty objects.” Upon returning home, he would wash his hands five times at night before (A) fluphenazine going about his activities. If he accidentally touched (B) risperidone anything prior to washing his hands, he would expe- (C) ziprasidone (Geodon) rience a vague uncomfortable feeling and would be (D) clozapine unable to perform his usual nightly activities. As a (E) perphenazine result of these symptoms he was unable to have a meaningful social life. 36. One of the leading theories for the efficacy of the above agent involves its action at which of 33. The most effective agent for this syndrome is the following receptors? which of the following? (A) D (A) clomipramine 2 (B) D (B) mirtazapine 3 (C) D (C) clonazepam 4 (D) 5-hydroxytryptamine-1 (5-HT ) (D) phenelzine 1 (E) 5-hydroxytryptamine-3 (5-HT ) (E) olanzapine 3 37. A 65-year-old man with major depression and 34. You started treating this patient with fluoxetine hypertension presents to the emergency depart- at 60 mg/day 4 months ago. He no longer ment because he fell to the ground after arising needs to wash his hands quite so often but still from a chair. He has no significant cardiac history has an uncomfortable feeling when he touches and his ECG, electrolytes, and neurologic exam- “dirty things.” Which of the following state- ination are unremarkable. His family reports that ments is most accurate regarding treatment at his physician recently started a new antidepres- this point? sant but they do not know its name. Which of the (A) Buspirone is not effective as an following antidepressants is generally associated augmenting agent. with this scenario? (B) Fenfluramine (Pendormin) has (A) bupropion documented efficacy as an augmenting (B) mirtazapine agent. (C) nefazodone (C) Lithium has documented efficacy as an augmenting agent. (D) imipramine (D) Cingulotomy should be considered next. (E) nortriptyline (E) Clozapine has been effective. 38. A 35-year-old man with a history of seizures and hypertension has been treated for major Questions 35 and 36 depression with sertraline at 200 mg/day for A 21-year-old college student has been hearing voices 2 months with no improvement in symptoms. telling him to hurt himself for more than a year. Trials He is not acutely suicidal but he is frustrated by with the following agents have reduced the voices: his lack of improvement. His primary care chlorpromazine 1800 mg/day for 1 month, then doctor has referred him to you for further eval- haloperidol 60 mg/day for 6 weeks, then olanzapine uation. He has not had a seizure for years and Questions: 33–42 93

is not taking any anticonvulsant medications. (C) lorazepam At this time you should consider which of the (D) orientation to her surroundings following? (E) risperidone (A) adding lithium to his regimen (B) adding thyroid hormone Questions 41 and 42 (C) switching to imipramine A 67-year-old woman presents to your office for eval- (D) switching to bupropion uation of a 20-lb weight loss over a 4-month period and (E) switching to venlafaxine insomnia for the past 4 weeks. Her daughter informs you that her mother no longer enjoys any of her hob- Questions 39 and 40 bies and has been speaking lately about dying. They have already consulted an internist who performed A 28-year-old woman whom you have treated for an extensive medical workup, including chemistries, bipolar disorder informs you that she is 3 weeks blood count, thyroid studies, gastrointestinal imaging, pregnant and is anxious about the baby’s health. You and endoscopy all of which were normal. are treating her with lithium and she has not had a manic episode in 3 years. 41. You would like to start an antidepressant med- ication but would like to minimize adverse side 39. Which of the following statements is correct effects while exploiting other side effects. regarding bipolar disorder and pregnancy? Which of the following agents has a side effect (A) Lithium is safe after the first trimester and profile that may be beneficial? can be safely continued through birth. (A) nefazodone (B) Divalproex sodium (Depakote) is a safe (B) fluoxetine alternative to lithium. (C) nortriptyline (C) Carbamazepine is a safe alternative to (D) mirtazapine lithium. (E) bupropion (D) Lithium substantially increases the risk of Ebstein anomaly of the tricuspid 42. Upon further history, you learn that antide- valves by 75%. pressants have failed in several previous trials. (E) Clonazepam does not carry an increased You would like to start a TCA. Which of the fol- risk of major fetal malformations. lowing would preclude using such an agent?

40. You are called to consult on an 85-year-old (A) lacunar infarcts patient who has become combative, yelling, (B) treatment with the antihypertensive punching staff, and pulling out her IVs. She amlodipine demands to leave but is too weak to get out of (C) left bundle branch block on ECG her hospital bed. Which of the following would (D) urinary retention be the most appropriate intervention at this time? (E) estrogen replacement therapy (A) diphenhydramine (B) donepezil 94 4: Somatic Treatment and Psychopharmacology

Questions 43 and 44 46. Further history reveals that he does not have a mood disorder. He tells you that often he A 72-year-old man is admitted to a general hospital’s simply cannot control his craving to have intensive care unit because of altered mental status. another drink. Which of the following agents His medical workup has revealed pneumonia and con- has shown some success in decreasing crav- gestive heart failure (CHF). On the second hospital ings for alcohol? day, he is agitated and pulls out his IV access. He also has been noted to speak out loud with no one in the (A) fluoxetine room. His level of consciousness seems to wax and (B) disulfiram wane. He does not have a psychiatric history and is not (C) naltrexone (ReVia) allergic to any medications. Besides his CHF and pneu- (D) bupropion monia, he does not have other comorbid conditions. (E) diazepam 43. Which of the following agents would be the most Questions 47 and 48 appropriate to administer for his agitation? (A) thioridazine A 36-year-old woman is referred to you by her pri- mary care physician for management of anxiety and (B) chlorpromazine fear that has persisted for several months. She reports (C) lorazepam that she was raped and held hostage on a boat for (D) olanzapine several days by two armed men. During this, she (E) haloperidol experienced intense fear for her life. Since then, she has had intense stress whenever she is near the water 44. Which of the following agents would be least and has frequent nightmares. She cannot recall details likely to cause orthostatic hypotension? of the ordeal but tries to avoid driving her car within sight of the ocean, which has been difficult because (A) haloperidol of the location of her home. She feels detached from (B) perphenazine her husband and family and has abandoned plans to (C) thioridazine pursue her career as a painter. She has difficulty (D) risperidone falling asleep and is easily startled by phone calls. She (E) quetiapine no longer goes to public places alone.

Questions 45 and 46 47. Which of the following agents is believed to help relieve the numbing symptoms she is A 32-year-old man presents to your clinic after losing experiencing? his job because he was intoxicated while working. He has been drinking daily since he was 16 years old. He (A) alprazolam was able to complete college and went to work full- (B) fluoxetine time right after graduation, but has lost several jobs (C) carbamazepine since. After the loss of his last job, his wife threatened (D) thioridazine to leave him if he does not get help. (E) naltrexone

45. Which of the following agents can be used to 48. Whenever she drives near the water, which is sensitize him to the effects of alcohol? unavoidable, she experiences intense anxiety, (A) metronidazole (Flagyl) fear, and palpitations, and feels that she is (B) fluoxetine reexperiencing her abduction. Which of the fol- lowing agents may help reduce these symptoms? (C) flumazenil (Romazicon) (D) disulfiram (A) olanzapine (E) naloxone (Narcan) (B) naltrexone Questions: 43–52 95

(C) perphenazine also noted that he speaks much less than he once did (D) divalproex sodium over dinner. He does not have any medical illnesses. (E) clonidine 50. Which of the following agents may play a role 49. You have been asked to evaluate an 80-year-old in improving his memory? male nursing home resident because of a change (A) fluoxetine in mental status. The nursing staff reports that he (B) trazodone has been a patient there for about a year without (C) aspirin any problems until about 1 month ago when he was having difficulty sleeping. The staff physi- (D) donepezil (Aricept) cian prescribed lorazepam 2 mg at bedtime; (E) pemoline (Cylert) however, he was still unable to sleep. He started wandering during the day and has been 51. On further questioning, his wife reports that he restricted to his room. Because he remains agi- does not sleep well at night and wanders tated during the day, they have been giving him around the house. He then sleeps for a large lorazepam 2 mg every 6 hours for agitation. His part of the day. Which of the following would other medications include hydrochlorothiazide be useful to control this behavior? 25 mg/day, digoxin 0.125 mg every other day, (A) trazodone diltiazem (Cardizem) 240 mg sustained release (B) haloperidol daily, and transdermal nitroglycerin 0.4 mg/h. His electrolytes are normal except for a potas- (C) lorazepam sium of 3.4 mmol/L. A recent digoxin level is (D) triazolam (Halcion) not available. The first intervention you should (E) chlorpromazine consider is which of the following? Questions 52 and 53 (A) stopping the room seclusion (B) replacing the potassium A 25-year-old man is brought into the emergency (C) tapering the lorazepam department lethargic and stuporous. He responds only to painful stimuli, wakes up briefly and yells, (D) checking the hydrochlorothiazide level then goes back to sleep. Ambulance personnel report (E) monitoring orthostatic vitals that they found him near a house known for drug traf- ficking. There is no evidence of physical injury. Questions 50 and 51

A 64-year-old man presents to your office, accompa- 52. Which of the following medications should he nied by his wife, complaining of memory loss. He is receive first? a retired stockbroker who can no longer recall stock (A) dextrose and flumazenil quotes like he did several years ago. His wife has (B) dextrose, flumazenil, and naloxone become both concerned and annoyed because he seems to immediately forget whatever she has told (C) dextrose, flumazenil, naloxone, and him. These memory problems have slowly pro- thiamine gressed over several years and recently he has had (D) dextrose and naloxone difficulty getting dressed. More than once, he has (E) dextrose, naloxone, and thiamine put his underwear on over his pants. His wife has 96 4: Somatic Treatment and Psychopharmacology

53. The patient is treated and a urine toxicology (C) amitriptyline screen is positive for the presence of opioids. (D) bupropion When he is more alert, he informs you that he (E) nefazodone has been using IV heroin daily for several weeks. His last use was about 8 hours ago. His 56. A 47-year-old woman presents to your office heroin use escalated several weeks ago from for psychiatric care because she has just relo- snorting to IV use when he no longer felt a cated to your town. She has a history of bipo- good “high” from snorting. He has never lar disorder, but her prior psychiatrist recently detoxified before. He has no other comorbid stopped her lithium because she developed medical conditions and he does not abuse alco- thyroid autoantibodies. She complains of being hol. To detoxify him, many clinicians might irritable and not able to sleep for several weeks. use which of the following? She tells you that her thoughts are racing and (A) clonidine you have a difficult time redirecting her answers. (B) propranolol However, she believes her biggest problem is her facial pain, which was diagnosed as trigem- (C) methadone inal neuralgia and has not improved with anal- (D) naloxone gesics. A medication that may potentially help (E) chlordiazepoxide both conditions is which of the following?

54. A 45-year-old woman presents to your practice (A) lithium with depression. Her mother recently passed (B) amitriptyline away from lung cancer and her father died (C) fluoxetine 4 years ago from lung cancer. She has seen ther- (D) gabapentin (Neurontin) apists to help with her grief but she continues to (E) thyroxine have symptoms of major depression 6 months after her mother’s death. Despite several 57. A 33-year-old woman with a history of schizo- attempts to quit smoking, she still smokes two phrenia has recently started treatment with packs of cigarettes a day. Which of the follow- olanzapine. She has tolerated the medication ing should be considered as pharmacotherapy well and is living in a group home, anticipating for her depression and smoking? moving into her own apartment. She no longer (A) imipramine hears voices and no longer has a desire to hurt (B) bupropion herself. Which of the following side effects of olanzapine is most likely to interfere with her (C) buspirone continued use of the drug? (D) trazodone (E) phenelzine (A) increased prolactin (B) increased excitability 55. A 52-year-old man who has poorly controlled (C) weight gain diabetes is referred to you for evaluation of (D) agranulocytosis medication noncompliance. You learn that he (E) development of cataracts cannot fall asleep at night and has no energy during the day. His appetite is gone and he 58. A 42-year-old male with schizophrenia is being does not enjoy activities as he once did. Overall treated for an acute exacerbation of his audi- he feels depressed. Additionally, he has been tory hallucinations. As is his pattern, he experiencing chronic pain in his feet, which is responds very favorably to medications while not relieved by analgesics. Which of the fol- hospitalized, but when he is discharged for- lowing may help his mood and his pain? gets to take them regularly. This results in a (A) alprazolam cycle of subsequent worsening of his psychosis (B) citalopram and rehospitalization. He is currently taking Questions: 53–64 97

oral risperidone and tolerating it well with responded well to nortriptyline and risperi- good efficacy. Which of the following would be done. Which of the following is a side effect of the most appropriate step in his treatment? risperidone that could possibly limit its use? (A) begin oral haloperidol (A) weight gain (B) begin Risperdal Consta (B) galactorrhea (C) continue the current regimen (C) sedation (D) cross-taper with clozaril (D) akathisia (E) cross-taper with olanzapine (E) parkinsonism

Questions 59 and 60 Questions 62 and 63

A 76-year-old man who has felt depressed for several A 58-year-old man arrives in your office complaining months presents to your office for evaluation. He has of impotence. He suffers from hypertension but does had weight loss, decreased appetite, and no interest not have cardiac disease, renal disease, glaucoma, or in his hobbies, and cannot concentrate on your cog- a history of peptic or duodenal ulcers. nitive examination. He has passing thoughts of dying. He admits that he frequently hears voices 62. Which of the following drugs is best for the telling him that he is going to die soon. You have treatment of impotence? decided to start an antidepressant and risperidone at (A) paroxetine 0.5 mg bid. (B) citalopram 59. Which of the following side effects of risperi- (C) molindone (Moban) done should be monitored for? (D) diphenhydramine (A) agranulocytosis (E) yohimbine (B) leukocytosis 63. If the patient is taking clonidine for hyperten- (C) anticholinergic side effects sion, which of the following drugs would (D) orthostatic hypotension directly inhibit clonidine’s mechanism of action? (E) weight loss (A) paroxetine 60. As his dose of risperidone increases, the likeli- (B) citalopram hood of which of the following can also be (C) molindone expected to increase? (D) diphenhydramine (A) extrapyramidal side effects (E) yohimbine (B) leukocytosis 64. A 45-year-old woman who travels often to (C) anticholinergic effects Europe from the United States presents to your (D) agranulocytosis office complaining of not being able to fall (E) weight loss asleep at night after she arrives in Germany. Which of the following is the first line of treat- 61. A 26-year-old woman who gave birth to her ment for insomnia? first child 2 months ago presents to your office (A) diphenhydramine on a referral for evaluation of depression. She is not breast-feeding. She has a history of (B) trazodone depression from before her pregnancy and her (C) clozapine depression is usually accompanied by voices (D) benzodiazepines that tell her to harm herself. In the past, she has (E) amitriptyline 98 4: Somatic Treatment and Psychopharmacology

Questions 65 and 66 68. A 24-year-old man is hospitalized because he has been hearing voices for 6 months telling A 60-year-old man presents to your office complain- him to kill himself and fears he may act on ing of the sudden onset of palpitations, sweating, them. He appears unkempt and lost his job shaking, shortness of breath, choking, and feeling 2 months ago. You decide to prescribe haloperi- that he is “going crazy.” He has been seen by his car- dol for this patient’s psychotic symptoms, diologist who has ruled out a cardiac etiology. The which resolve enough for him to be discharged. patient fears future attacks. On a follow-up visit to your office, he reports that his hallucinations have improved but that 65. Which of the following is the safest drug with he “can’t sit still” and feels like he needs to be which to treat panic disorder? constantly in motion. The patient intermittently (A) fluvoxamine (Luvox) stands up and walks around your office as you (B) imipramine interview him. This patient most likely has which of the following? (C) phenelzine (D) bupropion (A) acute dystonic reaction (E) selegiline (Eldepryl) (B) akathisia (C) NMS 66. If the patient’s symptoms were due to a cardiac (D) mania etiology and he subsequently became depressed, (E) obsessive-compulsive disorder (OCD) which antidepressant would be the most dan- gerous? 69. You are seeing a 19-year-old male college stu- (A) amitriptyline dent whose roommates say has been “creeping (B) sertraline them out.” He was always withdrawn, but has built himself a wall around his bed and fre- (C) amphetamines quently yells at his roommates for trying to (D) bupropion poison him. After taking a complete history, (E) venlafaxine you diagnose schizophrenia. What is the most important consideration in choosing antipsy- 67. A 33-year-old woman with a diagnosis of chotic medications? chronic paranoid schizophrenia has been main- tained on haloperidol since she was diagnosed (A) renal function at age 24. In your office, she says she has not (B) the presence of negative symptoms had her period in several months, has dimin- (C) gender ished sex drive, has been unable to conceive, (D) onset of action has been discharging milk from her breasts, (E) side effect profile and has suffered pain on sexual intercourse. Assuming that these complaints are caused by 70. You have been treating a 33-year-old woman her antipsychotic medication, which of the fol- with bipolar disorder for 3 years. In prepara- lowing is most likely to explain her symptoms? tion for a planned pregnancy, and because she (A) increased adrenocorticotropic hormone has never demonstrated behavior that would (ACTH) levels put her or others in danger, you have tapered (B) decreased prolactin levels off her medications but see her frequently in case of a manic episode. Two months into her (C) increased prolactin levels pregnancy, she presents to your office with (D) increased serotonin levels pressured speech and the belief that her baby is (E) increased dopamine levels Questions: 65–75 99

Jesus Christ. Her husband says he is concerned (A) benzodiazepines because she has stopped taking her prenatal (B) barbiturates vitamins and is acting bizarrely. Which of the (C) clonidine following is the safest intervention to consider (D) phenytoin to treat her symptoms? (E) disulfiram (A) lithium carbonate (B) divalproex sodium 74. You are seeing a 56-year-old man with a history (C) chlorpromazine of hepatitis C and alcoholism. He tells you that (D) electroconvulsive therapy (ECT) would like to quit drinking and wants to detox first. In a patient with alcoholism in whom you (E) diazepam suspect impaired liver function, which of the following is the most appropriate drug to treat 71. You are the consult-liaison psychiatrist on call withdrawal symptoms? for the trauma surgery team. You are asked for to see a man who was hit by a car and severely (A) phenobarbital (Luminal) injured. He is unable to give a history but an (B) chlordiazepoxide acquaintance who accompanied him to the hos- (C) lorazepam pital reports that he is a drug user. He has track (D) alprazolam marks on his arms. Which of the following classes of drugs do you tell the surgeons is the (E) clonidine most dangerous when abruptly withdrawn? DIRECTIONS (Questions 75 through 84): Each (A) opiates group of items in this section consists of lettered (B) sedative-hypnotics headings followed by a set of numbered words or (C) stimulants phrases. For each numbered word or phrase, select (D) psychedelics the ONE lettered heading that is most closely associ- ated with it. Each lettered heading may be selected (E) cannabinoids once, more than once, or not at all. Questions 72 and 73 Questions 75 through 79 A 44-year-old man with a history of alcoholism is brought to the emergency room. You get a report from For each vignette, pick the medication most likely to a family member of the patient that he “has been living be associated with the side effect. off booze and drugs and nothing else for weeks.” The (A) lithium emergency department nurse is about to go in to draw (B) divalproex sodium blood from the patient and offer him food. (C) paroxetine (D) haloperidol 72. What is the most important thing to do next for this patient? (E) gabapentin (F) pimozide (Orap) (A) Be sure liver function tests are included (G) ziprasidone (Geodon) in the blood draw. (H) perphenazine (B) Listen for a murmur. (I) clozapine (C) Administer lorazepam prophylactically for probable agitation. (J) carbamazepine (D) Administer thiamine. 75. A 14-year-old boy being treated for Tourette (E) Get a vitamin B level. 12 disorder is started on citalopram and is seen to have an increase of 11 msec in his QTc values 73. How will this man’s underlying condition be on ECG. best treated in the acute medical setting? 100 4: Somatic Treatment and Psychopharmacology

76. A 51-year-old woman being treated for schizo- DIRECTIONS (Questions 85 through 89): Each of phrenia develops diabetes. the numbered items in this section is followed by answer choices. Select the ONE lettered answer 77. An 11-year-old girl being treated for major that is BEST in each case. depressive disorder reports frequent suicidal ideation. Questions 85 and 86

78. A 34-year-old woman being treated for bipolar You are treating a 28-year-old single woman for a I disorder develops acute pain in her abdomen first episode of major depression with sertraline. Two and is diagnosed with pancreatitis months after starting treatment with you, the patient reports that she feels “back to normal” and asks when 79. The WBC count of a 27-year-old man being she can stop the medication you initially prescribed. treated for schizophrenia drops to 3000/mL. 85. You tell her that she can safely stop the Questions 80 through 84 medication (A) fluoxetine (A) now (B) alprazolam (B) in 2 months (C) trazodone (C) in 4 months (D) paroxetine (D) after 10 months (E) risperidone (E) after 5 years (F) haloperidol 86. She returns 3 months after starting treatment (G) olanzapine and reports a poor sexual relationship with her (H) lithium boyfriend. Which of the following antidepres- (I) citalopram sants could be considered in place of your orig- (J) divalproex sodium inal therapy if the sexual dysfunction becomes (K) sertraline intolerable? (A) bupropion 80. A 42-year-old man stops his antidepressant. (B) fluvoxamine Two days later, he experiences a feeling of hopelessness, has difficulty sleeping, and feels (C) fluoxetine quite anxious. (D) clomipramine (E) citalopram 81. A 34-year-old man has been noncompliant with medications in the past. You want to be able to Questions 87 and 88 give him a depot injection of an antipsychotic. You are treating a 28-year-old man with a 6-year his- tory of schizophrenia who lives with his parents. The 82. A 36-year-old woman being treated for bipolar patient has had a number of debilitating side effects, disorder finds herself urinating very frequently. such as severe extrapyramidal symptoms, from tra- ditional antipsychotics including haloperidol. His 83. You are treating a 45-year-old veteran with a family is worried about weight gain, which the history of alcoholism for posttraumatic stress patient has experienced on olanzapine. You discuss disorder (PTSD) and he asked you for some- clozapine with the patient and his family and they thing to help him sleep. agree to try the medication. You have initiated treat- ment slowly and he has shown some improvement; 84. Several months after starting on a new antipsy- however, he has been producing a large amount of chotic, a 32-year-old patient’s total cholesterol saliva and drool soaking his shirts. increases from 210 to 230 mg/dL. Questions: 76–92 101

87. You should 90. A 34-year-old electrical engineer complains to his doctor that he has been feeling suicidal. He (A) stop treatment with clozapine feels guilty over the breakup of his marriage, (B) initiate propranolol cannot concentrate, feels hopeless and help- (C) initiate propylthiouracil less, and has a subjective feeling of sadness (D) increase the dose of clozapine and depression. He has lost 25 lb in the last (E) initiate amantadine (Symmetrel) 3 months because his “appetite isn’t the same.” A diagnosis of depression is made and he is 88. In addition to the above side effect, the patient started on an antidepressant. Which symptom develops an increase in his resting heart rate. can be expected to improve most quickly? An ECG revealed a sinus tachycardia without (A) guilt other changes from his baseline. This has been (B) hopelessness and helplessness repeated several times over the past month and the heart rate remains elevated. You should (C) decreased concentration consider which of the following? (D) suicidality (E) decreased appetite (A) stopping treatment with clozapine (B) initiating benztropine 91. The patient is a 59-year-old female with chronic (C) checking creatinine phosphokinase schizophrenia, undifferentiated type, as well levels as poorly controlled diabetes mellitus and (D) initiating treatment with propranolol hypertension. She has been treated for over (E) initiating treatment with labetalol 35 years with various typical antipsychotics, and although she is on an adequate dose, she still 89. A 21-year-old single female college student is has residual psychotic symptoms and has now brought to the university health service by her begun to develop involuntary blinking and roommates. She makes it clear to you that she tongue rolling movements. She is agreeable to does not think there’s anything wrong with switching to an atypical antipsychotic. Which her, but her roommates tell you that they “can’t of the following medications would be the deal with her anymore” because she never most appropriate for this patient? sleeps and “never shuts up.” The woman tells (A) aripiprazole you that’s just because she has discovered an (B) clozapine important mathematical proof that she just must finish writing. She’s sure that the univer- (C) olanzapine sity will grant her tenure as soon as they see the (D) quetiapine proof, although she has trouble staying on the (E) risperidone topic when you start to ask questions about it. She has not seen a psychiatrist before, and a 92. Jane is a 20-year-old prostitute who has been urine toxicology screen, which she consents to using cocaine regularly for greater than 3 years. “to prove I’m not crazy,” is negative. The Over the past 12 months, she finds that she has most appropriate agent for initial and main- had to use increasing amounts of cocaine in tenance treatment of this patient is which of order to obtain the same level of “high.” Which the following? of the following terms best describes this effect? (A) clonazepam (A) tolerance (B) lithium (B) dependence (C) topiramate (Topamax) (C) addiction (D) haloperidol (D) withdrawal (E) gabapentin (E) abuse 102 4: Somatic Treatment and Psychopharmacology

93. A 25-year-old woman is sent to a psychiatrist no longer believes he is the Messiah, he still for evaluation. She has felt “tired” for the past wonders if cars are following him near his 1 2 /2 months, with associated crying spells, dif- group home. His Mental Status Examination is ficulty falling asleep, diminished appetite with notable for psychomotor retardation and a flat a 10 lb weight loss, and feelings of guilt over affect. He admits to suicidal ideation without many aspects of her life. She hasn’t been able to plan or intent and denies homicidal ideation. enjoy pursuits that had given her pleasure, and Consideration is given to switching to queti- she has been distracted at work, making many apine. Which of the following symptoms “silly” mistakes. Although she feels like she would be the most appropriate to target for has “no hope,” she denies any suicidal switching medication in this patient? ideation. She denies past psychiatric or medical (A) anhedonia history, and she drinks 2 glasses of wine per week. She admits both parents were diagnosed (B) hallucinations with “manic-depression” and are on unknown (C) obesity medications for it. Which of the following (D) paranoia would be the most appropriate treatment for (E) suicidality this patient? (A) aripiprazole 96. After discussion with the patient, it is decided to change medications to quetiapine, with a (B) carbamazepine target dose of at least 400 mg daily. Which of (C) ECT the following switching methods would be the (D) lamotrigine most appropriate? (E) venlafaxine (A) Initiate quetiapine at 50 mg bid and increase while tapering haloperidol. 94. A 40-year-old woman with a history of psy- chotic depression resistant to many trials of (B) Initiate quetiapine at 200 mg bid while antidepressant medication is being considered tapering haloperidol. for ECT. Which of the following conditions is (C) Initiate quetiapine at 200 mg bid and an relative contraindication to ECT? taper haloperidol after 4 weeks. (D) Simultaneously discontinue haloperidol (A) pregnancy while initiating quetiapine at 50 mg bid. (B) degenerative joint disease (E) Taper haloperidol and then initiate (C) hypertension quetiapine at 50 mg bid. (D) recent MI (E) psychotic depression 97. A 23-year-old patient with schizophrenia who was recently started on a neuroleptic complains Questions 95 and 96 that he cannot sit still and feels constantly rest- less. The nursing staff has noted that he often 95. The patient is a 23-year-old obese male newly paces. Which of the following is this known as? diagnosed with schizophrenia. He was first admitted to the inpatient unit 2 months ago (A) parkinsonism where he was stabilized on haloperidol 10 mg (B) dystonia bid and he is following up for his outpatient (C) tremor appointment. Upon evaluation, he is coopera- (D) TD tive but does not expand on his answers. He (E) akathisia denies feeling depressed but also denies enjoy- ment of activities. He sleeps approximately 98. A 35-year-old woman with a history of seizures 5 hours per night. He admits to lingering audi- is diagnosed with bipolar disorder and started tory hallucinations telling him derogatory on lamotrigene. Which of the following side things, but he denies any commands. While he Questions: 93–102 103

effects would be the most important to educate (C) fluoxetine the patient about? (D) mirtazapine (A) aplastic anemia (E) reassurance (B) hypertension 101. A 44-year-old married patient with a history of (C) memory loss recurrent major depressive disorder presents (D) rash to a psychiatrist after stopping his sertraline (E) TD 1 month ago. Although it has been effective in the treatment of his depression, he has had 99. A 19-year-old man is diagnosed with schizo- increasing difficulty maintaining an erection, phreniform disorder and is given an intramus- and this has caused friction and conflicts with cular injection of haloperidol. Later, the patient his wife. He is concerned, however, that he will is found arching forward. On physical exami- have a relapse of his depressive symptoms if nation, stiffness of the neck and back muscles not medicated. Which of the following would are noted. How is this dystonia best described? be the most appropriate treatment for this (A) opisthotonos individual? (B) torticollis (A) begin mirtazapine (C) laryngospasm (B) begin paroxetine (D) oculogyric crisis (C) begin venlafaxine (E) pleurothotonos (D) no treatment (E) restart sertraline 100. The patient is a 59-year-old woman, with a past medical history of hypercholesterolemia, dia- Questions 102 and 103 betes, and diabetic neuropathy, who presents to her primary care doctor with the chief com- 102. A 36-year-old man is started on bupropion plaint of “headaches.” Upon further interview, (Zyban/Wellbutrin) for depression. He returns she states her symptoms began after the death 2 weeks later and states that he feels that his of her husband 5 months ago. Since that time, mood has improved slightly and that he has more she has felt “empty,” with poor sleep, fatigue, energy. He has also noticed that his sexual decreased concentration, and wishes she desires have become insatiable. He notes a dra- “would die so I can join him.” Her review of matic increase in his sexual energy and is mas- systems is remarkable for the above com- turbating frequently despite having intercourse plaints plus ongoing “sharp pins and needles” up to two times per day with his wife. Which in her extremities. Her physical examination is of the following is the best term to describe unchanged and her blood sugars have been this patient’s insatiable sexual desire? relatively well-controlled. Which of the fol- (A) hypervigilance lowing would be the most appropriate treat- (B) hypermnesia ment for her symptoms? (C) satyriasis (A) bupropion (D) sexual sadism (B) duloxetine (E) sexual masochism 104 4: Somatic Treatment and Psychopharmacology

103. The patient is switched to fluoxetine (Prozac) (A) amitriptyline after the bupropion was stopped. One week (B) cognitive-behavioral therapy later, he calls complaining of an inability to (C) duloxetine sleep. He is able to fall asleep but awakens (D) ECT within 2 hours and is unable to return to sleep. The patient is started on trazodone (Desyrel) (E) fluoxetine for this problem. Three days later, he calls com- plaining of a painful erection that he cannot Questions 106 through 109 get rid of even after several hours. Which of the Use the list below to identify the proper term for following best describes the condition of the each question. patient following the initiation of trazodone? (A) inhalants (A) parapraxis (B) withdrawal (B) priapism (C) tolerance (C) nymphomania (D) abuse (D) satyriasis (E) dependence (E) libido (F) formication (G) intoxication 104. A 35-year-old married female is being followed by a psychiatrist for her first episode of major (H) cannabis depression. She is prescribed citalopram 40 mg (I) post-hallucinogen perception disorder daily and has been in remission for 3 months. (J) amotivational syndrome She now complains of the new onset of decreased (K) cannabis libido and this is causing her a moderate amount of distress. Which of the following 106. A 45-year-old man presents to the emergency would be the most appropriate to add to her department in delirium tremens. He complains current regimen? of bugs crawling on his skin. (A) bupropion 107. A 14-year-old boy is brought to you by his (B) desipramine mother. She states that he has no motivation and (C) lithium is failing in school. The boy admitted to his (D) tranylcypromine mother that he has been smoking marijuana. To (E) venlafaxine what class of substances does marijuana belong?

105. A 27-year-old married female, 13 weeks preg- 108. A 21-year-old man presents to the emergency nant, is referred to a psychiatrist for evalua- department after “huffing gas.” The patient tion. She has no past psychiatric or medical claims that he feels disconnected from reality history, but she believes that her mother also and is frightened. After about 20 minutes, these suffered from depression. Over the past 5 weeks, effects diminish. Petroleum fumes are in what she has become increasingly “sad,” with almost group of substances? daily crying spells, middle insomnia, anergia, and anhedonia. More concerning, however, is 109. A 54-year-old man states that he continues to that she has had “no interest” in eating, and she use alcohol despite having lost his family and has lost 10 lb. Although she denies a specific job. He often drives his car while intoxicated plan for suicide, she feels no hope for the future and recently was arrested in a barroom brawl. and doesn’t care “if I live or die.” Which of the The fact that this patient continues to drink following would be the most appropriate alcohol despite adverse effects indicates that treatment? he has which substance problem? Questions: 103–121 105

Questions 110 through 121 115. The same 43-year-old man was continued on haloperidol despite his side effects and after Use the following terms to match the correct response several years developed choreic movements of to the questions below. the trunk and limbs along with lip smacking (A) galactorrhea and tongue movements. (B) agranulocytosis 116. A 21-year-old man with psychosis was started (C) blepharospasm on haloperidol at high doses. Eight days into (D) opisthotonos his treatment, he developed fever, generalized (E) torticollis rigidity, diaphoresis, and altered mental status. (F) rabbit syndrome (G) NMS 117. A 32-year-old woman with bipolar disorder (H) obstructive jaundice suffers a severe depressive decompensation (I) gynecomastia 1 month after a minor decompensation. (J) pigmented retinopathy 118. A 26-year-old man being treated with high- (K) orthostatic hypotension dose thioridazine is found on ophthalmologic (L) pseudoparkinsonism examination to have peppery pigmentation of (M) akathisia the retina. Despite discontinuation of the med- (N) TD ication, the pigmentation continues and the (O) constipation patient eventually suffers total blindness. (P) retrograde ejaculation 119. A 22-year-old man complains of severe rest- (Q) double depression lessness and agitation after starting on a neu- (R) kindling roleptic. He complains that he cannot sit still and paces constantly. 110. A 27-year-old man with paranoid schizophre- nia notices the development of breasts, a 120. A 26-year-old patient with schizophrenia is decreased libido, and not ejaculating with started on chlorpromazine (Thorazine). After orgasm. He is being treated with thioridazine 1 week, he complains of flu-like symptoms. A (Mellaril). The development of breasts is due to CBC is obtained and is normal. After another the blockade of dopamine receptors in the week, the patient develops a yellowish color to tuberoinfundibular tract. his skin and sclera.

111. A 35-year-old man taking thioridazine finds 121. A 45-year-old patient with schizophrenia is himself unable to ejaculate. started on a high dose of chlorpromazine. The next day, he complains of being light-headed 112. A 35-year-old woman you are treating for an every time he stands. On examination, he is active dysthymic disorder develops major noted to have a significant decrease in blood depression. pressure and an increase in heart rate. You speculate that this is secondary to the alpha1- 113. A 22-year-old African American man on cloza- adrenergic blockade. pine (Clozaril) for 3 months reports a sore throat and fever. A CBC is obtained with a WBC count of 1000/mm3 with 1% granulocytes.

114. A 43-year-old man with schizophrenia is on haloperidol. He is noted to have a flexed posture, festinating gait, resting tremor, and bradykinesia. 106 4: Somatic Treatment and Psychopharmacology

Questions 122 and 123 (C) GABA-A (D) GABA-B 122. You are called in the middle of the night by the surgical service to see a 52-year-old man who (E) 5-HT2 was admitted to the hospital 36 hours earlier for an emergency appendectomy. The man is 123. This syndrome usually occurs at which of the very agitated and is talking nonsense to the following times? nursing staff who is unable to calm him down. (A) within the first 12 hours after admission You suspect alcohol withdrawal. Which of the (B) within the first 2 days after admission following is the receptor most closely associ- (C) within the first 5 days after admission ated with these symptoms? (D) within the first 10 days after admission (A) dopamine-2 (D2) (E) anytime during this admission (B) dopamine-4 (D4) Answers and Explanations

1. (C) Phenelzine and tranylcypromine are the only therefore, wait at least 14 days after discontin- nonselective monoamine oxidase inhibitors uing an MAOI before starting a serotonergic (MAOIs) currently approved for use in the agent. Rapid-cycling bipolar disorder is an United States. They are extremely dangerous in affective disorder and NMS is an idiosyncratic overdose and, after a brief asymptomatic period reaction to neuroleptic drugs. Akathisia and of 12–24 hours, may produce hyperpyrexia and opisthotonos are both extrapyramidal side autonomic excitability sufficient enough to cause effects of neuroleptics. rhabdomyolysis. Supportive care should be instituted. If delirium develops, small doses of 3. (C) This patient is likely suffering from a major IV benzodiazepines should be used. Lorazepam depressive episode. Because of her weight gain, is preferred because of its short elimination half- hypersomnia, and mood reactivity, her depres- life. Neuroleptics, especially short-acting agents sion has atypical features. Although atypical fea- such as chlorpromazine, should be avoided tures respond best to MAOIs such as phenelzine, because of their tendency to contribute to these medications are rarely used because of the hypotension. If ventricular arrhythmias develop, potential risk of a fatal hypertensive crisis when they can be treated with lidocaine. Bretylium eating certain tyramine-containing foods. SSRIs may contribute to the adrenergic crisis and such as fluoxetine and sertraline are also bene- should be avoided. Phenytoin may be used if ficial even though response rates to SSRIs may seizures develop. Meperidine should be avoided not match those of the MAOIs. Because of their because it may contribute to the adrenergic favorable safety profile, SSRIs have become a crisis. Other medications that are contraindi- first-line treatment in major depression. TCAs cated with MAOIs include stimulants, decon- such as nortriptyline are less effective than gestants, amine precursors such as L-dopa and MAOIs for atypical depression. Trazodone L-tryptophan, and the antihypertensives methyl- inhibits serotonin reuptake and may also be dopa, guanethidine, and reserpine. serotonomimetic.

2. (C) Blocking reuptake of catecholamines and 4. (B) Approximately 50% of patients who meet indolamines in patients already using an MAOI criteria for major depression recover after treat- can result in a potentially life-threatening drug ment for 6 weeks with an adequately dosed anti- interaction known as serotonin syndrome. depressant. Initially, sleep may improve within Medications that block such reuptake include the first 2 weeks; however, this response may not the SSRIs, TCAs, buspirone, and other antide- be predictive of efficacy. Substantial effects may pressants such as venlafaxine. Features of mild not be seen for 4 weeks, and maximal effects serotonin syndrome include tachycardia, flush- may require up to 8 weeks of treatment. ing, fever, hypertension, ocular oscillations, and myoclonic jerks. Severe serotonin syndrome 5. (A) Alcohol withdrawal should be strongly may result in severe hyperthermia, coma, auto- considered in this patient. Benzodiazepines nomic instability, convulsions, and death; are the drug of choice for control of alcohol

107 108 4: Somatic Treatment and Psychopharmacology

withdrawal symptoms and for prophylaxis 9. (B) Akathisia, an extrapyramidal side effect of against withdrawal seizures and the potentially neuroleptics, can be difficult to distinguish life-threatening delirium tremens. Long-acting from anxiety in a psychotic individual; how- benzodiazepines such as chlordiazepoxide and ever, based on observation and history from diazepam are appropriate; however, both are his parents, akathisia should be strongly con- extensively metabolized by the liver. Because this sidered and empiric treatment attempted. patient has elevated transaminases, oxazepam or Onset usually begins between 5 and 60 days lorazepam, which only undergoes glucuronida- after initiation of treatment but can occur after tion prior to elimination, is preferable. A high- just one dose. First, the neuroleptic dose should potency antipsychotic agent such as haloperidol be reduced as much as possible. In general, may be necessary to help control this patient. anticholinergic agents such as benztropine are However, low-potency antipsychotics, such as used to treat extrapyramidal symptoms. In the chlorpromazine, should be avoided because they absence of other extrapyramidal side effects, may lower the seizure threshold. Disulfiram is however, many clinicians would start treat- used to treat alcohol dependence not acute alco- ment with a beta-blocker such as propranolol. hol withdrawal. Antianxiety agents such as lorazepam are gen- erally not helpful. Diphenhydramine is helpful 6. (E) This patient has delirium tremens, which is in acute dystonias but not in akathisia. a medical emergency. He should be transferred to an intensive care setting where IV benzodi- 10. (D) Flooding is an exposure technique that is azepines can be administered with greater commonly used in behavior therapy. The prem- safety. The IV route is preferred in this situation ise is confrontation with the feared symp- to ensure adequate absorption. Haloperidol tom. Couples and family therapy are designed can be used for its sedative effect but it should to modify relationships between the patient not take the place of a benzodiazepine. Similarly, and others in his or her social constellation. IV labetalol may be useful in the intensive care Psychodynamic psychotherapy is a form of ther- unit but it should not replace a benzodiazepine. apy informed by psychodynamic processes. Hypnosis is a sleep-like state induced by a ther- 7. (A) The precise mechanism of action of SSRIs in apist that has been of some use in uncovering depression is not fully understood. Acutely, psychodynamic processes and in behavioral they enhance serotonergic neurotransmission modification such as stopping smoking. by permitting serotonin more time to bind at the postsynaptic terminal. However, because 11. (C) Rhabdomyolysis is the most common seri- therapeutic effect of SSRIs may take several ous complication of NMS, occurring in up to weeks, this acute effect is likely not responsible 25% of patients in one series. Dialysis may be for the improvement seen in depression. One required to protect patients from renal failure. leading theory is that they cause downregula- Creatinine phosphokinase is usually elevated

tion of postsynaptic 5-HT2 receptors. However, in NMS but there are no specific laboratory not all SSRIs have been shown to do this, so the findings. All of the other listed complications answer is likely much more complicated. SSRIs have been reported but with less frequency. can inhibit the dopamine system accounting for the anecdotal reports of SSRIs causing 12. (C) Retinal pigmentation is known to occur extrapyramidal side effects. However, this is when thioridazine is used in high doses (>1000 probably unrelated to their effectiveness in mg/day). It may not remit when thioridazine is depression. discontinued and it can lead to blindness. The other choices can occur with all neuroleptics. 8. (A) Anorgasmia is the most common reported sexual dysfunction side effect in women taking 13. (E) This patient is suffering from panic disorder SSRIs such as sertraline. with agoraphobia. There are many medications that have proven benefit for this condition Answers: 6–21 109

including TCAs, SSRIs, MAOIs, and high- sulindac, however, do not affect lithium levels potency benzodiazepines. Of the agents listed, and are therefore safe for use with lithium. only fluoxetine, an SSRI, is one of them. Bupropion, an antidepressant with dopamine 17. (A) Difficulty falling asleep and decreased activity, is not effective. Trazodone, an antide- appetite are the two most common side effects pressant with mixed serotonergic effects, has of methylphenidate. Stimulants have been shown conflicting results. Buspirone, a partial reported to slow growth; however, it is believed

5-HT1a antagonist with antianxiety activity, is to be less common with methylphenidate and not effective. Propranolol, a beta-adrenergic dose related. When drug holidays are given, a blocker, may help to alleviate the physical growth rebound is seen. Methylphenidate may symptoms such as tachycardia but does not cause an increase in systolic blood pressure. prevent the attack. 18. (B) Stimulants have been frequently reported to 14. (C) Clonazepam is the most potent benzodi- exacerbate the tics associated with Tourette dis- azepine listed. Using 1 mg of lorazepam for order and some studies have warned that they the equivalent dose in milligrams, the potency should not be given to children with tics or a relationship of some common benzodiazepines family history of Tourette disorder. However, is as follows: clonazepam 0.25 mg, triazolam some recent studies have identified frequent 0.25 mg, alprazolam 0.5 mg, diazepam 5 mg, comorbidity of attention deficit hyperactivity clorazepate (Tranxene) 7.5 mg, chlordiazepox- disorder (ADHD) and Tourette disorder. ide 10 mg, and flurazepam (Dalmane) 30 mg.

Flumazenil is a benzodiazepine antagonist. 19. (C) Dopamine D2 receptor antagonism pro- Buspirone is a nonbenzodiazepine anxiolytic vides the greatest relief of symptoms in children that is generally considered to be less potent. suffering from Tourette disorder. Pharmacologic treatment is strictly symptomatic and is not cur- 15. (A) These are the minimum initial laboratory ative. An alpha2-agonist, not antagonist, would studies required before initiating lithium treat- provide some relief but it is not as effective as

ment. Because lithium is a Group IA monova- D2 receptor antagonism. lent ion, the kidney handles it much as it does sodium. Ninety-five percent of lithium is 20. (B) Haloperidol is the only medication listed

excreted unchanged through the kidneys. that is a strong antagonist at the D2 receptor. Therefore electrolytes, creatinine, BUN, and Another agent that is nearly as effective as urinalysis are required to check kidney func- haloperidol is pimozide. Clonidine, which is tion. Thyroid studies are required because an alpha2-agonist, is also used in Tourette dis- lithium inhibits the synthesis of thyroid hor- order but it is clearly less effective than mone and its release from the thyroid. A CBC haloperidol. However, because clonidine is not is optional. Lithium may cause a benign eleva- a neuroleptic, it lacks extrapyramidal side tion in the WBC. Liver function tests should effects and the associated long-term sequelae. be performed prior to initiating treatment with Additionally, it may be useful in patients with divalproex sodium. Rare cases of fatal hepato- ADHD and Tourette disorder. toxicity have been reported with divalproex sodium. Divalproex sodium may also elevate 21. (C) The most likely syndrome is TD, a move- serum ammonia levels, but this is rare. ment disorder that may occur after long-term treatment with neuroleptic medication such as 16. (D) The recommendation should be to switch to haloperidol. It consists of a number of abnor- aspirin. In some individuals, nonsteroidal anti- mal and involuntary movements such as lip inflammatory agents (NSAIDs) cause an smacking, facial grimacing, and choreoa- increase in lithium levels. Therefore, these agents thetoid-like movements of the limbs and should be used with caution in patients taking trunk. In Huntington disease, a dominant lithium and avoided if possible. Aspirin and genetic disorder, the involuntary movements 110 4: Somatic Treatment and Psychopharmacology

are accompanied by a progressive dementia. TCAs carry a heightened risk in these patients Sydenham chorea, associated with rheumatic because of their potential lethality in overdose. fever, occurs in children. Meige syndrome is an oral facial dystonia involving blinking and chin 25. (A) This patient is suffering from bulimia ner- thrusting, sometimes lip pursing or tongue vosa, purging type, and several studies have movements, and occasionally shoulder move- shown that fluoxetine can help these patients. ment. Anticholinergic toxicity, which usually Many believe it is related to the high incidence presents as delirium, is not present. of comorbid depression in this population. TCAs, such as protriptyline, have not been as 22. (E) It was once believed that if patients devel- effective. Because electroencephalographic oped signs of TD, they would progress to abnormalities have been found in patients with severe dyskinesias. Recent evidence, however, bulimia, phenytoin has been tried but it does indicates that most patients stabilize and may not appear to be effective. spontaneously improve even if they continue taking the antipsychotic. However, the only 26. (B) Unless there is a comorbid psychiatric dis- medication that may improve these symptoms order such as OCD or depression, pharma- is clozapine. cotherapy generally has a limited role in the treatment of anorexia nervosa. Reports on 23. (A) This patient likely has generalized anxiety the usefulness of cyproheptadine to stimulate disorder (GAD). Benzodiazepines are the most appetite have been mixed. Generally, these effective medications for quickly reducing patients already have a good appetite. One study symptoms of GAD. All benzodiazepines are suggested that high-dose chlorpromazine is effective and the choice should be based on useful, but the results have not been dupli- potency, half-life, and side effects. Alprazolam cated. The TCA amitriptyline and the pro- has an intermediate onset, a half-life of 6–20 motility agent cisapride are not useful in anorexia hours, and does not have active metabolites. nervosa. In particular, cisapride should be avoided Buspirone is effective for reducing symptoms because patients with anorexia nervosa are but does not work as quickly as benzodi- already at high risk for arrhythmias, which can be azepines. The tricyclic agent imipramine and worsened by cisapride. SSRIs such as fluoxetine also have anxiolytic properties but also do not work as quickly as 27. (D) This patient likely has SSRI discontinuation benzodiazepines. Bupropion does not have syndrome, which may occur within 1–3 days of anxiolytic properties. abruptly stopping an SSRI. The most common physical symptoms are dizziness, nausea and 24. (B) This patient likely has the Axis II diagnosis vomiting, fatigue, lethargy, and flu-like symp- of borderline personality disorder. This disor- toms. Psychological symptoms are anxiety, irri- der can be extremely difficult to treat. SSRIs, tability, and crying spells. Paroxetine may cause particularly fluoxetine, have been shown to this because of its short half-life and lack of an reduce the impulsiveness and affective insta- active metabolite. Fluoxetine has an active bility in these patients, and antipsychotics such metabolite with a half-life ranging from 4 to 16 as haloperidol have been shown to decrease days, and, therefore, does not require tapering. the impulsiveness and psychotic thinking. One The discontinuation syndrome is not known recent study used low-dose clozapine in a small to be lethal; however, restarting the paroxetine sample of patients with severe borderline per- and tapering it is a reasonable choice. sonality disorder and found it to be beneficial. Anticonvulsants such as divalproex sodium 28. (C) This patient has major depression requiring may help to modulate intense fluctuations in antidepressant treatment. SSRIs are a good first affect but stimulants have not been shown to be choice; however, fluoxetine has been reported effective in borderline personality disorder. to increase carbamazepine levels. Paroxetine Answers: 22–35 111

has not been reported to increase carbama- tetracycline; and angiotensin-converting enzyme zepine levels. Nefazodone does increase carba- inhibitors. mazepine levels, perhaps through interaction with the cytochrome P-450 3A3/4 isoenzyme. 33. (A) Clomipramine, a TCA with potent sero- Clomipramine and amitriptyline are both TCAs tonin reuptake inhibition, is the most widely that have substantial anticholinergic side effects. studied agent with efficacy in OCD. Several TCAs are more dangerous in overdose than are SSRIs, including fluoxetine, fluvoxamine, ser- SSRIs as well. traline, and paroxetine, have demonstrated effi- cacy in OCD. Other tricyclic agents are not as 29. (C) Trazodone is an antidepressant medication, effective as clomipramine probably because unrelated to other antidepressant drugs (except they lack the potent serotonin reuptake inhibi- nefazodone), which affects the serotonin system tion of clomipramine. Clonazepam may be by weak reuptake inhibition and antagonist used for anxiety associated with OCD but it is

activity at 5-HT1a, 5-HT1c, and 5-HT2 receptors. not effective in treating the underlying disor- Some clinicians have questioned its efficacy in der. Mirtazapine is an antidepressant that is a severe illness but its use as an adjunct to help central alpha2-antagonist, leading to increased with sleep in depressed patients is growing. norepinephrine and serotonin release, and an

Some SSRIs can be activating, such as fluoxe- antagonist of both 5-HT2 and 5-HT3. However, tine, and trazodone can help with sleep in these it has not been used in OCD. Phenelzine is an patients. Bupropion does not cause significant MAOI that is not used in OCD. Olanzapine is drowsiness. Diazepam is a benzodiazepine and an atypical neuroleptic. Neuroleptics generally diphenhydramine is an antihistamine although are not effective in OCD. both have been used to help with sleep. Both should be avoided in the elderly if possible. 34. (A) Just like treatment of refractory depression, treatment of refractory OCD is largely based on 30. (A) Priapism, though rare, has been reported case reports and clinical preferences. There are with trazodone use. The manufacturer has very few controlled blinded trials in refractory reported the incidence of any abnormal erectile OCD. Several open label studies reported bus- function to be about 1 in 6000 men. This side pirone was effective as an augmentation to effect usually occurs within the first month of SSRIs or clomipramine; however, a double- treatment but can occur at any time. Any blind controlled study found no benefit to the abnormal erectile function should prompt dis- addition of buspirone. There are case reports of continuation of the medication. fenfluramine and lithium as effective aug- menting agents but these findings require val- 31. (C) Lithium often causes T-wave flattening or idation and fenfluramine has been withdrawn inversion on ECG, but the changes are usually from the U.S. market. Some studies found that not clinically significant. Sinus node dysfunc- clozapine in patients with schizophrenia wors- tion and first-degree atrioventricular (AV) ened OCD symptoms. Cingulotomy, a surgical block are rare but are the most commonly procedure, should be reserved for severe refrac- reported cardiac side effects of lithium. Lithium tory cases. toxicity can cause sinoatrial block, AV block, AV dissociation, bradyarrhythmias, ventricular 35. (D) This patient is suffering from command tachycardia, and ventricular fibrillation. auditory hallucinations, which have not suffi- ciently responded to adequate trials of two typ- 32. (C) Thiazide diuretics can increase lithium ical neuroleptic agents, chlorpromazine and levels owing to decreased lithium clearance. haloperidol, and one atypical neuroleptic Other agents that can increase lithium levels agent, olanzapine. Therefore, a trial of clozap- are the diuretics ethacrynic acid, spironolac- ine is warranted. Clozapine was the first atyp- tone, and triamterene; NSAIDs except aspirin ical neuroleptic and has been found to be and sulindac; the antibiotics metronidazole and superior to typical neuroleptics in reducing 112 4: Somatic Treatment and Psychopharmacology

both positive and negative symptoms. It may SSRI without response. Lithium and thyroid also improve cognitive deficits in patients with augmentation of antidepressants have been schizophrenia. Fluphenazine and perphenazine well worked out; however, this patient has are both typical neuroleptics like haloperidol and not even had a partial response to augment. chlorpromazine. Risperidone is considered an Additionally, these options are generally atypical neuroleptic but the rate of response in reserved for patients who have failed multiple refractory patients is less than that with cloza- trials of other medications. Imipramine, a TCA, pine. Ziprasidone, approved by the U.S. Food has the accompanying anticholinergic side and Drug Administration in August 2004, is effects typical of this class, which make it a less

like olanzapine in that it has 5-HT2a and D2 desirable choice. Bupropion is an effective receptor antagonist properties. agent; however, the risk of seizures in patients without a seizure history is 0.6–0.9% at doses 36. (C) Clozapine acts at many receptors, including greater than 200 mg/day. At low doses, ven-

D1, D2, and D4; histamine-1; muscarinic; alpha1- lafaxine inhibits only serotonin reuptake, but at adrenergic; and serotonin types 5-HT2, 5-HT2c, doses greater than 225 mg/day, it also inhibits and 5-HT3. Efforts to identify the exact mecha- norepinephrine reuptake distinguishing it from nism of antipsychotic action of clozapine have the SSRIs. In 3–13% of patients, venlafaxine has revealed at least two possibilities. Unlike typi- caused a sustained increase in supine diastolic cal neuroleptics, it has much more potent blood pressure.

antagonism at the D4 receptor compared to the D2 receptor, especially in the limbic system. 39. (E) All of the commonly used mood-stabilizing This has led to speculation that the D4 receptor medications, except clonazepam (a benzodi- may mediate psychotic symptoms. It also has azepine), appear to carry an increased risk of

activity at the 5-HT2 receptor, activity that typ- fetal malformations or a potential deleterious ical neuroleptics lack. effect on later cognitive development. Use of lithium during the first trimester increases 37. (D) TCAs, including imipramine and nor- the risk of cardiac malformations, including triptyline, block various receptors, including Ebstein anomaly, to 7.7%. A fetal echocardio- muscarinic, alpha1- and alpha2-adrenergic, gram should be done between weeks 16 and 18 dopaminergic, and histaminergic, to varying of pregnancy. Lithium may be used in the degrees. As a class, their blockade of alpha1- second and third trimesters but it should be receptors is generally believed to be responsi- stopped peripartum because of the rapid fluid ble for any orthostatic hypotension that may shifts and changes in glomerular filtration. occur especially in the elderly. Nortriptyline Carbamazepine increases the risk of neural has relatively less alpha1-blocking potency and tube defects and divalproex sodium increases some clinicians prefer it over other tricyclics in the risk of intrauterine growth retardation and the elderly. Bupropion and mirtazapine are neural tube defects. not associated with orthostatic hypotension. Trazodone can cause orthostatic hypotension 40. (E) It is not unusual for delirious patients to from alpha1-blockade; however, nefazodone become hostile or combative posing a risk to has a lower affinity and is expected to have a themselves or other hospital staff. Low-dose lower incidence of orthostatic hypotension. atypical antipsychotics, such as risperidone, are very effective in reducing agitation in deliri- 38. (E) Treatment of refractory depression remains ous patients. Diphenhydramine should be controversial. Unlike evidence for initial selec- avoided as the anticholinergic effects may tion of antidepressant therapy, there is little actually worsen the delirium and confusion. well-controlled evidence regarding treatment Donepezil is an anticholinesterase inhibitor of refractory depression. Selection is based used for dementias. Although benzodiazepines largely on clinical practice and side effect pro- can be used for agitation in delirium, they may file. This patient has had an adequate trial of an overly sedate or conversely disinhibit the Answers: 36–46 113

patient further. Orientation to surroundings is lower doses with fewer anticholinergic or ortho- often additionally helpful in delirium but it static side effects. Additionally, low-potency will not immediately calm the patient. agents such as chlorpromazine or thioridazine are not only associated with orthostatic hypoten- 41. (D) Mirtazapine is an antidepressant whose sion and anticholinergic side effects but also with pharmacologic profile is different from other prolongation of the QT interval. Olanzapine is available agents. It is a central alpha2-adrenergic not used in the intensive care setting partly

antagonist and an antagonist of 5-HT2 and because it is not available in a parenteral form. 5-HT3 receptors as well as H1 receptors. Because Lorazepam may help sedate the patient but it it blocks alpha2-receptors, it leads to enhanced will not help his psychosis.

serotonin release but blockade of 5-HT2 and 5-HT3 leads to relative enhancement of 5-HT1 44. (A) Of the typical neuroleptics listed (haloperi- activity giving it a different side effect profile dol, perphenazine, and thioridazine), haloperi- than SSRIs. For example, it tends to increase dol is the most potent and has the least activity appetite and cause weight gain compared to at alpha1-receptors. Therefore, it is the least

placebo. Also, its H1 antagonism causes sedation. likely to cause orthostatic hypotension. The However, at higher doses, alpha2-adrenergic atypical agents listed (risperidone and queti- blockade also leads to increased norepinephrine apine) have activity at alpha1-receptors and are

release, which may counteract the H1-mediated both associated with orthostatic hypotension. sedation. In this patient, use of mirtazapine to exploit its potential increased appetite and seda- 45. (D) The metabolism of ethyl alcohol involves a tion side effects should be considered. two-step enzymatic process. The first enzyme, alcohol dehydrogenase, metabolizes ethanol to 42. (C) TCAs produce several cardiovascular side acetaldehyde, which is quickly metabolized effects, the most significant being a quinidine- by aldehyde dehydrogenase. Disulfiram inhibits like effect slowing cardiac conduction. Some aldehyde dehydrogenase resulting in an accu- clinicians avoid TCAs if there are any ECG mulation of acetaldehyde. Acetaldehyde causes changes. Certainly, if there are changes in con- facial flushing, tachycardia, hypotension, nausea duction, such as a prolonged QT interval, and vomiting, and physical discomfort. Therefore, widening of the QRS complex, or AV conduc- a patient on disulfiram has an incentive to remain tion abnormalities, TCAs should be avoided. In abstinent. Obviously, disulfiram works only if overdose, they can produce heart block, widen patients continue to take it. Metronidazole is an the QRS complex, cause a bundle branch block, antibiotic that has a disulfiram-like reaction but and cause tachyarrhythmias; however, even at is not used for this purpose. Flumazenil is a ben- therapeutic concentrations, they may have zodiazepine antagonist. Fluoxetine does not sen- adverse effects on cardiac conduction. TCAs sitize against alcohol use. Naloxone is an opioid may be used in patients with prior cerebral antagonist. infarctions. Amlodipine and estrogen replace- ment are not contraindications. TCAs may 46. (C) Multiple neurotransmitter systems have exacerbate urinary retention from anticholin- been investigated in attempts to control alcohol ergic effects but this is not a contraindication. cravings. One system that may play a role is the opioid system. Naltrexone is an opioid antag- 43. (E) This patient is likely delirious and prompt onist that has been shown to decrease the identification and treatment of the underlying number of days a person with alcohol depend- cause is indicated. To help control the agita- ence drinks and to increase the time before tion that may accompany delirium, low-dose relapse of heavy drinking. Treatment of depres- haloperidol is frequently used. Haloperidol sion with an agent such as fluoxetine or bupro- does not treat the delirium, however. It is used pion may help control drinking if a comorbid most frequently because it is the most potent of depression exists but use of antidepressants in the typical antipsychotics, therefore, requiring the absence of a mood disorder has not been 114 4: Somatic Treatment and Psychopharmacology

effective. Disulfiram is an alcohol-sensitizing ability. Pemoline is a stimulant that does not agent. Diazepam is a benzodiazepine that may appear to be indicated in this case. treat withdrawal symptoms but has not been shown to decrease cravings. 51. (A) Trazodone is a serotonergic agent fre- quently used in the elderly to help with sleep 47. (B) This patient is likely suffering from PTSD. and in turn may help this patient’s occasional Several studies have found that SSRIs were wandering behavior. It can cause orthostatic useful in reducing the numbing symptoms of hypotension and should therefore be used with PTSD. In one trial with rape victims, fluoxetine caution in those with risk factors for bleed- reduced symptoms of reexperiencing, avoid- ing. Benzodiazepines should generally be ance or numbing, and hyperarousal. Further used with caution in the elderly. They may double-blind controlled trials are needed to con- actually make behavior worse and may con- firm these findings. Benzodiazepines have not tribute to delirium. If necessary, starting doses been useful for these symptoms. Antipsychotics should be much lower than recommended. such as thioridazine have generally not been Chlorpromazine and haloperidol are neu- useful, although some clinicians are beginning roleptics that should also be used carefully in to try the atypical neuroleptics. Anticonvulsants the elderly. Chlorpromazine, a low-potency are not routinely used but some studies found neuroleptic, has substantial side effects includ- that carbamazepine reduced the reexperienc- ing extrapyramidal symptoms and anticholin- ing and arousal symptoms. Naltrexone, an ergic properties that may exacerbate other opioid antagonist, has not been extensively problems such as urinary retention and confu- studied in PTSD. sion. Haloperidol is a high-potency neurolep- tic that may cause substantial extrapyramidal 48. (E) Open-label trials of the antiadrenergic agent symptoms. Additionally, elderly patients may clonidine (an alpha2-adrenergic agonist) have develop TD much more rapidly than younger demonstrated decreases in symptoms of reex- patients might. periencing and hyperarousal. Double-blind controlled trials are needed to confirm these 52. (E) Patients who present with altered levels of findings. consciousness need to be medically managed, evaluated, and treated for several reversible 49. (C) Elderly patients can be sensitive to the causes. These include hypoglycemia, opioid adverse effects of benzodiazepines. This patient overdose, and alcohol intoxication. Airway pro- is taking 8 mg/day of lorazepam, which is a tection and monitoring of air exchange and large dose. Slowly tapering the lorazepam cardiovascular status are required. Several while investigating other causes of delirium is treatments that should be immediately consid-

a reasonable first step. A digoxin level might be ered include IV dextrose, usually D50, to treat useful because high levels of digoxin can affect hypoglycemia; thiamine to guard against the mental status. Hydrochlorothiazide levels are development of Wernicke-Korsakoff syndrome not measured. when giving the dextrose to an alcoholic patient with thiamine deficiency; and naloxone, an 50. (D) This patient may be suffering from Alzheimer opioid antagonist, to reverse the effects of dementia. He has clear cognitive decline and, opioid intoxication. Flumazenil is a benzodi- although reversible causes of cognitive impair- azepine antagonist that should not be used ment should be investigated, there is no indica- before obtaining further history because it may tion that any reversible cause is present. Two theoretically lower the seizure threshold. cholinesterase inhibitors have been approved for use in Alzheimer dementia: donepezil and 53. (A) Detoxification from heroin can usually be tacrine (Cognex). There does not appear to be managed with clonidine. Clonidine, a central an indication for fluoxetine in this patient. alpha2-agonist, suppresses the sympathetic Aspirin or trazodone will not improve cognitive response to the heroin withdrawal and helps to Answers: 47–60 115

control agitation and autonomic instability, symptoms, it could potentially treat both such as elevated blood pressure and heart disorders. rate, seen in opioid withdrawal. It does not take away heroin craving. Some centers use 57. (C) Weight gain is a side effect of most neu- methadone for heroin withdrawal but some roleptic medications, traditional and atypical; authors advise against methadone for detoxi- however, it has been particularly troubling for fication in patients who have never detoxified olanzapine. For most patients, the weight gain before because of the success of clonidine can be modest (5–15 lb), but for some it can be detoxification. Propranolol is not used for particularly troubling, necessitating discontin- heroin detoxification. Naloxone would not be uation of the medication. Elevated prolactin is appropriate beyond the initial resuscitation a side effect of typical antipsychotics and less efforts. Benzodiazepines may be helpful as common with olanzapine. Agranulocytosis is a adjuncts to control anxiety, but they are not rare side effect of clozapine. Olanzapine is sufficient. likely to cause sedation and is usually admin- istered in the evening. As an antipsychotic 54. (B) Bupropion (Wellbutrin, Zyban) is an anti- with mood-stabilizing properties, it is useful depressant that has been shown to be effective for both psychotic disorders and bipolar dis- as part of a smoking cessation program. The order. Olanzapine is not associated with mechanism of action is unclear, but it is increased excitability or cataracts. believed to have an affect on dopaminergic transmission. Giving up cigarettes may be 58. (B) The patient is a good candidate for Risperdal extremely difficult for a patient who is also suf- Consta. He responds favorably to and tolerates fering from major depression; however, start- oral risperidone; however, he suffers exacerba- ing bupropion during the depression may help tions due to poor compliance. Depot haloperidol her to quit smoking after she has recovered is not as good a choice given his positive from her depression. response to risperidone and greater likelihood of TD. Continuing his current regimen or switch- 55. (B) Several TCAs have been used to treat ing to another oral atypical such as olanzapine chronic pain, including amitriptyline and nor- will likely only lead to a continuation of the triptyline. Using such an antidepressant in this vicious cycle of relapse. As he is not treatment patient may have the added benefit of reducing refractory, clozaril is not appropriate in this case. the chronic pain. However, TCAs, as compared with SSRIs, are more lethal in overdose and 59. (D) Risperidone is an atypical antipsychotic

generally have more side effects, particularly agent that has potent 5-HT2a antagonist anticholinergic and cardiovascular. SSRIs may properties as well as D2 antagonism and alpha1- play a role in chronic pain management so as a adrenergic blocking activity. Because of the first-line therapy they are a better choice. alpha1-blocking activity, it may cause orthosta- tic hypotension. It may cause weight gain not 56. (D) Gabapentin is an anticonvulsant that has weight loss. Unlike clozapine, hematologic been reported to be modestly effective in bipo- effects do not require routine monitoring. It does lar disorder as well as in neuropathic pain dis- not cause significant anticholinergic effects. orders, although it is important to note that it has not been approved for bipolar disorder. 60. (A) Atypical antipsychotic agents, like clozap- Gabapentin has not been studied as extensively ine and risperidone, are considered atypical as lithium, valproic acid, or even carba- because of their decreased propensity for caus- mazepine; however, it has been shown to be ing extrapyramidal side effects and presum- more efficacious than placebo in a few studies. ably a reduced risk of TD. Unfortunately, only Additionally, it has been used to treat neuro- clozapine appears to truly obey this rule. The pathic pain, such as trigeminal neuralgia. In other atypical antipsychotic agents all appear this patient, who is demonstrating hypomanic to have some degree of extrapyramidal side 116 4: Somatic Treatment and Psychopharmacology

effects. Risperidone seems to have the greatest phenelzine have all been shown to be effective in propensity for extrapyramidal side effects and the treatment of panic disorder. The first-line at doses greater than 6 mg/day may cause agents are the SSRIs because of their better side extrapyramidal side effects comparable to effect profile. Bupropion, an antidepressant with haloperidol. a dopaminergic mechanism, and deprenyl, an MAO “b” inhibitor, have both been shown to be 61. (B) Antipsychotic agents, in addition to block- ineffective in the treatment of panic disorder. ing dopamine receptors in the nigrostriatal system (causing extrapyramidal symptoms) 66. (A) The TCAs have been shown to increase and the mesolimbic system (decreasing hallu- mortality in cardiac patients because of their cinations), may also block dopamine in the quinidine-like effects and tendency to increase tuberoinfundibular system, thereby, increasing heart rate and decrease blood pressure. Sertraline prolactin levels and causing galactorrhea is safe for patients with cardiac disease and especially in women. Risperidone may raise affords the fewest drug-drug interactions of prolactin levels 100-fold. Olanzapine has not the SSRIs. As a class, the amphetamines have been demonstrated to raise prolactin levels been found to be safe in patients with cardiac significantly. illness. There may be minor changes in blood pressure and pulse; patients should be moni- 62. (E) Yohimbine, used to treat impotence, can tored closely. Bupropion is safe in cardiac cause elevated blood pressure and heart rate patients. Venlafaxine can increase the blood pres- and should not be used by patients with car- sure an average of 8–10 mm Hg but this effect diac or renal disease, glaucoma, or a history of is dose related. At doses less than 150 mg qd, ulcers. In fact, paroxetine and citalopram are clinically significant changes are relatively rare. antidepressants that can cause impotence. Doses above 300 mg/day are the highest risk Molindone is an antipsychotic. Diphenhydramine but most patients tolerate this dose without is an antihistamine. increases in blood pressure.

63. (E) Clonidine is a centrally acting alpha2-agonist. 67. (C) Haloperidol’s mechanism of action is

In contrast, yohimbine is an alpha2-antagonist. dopamine receptor blockade (D2). Dopamine Clonidine decreases norepinephrine by stim- inhibits prolactin secretion. Because haloperi- ulating the alpha2-autoreceptor. Yohimbine dol prevents dopamine from binding to the increases norepinephrine by blocking the alpha2- dopamine receptor, prolactin secretion is unop- autoreceptor. posed. The symptoms the patient is experienc- ing are common effects of elevated prolactin 64. (D) Benzodiazepines are considered the first- levels. Haloperidol does not increase ACTH, line agents for the treatment of primary insom- serotonin, or dopamine levels to clinically sig- nia. Short-acting agents such as temazepam are nificant levels and the symptoms described are useful in helping patients fall asleep and not not associated with ACTH, serotonin, or remaining drowsy the next day. Diphen- dopamine. hydramine and trazodone are often used for insomnia and are effective medications. 68. (B) This patient most likely has akathisia, However, these can leave patients feeling which is a neuroleptic-induced side effect. drowsy the next day. Clozapine and amitripty- Patients generally experience subjective feel- line are sedating but have serious side effects ings of restlessness and can be seen swinging and should not be used for primary insomnia. their legs, rocking back and forth while sitting, pacing, and rapidly alternating between sitting 65. (A) Fluvoxamine is an SSRI. As a group, the and standing. Acute dystonias are character- SSRIs have fewer side effects and are safer than ized by contraction of muscles resulting in the tricyclics and the monoamine oxidase (MAO) abnormal movements or postures, such as “a” inhibitors. Fluvoxamine, imipramine, and spasms of the jaw, abnormal positioning of the Answers: 61–74 117

head, or difficulty swallowing. The onset of remains controversial. ECT has been used in acute dystonia usually develops within the first pregnancy for more than 50 years and its safety week of initiating a neuroleptic medication. and efficacy is well documented. NMS is a life-threatening complication involv- ing muscle rigidity and dystonia as well as 71. (B) Cessation or a reduction in sedative, hyp- autonomic symptoms such as elevated tem- notic, or anxiolytic medications that have been perature, increased heart rate, and increased used heavily or for a prolonged period of time blood pressure. Akathisia can sometimes be may result in a withdrawal syndrome charac- mistaken for anxiety or mania, but in this case terized by symptoms that develop within hours there are no other signs (such as delusions of to a few days after cessation or reduction. grandeur, decreased need for sleep, increased Autonomic hyperactivity, orthostatic hypoten- energy, or hypersexuality) to indicate a manic sion, muscle weakness, tremor, insomnia, episode. There is no evidence of OCD, feeling nausea, vomiting, auditory/visual/ tactile hal- as though one must always be in motion is not lucinations, agitation, or anxiety may occur. By a compulsion. far, the most serious sequelae are grand mal seizures or delirium. As many as 75% of patients 69. (E) When starting a patient on antipsychotic may experience grand mal seizures on the medication (neuroleptics), the most important second or third day of withdrawal and two- consideration is the side effects the patient will thirds of these patients have more than one experience. Compliance with medication is seizure. Delirium may develop between the usually a major issue and there is a general ten- third and eight day of withdrawal. Minor dency to want to “get off on the right foot.” symptoms may persist for up to 2 weeks. Renal function is less important with neu- roleptics because they are cleared principally 72. (D) Serious chronic alcoholics often take in by the liver. The presence of negative symp- calories from little else besides alcohol and are toms may steer a clinician toward the atypical thus at risk for thiamine deficiency. If a thi- antipsychotics but today these are often chosen amine-deficient patient is given food, he or she because of their superior side effect profile. can develop Wernicke encephalopathy from Gender and onset of action do not typically the body’s attempts to metabolize glucose in guide the choice of neuroleptic. the absence of thiamine pyrophosphate. Although the other choices are important con- 70. (D) Lithium and valproic acid are indicated as cerns, none are of the acute significance of thi- first-line agents for the treatment of bipolar amine deficiency. manic episodes. Antipsychotic agents like chlor- promazine are important for the treatment of 73. (A) This man is probably suffering from alcohol psychosis, sometimes associated with mania, withdrawal. Benzodiazepines and alcohol have but are not considered effective as single agents. near identical modes of action in their modu- Fluoxetine is an antidepressant that may actu- lation of GABA receptors in the brain. This sim- ally activate patients with bipolar disorder and ilarity makes benzodiazepines a sensible and precipitate a manic episode. Ideally, all med- popular choice for treatment of alcohol with- ications should be discontinued during preg- drawal. Much less commonly (although more nancy, but in many instances they are common in the distant past), barbiturates necessary. Considering the risk-benefit ratio is (which also act on GABA) and clonidine important. Low-potency antipsychotics (chlor- (which modulates autonomic instability) are promazine), lithium, and valproic acid are all used. Phenytoin is not used and disulfiram is associated with an increased risk of congenital used to encourage alcohol abstinence by its abnormalities, especially when taken during inhibition of aldehyde dehydrogenase. the first trimester. The benzodiazepines are associated with an increased risk for oral clefts 74. (C) In cases of suspected liver impairment, it is after first-trimester exposure. However, this advisable to use a benzodiazepine minimally 118 4: Somatic Treatment and Psychopharmacology

metabolized by the liver of which there are question 82. Finally, alprazolam is an effective two: lorazepam and oxazepam. Other benzo- sleeping aid but it is wise to avoid using it and diazepines, such as those listed, are mostly other benzodiazepines in a patient with a his- metabolized by the liver and could thus quickly tory of alcoholism because of the affinity of such build to toxic levels. Phenobarbital and cloni- drugs to the GABA receptors implicated in alco- dine are no longer used to treat alcohol with- hol dependence. Trazodone, an antidepressant, drawal. is an effective sleeping aid that is not habit form- ing. Olanzapine has been shown to increase 75–79.[75 (F), 76 (I), 77 (C), 78 (B), 79 (I)] Psychiatric cholesterol levels in about 10% of patients after drugs can have serious side effects, some of 14 weeks of treatment. which are the subjects of “black box” warnings on prescribing information packets. Pimozide 85. (D) The recommended length of treatment of a is known to interact with a number of medica- first episode of unipolar depression is at least 6 tions including citalopram. Although haloperi- months and usually on the scale of 8–12 months, dol, perphenazine, ziprasidone, and clozapine and possibly longer, depending on patient fac- are all antipsychotics used to treat schizophre- tors such as family history of mood disorder, nia, only clozapine is a member of the class of severity and duration of the depressive episode, newer atypical antipsychotics known to cause and comorbid psychiatric symptoms such as hyperglycemia and lead to the development of anxiety and substance abuse. Discontinuation diabetes. After a controversial set of hearings within the first 16 weeks of treatment is associ- on the subject of children and antidepressants ated with a high risk of relapse. in the United Kingdom and United States in 2003 and 2004, regulators decided there was 86. (A) Bupropion is the one antidepressant listed enough evidence to show that some of the that does not cause sexual dysfunction. SSRIs, including paroxetine, may increase the Fluvoxamine, fluoxetine, and citalopram are all risk of suicidal thoughts in children. Although SSRIs that have been reported to cause varying lithium, carbamazepine, divalproex sodium, degrees of sexual dysfunction. Clomipramine and gabapentin are all used to treat bipolar dis- is a TCA that has a greater incidence of sexual order, only valproic acid carries a warning of an dysfunction than other TCAs, possibly due to its increased risk of pancreatitis. Finally, although potent serotonin reuptake inhibiting properties. carbamazepine and clozapine can both cause agranulocytosis, the only one of these drugs 87. (C) Sialorrhea is a common side effect of treat- that would be used primarily for schizophrenia ment with clozapine. It can be extremely both- is clozapine. ersome to the patient and others but does not usually require discontinuation of treatment. 80–84.[80 (D), 81 (E), 82 (H), 83 (C), 84 (G)] Although Often, behavioral measures are sufficient, such fluoxetine, trazodone, paroxetine, citalopram, as the use of lozenges or placing a towel on and sertraline are all antidepressants, paroxe- the patient’s at night. Anticholinergic tine is the only such medication with a short agents such as propylthiouracil often help to enough half-life that it produces a significant reduce the volume of saliva. Propranolol is withdrawal syndrome after just 2 days. useful in the treatment of akathisia and aman- Haloperidol, clozapine, risperidone, and olan- tadine may be helpful in the treatment of zapine are all useful in the treatment of schizo- parkinsonian symptoms. phrenia, but only haloperidol and risperidone, of these choices, are available in a depot injection. 88. (D) Patients on clozapine often develop a per- Risperidone is preferable because it is an atypi- sistent sinus tachycardia that does not require cal with a better side effect profile. Lithium and cessation of treatment. Often, the tachycar- divalproex sodium are both used to treat bipo- dia resolves without necessitating further lar disorder, but it is lithium that causes nephro- intervention; however, should it persist, it may genic diabetes insipidus, which is suggested by be treated with the beta-antagonist propranolol. Answers: 75–94 119

Clozapine has significant alpha-receptor block- 92. (A) The need for increasing amounts of a drug age, which often causes orthostatic hypoten- to achieve the same effect or a diminishing sion, requiring a gradual titration of the dose. effect achieved through the use of the same Because of this, labetalol would not be appro- amount of a drug describes tolerance. Withdrawal priate as it may exacerbate this side effect. is a syndrome of behavioral, emotional, and Benztropine is an anticholinergic agent that physiologic signs and symptoms that occur in a will not help the tachycardia. There is no reason setting of the discontinuation of a drug. to check creatinine phosphokinase levels, which Dependence and abuse both require more his- might be expected to be high in NMS. tory and clinical information than is offered in this question. Dependence on a drug requires 89. (B) This patient presents with signs and symp- that its use causes significant impairment, toms suggestive of a first manic episode. Lithium anguish, or devastation to the individual and, and divalproex sodium remain the mainstay of despite this, the patient continues using. The initial and maintenance treatment of bipolar I presence of withdrawal on discontinuation and disorder. The use of anticonvulsants such as tolerance frequently help make a diagnosis of gabapentin and topiramate are being utilized substance dependence. Abuse is similar in that for bipolar disorder but they are not drugs of use causes significant problems for the patient, first choice. Neuroleptics, such as haloperidol, but use of the drug is generally more infre- and benzodiazepines, such as clonazepam, quent than is found in dependence and conse- may be used with lithium or divalproex quently does not involve tolerance and sodium initially; however, they are not rou- withdrawal symptoms. tinely used as maintenance treatment. 92. (D) The patient is currently suffering from a 90. (E) The neurovegetative symptoms can be major depressive episode but her significant remembered by a mnemonic, SIGECAPS, family history of bipolar is likely to increase developed by Dr Carey Gross: Sleep disorder, her risk for mania as well. Antidepressants Interest deficit (anhedonia), Guilt, Energy such as venlafaxine or even ECT may pro- deficit, Concentration deficit, Appetite disor- voke a manic episode in vulnerable patients. der, Psychomotor retardation, and Suicidality. Lamotrigine, an antiseizure medication, has Such symptoms tend to respond first with anti- been approved and recommended for the treat- depressant treatment. This is why it is a ment of bipolar depression. Although atypical common wisdom in psychiatry that the person antipsychotics (e.g., aripiprazole) and mood who begins to recover from depression may be stabilizers (e.g., carbamazepine) may also be at greater risk for suicide than when he is at the used, they are not likely to be as effective as height of his depression. As he starts to func- lamotrigine in monotherapy for depression. tion better physiologically, he may be more capable of carrying out a suicidal impulse. 93. (A) All of the second-generation antipsychotics have a lower incidence of TD when compared 91. (A) All of the second-generation antipsychotics to the typical antipsychotics. Although they all have a lower incidence of TD when compared have warnings regarding the increased risk of to the typical antipsychotics. Although they all diabetes and hyperlipidemia (metabolic syn- have warnings regarding the increased risk of drome), they are believed to cause it at differ- diabetes and hyperlipidemia (metabolic syn- ent rates so this is often a factor in choosing a drome), they are believed to cause it at different particular antipsychotic medications. The least rates, so this is often a factor in choosing a par- likely are aripiprazole and ziprasidone fol- ticular antipsychotic medications. The least lowed by risperidone and quetiapine. The most likely are aripiprazole and ziprasidone followed likely are olanzapine and clozapine. by risperidone and quetiapine. The most likely are olanzapine and clozapine. 94. (D) ECT has relatively few contraindications and in some cases is preferred for its rapid 120 4: Somatic Treatment and Psychopharmacology

onset of action. However, because of the car- Dystonias are uncontrolled muscle contractions. diovascular effects of ECT, a history of a recent TD is a rhythmic, involuntary movement of MI (within the past 6 months) is a relative the tongue, jaw, trunk, or extremities appearing contraindication. Another relative contraindi- months to years after the initiation of neu- cation is the presence of a clinically significant roleptic therapy. intracranial space-occupying lesion because of the risk of brain stem herniation. ECT may be 98. (D) Although there are various side effects of performed during pregnancy. The most common lamotrigine, the most dangerous is that of a complaints patients have following ECT are severe rash, which can develop into Stevens- impairments in both anterograde and retrograde Johnson syndrome. Beginning at a low dose memory. Although most memory problems and slowly tapering reduces the risk signifi- resolve, some may persist indefinitely. cantly. Aplastic anemia is a rare side effect of carbamazepine. Hypertension can occur in 95. (A) Atypical, or second-generation, antipsy- higher doses of venlafaxine. Short-term memory chotics are being used first line in the treat- loss is not uncommon after receiving ECT. TD is ment of schizophrenia. While they are likely at a long-term side effect of antipsychotics although least as effective as typical neuroleptics for the much less likely with newer medications such as positive symptoms of schizophrenia (halluci- aripiprazole. nations, delusions, paranoia, disorganization), they are believed to be more effective for the 99. (A) All of these are types of dystonic reactions negative symptoms (such as anhedonia, amo- that are induced by the use of neuroleptics. tivation, flat affect, etc). The newer antipsy- Opisthotonos, also known as arc de cercle, is a chotics can also cause significant weight gain in spasm of the neck and back that causes the some patients. Only clozapine has been patient to arch forward. Torticollis is a spasm of demonstrated to reduce suicidality in individ- the neck muscles that usually brings the neck to uals with schizophrenia. one side or another but can also pull forward or backward. Pleurothotonos, also known as Pisa 96. (A) Beginning quetiapine at a low dose and syndrome, is a leaning posture induced by the gradually increasing while simultaneously spasm of the torso muscles. Oculogyric crisis is tapering haloperidol (a cross-taper) is the most simply the spasm of the extraocular muscles. appropriate switching strategy. Starting at the The most alarming of these dystonias is laryn- target dose may cause significant side effects gospasm, which is the spasm of the muscles con- such as sedation and orthostasis, which may trolling the tongue and the throat. This can lead lead to poor compliance. Discontinuing the to respiratory distress. haloperidol completely or tapering it before the quetiapine is at a therapeutic level may 100. (B) Although any of the listed antidepressants lead to a relapse or worsening of psychotic will treat her depression adequately, only dulox- symptoms. etine, a combined serotonin-norepinephrine reuptake inhibitor, which has demonstrated effi- 97. (E) A subjective feeling of restlessness and a cacy in and been approved in the treatment of need to constantly move about and pace is both depression and diabetic neuropathy. Given known as akathisia, a common side effect of her major depressive disorder, reassurance neuroleptic medication that can occur soon would not be appropriate or efficacious. after such medications are started. The other choices listed are also neuroleptic side effects. 101. (A) With the exception of mirtazapine, bupro- Parkinsonism refers to symptoms evoked by pion, and nefazodone, the antidepressants all neuroleptics that are commonly seen in can cause significant sexual dysfunction, includ- Parkinson disease. Parkinsonian symptoms ing decreased libido, erectile dysfunction, and include tremor (a 3–6-cycle/second motion), anorgasmia. Given the recurrent nature of his akinesia (or bradykinesia), and rigidity. illness, no treatment would likely lead to a Answers: 95–121 121

relapse of his major depression, and restarting Tolerance (C) is the need for higher doses of a sertraline would be likely to cause the same drug needed for intoxication. Abuse (D) is problems and lead to noncompliance. drug use in spite of adverse consequences related to the substance. Dependence (E) is a 102. (C) Satyriasis is a man’s insatiable need for physical or psychological need to continue sexual gratification. Hypervigilance is often used taking a drug. Formication (F) is the feeling of in anxiety disorders and is simply an increased bugs crawling on the skin. Intoxication (G) is awareness of internal or external stimuli. represented by behavioral, cognitive, or per- Hypermnesia is the ability to recall detailed ceptual changes taken in relationship to drug material that is not usually available to recall. ingestion. Hallucinogens (H) are a group of Sexual sadism and sexual masochism are deriving drugs administered by various routes that sexual pleasure from causing or receiving cause perceptual changes. Posthallucinogen per- mental or physical abuse respectively. ception disorder (I) is also known as a flashback. It is characterized by a distressing return of 103. (B) Priapism is a painful, prolonged erection perceptual changes without the ingestion of that can be seen as an uncommon side effect of hallucinogens. Amotivational syndrome (J) is a trazodone in men. Parapraxis, a slip of the lack of drive or motivation usually associated tongue, is also known as a Freudian slip. with drug use, especially marijuana. Marijuana Nymphomania is insatiable sexual desire in a has its own classification in the Diagnostic woman and satyriasis is its counterpart in men. and Statistical Manual of Mental Disorders, Libido refers to sexual energy. Fourth Edition, Text Revision (DSM-IV-TR), cannabis (K), and is not considered among the 104. (A) Sexual dysfunction (e.g., decreased libido, hallucinogens. erectile dysfunction, anorgasmia) is not uncom- monly seen with SSRIs and other antidepres- 110–121. [110 (I), 111 (P), 112 (Q), 113 (B), 114 (L), sants in the treatment of depression. Many other 115 (N), 116 (G), 117 (R), 118 (J), 119 (M), 120 psychotropic medications such as antipsychotics (H), 121 (K)] Antipsychotics, also known as or lithium may also cause sexual dysfunction. neuroleptics, can cause a wide variety of side Bupropion is one of the few antidepressants, effects. These side effects can be broken down which causes little to no sexual dysfunction, into some of the following categories for an and, in fact, it has been found to reverse the side easier approach. The dopaminergic side effects effect when added to treatment. include those that are due to the blockade of the natural dopamine inhibition of prolactin 105. (D) Given the time it takes for antidepressant release from the anterior pituitary causing medications to become effective, ECT is con- galactorrhea (lactation; A) and gynecomastia sidered the treatment of choice when rapid (breast development; I). The dopaminergic side antidepressant action is desired (e.g., refusal effects also include those that arise from the to eat/drink, severe suicidality, catatonia). dopamine blockade in the basal ganglia caus- There are no absolute contraindications and it ing extrapyramidal and other side effects such is very safe during pregnancy as well. The as blepharospasm (spasm of the eyelids; C), other antidepressants have not been proven opisthotonos (spasm of the neck and back caus- safe in pregnancy and cognitive-behavior ther- ing an arched posture; D), and torticollis (spasm apy alone would not be appropriate for treating of the sternocleidomastoid; E). The blockade

a severe depression disorder. of D2 receptors in the basal ganglia can also cause pseudoparkinsonism disorder (L) resem- 106–109. [106 (F), 107 (J), 108 (A), 109 (D)] Inhalants bling Parkinson disease, characterized by (A) are volatile substances that have rapid- muscle rigidity and a short festinating gait, and onset intoxicating effects. Withdrawal (B) is a TD (N), a disorder of abnormal involuntary set of signs and symptoms, physical and psy- movements caused by prolonged use of neu- chiatric, that are associated with drug cessation. roleptics. Akathisia (subjective or observable 122 4: Somatic Treatment and Psychopharmacology

restlessness; M) and rabbit syndrome (rapid Double depression (Q) occurs when a major movement of the lips; F) are also highly dis- depressive episode is superimposed on dys- tressing side effects related to the dopaminer- thymic disorder. It is thought to affect as many gic system. as 40% of patients with MDD and to carry a Another side effect that can be caused, espe- poorer prognosis. Kindling (R), based on a cially by high-potency neuroleptics such as model of neural activity in which repeated sub- haloperidol, is NMS (G). This is characterized threshold stimulation of a given neuron results by hyperthermia, change in mental status, and in discharge of an action potential, is a theory increased muscle tone. It can lead to renal fail- that suggests that one decompensation can lead ure following dehydration and a rise in muscle to a second episode. breakdown products (creatine kinase levels are typically elevated), pulmonary complications, 122. (C) In the CNS, GABA is an inhibitory neuro- and death. transmitter. At the postsynaptic GABA-A Clozapine and chlorpromazine can both receptor, GABA facilitates chloride ion influx cause agranulocytosis (B), a dangerous side into a cell via a chloride channel resulting in effect that sometimes produces symptoms of inhibition of that neuron. Alcohol and benzo- sore throat and fever. The drug must be diazepines both allosterically modulate the stopped immediately. Constipation (O) is a com- GABA-A receptor to facilitate GABAergic inhi- plicating side effect of some antipsychotics with bition. Abrupt removal of alcohol or benzodi- anticholinergic effects. This troublesome side azepines after prolonged use results in a effect can often be relieved with use of a laxa- relative deficit of GABAergic inhibition, which tive. Orthostatic hypotension (K), a drop in blood can lead to anxiety, insomnia, delirium, and pressure while standing, is a side effect from seizures. GABA-B is the site of action of the alpha1-adrenergic blockade. This is especially muscle relaxant baclofen. Alcohol does not act troublesome with chlorpromazine and clozap- at the dopamine or serotonin receptors. ine and is best handled by raising doses slowly. Obstructive jaundice (H) is rare and occurs 123. (C) Alcohol withdrawal can take the form of mainly with chlorpromazine. Symptoms are minor withdrawal symptoms such as tremu- fever, nausea, malaise, and pruritus. Pigmented lousness, withdrawal seizures, or withdrawal retinopathy (J) is observed with uses of high delirium, formerly known as delirium tremens. doses of thioridazine and is best detected with Minor withdrawal symptoms can last for several a good ophthalmologic examination. Another days. Withdrawal seizures are rarely focal and side effect that can decrease compliance with usually occur within 48 hours after alcohol con- antipsychotics is retrograde ejaculation (P). Orgasm sumption ceases. Delirium tremens most often may be achieved but ejaculation is abnormal occurs within 96 hours of cessation of drinking; with retrograde propulsion of the semen. therefore, 5 days is the most correct response. CHAPTER 5 Psychological Treatment and Management Questions

DIRECTIONS (Questions 1 through 42): Each of (C) couples therapy the numbered items in this section is followed by (D) family therapy answer choices. Select the ONE lettered answer or (E) paradoxical therapy completion that is BEST in each case. 3. A 43-year-old man is chastised at work. When Questions 1 and 2 he comes home, his friend asks him how his day went. He responds angrily saying that a A 34-year-old married man comes into your office “real friend wouldn’t be so nosy.” What is the complaining of being “overcome” with discrete name of this defense mechanism? episodes of palpitations, sweating, shortness of breath, “choking,” abdominal discomfort, and feel- (A) denial ings of impending doom. He goes on to say that he (B) displacement was a football player in high school and was never (C) isolation of affect scared of anything. Now, he says, he is starting to (D) intellectualization avoid driving for fear of being locked in the car when he has one of these “attacks” and not being able to get (E) humor out. All of his friends say that he is “losing it,” and he wants to know if you think he is. 4. A 20-year-old woman diagnosed with border- line personality disorder thinks of her therapist 1. What would be the most appropriate response? as “the best person in the world.” The next week, the therapist announces that he will be (A) “Yes, you are.” going on vacation. The patient then says that (B) “No, you are not.” he is “the worst person in the world.” What is (C) “Why do you want to know?” the defense mechanism that allows people to (D) “What do you think?” see others as either all good or all bad? (E) Ask him what he means by “losing it.” (A) splitting (B) intellectualization 2. He goes on to say that he will not take med- (C) repression ication for these episodes. What therapy is gen- (D) devaluation erally most effective in treating panic disorder? (E) idealization (A) psychodynamic psychotherapy (B) cognitive-behavioral therapy

123

Copyright © 2007 by The McGraw-Hill Companies, Inc. Click here for terms of use. 124 5: Psychological Treatment and Management

5. A 16-year-old boy has just lost his mother and 8. A 32-year-old psychiatry resident is in psycho- father in a car accident. In your office, he talks analysis. He is lying on the couch and saying philosophically about death and its implica- whatever comes to mind. This is an example of tions. When asked how he feels about his par- what analytic technique? ents’ death, he responds by saying that “it is the (A) repression nature of things to pass away.” This patient’s inability to talk directly of his emotional expe- (B) speakeasy rience concerning personal losses is an example (C) countertransference of which defense mechanism? (D) transference (A) lying (E) free association (B) projection 9. A psychotherapist finds herself getting angry at (C) intellectualization a patient after the patient reports beating up his (D) denial little brother. The psychotherapist is exhibiting (E) suppression what phenomenon? (A) countertransference Questions 6 and 7 (B) transference A 44-year-old woman with schizophrenia is strug- (C) resistance gling with paranoia, auditory hallucinations, and (D) interpretation delusions. She lives with her mother but has a poor (E) extinction relationship with her. She tells you that everyone wants her to spend a lot of money and buy drugs. 10. In an acute inpatient psychiatric ward, the res- ident psychiatrist is in charge of leading a 6. Which of the following would be the best group consisting of patients admitted to the response? ward. In the morning, she asks each patient in (A) “Well, deciding for yourself is best.” turn to comment on his or her goals for treat- (B) “If you buy drugs, I’ll call the police.” ment. What is the name of this technique of (C) “Perhaps we should look at what your group psychotherapy? mother would think about that.” (A) go-around (D) “Why do you think everyone wants you (B) free association to do that?” (C) feedback (E) “No, they don’t.” (D) interpretation (E) resistance 7. You decide that supportive therapy would be helpful to this patient. Which of the following Questions 11 and 12 is a goal of supportive therapy? (A) personality change A 29-year-old health professional entered group ther- apy wanting to know why he was unable to stay in (B) correcting faulty ideas a long-term relationship with a woman. A handsome (C) the reestablishment of psychic man, he said that women found him attractive and he homeostasis through the strengthening did not understand why he should restrict himself to of defenses one person. However, he was bored with the atten- (D) exploring the feeling of meaninglessness tion and felt empty inside. At work, he was very in life competitive with his male colleagues. In the group, (E) investigating the freedom of individuals members pointed out to him that whenever he was Questions: 5–16 125 criticized by other men in the group, he made group psychotherapy. Which of the following advances toward women in the group. These reasons is most consistent with why this is true? advances sometimes continued outside the group, (A) Wounds to self-image while in the but he lost interest when he felt better. group can be examined supportively in individual therapy. 11. What is the name of this defense mechanism? (B) These patients need as much attention (A) idealization as they can get. (B) acting out (C) These patients deplore the attention of (C) abreaction others and need exposure to overcome (D) ventilation this. (E) catharsis (D) These patients constantly believe others are conspiring against them. 12. Compared with inpatient groups, which of the (E) These patients need much structure to following is a treatment objective most charac- occupy their time. teristic of outpatient groups? (A) crisis intervention Questions 15 and 16 (B) return to previous level of functioning A 29-year-old woman has been depressed for (C) personality change and reconstruction 2 months prior to seeking medical attention. She (D) stabilization of psychotic symptoms believes that nobody likes her even though she is (E) discussing current feelings regarding always cordial and there is nothing that she can do to alcohol detoxification and heart disease change the way other people perceive her. She adamantly refuses to take antidepressants. You believe that cognitive therapy is indicated for this patient. Questions 13 and 14

You have been assigned to lead various groups that 15. Which of the following cognitive profiles most help patients who suffer from personality disorders. reflects the profile seen in depression? To accomplish this task, you need to know which (A) negative view of self, experience, and personality disorders are appropriate for group psy- future chotherapy. In addition, you need to know which (B) fear of being fat and unattractive personality disorders benefit from combined indi- vidual and group psychotherapy. (C) concern about serious, insidious medical disorders 13. Which of the following personality disorders is (D) view of others as biased, manipulative, best treated with combined individual and and devious group psychotherapy because of intense trans- (E) fear of physical or psychological danger ferences and countertransference that develop in a single mode of treatment? 16. According to cognitive psychology, what are (A) borderline personality disorder automatic thoughts? (B) avoidant personality disorder (A) thoughts that impede free association (C) schizotypal personality disorder (B) thoughts that cannot be changed (D) antisocial personality disorder (C) thoughts at the edge of conscious (E) schizoid personality disorder awareness that regularly precede unpleasant feelings 14. Patients with narcissistic personality disorder (D) genetically determined thoughts are best treated with combined individual and (E) thoughts that every human ought to have 126 5: Psychological Treatment and Management

Questions 17 and 18 his daily activities causing him to be constantly late to other appointments. You decide to help the patient A 45-year-old man presents to the outpatient clinic with behavior therapy. handicapped by a fear of parking lots and fields. This fear started 4 months prior to this visit. At the 19. Which of the following does behavior therapy beginning of cognitive therapy, he says that his attempt to change? behavior is constantly being scrutinized and criti- cized by other people. He says that he cannot change (A) maladaptive thoughts underlying a his behavior because if he does others will think that maladaptive behavior he is a fool. (B) neurobiology (C) how a patient perceives the environment 17. What would be the most appropriate response (D) maladaptive behaviors seen in a from the therapist? particular syndrome (A) “That’s silly. People will not think you (E) underlying conflicts in an attempt to are a fool.” change behavior (B) “Yes, I can see that.” (C) “What makes you think that others are 20. Which of the following best describes behavior constantly scrutinizing your behavior?” therapy? (D) “How can you possibly think other peo- (A) a long-term assessment of relational ple care enough about you to constantly issues scrutinize your behavior?” (B) a set of procedures designed to change (E) “Well, maybe you are a fool. Have you maladaptive thoughts ever thought of that?” (C) a set of procedures designed to enhance the learning of adaptive behaviors and 18. Which of the following statements best describes the unlearning of maladaptive behaviors the view of cognitive therapy regarding the role (D) hypnosis of in psychiatric disorders? (E) a technique used to resolve unconscious (A) Cognitive distortions are always the conflicts cause of various psychiatric syndromes. (B) Cognitive distortions are never the 21. A 39-year-old man presents to your office com- cause of various psychiatric syndromes. plaining of headaches, which you diagnose as (C) Correcting cognitive distortions is tension headaches. As part of this patient’s always sufficient to correct treatment, you decide to use behavior therapy. symptomatology. Which of the following behavior techniques is used in the reduction of tension headaches? (D) Although cognitive distortions are not necessarily the ultimate cause of psychi- (A) stress reduction atric syndromes, correcting distortions (B) stimulus control can be an effective intervention in (C) systematic desensitization psychiatric syndromes. (D) aversive stimuli (E) Correcting cognitive distortions never (E) negative reinforcement completely alleviates symptoms. Questions 22 and 23 Questions 19 and 20 A 23-year-old woman broke up with her boyfriend of An 18-year-old man presents to your office com- 8 months 1 month before presenting to your office plaining that he cannot pass a movie theater without complaining of depression. Currently, she is just start- stopping, going inside, and buying candy. This ing to date someone else. Given that her episode behavior is troublesome to him and interferes with Questions: 17–26 127 meets criteria for a major depressive episode, you (A) It is difficult for patients to sustain a decide to use both medication and interpersonal transference and keep their defenses psychotherapy. lowered with fewer sessions. (B) Analytic patients are more seriously 22. Which of the following is the major focus of mentally ill and therefore need more interpersonal psychotherapy? time. (A) character change (C) Analytic patients are typically more (B) clarifying communication patterns seriously mentally ill and therefore need less time. (C) interpreting transference (D) Analysis is based on a supportive model (D) hypnosis of therapy. (E) pointing out resistance (E) Analysis is a quick therapy. 23. Which of the following best represents the 25. What is the major tool of treatment in psycho- hypothesis on which interpersonal psycho- analysis? therapy rests? (A) interpretation of transference (A) In addition to any biological diathesis, certain interpersonal problems (B) altering maladaptive behaviors predispose one to depressive disorders, (C) altering cognitive errors complicate their course, or interfere (D) interpreting with recovery. (E) solving interpersonal problems (B) Unconscious conflicting psychic processes need to be resolved. Questions 26 and 27 (C) Maladaptive behaviors may be changed A 50-year-old man is in psychoanalysis because he by examining cognitive errors. feels ineffective in his professional and personal (D) Maladaptive behaviors may be changed relationships. by positive reinforcement. (E) Maladaptive behaviors may be changed 26. Which of the following best defines the analytic through the power of suggestion. term dynamic?

Questions 24 and 25 (A) the notion that all mental phenomena are the result of a continual interaction A 22-year-old college student presents complaining of forces that oppose one another of having no goals in life, not doing as well as he (B) the notion that mental phenomena could in school, and desiring but having no serious reveal themselves at various levels in long-term romantic relationship. He has not chosen the psyche a major and says that his indecision is paralyzing (C) the notion that personality and its him from moving on with the rest of his life. After subsequent development has a historical discussing various forms of psychotherapy with the aspect patient, you recommend psychoanalysis and he (D) the notion that the psyche is organized agrees. into functional units called the id, ego, and superego 24. Why does it take four or five treatment ses- sions per week to have a successful analysis? (E) fun, interactive, and pleasant 128 5: Psychological Treatment and Management

27. How are dreams useful in psychoanalysis? proceedings and are currently still living together. Also, 1 year ago, the patient’s younger brother was (A) Dreams are interpreted by the analyst to born. You suspect that there are significant environ- subliminally influence the minds of mental stressors that are contributing to this patient’s patients. depression and decide that a trial of family therapy (B) Dreams are used to alter cognitive is warranted. errors. (C) Dreams provide information about 30. Family therapy is based on which of the fol- psychic conflicts. lowing theories? (D) Dreams are used to change behavior. (A) Individual therapy is inappropriate for (E) Dreams represent the conscious frame- someone in a family. work for behavior. (B) When families are dysfunctional, look for the individual who is causing the Questions 28 and 29 trouble. You are seeing a 50-year-old man who suffers from (C) Families that live together fight together. generalized anxiety disorder for cognitive-behavioral (D) The familial dynamic can help to therapy. He is having a hard time with the restruc- explain individual psychopathology. turing of maladaptive thoughts. You decide that hyp- (E) Role playing can explore a patient’s nosis may be helpful as an adjunctive therapy for interpersonal relationships, personality this patient. makeup, and conflicts.

28. Which of the following is most needed for a 31. After further questioning, it becomes clear that successful hypnosis? the patient’s mother is also depressed. Which of (A) a skillful hypnotist the following is the greatest risk factor for (B) a patient who is highly responsive to depression for women with chronic depression? suggestion (A) marital status and satisfaction (C) a quiet room (B) being single (D) a skilled induction technique (C) age over 30 years (E) creative ability (D) low income (E) high-fat diet 29. Hypnosis is best used as an adjunctive ther- apy within a psychotherapeutic relationship. Questions 32 and 33 In which of the following is hypnosis a relative contraindication? A 9-year-old boy with attention deficit hyperactivity disorder (ADHD), predominantly hyperactive type, (A) intractable hiccups presents to your office accompanied by his mother. (B) anxiety disorders He is currently on methylphenidate (Ritalin) and (C) pain disorder they both wonder if there is something else that can (D) treatment of warts (specifically verrucae be done to help the patient. You recommend behav- planae juvenalis) ior therapy. (E) psychotic disorders 32. Which of the following best illustrates how Questions 30 and 31 behavioral therapy would help this patient?

A 14-year-old girl is brought by her mother to see (A) reinforcing attention to various tasks you. The mother says that the patient has been necessary for learning and ignoring depressed for the last year. You learn that 1 year disruptive behaviors ago, the patient’s mother and father began divorce (B) interpreting a transference Questions: 27–37 129

(C) positively reframing a negative (A) correcting cognitive errors experience (B) interpreting many different conflicts (D) focusing on the patient’s interpersonal (C) modifying maladaptive behaviors relationships (D) interpreting resistance (E) focusing on maladaptive communication (E) identification of a focal conflict techniques in the family Questions 36 and 37 33. Which of the following represents a behavioral tool that is helpful for patients with ADHD? A 20-year-old man is currently in college and has completed his first semester of the sophomore year. (A) a well-timed interpretation of maladap- Halfway into his second semester, his grades drop tive behaviors from As to Fs over a 3-month period. He becomes (B) a report card of various behaviors that increasingly isolated and paranoid, believing that can be rewarded at home by parents the government is after him because he has solved all (C) clozapine (Clozaril) theological problems through direct communication (D) methylphenidate with God. He is started on olanzapine (Zyprexa) 10 mg (E) positively reframing a negative by mouth (PO) daily and is tentatively diagnosed experience with schizophreniform disorder. He recovers after 4 months but has another relapse 6 months later in Questions 34 and 35 the absence of elevated, irritable, or depressed mood. He is given a diagnosis of schizophrenia. A 25-year-old man in his fourth year of medical school presents to your office complaining of not 36. Which of the following is most indicated for the being able to remain in a relationship with a girl- treatment of this patient’s schizophrenia? friend for longer than 3 months. He says that he would like an insight-oriented therapy, and you agree (A) psychoeducation alone that this form of therapy would help him. You decide (B) medication alone to treat this patient with brief psychodynamic psy- (C) family therapy chotherapy. (D) psychoanalysis (E) a combination of psychoeducation, 34. Which of the following factors best predicts a medication, and family therapy positive outcome with brief psychodynamic psychotherapy? 37. Which of the following therapies is usually (A) gender not indicated for the treatment of psychotic (B) age disorders? (C) motivation for change (A) behavioral therapy (D) marital status (B) psychoanalysis (E) socioeconomic status (C) supportive psychotherapy (D) family therapy 35. Which of the following most distinguishes brief (E) group psychotherapy psychodynamic therapy from other forms of therapy? 130 5: Psychological Treatment and Management

Questions 38 and 39 (C) Psychoanalytic theory is the most important underpinning of this A 33-year-old woman presents to the emergency treatment. department saying that she wants to kill herself. She (D) It is likely that he is in treatment for is hopeless and feels helpless. She has a plan to over- depression. dose on pills that she has stockpiled at home. (E) He is being confronted directly with the 38. Other than severe hopelessness, which of the feared situation and will eventually following cognitive difficulties do suicidal grow immune to the anxiety it patients characteristically exhibit? produces.

(A) personalization 42. During his session, a patient receives mainly (B) begging the question empathetic reassurances from his psychiatrist. (C) rigid, black-or-white, either-or thinking This best exemplifies which type of psycho- that limits problem-solving options therapy? (D) selective abstraction (A) existential (E) circular reasoning (B) dynamic (C) biological 39. Which of the following forms of psychotherapy (D) supportive would be most appropriate to help this patient think of alternatives to suicide? (E) cognitive-behavioral

(A) behavioral therapy DIRECTIONS (Questions 43 through 49): Each set (B) cognitive therapy of items in this section consists of a list of lettered (C) interpersonal psychotherapy headings followed by several numbered words or (D) hypnosis phrases. For each numbered word or phrase, select the ONE lettered option that is most closely asso- (E) paradoxical therapy ciated with it. Each lettered option may be selected once, more than once, or not at all. 40. A psychiatry consult is called on a patient who, in a variety of ways, is making the nursing staff angry at him. He became enraged during this Questions 43 and 44 hospitalization when he was told that he has a (A) conditioning terminal illness. What defense mechanism is (B) insight being demonstrated? (C) therapeutic alliance (A) denial (D) schema (B) sublimation (E) real relationship (C) intellectualization (F) anxiety (D) projective identification (G) posttraumatic stress disorder (E) reaction formation (H) projection (I) transference neurosis or transference 41. Which of the following statements best char- (J) countertransference acterizes a 34-year-old man being treated with systematic desensitization? (K) projective identification (L) biofeedback (A) Reciprocal inhibition is key to his treat- (M) undoing ment. (N) negative reinforcement (B) Allowing this patient to use his (O) punishment is far more effective than exposing him to the actual situation that (P) operant conditioning engenders anxiety. (Q) splitting Questions: 38–49 131

43. You are seeing a 26-year-old woman who com- 45. You prescribe disulfiram (Antabuse) to a 56- plains of migraines. She is otherwise healthy. year-old patient. You explain to her that you would like to try a technique in which you give her information 46. To train a 4-year-old boy to say “thank you,” on muscle tension and temperature. his mother gives him a sweet whenever he does. 44. You are seeing a 45-year-old man in analysis four times per week. After about 6 months, the 47. To train a 4-year-old boy not to bite his class- man tells you that you are the center of his mates, his mother puts soap in his mouth universe. whenever he does.

Questions 45 through 47 Questions 48 and 49 (A) negative reinforcement (A) Sigmund Freud (B) positive reinforcement (B) Franz Anton Mesmer (C) desensitization (C) Josef Breuer (D) avoidance (D) Jean-Martin Charcot (E) extinction (E) Friedrich Nietzsche (F) conditioned avoidance (G) flooding 48. Who is considered the father of psychoanalysis? (H) reciprocal inhibition 49. Who is considered the father of modern hypnosis? (I) hypnosis (J) modeling Answers and Explanations

1. (E) Although it would be tempting to reassure 3. (B) Displacement is a defense that transfers a this patient that he is not “going crazy,” it is feeling about, or a response to, one object onto better to find out what he specifically means by another object. Denial is a defense that keeps “losing it.” He could mention a specific fear out of conscious awareness an aspect of exter- that would be important for you to help him nal reality or subjective distress that is too clarify or address. Asking the patient why he uncomfortable for the person to accept. Isolation wants to know or asking “what do you think” of affect involves detachment of feelings from a is easily interpreted as an insulting and patron- particular idea or experience. Intellectualization izing statement that does nothing to enhance is a defense in which the individual favors doctor-patient trust. abstract thinking over dealing with the dis- turbing feelings of an idea or experience. 2. (B) Cognitive-behavioral therapy is actually the Humor is considered a defense mechanism that combination of two therapies: cognitive therapy emphasizes the amusing or ironic aspects of and behavioral therapy. Cognitive therapy the stressor. would help the patient to think about his symp- toms as misinterpretations. For example, his 4. (A) Splitting occurs when an individual is feelings of impending doom, although fright- unable to see others moderately; that is, ening, do not actually predict the end of his life. people’s actions are either all good or all bad. An example of behavioral therapy is exposure. Intellectualization is a defense that utilizes The premise is to expose patients to feared sit- excessive abstract thinking or generalizations to uations while maintaining a safe atmosphere. control threatening emotions. Repression is a For example, a therapist could accompany the defense that expels disturbing wishes, thoughts, patient to the car to precipitate symptoms. In or experiences from conscious awareness. The the controlled atmosphere, the patient can sep- feeling component may remain but be divorced arate the connection of feared symptoms with from the idea. Devaluation is a defense that the specific environment. In paradoxical ther- attributes excessive negative qualities to another. apy, developed by Bateson, the therapist sug- Idealization is a defense that attributes excessive gests that the patient engage in the behavior positive qualities to another. with negative connotations (e.g., a phobia or compulsion). The efficacy of cognitive-behavioral 5. (C) This is an example of intellectualization, a therapy is more established than that of para- defense that utilizes excessive abstract thinking doxical therapy. Couples therapy is not indi- or generalizations to control threatening emo- cated here because we have no evidence that the tions. Lying is not considered a defense mech- root of the patient’s disorder is a conflict with a anism. Denial is a defense that keeps out of significant other; similarly, family therapy is conscious awareness an aspect of external real- not indicated. Psychodynamic psychotherapy is ity or subjective distress that is too uncomfort- a form of therapy informed by psychodynamic able for the person to accept. Projection is a processes. defense whereby a thought, feeling, or idea

132 Answers: 1–10 133

that is unacceptable to the person is falsely analyst in the setting of psychoanalytic psy- attributed to another person. Suppression is a chotherapy. Countertransference is the ana- defense in which one intentionally avoids lyst’s response to this. However, these terms thinking of distressing thoughts, feelings, ideas, have come to mean the transferring of emo- or experiences. tions and feelings that one has from one’s past to the physician or care provider in the case of 6. (D) The best strategy in any communication is transference and the physician toward the to try to gain clarity. Asking this patient why she patient in countertransference. A speakeasy believes what she does may lead nowhere or it was an underground bar during Prohibition. may very much help understand this thought. Choice A may be helpful in some situations and 9. (A) Countertransference is the conscious or uncon- certainly has a positive ring to it. However, this scious emotional reaction of the therapist to the patient’s comment is an implicit question. She patient. Transference is the conscious or uncon- is asking for help and genuinely does not know scious emotional reaction of the patient to the what to do. Hiding behind pithy maxims is not therapist based on a past relationship the patient helpful. Choices B and C are coercive and block had with a significant other person. Resistance is communication. Giving supportive guidance is unconscious opposition to full disclosure of feel- appropriate in this situation, but coercion is ings or ideas. Interpretation is insight offered by never appropriate in a therapeutic relationship. the therapist regarding patterns of thought or Choice E is not coercive but blocks communi- behavior. Extinction is the reduction in frequency cation. This may be the content that needs to be of a learned response as a result of the cessation heard but it abruptly terminates communica- of reinforcement, and is a term frequently used tion and could sacrifice a more insightful, in learning theory. empowering response. 10. (A) The go-around is a technique frequently 7. (C) The goal of supportive therapy is not to used in inpatient groups. By having each break down the defenses to achieve a more opti- patient comment on a plan, theme, mood, mal level of functioning. This would be a long- behavior, or idea, active participation by all term goal of psychodynamic psychotherapy. members is ensured, and other members of the Supportive therapy’s goal is the strengthening of group can benefit from various ideas pre- current defense mechanisms. Through support- sented. The go-around also helps patients to ive care, the patient can learn to cope with diffi- feel that they are not the only ones with diffi- cult problems using already established abilities. culties. Free association is used in psychoana- Personality change is a goal of psychodynamic lytically oriented groups. These groups are psychotherapy, and correcting faulty ideas is the largely outpatient groups whose members can expressed goal of cognitive therapy. Exploring tolerate and effectively use this technique. the feeling of meaninglessness in life and inves- Members are encouraged to be spontaneous tigating the freedom of individuals are goals of with their thoughts and feelings, and the ther- existential psychotherapy. apist assists in pointing out common themes and possible connections. Feedback, a cardinal 8. (E) This is an example of free association. Freud feature of group psychotherapy, comes from believed that all mental activity is causally peers as well as from the leader. Feedback from related, meaning that free associations are not peers in the group can be a powerful stimulus truly random. They follow logically, but the for insight and change. Interpretation is the missing connection is being held down or offering of insight that allows patients to gain repressed unconsciously by defenses. Repression an understanding of conflicts and motivations is a defense mechanism that expels disturbing that contribute to their behavior. Resistance wishes, thoughts, or experiences from conscious is the unconscious opposition to full disclo- awareness. Transference, in strict terms, is the sure of thoughts or feelings. reexperiencing of past experiences with the 134 5: Psychological Treatment and Management

11. (B) Acting out is characterized by behaviors that magical thinking styles. Unlike the patient with distract a patient who needs to relieve tension or schizotypal personality disorder, the patient gratify needs through activity rather than ver- with avoidant personality disorder longs for balization of uncomfortable feelings in the relationships yet is morbidly shy and fears group. Idealization is a defense that attributes them. Schizoid personality disorder is marked excessive positive qualities to another. Abreaction by bizarre behavior and withdrawal. is a process by which repressed material, par- ticularly a painful experience of conflict, is 14. (A) This is one of the strongest reasons for com- brought back to consciousness. In the process, bining the two forms of therapy. In individual the person not only recalls the experience but therapy, patients can reflect on what their feel- relives the experience, which is accompanied by ings are in the group and the responses of others the appropriate emotional response. Ventilation to them from the group. Choice B is true of this (also known as self-disclosure) is the expression of personality disorder but is not a reason to com- suppressed feelings, ideas, or events to other bine therapies. Choices C, D, and E are not char- group members; this relieves a sense of sin or acteristic of narcissistic personality disorder. guilt. Catharsis is expression of ideas, thoughts, and suppressed material that is accompanied 15. (A) The cognitive triad of depression is a belief by an emotional response that produces a state that oneself is ineffective, the world is a hostile of relief in the patient. place, and nothing one can do will ever change anything. The other choices are cognitive pro- 12. (C) Personality change and reconstruction are files typical of various psychiatric syndromes. goals of psychoanalytically oriented outpatient Fear of being fat and unattractive is typical of groups. This type of significant change is vir- anorexia nervosa and bulimia nervosa. Concern tually impossible in an inpatient group. Crisis about serious, insidious medical disorders is intervention, return to previous level of func- typical of hypochondriasis. Seeing others as tioning, and preparation for outpatient therapy biased, manipulative, and devious is typical of are all objectives for inpatient group psy- paranoid states. Fear of physical or psycholog- chotherapy. Certain outpatient groups can also ical danger is typical of anxiety disorders. help with these objectives so that inpatient hos- pitalization may be averted. If a patient has 16. (C) Automatic thoughts are constant ways of per- heart disease and needs alcohol detoxification, ceiving situations and oneself in relation to the inpatient detoxification is indicated. environment. They can be thought of as a given way of evaluating events. Cognitive therapy 13. (A) Patients with borderline personality disor- aims to identify negative automatic thoughts der develop intense transferences in individual and then change them so that individuals do treatment. This can impede the work of psy- not constantly view themselves or their envi- chotherapy. Transference is usually diluted in ronment in a negative light. Automatic thoughts group psychotherapy because of the number of are not genetically determined. They have not people. The group can offer stability and mod- always been present and can be changed. There eration to the extreme ways of seeing others is no moral imperative from automatic thoughts that distort borderline perceptions. The com- and they do not impede free association. bined modes of treatment, individual and group, offer the patient with borderline per- 17. (C) One of the reasons that this is the best of the sonality disorder a higher probability for per- possible responses is that it is a question. sonality change. Patients with borderline Patients with cognitive distortions often take personality disorder are known for their their view of the world as a given. They do not intense personal relationships, self-destructive question their perspectives. Asking a question behaviors, anger management issues, and poor invites patients to examine their perspectives senses of self-image. Schizotypal patients are and thereby become open to the possibility that known for their quirkiness and seemingly their thoughts may be inaccurate. Choices D Answers: 11–24 135

and E are also questions but are authoritarian headache. They are equally effective in accom- and punitive not to mention insulting. To have plishing this goal. Stimulus control is used in the best outcome in any form of therapy, there the treatment of insomnia. Stimulus control must be a good relationship or alliance between procedures are used to narrow the stimuli asso- the patient and therapist. Choices A and B have ciated with sleep onset. Systematic desensiti- not been supported by any evidence. Choice B zation is a technique used in asthma, in which is ambiguous—to which part of the question are the patient becomes relaxed and visualizes typ- you responding? The patient may not think you ical situations that precipitate asthma attacks. are awkwardly trying to be supportive but may Substance withdrawal syndromes are exam- believe that you are insulting him. Choice A is ples of the aversive stimuli and consequently dangerous—a patient should not be called negative reinforcement that lead to decrease in “silly.” substance use.

18. (D) This question is meant to emphasize that 22. (B) The major focus of interpersonal psy- although correcting cognitive distortions may chotherapy is communication analysis. Indirect, have symptomatic benefit, it should not be unclear interpersonal communication patterns inferred that cognitive distortions are the ulti- are identified in the course of therapy. These mate cause of the psychiatric syndromes. The communication patterns are then altered, for other choices are wrong because of the “always” instance, by role playing or asking the patient to or “never” used in the question. The other try out different forms of communication choices can be true at times but are not always during the session. Interpreting transference, true or false. pointing out resistance, and character change are more the focus of psychodynamically ori- 19. (D) Behavior therapy is focused on behaviors, ented psychotherapy. Hypnosis is a sleep-like not on possible underlying factors that cause state induced by a therapist, which has been of behaviors. Behavior therapy is a here-and-now some use in uncovering psychodynamic approach to maladaptive behaviors. It may uti- processes and in behavioral modification such lize information surrounding the emergence of as stopping smoking. It is not part of interper- a particular maladaptive behavior to construct sonal psychotherapy. a behavior modification plan. Although behav- ior therapy can and does change neurobiology, 23. (A) Interpersonal psychotherapy focuses on this is not its expressed goal. Changing mal- the patient’s role and communication style adaptive thoughts underlying a maladaptive within important relationships and how that behavior and changing how the patient per- role may be causing or complicating the ill- ceives the environment involve perspectives ness, such as depression. If one or two specific more appropriate for cognitive therapy. Changing ways of communicating can be altered so that underlying conflicts in an attempt to change a relationship improves, the depressive illness behavior is a description of the goals of psycho- also improves. This therapy also has the added dynamic therapy. benefit of durability; once a communication pattern is improved and the patient better 20. (C) Behavior therapy is a set of procedures understands how his or her role in a relation- designed to enhance the learning of adaptive ship may create distress, the depressive illness is behaviors and the unlearning of maladaptive not as likely to recur. Although other processes behaviors. In a strict sense, it is not designed to such as those in the remaining answers may be uncover or alter unconscious conflicts using used, choice A best reflects the foundation of hypnosis or other methods. It is also a short- the therapy. term therapy. 24. (A) It takes 4–5 days a week to successfully 21. (A) Stress reduction and biofeedback are used complete an analysis because it is difficult for to reduce the frequency and the severity of each patients to sustain a transference and keep their 136 5: Psychological Treatment and Management

defenses lowered with fewer sessions. There is more than others; a patient can be taught self- no correlation between severity of mental ill- hypnosis. Needless to say, a skillful hypnotist ness and length of therapy. Psychoanalysis and induction technique are helpful. Neither a tends to be among the longest therapies and quiet room nor creative talent is a factor. involves the process of free association in which the patient lies on a couch, facing away 29. (E) Hypnosis should not be used in psychotic from the therapist, and says whatever comes to disorders. Patients with schizophrenia often mind. Psychoanalysis is based on an expressive have cognitive disorders or paranoid symp- or exploratory model of therapy, as opposed to toms that make the use of hypnosis problem- a supportive model aimed at stabilizing crisis atic. Often, these patients do not remember the and reestablishing a baseline level of function. changes that were supposed to be induced through the hypnosis, and there is controversy 25. (A) Psychoanalysis may accomplish many tasks, over the extent to which patients with schizo- including changing maladaptive thoughts and phrenia can be hypnotized. Hypnosis has been behaviors and “solving” interpersonal prob- used successfully for the treatment of anxiety lems, but these are the consequences of analysis, disorders and pain. There is also literature indi- not the instrument of change. Altering cogni- cating its efficacy in the treatment of warts. tive errors is more in the domain of cognitive therapy. The major tool of psychoanalysis is the 30. (D) Family therapy is based on the idea that careful interpretations of the transference neu- the interactional dynamics of the family can rosis. Certainly, interpreting dreams can lead to help to explain individual psychopathology. deep insights into a patient’s thoughts and feel- Individual psychotherapy can be very useful ings, but it is an adjunctive tool. in conjunction with family therapy. Role play- ing can explore a patient’s interpersonal rela- 26. (A) The term dynamic or psychodynamic refers tionships, personality makeup, and conflicts. to the notion that all mental phenomena are the However, this is not the basis for family ther- result of a continual interaction of forces that apy; it is the basis for psychodrama. oppose one another. Choices B, C, and D are related ideas; specifically, the notion that the 31. (A) A dysfunctional marriage is a greater risk psyche is organized into functional units called factor for depression than being single. Certainly, the id, ego, and superego was a construction many people are content and single. The other formulated by Freud. Fun, interactive, and three answers are not correlated risk factors pleasant is close to a lay definition of dynamo. for depression. Some studies have shown age greater than 45 to be a risk factor for depression in 27. (C) Dreams provide information about psychic general. Others have shown age greater than 65. conflicts. Dreams can lead to a change in behav- ior, but they are not used per se in analysis to 32. (A) Behavioral therapy helps patients who change behavior or cognitive errors. Dreams suffer from ADHD to derive satisfaction from are used to uncover psychic conflicts, which, learning and from ignoring disruptive behav- when interpreted appropriately (not sublimi- iors. The other answers can also be very helpful; nally), can lead to a productive change in however, they are not examples of behavioral behavior. According to the psychoanalytic therapy. theory, dreams represent the unconscious workings of the mind, not the conscious ones, 33. (B) A behavioral tool that is used quite effec- and may be symbolic in content. tively is a report card. This tool can function as a bridge between school and home. In addi- 28. (B) Far and away the most important factor in tion, a report card can serve as a positive incen- hypnosis is a patient who is highly responsive tive to good behavior when the report is good to suggestion. Some people are able to alter their and is therefore rewarded. The rewards can be subjective experience in response to suggestion adjusted for the degree of good behavior Answers: 25–40 137

obtained. So, a 40% increase in a desired behav- may cause more harm than good. All of the ior gets rewarded with a proportional prize. other treatments can be helpful. Methylphenidate is also effective in treating ADHD but is not a behavior therapy. Clozapine, 38. (C) Patients who are suicidal characteristically a newer-generation antipsychotic, is not effec- have severe hopelessness and think about tive in treating the disorder. Interpretive strate- events in rigid, black-and-white ways. They gies such as a well-timed interpretation of see very few options and become overwhelmed maladaptive behaviors and positively refram- with the hopelessness that is exacerbated by ing a negative experience are not helpful behav- seeing “no way out.” Personalization is a patient’s ioral tools. tendency to relate events to himself without any reason for doing so. 34. (C) Motivation for change goes beyond a desire for removal of symptoms and includes a will- 39. (B) Cognitive therapy is the psychotherapy that ingness to tolerate discomfort and to take risks would be most appropriate to help the patient in a search for understanding. Demographics solve problems more productively. Behavioral do not play a role. therapy is focused on behaviors, not on possi- ble underlying factors that cause behaviors. 35. (E) Brief psychodynamic therapy identifies a Interpersonal therapy is aimed at understanding single, focal area of conflict and spends the communication patterns and roles in relation- time of the therapy actively interpreting trans- ships. Hypnosis is a sleeplike state induced by ferences as they pertain to this identified focal a therapist, which has been of some use in conflict. Interpretations of resistance certainly uncovering psychodynamic processes and in take place in brief psychodynamic therapy. behavioral modification such as stopping However, the resistance is interpreted in light smoking. In paradoxical therapy, developed by of the identified focal conflict. Many types of Bateson, the therapist suggests that the patient therapy correct cognitive errors and modify engage in the behavior with negative connota- maladaptive behaviors, although cognitive tions (e.g., a phobia or compulsion). therapy and behavioral therapy, respectively, are specifically concerned with these outcomes. 40. (D) In projective identification, the patient attrib- utes unacceptable thoughts or feelings to those 36. (E) The treatment of the devastating illness of around him; the patient also recognizes the schizophrenia needs to involve not only the feeling in himself. The unacceptable feeling in patient but also the family who will care for the this case is the patient’s anger resulting from patient. This includes education of the patient his terminal diagnosis. It is common that the and his family, medication of the patient, and unacceptable feelings are experienced by the family therapy to help the family accommodate objects of the projective identification—in this the patient and his new illness. Recent evidence case, the nurses, who have become angry with supports reducing high levels of expressed the patient. The remaining defense mecha- emotion in the family in order to decrease psy- nisms may each play a role here, but we are chotic relapses. Psychoanalysis is not useful in given no direct clues to this. Denial occurs the treatment of schizophrenia and is generally when a patient selectively ignores an unac- considered contraindicated in the treatment of ceptable feeling or fact, as may occur in an alco- patients with psychotic disorders. holic who refuses to believe he has a problem. Intellectualization occurs when a patient explains 37. (B) Psychoanalysis is usually not indicated in away through intellectual reasoning the emo- patients who have difficulty with reality test- tional impact of some feature of his life. ing. The introspection and self-examination Sublimation is the channeling of unacceptable required in psychoanalysis is usually over- impulses into socially acceptable activities. whelming to psychotic patients and actually Reaction formation is the endorsement of some 138 5: Psychological Treatment and Management

opposite view of an unacceptable feeling or the intensity of the headaches by directly trying impulse. to relax and alter the known parameters. Undoing (M) is a concept described by Freud in which a 41. (A) In systematic desensitization, one response patient with obsessive-compulsive disorder per- to some situation stimulus is substituted for forms a compulsive act to prevent or undo the another response. This is known as reciprocal consequences from a feared impulse. Negative inhibition. Most commonly, the former response reinforcement (N) describes the delivery of an is anxiety—usually enough to interfere with averse stimulus based on a given response. someone’s ability to lead a fulfilling life—and Punishment (O) is the administration of an unpleas- the latter response is relaxation. This is accom- ant stimulus based on a given response; reward plished by gradually exposing the patient to describes a pleasant stimulus delivered to a given increasingly greater approximations of some response. In operant conditioning (P), a response is situation that causes great concern—say air rewarded or punished each time it occurs, so that travel—and pairing that experience with a more over time the response increases or decreases in pleasant response. The most effective way to frequency. Friedrich Nietzsche was a nineteenth- conduct systematic desensitization is to gradu- century philosopher. Splitting (Q), characteristic of ally expose the patient to the very situations patients with borderline personality disorder, that cause him extreme concern, not some imag- occurs when an individual is unable to see others ined facsimile. Systematic desensitization arises moderately; that is, people’s actions are either all from the teaching of the behaviorists, who pos- good or all bad. tulate that the behaviors are learned; the behav- iorists are less concerned that some unconscious 44. (I) Transference and transference neurosis are process might be at play as psychoanalytic synonyms. Transference can also be under- theory proposes. The most common applica- stood as a general phenomenon in which the tion of systematic desensitization is helping a person displaces onto all aspects of living the patient deal with a phobia. Depression is more emotions and forgotten memories of one’s past readily treated pharmacologically and/or with (particularly the earliest years of one’s life). psychotherapy. 45–47. [45 (F), 46 (B), 47 (A)] A patient with alco- 42. (D) Supportive psychotherapy attempts to for- holism who drinks alcohol while taking disul- tify psychological defenses by providing empa- firam will likely avoid this combination in the thetic reassurance as opposed to probing into a future. The goal of disulfiram therapy is to give patient’s psychological conflicts. Dynamic psy- patients negative associations with alcohol. chotherapy probes such conflicts to bring uncon- Nevertheless, patients learning to not drink scious material into consciousness. Existential while on disulfiram still illustrates the principal psychotherapy is based on existential philosophy of conditioned avoidance. The boy who receives and emphasizes personal feeling and experience a reward—a sweet—for something he is being over rational thinking. Biological psychiatry is a trained to do more often is undergoing positive branch of psychiatry that specifically focuses on reinforcement; the boy who receives soap in the genetic and the pharmacologic aspects of his mouth for something he is being trained to psychiatry. Cognitive-behavioral therapy, used do less often is undergoing punishment. often in mood and anxiety disorders, is a mix of Desensitization (C) refers to a behavioral modi- behavioral therapy and cognitive psychology. fication technique of increasing exposure to an unpleasant or phobic stimulus with the goal 43. (L) Biofeedback is the process of giving a patient of minimizing negative effects. Avoidance (D) is information on physiologic variables to alter learned cessation of behavior. Extinction (E) is those variables and ultimately the correspon- a learned response to removal of a stimulus. ding behavior. Patients with migraine headaches Positive reinforcement (B) is the process by which receive information on muscle tension or tem- certain consequences of behavior raise the perature. They use this information to decrease probability that the behavior will occur again. Answers: 41–49 139

Modeling (J) is a process by which behavior is of psychoanalysis that is practiced today. changed simply by observation. Flooding (G) is However, Freud did not work in a vacuum; he an exposure technique that is commonly used was influenced by an array of people and ideas, in behavior therapy. The premise is confronta- including the internist Joseph Breuer and the tion with the feared symptom. Hypnosis (I) is neurologist Jean-Martin Charcot. Incidentally, a sleep-like state induced by a therapist, which Freud began his career as a neurologist. Just as has been of some use in uncovering psycho- there have been many changes in physics since dynamic processes and in behavioral modifi- , there have been many changes in cation such as stopping smoking. psychoanalysis since Freud.

48. (A) Certainly, Sigmund Freud is one of the most 49. (B) Franz Anton Mesmer is considered the well-known figures of the twentieth century. father of modern hypnosis; the word mesmerism He is generally credited with the modern form is derived from his name. This page intentionally left blank CHAPTER 6 Legal and Ethical Issues in Psychiatry and Medicine Questions

DIRECTIONS (Questions 1 through 50): Each of (C) Inform the police of the content of your the numbered items in this section is followed by previous sessions and his recent threat answer choices. Select the ONE lettered answer or toward the case manager. completion that is BEST in each case. (D) Notify the case manager of the potential danger. Questions 1 and 2 (E) Attempt to contact the patient over the next several hours to discuss the intent The case manager for an 18-year-old man that you are of his parting comments. treating notifies you that your patient has been acting strangely lately. You learn from the case manager 2. The legal precedent that guides the appropri- that the patient makes provocative sexual comments ate course of action in this case is which of the toward her on a daily basis. During your session, the following? patient expresses concern that his case manager is somehow conspiring against him and he plans to do (A) Rogers v Commissioner of the Department something about it. On further questioning, he of Mental Health becomes increasingly anxious and abruptly storms (B) Tarasoff v Regents of University of out of your office. You hear him in the hallway, California exclaiming, “That woman, I’m going to stab her and (C) Durham v United States she won’t bother me anymore.” (D) Zinerman v Burch 1. Your first course of action should be which of (E) Kansas v Hendricks the following? (A) Respect the patient’s confidentiality and wait until your next scheduled appoint- ment with him to discuss his feelings. (B) Contact the patient’s family and let them know about his threats toward the case manager.

141

Copyright © 2007 by The McGraw-Hill Companies, Inc. Click here for terms of use. 142 6: Legal and Ethical Issues in Psychiatry and Medicine

Questions 3 and 4 6. The patient’s family hires an attorney to chal- lenge the integrity of her will. Which of the fol- Two years ago, you were the examining pediatrician lowing items, if present, would undermine this for a baby who was born with cerebral palsy. You had patient’s competence to make a will? heard from your obstetrical colleague that the family was planning to sue, and today you are working in (A) paranoid delusions regarding the your office when a process server delivers papers patient’s family notifying you that the family has brought an action (B) the presence of a diagnosable Axis I against you. disorder (C) the patient’s nondelusional explanation 3. In medical malpractice cases, the plaintiff must of why she wants to donate her estate to establish which of the following? the local humane society (A) burden of proof beyond a reasonable (D) inability to read and write doubt (E) refusal to undergo treatment with (B) clear and convincing evidence of wrong- chemotherapy doing (C) harm or damage Questions 7 and 8 (D) criminal intent A 23-year-old man with no prior psychiatric history (E) criminal mischief is charged with murdering his next-door neighbor. His friends note that he became increasingly isola- 4. In malpractice cases and others, as opposed to tive and suspicious of others in the weeks prior to fact witnesses, expert medical witnesses have the crime. The examining psychiatrist found him to which of the following characteristics? be having paranoid delusions regarding his neighbor and noted that his thought processes were too disor- (A) They are familiar with a particular body ganized to complete the examination. of evidence no more so than laypersons. (B) They are not allowed to charge a fee. 7. Which of the following legal standards is the (C) They can provide only factual informa- current basis for establishing an insanity tion. defense? (D) They can testify only for the defense. (A) M’Naghten rule (E) They may provide opinion testimony. (B) American Law Institute test (Model Penal Code) Questions 5 and 6 (C) Durham rule An 80-year-old widow with a history significant for (D) extreme emotional disturbance bipolar I disorder was recently diagnosed with end- (E) irresistible impulse rule stage hepatic cancer. She is concerned about the dis- position of her estate and does not want her family to 8. Criminal responsibility requires demonstration receive any of her money; she plans to donate her of the criminal act along with which of the fol- entire estate to the local humane society. lowing elements? 5. Which of the following is most important in (A) actus reus establishing a legally valid will? (B) diminished mental capacity (A) mens rea (C) mens rea (B) informed consent (D) product test (C) lack of mental illness (E) “wild beast” test (D) testamentary capacity (E) conservatorship of estate Questions: 3–12 143

Questions 9 and 10 11. A 19-year-old man with a history significant for bipolar I disorder is charged with assaulting An obese 54-year-old woman presents to the emer- a police officer. The patient has been able to gency department by ambulance complaining of work with his attorney in preparation for his severe substernal chest pain lasting 40 minutes, pro- defense. Which of the following items would fuse sweating, and nausea. Her vital signs are blood be consistent with the assertion that the patient pressure 195/96 mm Hg, heart rate 63/min, respira- is competent to stand trial? tory rate 18/min, temperature 98.8°F. An electrocar- diogram reveals 3-mm ST-segment elevations in (A) comprehension of the court proceedings leads V4, V5, and V6. Cardiac enzymes and laboratory (B) ability to read and write workup are pending. You suspect a lateral wall (C) presence of a diagnosable mental illness myocardial infarction and recommend immediate (D) testamentary capacity thrombolytic therapy. The patient states, “You’re (E) capability to provide informed consent crazy if you think I’m gonna let some intern care for me...my family will drive me across town to the pri- 12. A 35-year-old man is admitted to the locked vate hospital.” The patient then jumps from the psychiatric unit of the emergency department gurney and begins walking toward the exit. after threatening to unload his 9-mm pistol into his “blood brother’s” chest. Urine toxicology is 9. The appropriate next step is which of the negative for cocaine, phencyclidine (PCP), or following? opioids, and his blood alcohol level is 0.04. You (A) Sedate the patient and begin throm- learn from the patient’s family that he has been bolytic therapy. threatening to assault his brother for several (B) Allow the patient to leave against days and has even written notes about execut- medical advice. ing his plan. On Mental Status Examination (C) Admit the patient to the cardiology (MSE), the patient’s speech is loud, pressured, service on a physician emergency and very threatening toward the emergency certificate. department staff. Family history is significant for bipolar disorder. The patient states that he (D) Detain the patient until the results of the has not done anything wrong and wants to be cardiac enzymes are available. released immediately. The next appropriate step (E) Detain the patient until you can assess is which of the following? her ability to provide informed consent. (A) Notify the police about the patient’s 10. On further evaluation, the patient demon- homicidal threats. strates a thorough understanding of the infor- (B) Warn the patient’s “blood brother” mation you have given to her and appreciates about the threats. the consequences of not being treated immedi- (C) Arrange for a police hold given the ately. The appropriate next step is which of the patient’s potential for violence. following? (D) Involuntarily admit the patient to a (A) Sedate the patient and begin throm- locked psychiatric ward. bolytic therapy. (E) Medicate the patient with a mood (B) Allow the patient to leave against med- stabilizer and arrange for outpatient ical advice. follow-up. (C) Admit the patient to the cardiology serv- ice on a physician emergency certificate. (D) Detain the patient until the results of the cardiac enzymes are available. (E) Detain the patient until you can assess her ability to provide informed consent. 144 6: Legal and Ethical Issues in Psychiatry and Medicine

Questions 13 and 14 persistent cough. She is withdrawn and complains of a “scratchy throat.” Vital signs are stable and the You are the psychiatrist on duty one night when a patient is afebrile. On examination, the lungs are man arrested by police for stabbing his lover is clear to auscultation bilaterally and the posterior brought into the emergency room for evaluation. The oropharynx is clear. There are multiple bruises on man’s behavior is wild and unpredictable; at one her buttocks and back. A chest x-ray demonstrates point he starts to pick up a chair and look menacingly several rib fractures in different stages of healing. at a nurse before being subdued by police officers. Her parents report that their daughter is quite active Detectives tell you that the man’s only chance of stay- with her younger siblings and often gets into fights ing out of a jail cell that night is to be admitted to the with them. The patient agrees with her parents. locked unit, but the man makes it clear that he does not want to stay in the hospital. 15. The most likely diagnosis in this case is which of the following? 13. Criteria for involuntary admission to the hos- pital include which of the following? (A) somatization disorder (B) sexual abuse (A) medication noncompliance (C) major depressive disorder (MDD) (B) inability to care for self (D) age-appropriate “rough play” (C) permission from the conservator of person (E) physical abuse (D) presence of at least one diagnosable 16. The next appropriate step is which of the mental disorder following? (E) clinical decompensation (A) discharge the patient with follow-up in You decide to admit the man who is released after a 1 week to reevaluate her bruises few days. Six months later, you receive a call from the (B) refer for family therapy to address the man’s attorney who says he would like you to testify issue of rough play at the upcoming trial about the man’s mental state at (C) treat the patient with penicillin 2.5 mil- the time of admission. lion units intramuscularly (IM) (D) notify Child Protective Services 14. Which of the following is true of individuals (E) confront the siblings about their behavior found not guilty by reason of insanity? (A) have to serve their original criminal sen- Questions 17 and 18 tence once deemed sane A 17-year-old girl presents to your office complaining (B) are barred from using the defense in the of a burning vaginal discharge. She informs you that future she has been involved in a consensual sexual rela- (C) typically serve less time than those tionship with her boyfriend and is worried that she found guilty has contracted a sexually transmitted disease (STD). (D) generally spend at least the same You start the patient on antibiotics. Three days later, amount of time or more than those you discover that the culture of her vaginal discharge found guilty grew Neisseria gonorrhoea. (E) are immediately released without fur- ther psychiatric evaluation 17. A few days later, you receive a call from the patient’s parents demanding to know why Questions 15 and 16 their daughter was seen in your office. Because the patient is a minor, the appropriate next step A 9-year-old girl is brought to the pediatric emer- is which of the following? gency department by her parents for evaluation of a Questions: 13–20 145

(A) Explain to the parents that their daugh- longer wishes to be dialyzed. You obtain a psy- ter has contracted an STD and requires chiatric consult that concludes no evidence of immediate antibiotic treatment. mood or thought disorder. The family is upset (B) Maintain confidentiality by disclosing with the patient’s decision and demands that no information, but encourage the you continue to administer dialysis until a patient to discuss this issue with her transplant is available. The most appropriate parents. next step is which of the following? (C) Ask the parents about their daughter’s (A) Respect the patient’s wishes and discon- sexual history. tinue dialysis. (D) Alert the parents that their daughter is (B) Coerce the patient into continuing treat- at risk for human immunodeficiency ment. virus (HIV) and should be tested. (C) Continue dialysis until you convince the (E) Find out the boyfriend’s name and tele- ethics committee to support the family’s phone number to confirm the history. decision. (D) Tell the patient you’re sure a transplant 18. At the patient’s next follow-up visit, she requests will arrive soon and encourage him to to be tested for HIV. She reports that her remain in treatment. boyfriend recently tested negative, but given (E) Place the patient on a physician’s emer- her promiscuity, she wonders if she has con- gency certificate because he is a danger tracted HIV. Before administering the HIV test, to himself. you should do which of the following? (A) Notify her parents about the HIV test. 20. A 40-year-old surgical attending is admitted to (B) Directly inform the patient’s boyfriend the medical unit after developing severe right that he is at risk for HIV. flank pain. Further workup confirms a diag- (C) Obtain a complete blood cell count to nosis of nephrolithiasis. One of the surgical res- determine if the test is necessary. idents asks you about the attending’s diagnosis so that coverage can be arranged. The most (D) Explain how the test is performed and appropriate next step is which of the following? interpreted, along with information on confidentiality and how you will pro- (A) Reveal only the estimated length of stay. ceed if the result is positive. (B) Inform the chief surgical resident of the (E) Perform a pregnancy test. attending’s condition and length of stay. (C) Tell the surgical resident that you will 19. A 44-year-old man with insulin-dependent dia- not say anything, but she could take a betes mellitus and end-stage renal failure has look in the attending’s chart. been on dialysis several years while awaiting a (D) Tell the resident to address the ques- kidney transplant. He feels as though he has tions directly to the attending. “waited long enough” and does not want to (E) Arrange a conference between your continue “living tied to a machine.” After sev- medical attending and the surgical eral family meetings and consultations with house staff. other physicians, he informs you that he no 146 6: Legal and Ethical Issues in Psychiatry and Medicine

21. A 43-year-old man is referred for continued 90/min. Hematocrit is 18%. On MSE, cogni- treatment of depression after release from jail. tion is intact and there is no evidence of a mood The court mandated psychiatric treatment or psychotic disturbance. The patient states that while he is on probation. The patient’s proba- she is a Jehovah’s Witness. She refuses any tion officer calls you for information regarding blood transfusion on the basis of her religious his condition, progress, and treatment compli- beliefs. Which of the following best applies to ance. You should do which of the following? your next step in the treatment of this patient? (A) Discuss the case with the probation (A) administering packed red blood cells officer because the treatment is court (B) persuading the patient that she must mandated. accept the transfusion (B) Limit your discussion with the proba- (C) explaining the implications of nontreat- tion officer to only treatment compli- ment but respecting the patient’s refusal ance because the rest of the information for treatment is confidential. (D) utilizing a “confrontation method” to (C) Obtain a confidentiality waiver from the secure lifesaving treatment patient before speaking to the probation (E) referring the patient for involuntary officer. psychiatric treatment based on her life- (D) Ignore the request altogether because threatening decision psychiatric treatment bears no relation to law enforcement. 24. The parents of a newborn with Down syn- (E) Correspond with the probation officer drome find their daughter to be lethargic and only through written documents. minimally responsive. Medical evaluation is significant for the following cerebrospinal fluid 22. A 55-year-old woman is admitted for intra- findings: opening pressure of 100 mm Hg, venous (IV) antibiotic treatment for acute white blood cell count of 5000/µL (predomi- hematogenous osteomyelitis. The nurse informs nantly neutrophils), protein >40 mg/dL, glu- you that the patient was accidentally given the cose content <40 mg/dL, and Gram stain wrong antibiotic but has suffered no adverse positive for bacteria. You suspect group B strep- reaction. Which of the following best applies in tococcal meningitis and recommend IV antibi- this case? otic therapy. The parents feel as though their Down child will ultimately have a poor quality (A) the “no harm, no disclosure” rule of life and request that treatment be withheld. (B) informing the patient that she was given You should do which of the following? the wrong medication (C) first reporting the mistake to the hospi- (A) Respect the parents’ wishes because tal advisory committee they are the primary decision makers. (D) notifying the patient’s family of the (B) Refer the case to the ethics committee for mistake review at their next scheduled meeting. (E) encouraging the patient to seek legal (C) Inform the parents that after reviewing action because a critical mistake has the laboratory data you realize the occurred patient has viral meningitis and care will be mostly supportive. 23. A 38-year-old woman is admitted to the oncol- (D) Start ampicillin and cefotaxime against ogy unit with severe aplastic anemia. She the parents’ wishes. appears pale and weak. Vital signs indicate a (E) Repeat the lumbar puncture to verify blood pressure of 110/75 mm Hg and a pulse of the diagnosis. Questions: 21–28 147

25. You are called by a local news station to offer a (C) Abide by the son’s wishes because he is specific psychiatric diagnosis on a prominent the oldest. politician. There is considerable information in (D) Abide by the daughters’ wishes because the print and television media that suggests the the patient’s quality of life is already politician has bipolar I disorder. Which of the poor. following best applies? (E) Seek the hospital ethics committee’s (A) Stating your opinion publicly is legiti- opinion. mate as long as the politician is not your private patient. Questions 27 and 28 (B) Offering a psychiatric diagnosis in such A 25-year-old woman is referred to you for evalua- instances is unethical. tion of insomnia, frequent crying spells, and weight (C) Comment in written form only. loss. She informs you that her symptoms began sev- (D) Agree to an “off-the-record” or anony- eral weeks ago after she lost her job and that she now mous interview. experiences bouts of helplessness and hopelessness. (E) State your diagnosis but indicate that On MSE, she describes her mood as “depressed” and other problems may account for the notes that she occasionally has thoughts of taking an symptoms. overdose of aspirin.

26. An 84-year-old widowed man with severe 27. The most significant predictor for suicidal dementia, for whom you have been the pri- behavior is which of the following? mary care physician for years, is diagnosed (A) prior serious suicide attempt with hepatocellular carcinoma. The patient (B) current alcohol use does not have an advance directive and never designated a power of attorney. Psychiatric (C) current diagnosis of MDD consultation concludes that the patient is (D) physical pain unable to make an informed decision regarding (E) recent loss of a family member treatment options, and you turn to his family for guidance. The patient’s oldest son is adamant 28. Which category of mental disorders is most that his father receive chemotherapy, while the common in individuals who have completed two younger daughters feel that he should not suicide? suffer the adverse effects of chemotherapy (A) affective disorders “especially because he’s so demented.” Which of (B) psychotic disorders the following is the next appropriate step? (C) alcohol-related disorders (A) Ask the patient which family member (D) anxiety disorders he would like to designate power of (E) toxic metabolic states attorney. (B) Let the patient decide whether or not to proceed with treatment. 148 6: Legal and Ethical Issues in Psychiatry and Medicine

Questions 29 and 30 and he has experienced multiple respiratory complications that likely will require a tra- You have been treating a patient for a minor phobia. cheotomy. Severe muscle weakness and atro- At the end of a suggested course of treatment, he has phy are apparent in all limbs. The patient states been cured of his phobia. He proceeds to tell you that there is no meaning in continuing without that he finds you attractive and would like to have his physical capacities. He asks for your help in dinner with you the following Saturday night. ending his life in a humane and dignified manner. MSE is at baseline and there is no evi- 29. Which of the following statements is most dence of any psychiatric disorder. You discuss accurate in regard to sexual relations with psy- the patient’s request with his family and they chiatric patients? unanimously support his desire to “end the suf- (A) Sexual relations with a former patient is fering.” The most appropriate course of action unethical. would be which of the following? (B) Sexual relations with a former patient is (A) Respect the patient’s wishes by helping permissible as long as the physician him end his life in a painless and does not exploit his or her past position respectful manner. of authority. (B) Refuse to participate in assisting the (C) Sexual relations with a current or former patient with suicide and focus on patient is ethical. responding to the patient’s end-of-life (D) Terminating the physician-patient rela- issues. tionship is not required prior to dating a (C) Provide the patient with plenty of med- patient. ication refills to provide a lethal dose. (E) Sexual relations between consenting (D) Provide the patient with information adults is not bound by any professional regarding how to effectively end his life. code of ethics. (E) Ignore the patient’s request.

30. Which of the following would be true if the 32. You are asked to see an incarcerated man for patient had been one of your medical or surgi- evaluation. He is serving a sentence for murder cal patients? and often has trouble with guards, who pun- (A) Sexual relations with a former patient is ished his behavior with solitary confinement. unethical. Antisocial personality disorder is estimated to (B) Sexual relations with a former patient is exist in what percentage of the male prison permitted as long as the physician does population? not exploit his or her past position of (A) 80–100% authority. (B) 55–75% (C) Sexual relations with a current or former (C) 30–50% patient is ethical. (D) 10–20% (D) Terminating the physician-patient rela- (E) <10% tionship is not required prior to dating a patient. Questions 33 and 34 (E) Sexual relations between consenting adults is not bound by any professional You receive a subpoena from an attorney represent- code of ethics. ing a party that has filed a lawsuit against one of your patients. The subpoena pertains to releasing the 31. A 52-year-old man, for whom you have been medical records of your patient. the primary care physician for the last 20 years, was recently diagnosed with amyotrophic lat- 33. Which of the following is the appropriate next eral sclerosis. The disease has rapidly progressed step? Questions: 29–37 149

(A) Contact the attorney who obtained the (C) oppositional defiant disorder subpoena to discuss the process of (D) antisocial personality disorder releasing the medical information. (E) paranoid personality disorder (B) Release the medical records upon receiving the subpoena. 36. A 28-year-old woman with dysthymic disorder (C) Sign a release of information form and has been seeing you in weekly psychotherapy turn over the records. and has failed to pay her bill for 2 months. The (D) Not release the information and contact appropriate next step is which of the following? your patient regarding the subpoena. (A) Contract with a billing collector to (E) Release the medical records directly to demand immediate payment. the presiding judge. (B) Terminate the treatment. (C) Contact the patient’s family to deter- 34. A court hearing has been organized by your mine if the patient is financially stable. patient’s attorney to quash the subpoena you have been issued. At the hearing, the judge (D) Inquire as to the reasons she has been rules that you should release the medical avoiding payment at the patient’s next records even though your patient has not con- visit. sented to the release of information. Which of (E) Inform the patient that you will not see the following statements is most accurate her if she doesn’t pay for her treatment. regarding the release of this patient’s records? 37. A wealthy 46-year-old male banker is in psy- (A) Disclosing this patient’s medical records chotherapy with you for treatment of a single is unjustifiable. episode of depression. After significant improve- (B) Releasing a patient’s medical records is ment in his symptoms, he offers you the oppor- always unethical regardless of court tunity to take part in one of his financial orders. ventures. The investment appears to be sound. (C) It would be appropriate to release this The most appropriate comment to make to the patient’s records. banker is which of the following? (D) Disclosing a patient’s medical records (A) “Thanks for thinking about me . . . I’d when there is potential harm to the ther- be honored to invest with you.” apeutic alliance is unethical. (B) “Because your depression is improved, (E) Work out a plan of legal action with it would be appropriate for us to be your patient. business partners.” (C) “I can invest with you only when our 35. A 12-year-old boy has a 1-year history of van- treatment is nearing its end.” dalism, violating parental curfews, starting fights, and stealing. Which of the following (D) “It is probably a bad idea; I’m already personality disorders is this patient most likely committed in other investments.” to develop? (E) “I have to decline; it potentially may interfere with our treatment (A) borderline personality disorder relationship.” (B) narcissistic personality disorder 150 6: Legal and Ethical Issues in Psychiatry and Medicine

38. A 23-year-old Caucasian Catholic woman with Questions 41 and 42 a history of schizophrenia presents to your office after arguing with her husband. She is A 48-year-old man is involuntarily admitted to the unemployed and has been staying home taking hospital after an acute manic episode. The patient is care of the couple’s children. She notes feeling pressured, demanding, and increasingly talkative on sad and reports thoughts of killing herself. She the unit. He becomes intrusive, not able to be redi- has a past history of an overdose attempt with rected, and demands immediate release. You explain aspirin. Which of the following places this to him that you feel he is gravely disabled and unable patient in a high-risk category for suicide? to care for himself. He respectfully disagrees with you and demands “due process.” (A) age (B) past history of suicide attempt 41. The patient’s request for a hearing is based on (C) gender which of the following legal principles? (D) marital status (A) parens patriae (E) terminal medical illness (B) actus reus (C) habeas corpus Questions 39 and 40 (D) rights under the Fourth Amendment A 36-year-old man with a history of bipolar I disorder (E) mens rea is brought to the emergency department by police after stabbing a patron in a barroom brawl. His blood 42. The court agrees that the patient is severely alcohol level was 0.320 upon arrival, and the patient disabled and in need of acute medical man- required intramuscular (IM) droperidol (Inapsine) for agement. You load the patient with divalproex agitation. The patient has no recollection of the event, sodium 20 mg/kg, and 3 days later his symp- and the victim died 2 hours later. The patient has a toms of mania are markedly diminished. The history of assault while psychotic and is currently in patient requests immediate discharge and agrees weekly psychotherapy. His lawyer has chosen to to follow up with a partial hospital program. assert an insanity plea in defense of the patient. You feel that he would benefit from further inpa- tient treatment but is no longer gravely disabled 39. Which factor may undermine this patient’s or a threat for self-harm. The next appropriate assertion of the insanity defense? step is which of the following? (A) voluntary intoxication (A) file for another court hearing to detain (B) mental disease or defect the patient further (C) inability to recall the event (B) ignore the patient’s request because he (D) involuntary intoxication has been committed by the court (E) prior conviction of a violent crime (C) release the patient to the partial hospital program 40. A psychiatrist called as an expert witness is pri- (D) persuade the patient to stay for a few marily responsible for which of the following? more days (E) appease the patient by increasing smok- (A) obtaining a not guilty by reason of ing privileges insanity verdict (B) rendering an opinion based on reason- Questions 43 and 44 able medical certainty (C) establishing reasonable doubt You are treating a woman on the medical unit who (D) evaluation, diagnosis, and initiation of suffers from dementia. In your opinion, she requires treatment of the accused a central line for fluids and medication, but you are not sure if she can fully comprehend the risks and (E) countering evidence of criminal benefits of the procedure. You call the family for a responsibility meeting and request a psychiatric consult. Questions: 38–47 151

43. Which of the following is most important in (C) flashbacks while awake discussing informed consent? (D) antisocial personality disorder (A) raising alternative treatment options (E) duration of symptoms (B) involving family members in this discussion Questions 46 and 47 (C) petitioning a court to establish the A 35-year-old woman is referred by her plastic sur- patient’s competence geon for a psychiatric evaluation in preparation for (D) capacity to read and write sexual reassignment surgery. The patient recalls (E) absence of mental illness dressing in her brother’s clothes since she was 4 years old and has always preferred opposite-gender play- 44. Which of the following best characterizes the mates. She insists that she is “a man trapped inside intent of a living will? of a woman’s body.” She has fully assumed the male gender role for the last 15 years. On MSE, (A) establishing personal preferences there is no evidence of psychosis, delusions, or cog- regarding end-of-life issues nitive impairment. She does report a mildly dys- (B) requesting physician-assisted suicide if phoric mood, which she attributes to her lack of the patient were to become terminally ill male genitalia. Complete blood count, serum elec-

(C) arranging for funeral services and trolytes, folate, vitamin B12 level, and liver func- distributing the estate to loved ones tion tests are within normal limits; rapid plasma (D) preventing patients from changing their reagin/Venereal Disease Research Laboratory minds about life support as they (RPR/VDRL) are nonreactive. Karyotype analysis become ill demonstrates XX genotype. (E) absolving personal responsibility 46. Which of the following is the most appropriate 45. A 28-year-old man involved in a motor vehicle diagnosis in this case? accident brings a lawsuit against the driver. (A) homosexuality Emergency department records do not show (B) transvestite fetishism any physical injuries, but the patient is claim- (C) frotteurism ing to suffer from posttraumatic stress disorder (PTSD). You are asked to evaluate the patient’s (D) schizophrenia symptoms. He complains of distressing dreams (E) gender identity disorder of the accident, having flashbacks while awake, and avoiding the intersection where the acci- 47. The plastic surgeon and patient decide to pro- dent occurred. His symptoms have lasted for 3 ceed with the surgery. Which of the following is months. Past psychiatric history is significant most important in obtaining informed consent? for major depression, impulsivity, and violent (A) contingency analysis behavior. He has been incarcerated numerous (B) discussing potential complications of times, showing a blatant disregard for the law. the procedure On MSE, the patient is likable and cooperative. His mood is reported as being depressed and (C) establishing the lack of any mental he currently denies any hallucinations or delu- illness sions. Which of the following items from your (D) seeking approval from family members evaluation of the patient raises your suspicion (E) exploring the patient’s dreams and regarding a diagnosis of malingering? desires (A) history of major depression (B) lack of physical injuries 152 6: Legal and Ethical Issues in Psychiatry and Medicine

48. One of your patients consistently misses appoint- of the following is the most important factor in ments without giving you advance notice. After determining which parent should get custody? numerous failed attempts at resolving this issue (A) Who is the biologic parent? with the patient, you realize that the patient’s behavior is not changing and you decide to dis- (B) Who can provide for the best interests of charge the patient from your care. Which of the the child? following most appropriately describes the dis- (C) Who is most financially stable? charge process? (D) Who has the highest level of education? (A) Write a letter to the patient notifying her (E) Who is in the best mental and physical that she has been discharged due to her health? failure to comply with treatment. 50. You discover that one of the physicians in your (B) Notify your staff that the patient is not hospital is abusing narcotics. The appropriate to be given further appointments. course of action is which of the following? (C) Contact the patient’s family/friends to request their help in improving the (A) Notify the local police. patient’s attendance. (B) Contact the Drug Enforcement (D) Write a letter to the patient stating that Administration to rescind his license. she will be discharged in 60 days. (C) Do nothing so as to avoid personal (E) Notify the patient’s insurance company liability. that she is discharged from your care. (D) Admonish him for using but do not report. 49. You are asked by the court to provide a foren- (E) Notify the hospital’s committee for sic evaluation for a child custody case. Which impaired physicians. Answers and Explanations

1. (D) A physician or therapist has a duty to pro- Tarasoff I (1974), established a “duty to warn.” tect identifiable victims from imminent danger. The California Supreme Court revised its pre- This obligation may be fulfilled by directly vious holding in Tarasoff II (1976) with a “duty contacting the party at risk, notifying the to protect.” In Rogers v Commissioner of the police, or taking other appropriate measures to Department of Mental Health, the court expounded protect the victim. Contacting the patient’s opinions regarding the treatment of involuntar- family fails to protect the intended victim. ily committed patients. Durham v United States Although notification of the police is an accept- held that an individual “is not criminally respon- able intervention in this case, disclosing unre- sible if the unlawful act was the product of lated confidential information about the patient mental disease or mental defect.” The holding of is inappropriate. When notifying third parties, this case has become known as the “product test” care should be taken to release only informa- for determining an insanity defense and ulti- tion necessary to protect the potential victim. mately was replaced by the American Law The remaining choices fail to protect the poten- Institute standard. Zinerman v Burch and Kansas v tial victim. Hendricks involve court decisions regarding civil commitment. 2. (B) Tarasoff v Regents of University of California is the landmark legal precedent establishing 3. (C) The 4 Ds of establishing medical malprac- liability to third parties. It states “when a psy- tice cases involve proving that a dereliction of chotherapist determines...that his patient pres- duty directly causes damage. The plaintiff must ents a serious danger of violence to another establish the presence of a fiduciary relation- he incurs an obligation to use reasonable care ship (a duty), negligence (dereliction of duty), to protect the intended victim against such and damages directly caused by that negli- danger.” Therefore, the physician may ethi- gence. Typically, the burden of proof in medical cally break confidentiality in cases involving malpractice cases (civil cases) is by a prepon- imminent harm to others. In the Tarasoff case, a derance of the evidence (greater than 50%). student at the University of California (Prosenjit Proof beyond a reasonable doubt refers to crim- Poddar) expressed intentions to kill a fellow inal cases; clear and convincing evidence is student (Tatiana Tarasoff) to his therapist. reserved for special cases decided by the court. Poddar tragically followed through on his Neither criminal intent nor criminal mischief is threat and murdered the young woman at her required to prove medical malpractice. home. Tarasoff v Regents of University of California holds legal precedent only in the state of 4. (E) An expert medical witness may be called to California; however, the case has been adopted testify in both civil and criminal trials. Expert by physicians/therapists as the standard of witnesses have a familiarity with a body of care and has led to nationwide legislative knowledge that exceeds that held by layper- changes. There have been two rulings regard- sons and are often called to provide opinion ing the Tarasoff case. The original decision, testimony, making choices A and C incorrect.

153 154 6: Legal and Ethical Issues in Psychiatry and Medicine

Choices B and D are incorrect because expert defect should not be held criminally responsible witnesses commonly charge a fee for their serv- for an act if the person (1) could not appreciate ices and provide testimony for either defense or the wrongfulness of the conduct and (2) could plaintiff. not conform the conduct to the requirements of the law. The American Law Institute test 5. (D) Testamentary capacity refers to the level of expanded upon the 1843 M’Naghten rule, competence required to make a legally valid which held that an individual was not guilty by will. Case law and statutes differ across states. reason of insanity if, as a result of a mental However, the central elements of testamentary defect, the person did not know the nature of capacity include: (1) comprehension of the act what he or she was doing or did not know of writing/signing a will, (2) knowledge of that what he or she was doing was wrong potential heirs, (3) understanding the extent of (“right-wrong test”). The Durham rule arose one’s assets, (4) lack of undue influence, and from a 1954 District of Columbia case in which (5) the absence of delusions compromising the court held that “an accused is not crimi- rational thought. Mens rea refers to criminal nally responsible if his unlawful act was the intent and has nothing to do with drafting a product of a mental disease or mental defect.” will. Informed consent is the process of obtaining This is also known as the “product test.” permission for medical treatment. Important Extreme emotional disturbance and the irre- elements include the ability to make a volun- sistible impulse rule are not modern standards tary decision regarding treatment, explanation for proving an insanity defense. Extreme emo- of risks and benefits, and competency to make tional disturbance is an affirmative defense used an informed decision. The presence or absence to render the accused less culpable for the of a mental illness does not necessarily impair crime (i.e., reduce the charge from murder to an individual’s competency to make a will. manslaughter). The irresistible impulse rule, Therefore, the lack of mental illness is incorrect. established in England in 1922, states that an Conservatorship of estate refers to an individual’s individual is not responsible for a criminal act ability to manage his or her finances. if he was unable to resist that act due to a mental illness. 6. (A) Paranoid delusions regarding the patient’s family may significantly undermine this patient’s 8. (C) Criminal responsibility requires (1) criminal assertion that she was competent to make a will. intent (mens rea) and (2) a criminal act (actus The absence of delusions, which may compro- reus). Diminished mental capacity serves to mise rational decision making, is one of several avoid criminal responsibility; certainly, it is not criteria required for testamentary capacity. The required to prove criminal responsibility. The presence of a diagnosable Axis I disorder, the product test and the wild beast test are anti- inability to read and write, and refusal to quated standards of the insanity defense. undergo treatment with chemotherapy do not automatically undermine a patient’s compe- 9. (E) Before discharging or transferring the patient, tence to make a will as long as the criteria for it is important to determine whether she can testamentary capacity are met. An explanation make an informed decision regarding her care. of why the patient wants to donate her estate to Informed consent requires that the patient (1) the local humane society does not undermine comprehend the issues and/or choices furnished the patient’s competence to make a will unless by the physician, (2) appreciate the consequences such an explanation demonstrates that the of making a certain decision, and (3) arrive at a patient is unable to meet any of the specified voluntary decision after weighing the facts. At criteria. this point in the vignette, there is not enough information to establish the patient’s ability to 7. (B) The American Law Institute test is the make an informed decision regarding her care. modern standard for the insanity defense. It Exceptions to informed consent include incom- holds that a person with a mental disease or petence, emergency situations (e.g., comatose, Answers: 5–16 155

unresponsive patient), a competent patient’s protector of public safety) and parens patriae (the waiver of informed consent, and therapeutic state as parent for those unable to care for them- privilege. The other choices are incorrect because selves), respectively. State laws differ on the this patient’s ability to make an informed deci- length of time a person can be committed to the sion has not been assessed. hospital on the basis of either criteria. Permission from the conservator of person is not required for 10. (B) A competent patient has the right to involuntary commitment. The other choices may refuse treatment, even if the consequences be important factors, but are not requisite ele- are life-threatening. In the extension of this ments for involuntary hospitalization. vignette, the patient has adequately demon- strated the elements of an informed decision. 14. (D) Contrary to popular opinion, the insanity The other choices are incorrect because they defense (1) is seldom asserted, (2) is usually would prevent a competent patient from making unsuccessful, and (3) typically leads to a greater an informed decision in her treatment. number of years of incarceration. Individuals found not guilty have no obligation to serve a 11. (A) In Dusky v United States, the Supreme Court criminal sentence. There is no limit to the number ruled that competency to stand trial requires of times one may assert the insanity defense. the ability to (1) rationally consult with one’s Releasing an individual found not guilty by attorney and (2) understand the proceedings in reason of insanity without an appropriate psy- both factual and rational terms. Illiteracy does chiatric evaluation and treatment is unethical not constitute grounds for incompetency. A and fails to protect society. person can suffer from a mental illness and be competent to stand trial. Testamentary capacity 15. (E) Typical signs of physical abuse include burns refers to the level of competence required to with peculiar patterns (e.g., cigarette marks, geo- make a valid will. Informed consent is impor- metric designs, bilateral immersion patterns), tant in making decisions related to medical bruises in low-trauma areas (e.g., buttocks, gen- treatments but has little bearing on one’s com- ital areas, or back), retinal hemorrhages, and petency to stand trial. multiple fractures at different stages of healing. Although there is a high rate of comorbidity 12. (D) Involuntary commitment is used when a between physical and sexual abuse, there is no physician believes that an individual is a evidence to suggest sexual abuse in this partic- threat to self or others or is gravely disabled. ular case. Sexual or physical abuse may predis- Medicating the patient and arranging for out- pose an individual to the development of other patient treatment when the patient is actively psychiatric disorders such as depression, PTSD, homicidal is an improper course of action. and borderline personality disorder. Although Notification of the police or warning the the differential in this case should include patient’s blood brother is not immediately nec- depression and somatization disorder, history essary because this patient is well-contained and mental status do not provide sufficient evi- in a locked unit. Arranging a police hold is dence to diagnose either. Age-appropriate “rough incorrect. Police holds are issued when an indi- play” is not relevant. vidual who is arrested requires a psychiatric evaluation. If the arrested individual does not 16. (D) Each state requires mandatory reporting require inpatient psychiatric care, he is released of abuse or neglect to Child Protective Services. back into the custody of the police. In cases in which there is a reasonable suspicion of abuse, a report should be filed even if the 13. (B) Involuntary admission to the hospital is jus- patient and family deny the allegations. In tified when the patient is judged to be (1) a cases of suspected abuse, steps to ensure the danger to self or others or (2) gravely disabled. safety of the minor should be taken immedi- The criteria for involuntary commitment derive ately. Discharging the patient with follow-up in from the doctrines of police power (state as 1 week to reevaluate her bruises disregards her 156 6: Legal and Ethical Issues in Psychiatry and Medicine

safety and potentially subjects the minor to used to explore the patient’s attitudes toward continued abuse. Considering the extent of probation and incarceration. Obtaining a con- physical injuries, it is unlikely that they were fidentiality waiver from the patient should be inflicted by the patient’s younger siblings. done in a thoughtful manner and is essential Consequently, confronting the siblings about before releasing any information to the proba- their behavior serves no purpose. tion officer. Ignoring the probation officer’s request is inappropriate in such circumstances. 17. (B) Confidentiality in the treatment of minors The other choices are incorrect because they should be maintained unless specifically pro- violate physician-patient confidentiality. hibited by state law (e.g., in cases of abortion) or when the parents’ involvement is necessary to 22. (B) The patient should immediately be informed make complicated or life-threatening treatment that she was accidentally given the improper decisions. The other choices all constitute vio- medication. The physician should also evaluate lations of confidentiality to your patient. for potential adverse reactions. The fact that the patient did not suffer harm is no reason to con- 18. (D) Informed consent for HIV testing involves ceal the medication error. Although many hos- a thorough explanation of the test’s interpreta- pitals require documentation of such errors or tion, information regarding confidentiality, and reporting of them to the appropriate person or further evaluation needed if the test is posi- committee, this should not be the first course of tive. The patient should be aware of the risk of action. Evaluating the patient for any adverse false positives and negatives, and the need for reaction and explaining the mistake is the appro- confirmatory testing if the test result is positive. priate first step. Notifying the patient’s family of Choices A and B are incorrect because they vio- the mistake is a breach of confidentiality and late patient confidentiality. When a patient tests inappropriate. Encouraging the patient to seek positive for HIV, state laws may require notifi- legal action is both alarmist and inappropriate in cation of the sexual partner (this is usually done such a case. without revealing the patient’s identity). There is no reason to first perform a complete blood 23. (C) Explaining the implications of nontreatment count or a pregnancy test before administering but respecting the patient’s refusal for treat- an HIV test. ment is the most appropriate course of action. This allows the patient to make an informed 19. (A) The patient’s desire to stop dialysis should decision with regard to her treatment. The ele- be respected because competent adults are able ments integral to informed consent include (1) to determine which treatment to accept or deny. competency to make a decision; (2) an adequate The American Medical Association (AMA) Code explanation of the risks, benefits, rationale for of Ethics notes that there is no ethical distinction treatment, and alternative treatments (including between withdrawing and withholding medical no treatment at all); and (3) a voluntary deci- treatment. The remaining choices undermine the sion. In this vignette, there is no evidence that patient’s right to make an informed decision. the patient suffers from a condition that might impair her competency. Further, her stated reli- 20. (D) Physician-patient confidentiality should be gious belief is consistent with a Jehovah’s maintained except in certain cases (i.e., duty Witness’s refusal of human blood products. to protect others, mandatory reporting, or Administering packed red blood cells infringes emergencies). The remaining choices violate on the patient’s right to refuse a blood transfu- physician-patient confidentiality. sion and consequently is incorrect. Strongly persuading the patient to arrive at a decision 21. (C) Confidentiality between patient and physi- for treatment or utilizing a confrontational cian should be respected even in cases of court- method to secure lifesaving treatment is coercive mandated treatment. In such cases, issues in nature and could prevent the patient from around disclosure to a probation officer can be making a voluntary decision. It is inappropriate Answers: 17–32 157

to involuntarily hospitalize this patient if she 28. (A) Affective disorders (e.g., MDD, bipolar I/II has arrived at her decision in compliance with disorder) account for 40–70% of completed sui- the elements of informed consent. cides. Alcohol-related disorders account for 20–30% of completed suicides. Psychotic and 24. (D) According to the AMA Code of Medical anxiety disorders are less frequently associated Ethics, treatment decisions must be made in with completed suicides, as are toxic metabolic the best interest of the child or neonate regard- states. However patients with schizophrenia less of the desires of the parents. Treatment can are at high risk for suicide. be withheld or withdrawn only in cases in which the risks far outweigh the benefits, there 29. (A) Sexual or romantic relationships between is low potential for success, or when treatment current and former psychiatric patients are con- extends the child’s suffering without potential sidered unethical, making the other choices for a joyful existence. The lumbar puncture incorrect. results unequivocally demonstrate bacterial meningitis. In this particular case, the neonate 30. (B) The AMA Code of Medical Ethics cautions suffers from a serious, but treatable, central that sexual and/or romantic relations with nervous system infection and treatment should former patients may be influenced by the not be withheld. Withholding treatment until previous physician-patient relationship; care the next regularly scheduled ethics meeting should be taken to not exploit the power would likely result in the neonate’s demise. derived from the professional relationship. Sexual relations with current patients is uneth- 25. (B) It is unethical to offer specific psychiatric ical and constitutes sexual misconduct. At the diagnoses for individuals that you have not very least, current physician-patient relation- personally examined. Moreover, commenting ships should be terminated prior to initiating on an individual that you have evaluated any romantic or sexual relationships. would be a breach of confidentiality. 31. (B) According to the AMA Code of Medical 26. (E) Family and/or close associates should be Ethics, “physician-assisted suicide is funda- consulted to aid in reaching treatment deci- mentally incompatible with the physician’s role sions if there is no advance directive or desig- as healer,” and the physician should aggres- nated power of attorney. Consultation with the sively respond to a patient’s end-of-life needs ethics committee should be sought when the rather than assist in suicide. Physician-assisted patient’s prior wishes are unclear or family suicide is defined as the act of facilitating a members are unable to agree on treatment patient’s death by “providing the necessary options. Ethics committee consultations are means and/or information to enable the patient also useful when the power of attorney’s deci- to perform the life-ending act.” Simply ignoring sion is unreasonable or contrary to the patient’s the patient’s request is inappropriate because it prior wishes. It would be inappropriate to fails to recognize and address potential end- allow a severely demented patient to designate of-life issues. End-of-life issues that should be a power of attorney or make treatment deci- addressed with the patient include adequate sions. Choosing sides in a family conflict would pain control, emotional/family support, comfort be arbitrary; consultation with the hospital care, respect of patient autonomy, and appropri- ethics committee would help clarify issues in a ate multidisciplinary referrals (hospice, counsel- complicated case such as this. ing, religious issues). The remaining choices are consistent with the definition of physician- 27. (A) The most significant predictor of suicidal assisted suicide and are therefore unethical. behavior is a prior serious suicide attempt. The other choices are all associated risk factors for 32. (C) The rate of antisocial personality disorder in suicidal behavior but are not the most signifi- the male prison population has been estimated cant predictor of it. at 30–50%. 158 6: Legal and Ethical Issues in Psychiatry and Medicine

33. (D) When a physician is served a subpoena to 36. (D) Inquiring about possible reasons the patient release medical information, the appropriate first may be avoiding payment may help uncover step is to contact the patient to find out if he or important treatment issues. Such inquiry often she consents to the release. Discussing the impli- leads to how the patient feels about the therapy cations of the lawsuit with your patient and clar- and/or the therapist. These feelings can be uti- ifying whether the patient has the proper legal lized to address factors related to her symp- counsel for the lawsuit also are important. toms. Exploring such issues should be done Contacting the attorney of the party requesting in a sensitive manner to avoid alienating the the subpoena and releasing the medical records patient. Requesting that the billing collector without consent from the patient constitute demand immediate payment and terminating unethical breaches of physician-patient confi- treatment would be far too aggressive at this dentiality. The confidentiality privilege belongs point. Contacting the patient’s family violates to the patient, not the physician. Releasing the physician-patient confidentiality. At this point, medical information directly to the judge at this informing the patient that you will not see her stage makes no sense and would likewise be a if she doesn’t immediately pay her bill may be breach of confidentiality. perceived as confrontational and threatens the therapeutic alliance. 34. (C) Turning over medical records to comply with a direct court order is ethical. Working out 37. (E) Becoming involved in the business venture a plan of legal action is not the role of the physi- of a psychotherapy patient could have signifi- cian, but rather the patient’s attorney. The other cant adverse effects on the patient’s transfer- choices are incorrect because there are cases in ence (feelings and memories experienced by which disclosing the patient’s medical records the patient that are aroused by the therapist) as without consent is ethically permissible (e.g., in well as on the physician’s countertransference certain criminal proceedings, malpractice cases (feelings aroused in the physician while work- initiated by the patient, contestation of a will). ing with the patient); therefore, it should be avoided. Anger, distrust, and guilt are some of 35. (D) Antisocial personality disorder is an Axis II the feelings that may occur if a business ven- psychiatric disorder (i.e., personality disorder) ture such as the one described were to fail. characterized by a pervasive disregard for the Further, a physician’s business involvement rights of others, which often develops out of the with a patient might affect the neutral stance symptoms described in this patient. Behaviors essential to effective psychotherapy. A central associated with antisocial personality disorder problem is that involvement in a business ven- include repeated unlawful acts, lying, stealing, ture might interfere with treatment. This is best contempt for the safety of others, and lack of explained to the patient honestly. remorse for having hurt or mistreated others. A diagnosis of antisocial personality disorder 38. (B) Suicide risk factors include: advancing age requires evidence of conduct disorder with (>40); gender (M > F); marital status (separated onset before 18 years of age. Conduct disorder or divorced > married); past history of suicide describes the clinical picture of the patient pre- attempt or threat; race (Native Americans/ sented in the vignette; further, it is not classified Alaskan natives > Whites > Blacks/Hispanics); as a personality disorder. Oppositional defiant religion (Catholics are lowest risk); employment disorder is an Axis I disorder characterized by status (unemployed or retired > employed); a pattern of hostile, defiant, or negative behav- support system (living alone > living with iors, generally of less severity than that of others); and health status (serious medical con- conduct disorder. Narcissistic and paranoid dition or terminal illness > good health). personality disorders might develop in this patient but are not as likely to occur as antiso- 39. (A) An individual who commits a crime while cial personality disorder. voluntarily intoxicated cannot assert an insanity Answers: 33–45 159

defense because the basis of such behavior promptly discharged to the partial hospital stems from a rational decision to drink. program. Ignoring the patient’s request because Involuntary intoxication may be used effec- the court has committed him is incorrect. Filing tively as an insanity defense if the individual for another court hearing is appropriate only if had no choice in becoming intoxicated (e.g., the patient is still gravely disabled or a danger the person was drugged). A mental disease or to self or others. Persuading the patient to stay defect or inability to recall the event might be a few more days even though he wishes to used to enhance an insanity defense, not under- leave the hospital and no longer meets criteria mine it. Prior conviction of a crime in itself for involuntary commitment is inappropriate. should have no bearing on an insanity defense. Appeasing the patient by increasing smoking However, if prior violent behavior has been privileges fails to address the key issue in this associated with mental illness and the inability question. to appreciate the criminality of one’s conduct or conform to the law, this might be used to 43. (A) The key elements to informed consent strengthen an insanity defense. include competency to make a decision; a vol- untary choice; and an adequate explanation of 40. (B) Medical expert witnesses are called to pres- the rationale, risks, benefits, and alternative ent their professional opinions based on rea- treatment options. The remaining choices are sonable medical certainty. Expert witnesses are not necessary for informed consent. expected to perform an impartial evaluation and testify as to their findings; it is not the 44. (A) Living wills, or advance directives, pre- responsibility of the expert to ensure a partic- serve patient autonomy by establishing per- ular outcome for either side. Unlike the tradi- sonal preferences regarding end-of-life issues. tional physician-patient relationship, the expert If patients lose capacity in the future, advance witness does not typically initiate treatment of directives outline what procedures they do or patients and has limits on confidentiality. do not desire without relying on physician or family member opinions. The remaining choices 41. (C) Habeas corpus refers to the U.S. Constitution’s are not characteristics of a living will. Fourth Amendment’s clause that the state shall not “deprive any person of life, liberty, or prop- 45. (D) Malingering is characterized by intentional erty, without due process of law.” Under habeas feigning of symptoms for secondary gain (i.e., corpus, citizens have the right to petition the court financial remuneration, disability benefits, avoid- when detained to decide whether they are being ing criminal prosecution or military duty). A held lawfully. Parens patriae refers to the state’s diagnosis of malingering should be suspected function as parent for those unable to care for when there is the presence of one or more of the themselves. This is often used for justification of following factors: (1) litigation (especially when involuntary commitment. Establishing criminal the patient is referred by a lawyer), (2) discrep- responsibility requires the presence of both ancy between objective findings and the patient’s actus reus and mens rea. Actus reus refers to the complaints, (3) a diagnosis of antisocial person- voluntary act of committing a crime, and mens ality disorder, and (4) a failure to comply with the rea refers to criminal intent. They are both evaluation and recommended treatments. A incorrect because they have nothing to do with past history of major depression does not raise the patient’s right to due process. The Fourth one’s suspicion of malingering. The lack of Amendment is incorrect; it protects citizens physical injuries or the presence of flashbacks in from unreasonable searches and seizures. a claim of PTSD is not sufficient to raise suspi- cions of malingering. PTSD occurs in individu- 42. (C) It would be unethical to involuntarily detain als exposed to actual or threatened death or a patient in treatment if he or she is no longer injury and consequently may have no physical deemed to be gravely disabled or a threat to injuries. Classic symptoms of PTSD include self or others. In this case, the patient should be flashbacks, distressing dreams, intrusive 160 6: Legal and Ethical Issues in Psychiatry and Medicine

memories, avoidance of trauma-related stim- consideration their opinions in seeking an uli, and increased autonomic activity. The dura- informed decision. Exploring a patient’s dreams tion of symptoms in this case has no bearing on and desires, while perhaps useful in under- a diagnosis of malingering. standing possible reasons behind any decision, is not an essential element in determining 46. (E) Gender identity disorder is characterized by informed consent. a persistent desire/claim to be the opposite sex, a desire to participate in games/play of the 48. (D) According to the AMA Code of Medical opposite gender, preference for opposite- Ethics, physicians are obligated to provide con- gender playmates, wearing cross-gender cloth- tinuity of care for their patients. When dis- ing, and the desire to change one’s physical charging a patient from your care, advance characteristics to conform with his or her notice of the pending discharge must be given gender identification. Generally, homosexuals with sufficient time to obtain another physician. lack the desire to assume the opposite gender Contacting the patient’s family breaches confi- role. Rather, they are attracted to members of the dentiality and is not the appropriate procedure same gender. Transvestite fetishism is character- for discharging the patient. Notifying the insur- ized by cross-dressing behaviors for sexual ance company fails to inform the patient of her excitement without an underlying desire to discharge and does not provide her time to assume the opposite gender identity. Frotteurism obtain a new physician. The remaining choices is the act of rubbing or touching nonconsenting fail to provide the patient with sufficient notice individuals for sexual excitement. Psychotic dis- to obtain proper medical follow-up. orders such as schizophrenia are associated with disorganized speech/behavior, hallucinations, 49. (B) The prevailing concept in determining child delusions, or impaired reality testing. In this case, custody is based on which individual(s) can the patient’s desire to become a male is not delu- provide for the best interests of the child. sional in nature. She acknowledges that she is Factors that can be used to determine the best genetically a female but identifies herself as the interests of the child include assessing the indi- opposite gender without evidence of psychosis. vidual’s biologic relation, financial stability, level of education, and health; however, it is the 47. (B) Elements essential to informed consent aggregate best interest, rather than any single include (1) voluntary decision making, (2) factor, that determines what is in the best inter- explanation of risks and benefits of treatment, est of the child. (3) discussion of alternative treatment options (including consequences of withholding treat- 50. (E) Physicians are ethically obliged to report ment), (4) ability to understand/appreciate the impaired colleagues. Reporting procedures vary information presented, and (5) absence of any from state to state. However, the AMA Code of condition that would impair an individual’s Medical Ethics suggests initial reporting to a competency to make a decision. Contingency hospital’s in-house impaired physician program. analysis is not relevant to informed consent. If no such program exists, the chief of staff or The presence of a mental illness does not nec- clinical service director should be notified. essarily preclude one from making an informed For physicians without hospital privileges, an decision. The severity of impairment caused impaired physicians program run by a medical by a mental illness is vital in determining society or the state board licensing committee whether the patient is capable of providing should be made aware of the impaired physi- informed consent. Although the patient may cian. The other choices do not meet the AMA desire to discuss the issue with members of her guidelines and fail to initiate proper treatment family, the physician is not obliged to take into and supervision of the impaired physician. CHAPTER 7 Differential Diagnosis and Management Questions

DIRECTIONS (Questions 1 through 61): Each of (C) ziprasidone (Geodon) and sertraline the numbered items in this section is followed by (Zoloft) answer choices. Select the ONE lettered answer or (D) haloperidol (Haldol) and perphenazine completion that is BEST in each case. (Trilafon) (E) divalproex sodium and lorazepam Questions 1 and 2 (Ativan) A 37-year-old man presents to your office complain- Questions 3 and 4 ing of auditory hallucinations that have worsened over the last several months. He notes that the Devil A 20-year-old single woman presents to your office has been telling him that he is “no good,” and that he complaining of “episodes of complete terror.” She will not “amount to anything.” During the last several states that “totally out of nowhere” she became months, the patient also reports feeling “depressed” extremely anxious, experienced heart palpitations, and has been sleeping poorly. He has no desire to get felt short of breath, began sweating profusely, and out of bed and has lost interest in even watching believed that she was going to die. She reports that sports (normally one of his favorite activities). The these episodes last approximately 10 minutes and patient states that even when his mood is improved, that they have occurred several times over the last 3 he still cannot “get the voices out of my head.” He months. She constantly worries that they will occur denies using any drugs or alcohol. again.

1. Which diagnosis best accounts for this patient’s 3. What general medical condition should be con- symptoms? sidered in the differential diagnosis for this (A) major depression patient’s cluster of symptoms? (B) schizophrenia (A) hypothyroidism (C) schizoaffective disorder (B) systemic lupus erythematosus (SLE) (D) bipolar II disorder (C) pheochromocytoma (E) schizoid personality disorder (D) folate deficiency (E) hypertension 2. What medication combination could be used to adequately treat this patient’s symptoms? (A) mirtazapine (Remeron) and citalopram (Celexa) (B) fluoxetine (Prozac) and diazepam (Valium)

161

Copyright © 2007 by The McGraw-Hill Companies, Inc. Click here for terms of use. 162 7: Differential Diagnosis and Management

4. Which of the following is the most likely diag- that people in his neighborhood have organized nosis for this patient’s symptoms? against him in an effort to ruin his life. According to the patient, people are tape-recording his telephone (A) generalized anxiety disorder (GAD) conversations and videotaping him with cameras (B) social phobia concealed in his house, and have recently recruited (C) panic disorder the police in their efforts. Although he is intensely (D) obsessive-compulsive disorder (OCD) distraught over this invasion of his privacy, he is per- (E) agoraphobia forming well at his job, exercising daily, and eating and sleeping well. He is well-groomed and well- Questions 5 and 6 related in his conversation. His thinking seems oth- erwise logical. A 64-year-old woman is brought to the emergency department by her neighbor, who says “my friend 7. What is the most appropriate treatment? isn’t acting right.” The patient requires the support of a nurse while walking to an examination table. (A) fluoxetine Examination reveals that she cannot correctly identify (B) alprazolam (Xanax) the season or the town she is in. She does not recog- (C) olanzapine (Zyprexa) nize her neighbor. She is inattentive and seemingly (D) electroconvulsive therapy (ECT) apathetic to the activity around her. She dozes off (E) lithium repeatedly during the interview, but each time is arousable and resumes answering questions. Her 8. Which statement regarding this illness is true? answers are illogical and inconsistent. Vital signs are within normal limits and she is nei- (A) Auditory hallucinations are common. ther tremulous nor diaphoretic. Neurologic exami- (B) The delusions are usually bizarre. nation finds bilateral sixth nerve palsy and horizontal (C) Negative symptoms predominate. nystagmus. Urine toxicology screen and blood alco- (D) The delusion is plausible but unlikely. hol level are negative. (E) Persecutory delusions are relatively rare. 5. What is the most likely diagnosis? Questions 9 and 10 (A) subdural hematoma A 29-year-old man with a history of chronic paranoid (B) alcohol withdrawal schizophrenia comes to the emergency department (C) Wernicke encephalopathy with a temperature of 102.9°F, labile blood pressure (D) folic acid deficiency rising to 210/110 mm Hg, a pulse of 110/min, and a (E) normal pressure hydrocephalus (NPH) respiratory rate of 22 breaths/min. This patient’s medications include haloperidol, benztropine 6. What is the most important first step in man- (Cogentin), and clonazepam (Klonopin). He cannot aging this patient? correctly identify the day, date, or year, and believes himself to be in a city from which he moved 10 years (A) administration of a benzodiazepine ago. A family member indicates that 3 days ago he (B) computed tomography (CT) scan of the was healthy and completely oriented and that he has head no significant medical or surgical history. (C) administration of folic acid (D) administration of thiamine 9. Physical examination reveals that he is in (E) intravenous (IV) fluids and observation acute distress with hypertonicity. Laboratory examination reveals a creatine phosphokinase Questions 7 and 8 (CPK) of 45,000 Iµ/L, a white blood cell count of 14,000/µL and no left shift, a sodium of A 52-year-old man with no psychiatric history is 145 mEq/L, and a creatinine of 2.5 mg/dL. brought in by the police to the emergency department Lumbar puncture produces clear fluid with a after threatening a neighbor with a knife. He says Questions: 4–14 163

slightly elevated protein count. What is the 12. What medication(s) would be most appropriate? most likely diagnosis? (A) tricyclic antidepressant (TCA) only (A) anticholinergic syndrome (B) lithium and a selective serotonin reup- (B) malignant hyperthermia take inhibitor (SSRI) (C) central nervous system (CNS) infection (C) an SSRI and a neuroleptic (D) prolonged immobilization (D) a benzodiazepine only (E) neuroleptic malignant syndrome (NMS) (E) lithium only

10. With appropriate treatment, the patient recov- Questions 13 and 14 ers completely and returns home. In a month’s time, he comes to the emergency department A 21-year-old man is brought to the emergency stating that the “voices in the walls” are telling department by police after an episode in which he him to kill himself. He has taken no medica- ransacked the office where he works looking for “evi- tions since he left the hospital. His vital signs dence.” He started this job 2 months ago after grad- are stable and a medical workup is negative. uating from college. He lives with four roommates Initial therapy may include which of the and he believes they are jealous of him because of his following? job and have been poisoning his food. His family reveals that once before when he went away to col- (A) ECT lege he went through a period of “acting crazy” but (B) haloperidol depot injections got better without treatment and has done well since. (C) olanzapine In the emergency department, he is shouting that he (D) safety monitoring only has been up for a week writing a “classic” book about accounting and someone at the office stole it from (E) physical restraints him. He needs to be physically restrained by emer- gency department security. Physical examination and Questions 11 and 12 complete laboratory workup and toxicology screen A 39-year-old woman comes to the emergency prove to be negative. department and complains that since her boyfriend broke up with her 3 months ago, she has been sleep- 13. Initial medications should include which of the ing and eating poorly, has lost all interest in her work, following? and feels guilty that she drove her boyfriend away. In (A) lithium the past month, she has begun to feel hopeless, help- (B) a neuroleptic and a benzodiazepine less, and that “life may not be worth it.” In the past 2 weeks, she has developed a belief that a rare disease (C) an SSRI is rotting her heart, and in the past week, a voice in (D) carbamazepine (Tegretol) her ear tells her she is no good and that she should (E) buspirone (BuSpar) take an overdose of a heart medication she is pre- scribed. 14. Three months later, the patient sees his doctor for follow-up. He is taking lithium and haloperidol. 11. What is the most appropriate treatment? He is doing well, except he complains of muscle stiffness. His lithium level is 0.8 mEq/L. A rea- (A) medication and discharge to a close sonable intervention is which of the following? family member (B) referral to an outpatient psychiatrist (A) increase the lithium dose (C) admission to a psychiatric unit (B) decrease the lithium dose (D) admission to a medical unit (C) start baclofen (Lioresal) (E) restraints and medication in the emer- (D) decrease the haloperidol dose gency department (E) increase the haloperidol dose 164 7: Differential Diagnosis and Management

Questions 15 and 16 (A) torticollis (B) trismus A 29-year-old woman tells her doctor that about 3 weeks ago a child she was caring for ran into the (C) oculogyric crisis street and was killed by a bus. Since then, she cannot (D) retrocollis get the image of the accident out of her mind. Even (E) NMS in sleep, she dreams about it. She used to take a bus to work but she now drives because she cannot bear Questions 18 and 19 to be near buses. In the past week, she has begun missing work because she is uncomfortable leaving A 59-year-old woman with a long history of GAD her house. She feels guilty, believing the accident tells her primary care doctor that in a crowded super- was her fault. market 2 days previously she felt dizzy, with associ- ated heart palpitations, pressure on her chest, and a 15. What is the most likely diagnosis? frightening sense of doom. Shortly thereafter, she fell unconscious and woke up minutes later with a crowd (A) posttraumatic stress disorder (PTSD) around her. She felt somewhat better and rejected (B) acute stress disorder others’ advice that an ambulance be called. She (C) major depressive disorder (MDD) quickly made her way home. (D) panic disorder with agoraphobia 18. What is the most appropriate next step? (E) adjustment disorder (A) short-acting benzodiazepines 16. The patient decides against any medication and (B) an SSRI follows up with psychotherapy. A year later, (C) cognitive-behavioral therapy although she is no longer having distressful (D) electrocardiogram (ECG) symptoms relating to the accident, she com- plains she feels sad and tearful most of the (E) reassurance that her condition is benign time, is having trouble eating, has lost interest 19. What is the most likely diagnosis? in gardening, and wakes up at 4 AM every morning, unable to get back to sleep. What is (A) cardiovascular disease the most likely diagnosis? (B) panic attack (A) PTSD (C) GAD (B) acute stress disorder (D) acute stress disorder (C) MDD (E) anxiety disorder due to cardiovascular (D) panic disorder with agoraphobia disease (E) adjustment disorder 20. A 41-year-old woman you have been following for depression comes for a follow-up visit. She 17. A patient is admitted to a psychiatric hospital. has taken increasing doses of an SSRI over the Her medications include an SSRI and a benzo- past 10 weeks, is now at the recommended diazepine, which are both discontinued on maximum dose, and has shown little, if any, admission. A neuroleptic and a mood stabilizer improvement. She is having no side effects to are started. Two days after admission, she calls the medication. What is the appropriate next the nursing staff to her bed. She is extremely step in her treatment? frightened and complains excitedly that she cannot stop looking up. On examination, her (A) Add lithium. eyes are noted to be deviated upward bilater- (B) Stop the SSRI and try a second antide- ally. This specific reaction is called which of pressant. the following? (C) Add a second antidepressant. Questions: 15–25 165

(D) Continue at the current dose and follow dormitory room naked, and accusing the roommate up in 2 weeks. of calling the Federal Bureau of Investigation (FBI) to (E) Discontinue all antidepressant treat- have him monitored. The patient’s vital signs are all ment. within normal limits and the neurologic examina- tion shows no deficits or abnormalities. Questions 21 and 22 23. The most important first test is which of the A 19-year-old college junior goes to the university following? health care office because she has stopped menstru- ating. Physical examination reveals a lean, athletic (A) a noncontrast CT scan of the brain woman in no apparent distress, but is notable for (B) a toxicology screen poor dentition and erosion of the enamel of her teeth. (C) an erythrocyte sedimentation rate (ESR) With respect to her body habitus, age, and height, (D) a complete blood count (CBC) her weight is below average but within normal limits. (E) liver function tests She admits to being “a little weird” about her weight and exercises vigorously in response to “pigging out.” 24. If this test were negative, the most likely diag- nosis would be which of the following? 21. Associated laboratory and physical examina- (A) major depression with psychotic tion findings of this patient might include features which of the following? (B) delusional disorder (A) low serum amylase (C) schizophreniform disorder (B) salivary gland atrophy (D) chronic paranoid schizophrenia (C) hyperkalemia (E) substance-induced psychosis (D) hypermagnesemia Questions 25 and 26 (E) normal thyroid function You have been asked by the surgery team to evaluate 22. Which of the following statements regarding a 35-year-old man who had surgery to repair his frac- the treatment of this patient is true? tured right wrist 24 hours ago and is now complain- ing of anxiety. The patient has been in the hospital for (A) A reasonable intervention is to prescribe 2 days. His heart rate is 120/min, and his blood pres- an SSRI, with follow-up in 1 month. sure is 160/106 mm Hg. He is afebrile and reports (B) Anorexia nervosa tends to be more that he has never suffered anything like this before. responsive to pharmacotherapy than is He is not in any pain and has no previous psychiatric bulimia nervosa. history. His medication includes only acetaminophen (C) Hospitalization is typically indicated. for pain control. You note that he is diaphoretic and (D) Treatment is necessary because she is at tremulous. risk for seizures. (E) Cognitive-behavioral therapy is of little 25. The most appropriate single-agent therapy is value. which of the following? (A) clonidine (Catapres) Questions 23 and 24 (B) methadone A 19-year-old man with no previous psychiatric (C) lorazepam history is noted by his college roommate to be (D) carbamazepine acting bizarrely for the past month and a half, talk- (E) naltrexone (ReVia) ing to people who are not there, walking around the 166 7: Differential Diagnosis and Management

26. Without this treatment, he is at risk for devel- distress. You also notice that her pupils are pin- oping which of the following? point. After a few hours, she becomes more interactive and is complaining of diffuse, (A) fatty liver crampy abdominal pain and symptoms of “the (B) abdominal pain flu.” Her pupils are now slightly dilated, and she (C) seizures is yawning. Her vital signs are a temperature of (D) muscle cramps 99.3°F, a heart rate of 99, and blood pressure of (E) appendicitis 142/90 mm Hg. After the second interview, your diagnosis is which of the following? Questions 27 and 28 (A) cannabis abuse You are working in the psychiatric emergency (B) opiate withdrawal department of a large metropolitan hospital. A 20- (C) cocaine withdrawal year-old man with unknown psychiatric history is (D) cocaine intoxication brought in by the police after being found stumbling (E) opiate intoxication naked around a local college campus. He is markedly agitated, pacing, and appears to be responding to Questions 30 and 31 internal stimuli. On examination, you note that he is tachycardic with a heart rate in the 110s, he has tics A 70-year-old widow is admitted for an evaluation of or spasms in his face, and he has vertical nystagmus. depression and anxiety. She tells you that for the past 15 years her family doctor has prescribed “some 27. The test most likely to be diagnostic is which of pills” that have helped her sleep and feel less nerv- the following? ous. She says that she ran out of them yesterday and since that time has felt increasingly anxious and jit- (A) a noncontrast head CT tery. She also notes that she’s now having tremors in (B) a magnetic resonance image (MRI) of her hands that have not been there before. the brain (C) an electroencephalogram (EEG) 30. Your initial concern would be greatest if you (D) a CBC found that the medication was which of the (E) a toxicology screen following? (A) nortriptyline (Pamelor) 28. The most appropriate treatment is which of the (B) triazolam (Halcion) following? (C) thioridazine (Mellaril) (A) placing the patient in a low-stimulus (D) fluphenazine (Prolixin) environment (E) imipramine (Tofranil) (B) phenytoin (Dilantin) (C) methylphenidate (Ritalin) 31. The most worrisome side effect of abruptly (D) benztropine stopping this medication is which of the fol- (E) propranolol (Inderal) lowing? (A) autonomic hyperactivity 29. A 40-year-old divorced woman is brought in to (B) seizures the emergency department after being found sleeping in a pile of leaves. Initially, she is diffi- (C) hallucinations cult to arouse and has trouble answering your (D) worsening anxiety questions, but she appears to be in no respiratory (E) vomiting Questions: 26–35 167

Questions 32 and 33 Questions 34 and 35

A 73-year-old man is admitted to the hospital for A 75-year-old widow is brought to the hospital by her community-acquired pneumonia and dehydration. son, who says that his mother has been having pro- On day 2 of hospitalization, you are asked to evaluate gressive difficulty with her memory since her hus- the patient for depression; the staff has noted that he band died 3 years ago. The memory difficulties, seems very withdrawn. He is not eating or sleeping however, have worsened in the last week. Prior to a well. Last night, the nursing staff reports, he was week ago, the patient and her family noted that she angry and requested to leave the hospital. You talk had difficulty remembering new names, often could with the patient’s family and find that the patient not remember where she had left her purse, and was has no previous psychiatric history. Prior to the onset having more problems paying her bills. One week of this illness 5 days ago, he had no depressive symp- ago, however, her family noted that she was having toms and no difficulties with cognition. On exami- difficulty dressing and feeding herself. On examina- nation, his vital signs are temperature 98.2°F, heart tion, her vital signs are significant for a blood pressure rate 87, blood pressure 130/86 mm Hg, and a periph- of 170/100 mm Hg, a heart rate of 80, a temperature of eral oxygen saturation of 95% on room air. He is 98.8°F, and a peripheral oxygen saturation of 99% on drowsy and oriented only to person. His Mini-Mental room air. She is alert, pleasant, and oriented to person, Status Examination (MMSE) score is 24/30. place, and time. She scores a 24/30 on the MMSE.

32. The most likely diagnosis is which of the fol- 34. The next step in the diagnosis is which of the lowing? following? (A) MDD (A) physical examination (B) anxiety disorder (B) noncontrast head CT (C) dementia (C) lipid profile (D) delirium (D) carotid ultrasound (E) factitious disorder (E) transesophageal echocardiogram

33. The medical team asks for medication recom- 35. The most concerning part of the history and mendations if the patient becomes agitated. examination is which of the following? You recommend which of the following? (A) history of progressive memory (A) lorazepam difficulties (B) benztropine (B) recent sudden decline in function (C) thioridazine (C) hypertension (D) diphenhydramine (D) fact that she is fully oriented (E) haloperidol (E) low score on the MMSE 168 7: Differential Diagnosis and Management

Questions 36 and 37 he comes home each weekend. His parents have noticed that over the past 2 months the patient has A 65-year-old woman with a past medical history of become increasingly withdrawn and does not seem non-insulin-dependent diabetes mellitus and depres- to be taking care of himself. The parents were called sion is admitted with increasingly depressed mood by the patient’s roommate who says that the patient over the last month. She is unable to complete her has not been going to classes for the last 2 months, crossword puzzles because of difficulty concentrat- has not been eating or bathing, and has been talking ing. She has difficulty falling asleep and also wakes about how people in his classes are trying to kill him. up in the middle of the night. She denies suicidal On examination, the patient’s vital signs are all ideation, but does feel guilty that she is depressed. within normal parameters. He is pacing and appears Prior to this episode, she was doing well and was to be responding to internal stimuli. His mood is actively engaged in community volunteer groups. In euthymic and his affect is flat. He denies any drug the last month, she has lost 13 lb due to poor intake. use and is not on any medication. He is fully ori- When you ask why she is not eating, she tells you ented. He says that he hears a number of people talk- that she is worried that she will become infected with ing to him, “maybe in my head.” They seem to be bacteria. She has been to her primary physician for an saying bad things about him. He looks frightened evaluation, but she says everything was normal. Her and asks, “What is happening to me?” husband confirms that the patient has been very wor- ried about “getting a disease” to the point where she 38. If this condition had been occurring for less will eat only food in sealed containers. He also con- than 1 month, your diagnosis would be which fides in you that she has been worried that she might of the following? have cancer and despite reassurances from her pri- mary care physician, she continues to voice her con- (A) delusional disorder cerns to her husband that “my intestines are not (B) chronic undifferentiated schizophrenia working.” (C) brief psychotic disorder (D) schizophreniform disorder 36. The diagnosis that best describes her illness is (E) substance-induced psychotic disorder which of the following? (A) somatization disorder 39. The most helpful laboratory test to make this (B) MDD without psychotic features diagnosis is which of the following? (C) dysthymia (A) toxicology screen (D) MDD with psychotic features (B) EEG (E) OCD (C) noncontrast head CT (D) CBC 37. Which of the following is the treatment of (E) electrolytes choice? (A) sertraline Questions 40 through 41 (B) sertraline and perphenazine A 35-year-old woman with no previous personal or (C) lithium and sertraline family psychiatric history is brought to the emer- (D) divalproex sodium gency department by her husband, who reports that (E) nortriptyline and lorazepam his wife was attempting to kill herself by cutting her wrists. Her husband tells you that 6 months ago the Questions 38 and 39 patient’s grandmother died. Since that time, her hus- band thinks the patient has been becoming more An 18-year-old man is brought to the psychiatric depressed. She has difficulty falling asleep and has lost emergency department by his parents for evaluation 15 lb in 2 months. She was recently fired from her job of his behavior. Three months ago, the patient started as a paralegal because she was unable to concentrate. classes at the state university in a different city, but Questions: 36–45 169

She feels guilty that she cannot feel better and endorses 43. You choose to begin treatment with which of feelings of hopelessness and worthlessness. She the following? believes that her only way out of this is to kill herself. (A) divalproex sodium only 40. You choose to treat her with which of the fol- (B) haloperidol only lowing? (C) fluoxetine (D) divalproex sodium and lorazepam (A) flumazenil (Romazicon) (E) lorazepam only (B) fluoxetine (C) fluphenazine Questions 44 and 45 (D) triazolam (E) phenelzine (Nardil) You are asked to see an 18-year-old woman with no previous psychiatric or medical history because the 41. A common side effect of this class of medica- head of her college is concerned about her and wor- tion that may mimic a symptom of depression ried that she may be depressed. The patient tells you is which of the following? that despite being far away from her home for the first time, she was enjoying school and her new (A) insomnia friends until 2 months ago, when she learned that her (B) nausea parents were getting divorced. Since that time, her (C) constipation grades have gone from As and Bs to Bs and Cs because (D) akathisia she is worried about her parents and her sister at home. She says she feels “bummed out” most of the (E) diarrhea time, and her friends note that she seems unhappy and occasionally becomes tearful when talking about her Questions 42 and 43 family. She denies difficulty sleeping or changes in A 25-year-old man is brought to the emergency appetite or weight and continues to enjoy daily trips department by his fiancé, who says that over the last to the gym to work out. week the patient has been increasingly angry. He has been yelling about minor things like taking out the 44. The diagnosis that best captures her symptoms trash. His behavior has become so erratic that his is which of the following? boss is going to fire him unless he gets some help. He (A) GAD has not slept more than 3 hours a night for the past (B) acute stress disorder week, instead staying up buying over $50,000 worth of stocks. On examination, his vital signs are within (C) MDD normal limits. His speech is rapid, and you are (D) bereavement having great difficulty following his conversation (E) adjustment disorder with depressed because he jumps from topic to topic. He reports mood increased energy, but also endorses a depressed, anx- ious mood and suicidal ideation. 45. For treatment, which of the following would you recommend? 42. The most helpful test in this person would be (A) risperidone (Risperdal) which of the following? (B) fluoxetine (A) an ESR (C) psychotherapy (B) a noncontrast head CT (D) nortriptyline (C) a Venereal Disease Research Laboratory (E) alprazolam (VDRL) test (D) a toxicology screen (E) serum electrolytes 170 7: Differential Diagnosis and Management

Questions 46 and 47 touch anything that might infect her without using a handkerchief. She also admits to being very tidy at A 26-year-old woman with no previous psychiatric home as well. She tells you that she spends about history is referred to you by her primary care physi- 1 2 /2 hours in the morning getting showered. She real- cian for evaluation of an episode of anxiety. She tells izes her fears of contamination are irrational, but you that approximately 2 months ago she began every time she tries to stop she becomes increasingly having episodes lasting 10 or 15 minutes during anxious. which, she says, “I feel like I’m going to die.” During these episodes, her heart races, she feels as though 48. You discuss which of the following diagnoses she cannot catch her breath, she is dizzy and worried with the patient? she may pass out, and has tingling and tremors in her hands. She is concerned because she is now having (A) MDD problems leaving her house because she is worried (B) GAD these episodes will occur and make it impossible for (C) OCD her to get home. She cannot identify any triggers (D) panic disorder leading to the episodes. She is not on any medication (E) somatization disorder and has no medical problems. 49. Which of the following treatments do you rec- 46. You diagnose her with which of the following? ommend for the patient? (A) GAD (A) cognitive-behavioral therapy (B) panic disorder with agoraphobia (B) risperidone (C) separation anxiety disorder (C) ECT (D) social phobia (D) lithium (E) panic disorder without agoraphobia (E) psychodynamic psychotherapy 47. You initiate treatment with which of the fol- 50. A 47-year-old secretary comes to your office lowing? complaining, “I’m always worried.” She says (A) sertraline that she worries about her job, her kids, her (B) alprazolam housework, and her husband. She is seeking (C) lithium help now because she has been having an increasingly difficult time concentrating at work (D) propranolol and has been more irritable with people around (E) tranylcypromine (Parnate) her. Her sleep has been “okay,” but she does not feel rested when she gets up in the morning. She Questions 48 and 49 has been more aware of these feelings over the A 36-year-old woman with no former psychiatric his- last 2 years and they occur almost every day. tory is referred to you by a dermatologist for evalu- She denies any discrete episodes of increased ation of her chronically chapped hands. She says she anxiety. You discuss which of the following has been seeing her dermatologist for about 5 years diagnoses with the patient? for this problem and treated with a variety of topical (A) GAD agents with limited success. Over the past 3 weeks, (B) OCD her hands have become worse, to the point where (C) social phobia disorder they are always cracked and bleeding. Reluctantly, she confides in you that she has had a fear of germs, (D) panic disorder but since her colleague has been sick, she has been (E) schizophrenia washing her hands at least 40 times a day because she is afraid of contracting the disease. She also refuses to Questions: 46–54 171

51. A 28-year-old woman with no previous psy- (A) malingering chiatric or medical history is admitted to the (B) somatization disorder neurology service for evaluation of acute onset (C) hypochondriasis of numbness and weakness of the right side of (D) conversion disorder her face and right arm and leg. Physical exam- ination shows symmetrical 2/4 reflexes in all (E) factitious disorder distributions, downgoing plantar reflexes bilat- erally, and 2/5 strength in the right upper and Questions 53 and 54 lower extremity in all muscle groups. No atro- An 18-year-old woman in her first year in college phy or fasciculations are noted. Her gait is comes to see you for evaluation of depression. Her ataxic and staggering with extreme exagger- roommates also encouraged her to see someone. She ated movements of her arms; however, she reports difficulty falling asleep and early morning does not fall when ambulating without assis- awakenings, poor concentration, fatigue, and anxiety tance. Extensive neurologic workup is nega- over the past month since she arrived for the fall tive. Given the severity of her deficits, she semester. She tells you that her parents are very strict seems unconcerned by her level of disability. and she is worried that she will not get straight As. The diagnosis that best captures the patient’s On examination, she is a calm, thin woman dressed symptoms is which of the following? in a very baggy jogging suit. You comment on her (A) factitious disorder thinness, and she tells you she prides herself on her appearance and tries to stay slim by exercising about (B) undiagnosed neurologic disease 4 hours a day along with a good diet. She denies (C) conversion disorder problems with eating too much or too little and says (D) malingering just diet and exercise help her to control her weight. (E) somatization disorder Despite your concerns about her thinness, she tells you she would like to lose a few more pounds. She 52. A 54-year-old woman with a past medical his- tells you she has not had a regular menstrual period tory of hypothyroidism is admitted with a for over a year. Before you proceed with therapy, you septic right knee. The surgery team asks you to discuss the need for further testing today. evaluate the patient because they found that the fluid aspirate from the knee was growing a 53. The test that would help most in making the pathogen found primarily in the human mouth. diagnosis would be which of the following? They suspect the patient was injecting saliva into her knee. You evaluate the patient and find (A) serum potassium level her to be pleasant and cooperative. She tells (B) weight and height you that she has had a very tough time lately (C) ECG because her husband has recently been sick. (D) serum amylase Fortunately, she is a nurse and has been able to care for him at home. Lately, she admits to feel- (E) serum magnesium level ing overwhelmed and not appreciated. She has no idea what has caused the problem with her 54. Even before this test is completed, your work- knee. You talk to the family and they tell you ing diagnosis is which of the following? she is in no financial difficulty and she contin- (A) bulimia nervosa ues to enjoy work as a nurse. After working (B) body dysmorphic disorder with you for a few weeks, she admits to inject- (C) MDD ing her knee but she cannot understand why she did. You tell the surgery team that you have (D) anxiety disorder diagnosed her with which of the following? (E) anorexia nervosa 172 7: Differential Diagnosis and Management

Questions 55 and 56 swings his legs and has poor eye contact. He fre- quently clears his throat, sniffs, and blinks his eyes. A 5-year-old boy is referred to you by his pediatrician Further examination reveals that he suffers from fre- for evaluation of aggressive behavior. A pediatric quent episodes of counting numbers in his head that neurologist’s extensive workup was negative. Interview do not stop until he does it six times. He is very fussy with parents and the patient reveals a restless boy who about his clothes, requiring them to feel just right. He is able to engage in conversation. He tells you he gets often feels the need to touch smooth surfaces to see angry and frustrated in school and thinks it is boring. how they feel. He admits these habits are bother- His parents report that he has never been a strong stu- some but he cannot stop them because when he does dent. He is currently repeating kindergarten due to he gets very anxious. His mother reports that his poor performance and difficulty socializing with other father has a number of mannerisms similar to her children. His mother tells you that she still has to help son’s, such as clearing his throat and blinking his him pick out his clothes for school and get dressed. eyes, but they are noticeable only when he is under Aggressive outbursts at school seem to occur at times stress. when he does not understand the schoolwork. His family history is positive for two paternal uncles with 57. The most likely primary diagnosis is which of learning disabilities. the following? 55. You recommend that which of the following (A) Tourette disorder tests be done first in the evaluation? (B) attention deficit disorder (ADD) (A) Kohs Block Test (C) OCD (B) Minnesota Multiphasic Personality (D) normal 12-year-old behavior Inventory-2 (MMPI-2) (E) childhood schizophrenia (C) Wechsler Intelligence Scale for Children (WISC) 58. You decide to treat the patient with pimozide (Orap). Prior to starting this medication and (D) Peabody Vocabulary Test while on therapy, you will need to monitor (E) Goodenough Draw-a-Person Test which of the following? 56. A score of 69 on this test is consistent with (A) liver function which of the following? (B) CBC (A) borderline intellectual functioning (C) serum dopamine levels (B) mild mental retardation (D) blood urea nitrogen (BUN) (C) moderate mental retardation (E) ECG (D) severe mental retardation 59. A 17-year-old boy with a history of attention (E) profound mental retardation deficit hyperactivity disorder (ADHD) pres- ents with odd behavior, confusion, a blood Questions 57 and 58 pressure of 128/85 mm Hg, and a heart rate A 12-year-old boy is brought in by his mother for of 68. His parents reported that at times they evaluation of hyperactivity. She has been told by have observed him repetitively touching his teachers and has observed herself that her son has stomach and rapidly blinking his eyes. On marked difficulty sitting still and concentrating. On examination, he appears dazed and unable to examination, you note a well-developed boy who is concentrate. The most likely diagnosis is which highly distractible, walking around the room playing of the following? indiscriminately with toys for a few seconds, then (A) somatization disorder moving to the windows and the chairs. His mother (B) hypoglycemia has a hard time getting him to sit down so that you (C) amphetamine toxicosis may talk with him. When he does sit down, he Questions: 55–63 173

(D) epilepsy DIRECTIONS (Questions 62 through 73): Each set (E) MDD of items in this section consists of a list of lettered headings followed by several numbered words or 60. A 50-year-old man with a long history of IV phrases. For each numbered word or phrase, select drug use is brought to the hospital by police the ONE lettered option that is most closely associ- after a local homeless shelter worker noted him ated with it. Each lettered option may be selected to be confused and “walking funny.” While at once, more than once, or not at all. the shelter, he became very suspicious of the workers and accused them of taking his belong- Questions 62 through 67 ings. On examination, you observe that he has (A) SLE a left pupil that accommodates but does not react, depressed deep tendon reflexes in all dis- (B) human immunodeficiency virus (HIV) tributions, and loss of position sense at the great (C) hepatic encephalopathy toes bilaterally. Although the patient was treated (D) conversion disorder with antipsychotic medications in the past, he (E) pheochromocytoma denies currently taking any medication. Which (F) somatization disorder of the following items best accounts for all of this (G) hypoglycemia patient’s neurologic symptoms? (H) hyperthyroidism (A) Wernicke encephalopathy (B) Korsakoff psychosis 62. You are asked to see a 55-year-old man who (C) NMS was admitted to the hospital today. Initially, he was cooperative, but now he is very agitated (D) neuroleptic-induced dyskinesia and wants to leave. His wife reports that he (E) neurosyphilis has periods during which he becomes very upset. She also tells you that he’s a completely 61. A 57-year-old woman with no previous psy- different person from a year ago and is becom- chiatric history complains of increasing anxiety ing very forgetful. On examination, you observe over the last 2 months. Today, she reports, it that he has significant memory impairment, became “very bad.” She also notes that with asterixis, palmar erythema, and a large ecchy- these periods of anxiety she gets a pounding motic area on his right scapula. headache and once fainted. She continues to feel “shaky.” When you check her vital signs, 63. A 52-year-old man with a long history of IV her heart rate is 170 and her blood pressure is drug use is brought to the emergency depart- 230/130 mm Hg. She is diaphoretic and tremu- ment for evaluation after a worker at a home- lous. Which of the following conditions is most less shelter found the patient having a difficult likely given this patient’s physiologic symp- time trying to figure out how to use his eating toms? utensils. The worker, who has known the patient (A) hypothyroidism for the last 10 years, says that she has seen a (B) hypercalcemia drastic change in the patient over the last year. (C) pheochromocytoma He previously was jovial and interactive with the staff, but now seems disengaged and sub- (D) acute alcohol intoxication dued. She also says that he has been increas- (E) posterior circulation stroke ingly forgetful. She is concerned about other health issues as she says he has lost at least 40 lb in the last 6 months and has complained about feeling weak and losing his balance. On examination, he is withdrawn and on MMSE he scores a 20/30. 174 7: Differential Diagnosis and Management

64. A 33-year-old woman is brought to the emer- to be drawn. She is not oriented to time or gency department by her husband, who tells place. She knows she takes medications for “a you that in the last week his wife has been condition,” but cannot elaborate further. The increasingly tearful and easily upset. Over the only set of vital signs that were taken show a last few months, she has been increasingly heart rate of 140 and a blood pressure of 172/98 irritable and has a hard time sitting still. mm Hg. Physical examination is remarkable Additionally, she had told him she was having for a fine tremor of her hands bilaterally and some difficulty doing her usual tasks like diaphoresis. Five minutes after you leave the paying her bills and balancing her checkbook. room, a nurse calls to tell you that the patient She has occasional “hot flashes.” On examina- has just had a seizure. tion, she is alert and oriented, but continuously changes position in her seat and gets up fre- Questions 68 through 73 quently. Her speech is normal for rate, volume, (A) cocaine withdrawal and production. She has difficulty with simple arithmetic and short-term memory. Her deep (B) phencyclidine (PCP) intoxication tendon reflexes are brisk symmetrically through- (C) cannabis abuse out. She has a fine resting tremor of her hands (D) opiate intoxication and has difficulty rising from a seated position. (E) 3,4-methylenedioxy-methamphetamine (MDMA) intoxication 65. A 33-year-old woman presents with a 1-year (F) alcohol withdrawal history of headaches. She was diagnosed with (G) opiate withdrawal tension headaches by a neurologist after an (H) alcohol intoxication extensive evaluation that was negative. Her headaches are worsening and now she also 68. A 35-year-old woman with no significant past reports blurry vision. Despite a repeat negative medical or psychiatric history is brought to the neurologic evaluation, the patient is concerned hospital by police after crashing her car. You that she has not been treated appropriately. She notice that she is slurring her speech, her gait is volunteers that she has been sick most of her life. unsteady, and she demands her car keys so she The real trouble began for her at age 16 when can drive home. On cognitive examination, she she had surgery for presumed endometriosis, has difficulty paying attention, and tests of her which has left her unable to have sexual rela- memory show difficulty on immediate recall. tions. Since the surgery, she has intermittent Her vital signs are all within normal limits. abdominal cramping, bloating, and diarrhea. The physical and neurologic examination is Three months ago, she saw a rheumatologist for unremarkable. knee, back, and eye pain. 69. A 25-year-old graduate student is brought to the 66. A 30-year-old woman is brought to the hos- emergency department by his roommate, who pital by her fiancé. He says that over the last says that since a big party this weekend he has 2 weeks, she has been “a totally different person.” been acting unusual. This morning he has been She has not been eating or sleeping and has been walking around the room “in a ” and has irritable and angry. Earlier in the month, she com- not been eating or going to class. He is quite plained of headaches, pain in her hands and feet, disheveled, and periodically makes odd facial and a fever. On examination, she looks tired; she grimaces. Examination shows mild increased is fully oriented but has difficulty relating the muscle tone, vertical nystagmus, and a large lac- events of the last month. Occasionally, she seems eration on his upper arm that the patient reports to become confused. is not painful. Vital signs are remarkable for a blood pressure of 145/95 mm Hg and a heart 67. You are asked to evaluate a 42-year-old woman rate of 105/min. Just after you leave the room, he in the emergency department because she starts screaming at someone and throwing chairs. appears confused and will not allow any blood Questions: 64–76 175

70. A 32-year-old woman with a history of poly- (A) Quantify the average amount he drinks. substance abuse refers herself to the hospital (B) Ascertain how often he drinks. because she is feeling depressed and intensely (C) Ask him how frequently he gets drunk. suicidal and is worried she may hurt herself. (D) Ask him what his family and friends While in the emergency department, she asks say about his drinking. for something to eat and then falls asleep for the next 4 hours. Upon awakening, the patient (E) Perform a complete laboratory is no longer suicidal but does report intense, investigation. unpleasant dreams. 75. You are the consulting doctor for a corporate 71. A 17-year-old boy is brought to the emergency employee assistance program. A 53-year-old department by his parents. They are concerned man is referred to you because he has difficulty because he was previously an A student and is concentrating and is falling behind in his work. now failing all of his classes. They are worried He tells you that he feels restless and has not because the patient’s uncle was diagnosed with been able to get a good night’s sleep recently. “I schizophrenia at about the same age. You find can’t shake this feeling,” he tells you. The the patient difficult to engage in conversation symptoms have been present for about a year but otherwise without evidence of psychosis. but have worsened as the company took on a He tells you he has been “partying” more in the new client that has been very demanding of past few months but denies drug use. He has the man’s time. Which of the following is the marked conjunctival injection. He also asks if most likely diagnosis? he will be able to leave soon because he is very (A) panic disorder hungry. His only complaint is a dry mouth. (B) GAD (C) MDD 72. A 46-year-old woman complains of diffuse (D) separation anxiety muscle pain and vague abdominal cramps. She continuously yawns, blows her nose, and has (E) dependent personality disorder “goose bumps.” Questions 76 and 77

73. A 16-year-old boy is brought to the emergency You are interviewing a 54-year-old married woman department by friends after being out all night who has been urged to “see a shrink” by her family. partying. The patient is found to be diaphoretic She describes symptoms of feeling ineffectual, believ- and exquisitely sensitive to tactile sensations. ing that the world is always hostile to her, and know- Vital signs are heart rate 98, blood pressure ing that things will never change. 200/100 mm Hg, and temperature of 103°F. The patient is also noted to be wearing a pacifier. 76. This triad of symptoms is associated with which of the following disorders? DIRECTIONS (Questions 74 through 81): Each of the numbered items in this section is followed by (A) depressive disorders answer choices. Select the ONE lettered answer or (B) GAD completion that is BEST in each case. (C) panic disorder (D) schizophrenia 74. A 56-year-old man presents to your office at the (E) dissociative disorder request of his wife, who says that he drinks too much. What would be the most important strat- egy in evaluating this patient for alcoholism? 176 7: Differential Diagnosis and Management

77. Which of the following treatments would best (C) subacute combined degeneration target these symptoms? (D) alcoholic peripheral neuropathy (A) behavioral therapy (E) Ménière disease (B) paradoxical therapy 81. In Korsakoff dementia (alcohol-induced per- (C) interpersonal therapy sisting amnestic disorder), which of the fol- (D) couples therapy lowing is true? (E) cognitive therapy (A) the treatment is vitamin B12 Questions 78 and 79 (B) one often sees focal neurologic signs (C) lesions are seen in the mammillary bod- You are seeing an 18-year-old White man who is ies and thalamus brought in by police after being found passed out (D) confabulation is a rare finding on the street. He regains consciousness long enough to answer “yes” when you ask him whether he uses (E) Lewy bodies are often found drugs, and then refuses to answer your questions about which drugs. 82. A 27-year-old man complains that he has felt “down in the dumps” for months and is feeling 78. In general, more severe drug withdrawal syn- guilty because he has been having an extra- dromes are seen with which of the following? marital affair. In recent weeks, he has started to believe that his wife is poisoning his food and (A) drugs with short half-lives the rest of his family is involved in an elaborate (B) drugs with long half-lives plot to drive him from the house. Assuming (C) drugs that are more lipophilic his thinking is delusional, how would his delu- (D) drugs that are less lipophilic sions be best characterized? (E) IV drugs (A) ego-syntonic (B) bizarre 79. In general, drugs of abuse are more addictive if (C) somatic they have which of the following characteris- (D) mood congruent tics? (E) mood incongruent (A) cross the blood-brain barrier (B) have antidepressant effects 83. What diagnosis would these delusions most (C) have a shorter onset of action likely accompany? (D) have a longer onset of action (A) schizophrenia (E) can be smoked (B) mania (C) MDD Questions 80 and 81 (D) dysthymia A 52-year-old man is admitted to the hospital (E) adjustment disorder requesting alcohol detoxification. You request some of his old charts and find that he has been admit- 84. A 45-year-old man is in the emergency depart- ted for similar reasons at least six times in the past ment because of a diabetic foot ulcer. In gath- 10 years. On neurologic examination, the patient is ering a history, the physician learns that this found to have nystagmus, confusion, and ataxia. man lives alone and works nights as a security guard. He says he has no friends but that this 80. The most likely diagnosis is which of the fol- does not bother him. He has never been hospi- lowing? talized or received any psychiatric help. His affect is flat. Although he answers questions (A) Wernicke encephalopathy and seems to trust the judgment of the doctors, (B) alcoholic cerebellar degeneration Questions: 77–88 177

he has little interest in the interview. He a diagnosis of schizophrenia. When he returns exhibits no signs or symptoms of psychosis or to the hospital a month later for follow-up, the depression. What is the most likely diagnosis? patient’s speech is logical and he is able to sit and talk with the interviewer becoming rela- (A) schizotypal personality disorder tively engaged in the examination. He says that (B) schizoid personality disorder he has been hearing voices since his discharge (C) paranoid personality disorder and he believes his every word is being (D) MDD recorded by a tape recorder inside his mouth. (E) schizophrenia What is the most likely type of schizophrenia in this patient? 85. The identified patient is a 30-year-old sepa- (A) paranoid rated female brought into the emergency room (B) disorganized by her identical twin sister. The patient’s his- tory is notable for a prior episode of depression (C) catatonic 5 years ago successfully treated with venlafax- (D) undifferentiated ine. The patient has been staying with her sister (E) residual since her separation 1 month ago. For the past 2 weeks, she has been pacing around the house, 87. A 33-year-old woman with a history of paranoid not sleeping more than 2–3 hours per night. schizophrenia tells you she has been hearing Despite her feeling “sad” immediately after the voices. What is the most important information separation, the patient now feels “wonderful, to obtain regarding her auditory hallucinations? like I can accomplish anything!” In fact, she (A) whether the voices come from inside or has been attempting to remodel her sister’s outside her head bathroom, even though she has no training or experience. Her sister has been extremely con- (B) how loud the voices are cerned about her, but she has been unable to (C) how long she has been hearing voices talk to her about is as, “I can’t get a word in (D) what the voices are saying edgewise, and she doesn’t always make sense.” (E) whether she recognizes the voices The patient is only taking oral contraceptives and omeprazole for acid reflux. Her sister is 88. A 38-year-old man complains that for the past concerned that she, herself, may eventually 2 years he has required several over the develop this illness. What is her approximate course of the day; he finds the naps quite risk of developing this disease? refreshing, but sees his doctor because lately, as (A) 40–50% he is waking up, he feels momentarily “para- lyzed.” What is the most likely diagnosis? (B) 50–60% (C) 60–70% (A) cataplexy (D) 70–80% (B) hypnagogic hallucinations (E) 80–90% (C) idiopathic hypersomnolence (D) narcolepsy 86. A 19-year-old man is admitted to a psychiatric (E) advanced sleep wave syndrome hospital for the first time and discharged with 178 7: Differential Diagnosis and Management

89. A 65-year-old woman is admitted to the hospi- 91. A 57-year-old man who comes to see you for a tal for cellulitis and you are asked to evaluate consultation has diabetes that is difficult to her because the intern on duty suspects that control. she is in delirium tremens. Which of the fol- lowing statements should you keep in mind 92. A 12-year-old boy has an intelligence quotient while evaluating her? (IQ) of 72. (A) Delirium tremens is expected to occur 93. A 34-year-old woman has just gone through a 2 or more weeks after the last drink. divorce. (B) Her symptoms might be confused by a comorbid diagnosis of dementia. 94. A 45-year-old woman carries a diagnosis of (C) Auditory hallucinations are common. borderline personality disorder. (D) It is important to look for signs of auto- nomic instability. 95. A 32-year-old man carries a diagnosis of schiz- (E) As many as half of patients in alcohol ophrenia. withdrawal experience delirium tremens. 96. A 46-year-old man is diagnosed with MDD.

DIRECTIONS (Questions 90 through 101): Each of 97. A 43-year-old woman has just been diagnosed the numbered items in this section is followed by with late-stage breast cancer. answer choices. Select the ONE lettered answer or completion that is BEST in each case. For each 98. A 34-year-old woman carries a diagnosis of description, choose the axis used. bipolar I disorder.

Questions 90–101 99. A 32-year-old man who suffers from paranoid delusions is given a diagnosis of paranoid per- (A) Axis I sonality disorder. (B) Axis II (C) Axis III 100. A 78-year-old man is being seen after the death (D) Axis IV of his wife. (E) Axis V 101. An 18-year-old man has a history of heroin 90. A 32-year-old woman has been sleeping on the dependence. street for 6 months. Answers and Explanations

1. (C) The most likely diagnosis in this case is the differential diagnosis of panic disorder. schizoaffective disorder. The patient has promi- Pheochromocytomas can produce similar symp- nent psychotic symptoms, including auditory toms to panic disorder, such as flushing, hallucinations and paranoid ideation, but also sweating, trembling, and tachycardia. Cardiac has concurrent mood symptoms, including arrhythmias should obviously be considered in depressed mood, decreased sleep, anhedonia, a patient complaining of palpitations. Substance and decreased motivation. Also important, the abuse with caffeine, cocaine, or amphetamines patient’s hallucinations have occurred in the can cause symptoms similar to a panic attack absence of mood symptoms, and his mood as can withdrawal from alcohol, barbiturates, symptoms appear to have been present for a or benzodiazepines. Hypertension, folate substantial portion of the total duration of his deficiency, hypothyroidism, and lupus are not illness. The presence of prominent psychotic generally thought of as causing an anxiety-like symptoms in the absence of mood symptoms syndrome. makes major depression unlikely. A diagnosis of schizophrenia alone would not adequately 4. (C) The most likely diagnosis in this case is account for this patient’s mood symptoms. panic disorder. This patient reports some clas- Bipolar II disorder is not a valid choice since sic symptoms of a panic attack, namely an there is no clear history of a hypomanic episode. intense fear in a very short amount of time, Schizoid personality disorder is unlikely because associated with palpitations, sweating, short- that diagnosis would not account for the ness of breath, and a feeling of being unable to patient’s psychotic and mood symptoms. breathe. The patient has also related past mul- tiple episodes and that she spends a lot of time 2. (C) In schizoaffective disorder, it is important to worrying she will have another episode. These treat both the mood and psychotic symptoms. symptoms fit criteria for a panic disorder and Because the patient’s symptoms mainly con- not GAD, social phobia, or OCD. Agoraphobia is sist of depression and psychosis, a combina- the fear of crowded spaces. tion of an antipsychotic and antidepressant medication would be a reasonable approach. 5. (C) This patient presents with the signs of Mirtazapine, citalopram, fluoxetine, and ser- Wernicke encephalopathy, which results from

traline are antidepressant medications, while thiamine (vitamin B1) deficiency. It is charac- ziprasidone, haloperidol, and perphenazine are terized by bilateral abducens nerve palsy, antipsychotic medications. Lorazepam and horizontal nystagmus, ataxia, and a global con- diazepam are benzodiazepines. Divalproex fusion accompanied by apathy. Each of the sodium is the only mood stabilizer listed. other choices are possible and should be actively ruled out. Alcohol withdrawal usually 3. (C) Hyperthyroidism, hyperparathyroidism, presents with unstable vital signs, tremulous- pheochromocytoma, cardiac arrhythmias, and ness, and agitation. Folic acid deficiency pres- substance abuse should all be considered in ents with diarrhea, cheilosis, and glossitis;

179 180 7: Differential Diagnosis and Management

neurologic abnormalities are usually not seen. neuroleptic exposure, makes the diagnosis NPH is associated with the classic triad of more or less likely. Anticholinergic syndrome, dementia, incontinence, and gait disturbance. resulting from overdosing on anticholinergic medications, does not produce rigidity and an 6. (D) If Wernicke is suspected, emergent admin- elevated CPK. Malignant hyperthermia, an istration of IV thiamine is essential, because acute muscular pathologic process, resembles many sequelae of thiamine deficiency are NMS, but follows the administration of inhaled reversible with this treatment; mistaken admin- anesthetic agents, as in general surgery. A diag- istration of thiamine is rarely harmful. A CT nosis of CNS infection would be better sup- scan of the head should be ordered immedi- ported by findings on the lumbar puncture and ately in this case; optimally, the IV thiamine CT or MRI scan; an elevated CPK would be would be started as the CT was being arranged. possible if the CNS infection caused seizures. The remaining choices do not address the Prolonged immobilization could result in an urgent need to replete thiamine. elevated CPK, but would not account for the other findings. 7. (C) This patient is clearly psychotic and the delusion has progressed sufficiently far as to 10. (C) Command auditory hallucinations are a psy- render the patient potentially dangerous to chiatric emergency and most clinicians would others. Therefore, most clinicians would prefer agree that this patient should be restarted on a to address his delusional thinking with an neuroleptic. Because they are long-acting (i.e., antipsychotic, in this case olanzapine, a newer hard to stop), depot injections of a neuroleptic atypical antipsychotic. Although alprazolam, are not a first choice. Because the patient is not a short-acting benzodiazepine might provide violent, physical restraints should be avoided. temporary relief from the stress of the delu- ECT has some efficacy in treatment-resistant sion, it would provide little long-term benefit. schizophrenia but is not a first-line therapy. ECT, lithium, and SSRIs have been shown to be helpful in some cases of delusional disorder, 11. (C) Command auditory hallucinations to per- but they would not be a first choice in this form suicidal attempts such as overdosing on patient with severe psychosis. medications are a psychiatric emergency and in virtually all cases require a psychiatric admis- 8. (D) Although tactile and olfactory hallucina- sion. We are given no clues that there is a neces- tions may be present, the presence of auditory sity to admit the patient to a medical unit; hallucinations would suggest a diagnosis such however, she will need a complete medical as schizophrenia. In delusional disorder, the workup to rule out organic causes of depres- delusions are not bizarre (logically impossible); sion. This can usually be accomplished on a usually, they have some internal logic and psychiatric unit. We are given no indication, could conceivably be possible. As in this case, such as violent behavior, that this patient needs negative symptoms are not generally a part of to be restrained. the presentation. The most common delusions are of the persecutory type. 12. (C) This patient requires an antidepressant. Treating her depression is likely to relieve her 9. (E) NMS is characterized by severe (“lead psychotic symptoms as well, but the antide- pipe”) rigidity, change in mental status, auto- pressant cannot be expected to start taking nomic instability, elevated CPK, and elevated effect for at least a week and likely longer. In white blood count; a slight elevation in cere- fact, either an antidepressant or an antipsy- brospinal fluid protein count is possible. NMS chotic would not treat this condition as well may be induced by any neuroleptic including as both together. Therefore, many clinicians the newer atypicals. No one symptom is nec- would add a neuroleptic, a class of drugs that essary for the diagnosis; instead, a constellation is relatively quick acting, to treat her auditory of symptoms and their severity, in a setting of hallucinations. A TCA would not be the first Answers: 6–19 181

choice. First, there is a suggestion that this hyperarousal states, dissociative states, and patient has some cardiac history and the tri- intrusive reexperiencing of the event (e.g., flash- cyclics can have a proarrhythmia effect. Second, backs and avoidance behaviors). PTSD is simi- unlike SSRIs, tricyclics can be lethal in over- lar in most respects to acute distress disorder, dose and this patient has risk factors for over- but in PTSD the symptoms persist for at least 4 dose. Lithium does have some antidepressant weeks following the trauma; a shorter duration effects but in the absence of convincing evi- suggests a diagnosis of acute distress disorder. dence that this patient has a bipolar disorder The patient does not appear to be suffering from lithium is not used. (If this patient has only a any of the typical criteria for MDD or adjust- partial response to the SSRI, lithium may at ment disorder, except for guilt. Although she is some point in the future be added for aug- suffering from agoraphobia, a symptom con- mentation.) Although a benzodiazepine may sistent with an acute stress reaction, there is no help this patient’s irritability, as a sole agent it evidence of panic disorder. is a poor choice for treating her depression and psychosis. 16. (C) Although MDD, panic disorder with ago- raphobia, and adjustment disorder are all in 13. (B) Initial medications in this case are aimed at the differential diagnosis at the initial pre- the target symptoms of agitation, delusional sentation and should be carefully considered, thinking, and disruptive behavior and could MDD does not emerge as a clear diagnosis until reasonably include a neuroleptic and a benzo- this question’s additional information is pre- diazepine. Although he clearly needs a mood sented. At this time, the principal symptoms of stabilizer such as lithium or carbamazepine as the acute stress disorder have remitted, and well, these are not expected to start showing an the classic symptoms of depression—feelings effect for at least a week and are not the critical of sadness, loss of interest, a change in sleep medications to start emergently. SSRIs must be habits, tearfulness, and change in appetite are used with extreme caution in bipolar illness apparent. These outweigh the earlier working because they can incite or exacerbate mania. diagnoses. Buspirone is an anxiolytic used mostly in GAD. It is of extremely limited use in bipolar disorder 17. (C) Oculogyric crisis is a specific example of an and does not adequately address the target acute dystonic reaction in which there is spasm symptoms seen in this case. of the muscles of extraocular motion. Torticollis and retrocollis refer to muscle spasms that cause 14. (D) When the patient returns for his first abnormal positioning of the head. Trismus is a follow-up visit, his lithium level is therapeutic spasm of the jaw muscles. NMS, also a reaction at 0.8 mEq/L and is not in need of adjustment. to neuroleptic medications, is characterized by However, his muscle stiffness is a parkinsonian dystonia, autonomic instability, and usually side effect of haloperidol. This is normally some degree of delirium. treated by lowering the dose of the neuroleptic or with anticholinergic medication not with a 18. (D) Benzodiazepines, SSRIs, and cognitive- muscle relaxant such as baclofen. The patient behavioral therapy each have a place in treat- would have to be followed closely for reemer- ment of the anxiety disorders. In this case, gence of psychotic symptoms. however, these should not be considered until an organic cause is ruled out. 15. (B) Acute stress disorder is a reaction that causes clinically significant disruption or distress in a 19. (A) Regardless of a patient’s past psychiatric patient who witnesses or somehow partici- diagnosis, organic causes of physical signs and pates in a traumatic event. The event must be symptoms must always be ruled out. The anx- quite horrible, invoking in the patient intense iety disorders, although intensely distressing fear, and usually involving death or threat- and uncomfortable to patients, rarely, if ever, ened death. The typical symptoms include result in unconsciousness. Many aspects of the 182 7: Differential Diagnosis and Management

patient’s story seem to be related to anxiety and PCP. Amphetamines, cocaine, and PCP and could in fact occur during a panic attack, intoxication can mimic symptoms of schizo- but this patient’s presentation is more consis- phrenia. Although a CT scan is helpful to rule tent with ischemic heart disease and should out a mass lesion or bleeding, it is not likely in prompt a cardiac workup. this age group nor is it likely, given the nonfo- cal neurologic examination, to provide further 20. (B) Most clinicians agree that after 10 weeks at information influencing treatment. Liver func- a reasonable dose, if one SSRI is not working, tion tests, CBC, and ESR are unlikely to aid in it is time to stop the medication and try a new the diagnosis. one. Monoamine oxidase inhibitors (MAOIs), however, require a 2-week washout period 24. (C) Delusions, hallucinations, and disorganized before initiation. Lithium is usually used only behavior are some of the hallmark symptoms as augmentation to antidepressant medication of schizophrenia. Because the symptoms have when at least a partial response is obtained been present for more than 1 month but less from the antidepressant. In the absence of evi- than 6 months and there is no previous psy- dence that the depression has remitted, there is chiatric history, the most appropriate diagnosis no advantage gained in discontinuing all med- is schizophreniform disorder. There is no his- ications. tory suggestive of a mood disorder and thus major depression with psychotic features is not 21. (E) Despite the tremendous toll binge eating correct. The toxicology screen is negative, so and purging takes on the body, thyroid function the diagnosis of substance-induced mood dis- tends to remain intact. However, each of the order is less likely. Delusional disorder usually other choices offers the opposite trend in what manifests in a person who is functional in soci- one tends to find in bulimia. Laboratory abnor- ety, with the exception of a circumscribed delu- malities include hypokalemic, hypochloremic sion, which can be of the jealous, erotomanic, alkalosis; high serum amylase; and hypomag- grandiose, persecutory, or somatic type. The nesemia. The salivary and parotid glands and patient is also suffering hallucinations, which the pancreas tend to enlarge. in combination with delusions exclude the diagnosis of delusional disorder. 22. (D) The fluid shifts and electrolyte imbalances introduced by excessive vomiting may cause 25. (C) Benzodiazepines are the treatment of choice seizures. Hospitalization is only required if for alcohol withdrawal. Any benzodiazepine patients become severely underweight or phys- can be used—the choice depends on the ically compromised. Medications have been of desired route of delivery or the presence of limited success in the treatment of anorexia. liver dysfunction. With abnormal liver func- However, many studies have documented the tion tests, lorazepam or oxazepam (Serax) benefits of SSRIs and TCAs in controlling would be chosen because they are primarily bulimic behaviors. Simply prescribing an SSRI to metabolized and eliminated renally instead of this patient, however, would be a woefully inad- through the liver as are the other benzodi- equate intervention. Typically, bulimic patients azepines. Additionally, lorazepam may be are steered toward a combination of pharma- chosen because it can be given IV, intramuscu- cotherapy and some form of talk therapy (e.g., larly (IM), or by mouth (PO). Methadone or cognitive-behavioral therapy, group therapy). clonidine may be used for opiate withdrawal. Carbamazepine in conjunction with propra- 23. (B) The key points in this case presentation are nolol has been advocated in some settings for the absence of a previous psychiatric history alcohol withdrawal, but carbamazepine is not and the duration of symptoms. Before diag- used as single-agent therapy. Naltrexone has nosing a mental illness, it is necessary to rule been used to help maintain abstinence from out the effects of a substance. A urine toxicol- alcohol but has no efficacy in withdrawal. ogy screen usually includes opiates, cocaine, Answers: 20–32 183

26. (C) Alcohol withdrawal can be life-threatening. signs may occur; however, tachycardia and Tremors begin about 6–8 hours after cessation, hypertension should always signal possible followed within 8–12 hours by psychotic and alcohol withdrawal. Pharmacologic manage- perceptual abnormalities (hallucinosis). Seizures ment of withdrawal can include clonidine, a may follow anywhere from 12 to 72 hours centrally acting alpha-2-agonist; buprenor- after cessation. Delirium tremens (DTs) may phine, a mixed opiate agonist/antagonist; and occur any time in the first week of abstinence. methadone, a synthetic long-acting opiate. Untreated DTs have a mortality rate of approx- Lorazepam can be used to treat elevated vital imately 20%. Fatty liver is a long-term conse- signs. quence of alcohol abuse. Abdominal pain and muscle cramps can be symptoms of opiate 30. (B) Your concern is greatest with sudden dis- withdrawal. continuation of any sedative, hypnotic, or anxi- olytic agent, which can cause seizures. Triazolam 27. (E) A toxicology screen is the best way to diag- is a short-acting benzodiazepine. Thioridazine nose this condition. Often, a urine toxicology and fluphenazine are neuroleptics, and nor- screen include opiates, cocaine, and PCP, but triptyline and imipramine are TCAs. Neither this varies depending on individual institu- TCAs nor neuroleptics cause life-threatening tions. There is no indication for an EEG at this withdrawal symptoms. time because no seizure history is elicited and the patient does not appear postictal. Although 31. (B) As with the sudden discontinuation of alco- structural brain lesions can cause psychotic hol, benzodiazepine or barbiturate withdrawal symptoms, substance-induced symptoms are can be life-threatening. Withdrawal symptoms more common in this age group. A nonfocal include tremors; anxiety; auditory, visual, and neurologic examination would further decrease tactile hallucinations; autonomic hyperactivity; the likelihood that imaging studies would alter and seizures. The onset of symptoms depends the diagnosis or treatment plan in this case. on the half-life of the medication, the dose, and duration of use. The most concerning discon- 28. (A) Unless the patient is extremely agitated tinuation side effect is seizures. and at risk for hurting himself or others, the best choice for treatment would be to place him 32. (D) The patient is exhibiting typical signs of in a quiet, dark (low stimulation) room. If the delirium, including impaired consciousness patient is violent and/or psychotic, a combi- and difficulty with cognition, manifested as nation of neuroleptics and benzodiazepines drowsiness and disorientation. Classically, could be used to keep the patient safe. these symptoms may fluctuate during the day. Methylphenidate is a stimulant; benztropine, Patients who meet criteria for delirium may used to prevent extrapyramidal symptoms appear depressed with a decreased level of associated with neuroleptic use, might worsen arousal or may appear overly active and ener- the patient’s delirium. Because the patient is getic, with the depressed type occurring more not suffering a seizure or at risk for seizures, commonly. Patients also show cognitive and phenytoin is not indicated. memory impairments, which may be mani- fested as disorientation or decreased capacity to 29. (B) The first part of the vignette illustrates some register, retain, and recall information. An of the features of opiate intoxication, namely MMSE during periods of delirium may be mis- pupillary constriction, drowsiness, impaired leadingly low, suggesting a dementing or pseu- attention and memory, and slurred speech. In dodementing process. In this setting, it is the second interaction, the patient shows signs important to gather information from the and symptoms of opiate withdrawal. Yawning, people who know the patient to evaluate for a muscle aches, diarrhea, lacrimation or rhinor- possible coexisting depression or dementia. Risk rhea, and fever are all typical of opiate with- factors for delirium are extremes of age, prior drawal. Additionally, mildly elevated vital history of delirium, alcohol dependence, sensory 184 7: Differential Diagnosis and Management

impairment (auditory or visual), and preexisting time and may be associated with periods of brain damage. There is a broad etiologic differ- agitation and confusion. Alzheimer dementia is ential for delirium. In this case, causative factors the most common form of dementia, followed may be infection, hypoxia, metabolic abnormal- by vascular dementia. Together, these account ities, and current medications. for between 70% and 80% of all dementias.

33. (E) Haloperidol is the drug of choice for man- 36. (D) This patient is exhibiting signs and symp- aging the agitated or confused patient with toms consistent with MDD with psychotic fea- delirium. Thioridazine, benztropine, and diphen- tures. For more than 2 weeks, the patient has hydramine all have anticholinergic properties displayed depressed mood, loss of interest in that may exacerbate delirium. Lorazepam, pleasurable activities, weight loss, inability to especially in older patients, may have a para- concentrate, poor sleep, and guilt about her doxical effect and exacerbate agitation and con- current depressive state. She is also suffering fusion. Haloperidol in low doses, such as 0.5 mg delusional-type concerns about bodily func- every 8 hours, helps to manage the agitated tioning and possible infection. If the patient patient. It has the advantage of IM, IV, or PO were complaining of contamination fears and administration. performing multiple compulsive tasks to rid herself of the pathogens, a diagnosis of OCD 34. (A) The most important next step is to perform with comorbid depression could be considered. a physical examination. Particular attention Somatization disorder is characterized by more should be concentrated on the neurologic somatic complaints as well as at least one neu- examination looking for any evidence of focal rologic and pain symptom. Dysthymia presents deficit suggesting an acute neurologic event with a chronic moderate level of depressive that may have led to the sudden decline. The symptoms, but this patient is describing an remainder of the test may be helpful in assess- episodic decompensation over the last month. ing the etiology of the deficit. 37. (B) Because of the delusional quality to her 35. (C) A sudden decline in an otherwise slowly presentation, it is reasonable to treat this patient progressive course should alert the clinician to with a neuroleptic such as perphenazine for a evaluate for an underlying cause. In this case, few days prior to beginning a trial of an antide- the patient is hypertensive, which increases the pressant. Antidepressants alone or with benzo- risk for stroke and therefore vascular dementia. diazepines would not help with the delusional Further workup for reversible causes of demen- aspects of her symptoms. Similarly, lithium and tia should include a CBC with differential; elec- divalproex sodium are used as mood stabilizers

trolytes; BUN; creatinine; B12; folate; VDRL; in bipolar depression or mania but not com- calcium, magnesium, and phosphorus; urinal- monly in the treatment of unipolar depression. ysis; ESR; urine toxicology screen; thyroid stud- ies; liver function tests; ECG; and chest x-ray. If 38. (C) Brief psychotic disorder is diagnosed when focal neurologic deficits are found on the neu- symptoms have been persistent for more than rologic examination, a noncontrast head CT 1 day but less than 1 month. Delusional disorder could be helpful in the assessment. The differ- is a rare condition in which the primary symp- ential diagnosis of dementia includes pseudo- toms are delusions that are usually nonbizarre; dementia (depression) and delirium. Depressed that is, they could occur. Delusional disorder individuals may show evidence of cognitive is further classified into subtypes, including impairment that can appear identical to demen- grandiose, jealous, persecutory, somatic, ero- tia, which makes obtaining a history from those tomanic, and unspecified. There are no symp- who know the patient crucial. Similarly, delir- toms of a mood disorder and the other ium may also present with cognitive deficits. In associated symptoms of schizophrenia are not delirious states, these tend to fluctuate over present. Answers: 33–46 185

39. (A) The most helpful test is a urine toxicology 3–5 days. Prior to the onset of mood-stabilizing screen. Intoxication with a variety of substances properties, a benzodiazepine such as lorazepam including cocaine, PCP, or amphetamines may can be useful in keeping the patient calm. An mimic the symptoms of schizophrenia, and SSRI such as fluoxetine is not indicated in bipo- thus it is necessary to rule out these intoxication lar disorder and can precipitate or worsen mania. syndromes. A CT scan helps to rule out a mass lesion or a bleed, but in this age group and 44. (E) Adjustment disorder is characterized by the with a nonfocal neurologic examination, it is onset of emotional or behavioral disturbances unlikely to contribute to the diagnosis and within 3 months of a significant life event that treatment. Because the patient does not present may manifest as marked change in an individ- with signs or symptoms of a seizure disorder, ual’s ability to function in school, work, or an EEG is unlikely to provide information that interpersonal relationships. The disturbances will impact the diagnosis in this case. There is are not so severe, however, as to suggest the no diagnostic finding on CBC or electrolytes diagnosis of another disorder such as MDD. specific for any of the psychotic disorders. The differential diagnosis for adjustment dis- order includes mood disorders such as MDD 40. (B) There are two choices for antidepressants in and anxiety disorders such as PTSD, GAD, and this group. Phenelzine, an MAOI, is not the acute stress disorder. In this case, the patient first choice of antidepressant because the SSRIs may be suffering from depressed mood but does have a more favorable side effect profile, are not have other neurovegetative signs of depres- safer in overdose, and have similar efficacy in sion. Acute stress disorder is usually diagnosed the treatment of MDD. Fluoxetine, therefore, is when an individual who has experienced an the best choice at this time. Triazolam is a ben- extreme stressor has dissociative experiences zodiazepine, flumazenil is an opioid antago- and anxiety after the stressful experience. This nist, and fluphenazine is an antipsychotic. patient is not describing these symptoms. Similarly, PTSD usually requires exposure to 41. (A) SSRIs have a number of side effects, includ- or witnessing of a life-threatening event, with ing diarrhea, constipation, insomnia, nausea, episodes of reexperiencing the event and headache, sexual dysfunction, and agitation. making efforts to avoid settings that may The side effect shared by the symptoms of remind the patient of that experience. In this depression is insomnia. Constipation and case, the patient has experienced a significant akathisia are also common side effects of stressor, but it is not a life-threatening event antipsychotics. that is usually required for the diagnosis of acute stress response or PTSD. Bereavement is 42. (D) It is essential to evaluate whether the the normal response to the loss or death of an behavioral disturbance is caused by an exoge- individual close to the patient. nous substance. The symptoms of bipolar ill- ness can be mimicked by intoxication with a 45. (C) The diagnosis of adjustment disorder does number of substances, including ampheta- not usually require pharmacotherapeutic inter- mines and cocaine. A CT scan, VDRL, ESR, and vention, but instead may respond to supportive electrolytes may all be appropriate to evaluate individual or group psychotherapy. Risperidone initially but are unlikely to change the diagno- is an atypical antipsychotic. Fluoxetine is an sis, and therefore are of limited utility in SSRI, nortriptyline is a TCA, and alprazolam is making the diagnosis of bipolar illness. a benzodiazepine.

43. (D) Given the severity of this patient’s symp- 46. (B) This patient is describing symptoms asso- toms, it is likely that he will require the use of ciated with panic disorder. This disorder occurs both lorazepam and valproic acid to control in late adolescence or the early twenties and is his illness. Divalproex sodium can be loaded more common in women than men. People rapidly to achieve therapeutic blood levels in typically describe feelings of fear, dread, or 186 7: Differential Diagnosis and Management

intense discomfort associated with a variety of Patients realize that the obsessions and compul- somatic symptoms, including tachycardia, sions are intrusive, unreasonable, and unwanted chest pain, shortness of breath, tremulousness, and cause marked impairment in their lives. diaphoresis, nausea, fear of dying, paresthe- Many patients with OCD may hide their symp- sias, light headedness, and hot or cold flashes. toms and may first come to the attention of The diagnosis of agoraphobia is made when other medical specialists. Somatization disor- fears of having an attack or being unable to der presents with numerous somatic complaints leave a place where they are having an attack that cannot be fully explained by the results of force patients to remain in familiar places. The medical investigations. GAD is an appropriate medical workup for panic disorder includes diagnosis in people who have worries and con- CBC, electrolytes, fasting glucose, calcium, liver cerns that are not limited to any particular function tests, BUN, creatinine, urinalysis, tox- aspect of their lives, with the anxiety signifi- icology screen, ECG, and thyroid function stud- cantly impairing their ability to function. Panic ies. A careful substance abuse and caffeine disorder occurs as recurrent, discrete episodes intake history should also be taken. Social of extreme anxiety and fear that occur unex- phobia or social anxiety disorder occurs in the pectedly and remit spontaneously. context of public appearances or performances. Individuals may experience anxiety and fear. 49. (A) A common intervention for OCD is cognitive- These feelings cause great distress and may behavioral therapy, during which the patient is cause people to limit or avoid these settings. exposed to the threat or fear (obsession) with This disorder differs from panic disorder by the response (hand washing) prevented for having an identifiable trigger. Separation anx- progressively longer periods of time until the iety disorder is commonly diagnosed in young behavior is extinguished. SSRIs have been suc- children when separated from their primary cessful in treatment of OCD. Risperidone is an caregiver. GAD occurs in individuals expe- atypical neuroleptic that is used to treat psy- riencing continued anxiety and worry for periods chotic disorders. Lithium is used in the treat- greater than 6 months, occurring throughout ment of bipolar disorder and has not been the day. Unlike panic disorder, there do not shown to be effective in the treatment of OCD. seem to be discrete episodes. Psychodynamic psychotherapy is intensive, long-term therapy that may be helpful for 47. (A) A variety of medications, including SSRIs people who wish to better understand the (sertraline), TCAs, MAOIs (tranylcypromine), nature of their relationship with others. and benzodiazepines (alprazolam) have been used to treat panic disorder. Although all have 50. (A) Symptoms of excessive worry and anxiety shown some efficacy in the treatment of panic, occurring for over 6 months without discrete SSRIs are usually the first agent of choice episodes are best characterized by the diagno- because they are generally well-tolerated, with sis of GAD. Other symptoms of GAD include fewer side effects than TCAs or MAOIs, and sleep difficulties, irritability, and difficulty con- lack the dependency potential of benzodi- centrating. Patients with GAD may also suffer azepines. Common side effects include sexual from muscle tension, fatigue, and restlessness. dysfunction, gastrointestinal disturbances, and Although this patient does admit to anxiety, it insomnia. Lithium is used in the treatment of does not occur in episodic fashion or in bipolar disorder, and propranolol is used for response to public performances, as with panic social phobia. disorder or social phobia. Similarly, although the symptoms of anxiety and worry are in 48. (C) The patient has OCD in which she suffers excess of what would be expected, they do not from recurrent, intrusive, unwanted thoughts reach the magnitude characteristic for OCD, or images (obsessions) and either repetitive nor are any compulsions elicited. There is no behavior or mental acts (compulsions) that are evidence of psychosis or a thought disorder in performed in response to these obsessions. this patient. Answers: 47–55 187

51. (C) The diagnosis of conversion disorder is form of psychotherapy or pharmacotherapy most likely given that the primary symptoms has been identified for the treatment of facti- are neurologic and exhaustive workup is neg- tious disorder. ative for evidence of pathology that accounts for the deficits. Neurologic deficits in conver- 53. (B) Obtaining height and weight is crucial for sion disorder involve either motor or sensory establishing the diagnosis of anorexia nervosa. modalities and are felt to be a result of psy- Patients with anorexia nervosa are unable to chological stressors. Had the symptoms been achieve or maintain greater than 85% of expected intentionally produced, the diagnosis of facti- body weight for height and frame structure. tious disorder or malingering would be appro- Additionally, they have continued concerns priate. If the primary incentive for the behavior about thinness and body image despite being was to assume the sick role, the diagnosis underweight. In contrast, most patients with would be factitious disorder. If the symptoms bulimia nervosa are of normal height and are feigned or exaggerated for another reason, weight or are slightly overweight. Some people such as to gain monetary reimbursement, the with bulimia nervosa engage in recurrent binge diagnosis would be malingering. Somatization eating with compensatory purging (e.g., via disorder is not appropriate in this case because vomiting or laxatives) or other behaviors to the number and types of complaints do not control weight in the absence of binge eating. extend past the neurologic complaints. An Both disorders occur more commonly in undiagnosed neurologic condition is a possi- women than men. Anorexia tends to begin in bility, but given the extensive workup, it is less the teens to twenties, whereas bulimia’s onset likely. It is important to note that as many as is somewhat later in the twenties. Patients with 50% of those diagnosed with conversion dis- anorexia have a higher risk of comorbid disor- order are eventually diagnosed with a neuro- ders, including depression and anxiety. logic condition that could have produced the initial symptoms. 54. (E) Patients with eating disorders may have a variety of medical complications including (1) 52. (E) Factitious disorder is an appropriate diag- amenorrhea secondary to starvation and alter- nosis when physical or psychological symp- ations in hormonal regulation, (2) electrolyte toms are intentionally produced in order to abnormalities (low serum magnesium and assume the sick role. It must be clear that these potassium) due to purging, (3) elevated amy- symptoms are not intentionally produced for lase, and (4) loss of cardiac muscle with result- other reasons, such as avoiding work, gaining ing conduction abnormalities detected by ECG. monetary compensation, or avoiding legal Body dysmorphic disorder is a somatoform issues. If any of these reasons are present, the disorder with a preoccupation of physical diagnosis is malingering. Somatization disor- imperfection. Anorexia and bulimia are con- der presents with complaints of pain, gas- sidered eating disorders and not somatoform trointestinal symptoms, sexual dysfunction, disorders. and pseudoneurologic symptoms. The diag- nosis of hypochondriasis is incorrect because 55. (C) Given the patient’s history of angry, aggres- this patient is not preoccupied with the idea of sive outbursts around not understanding having a severe disease that is unrecognized schoolwork and difficulties with socialization despite repeated reassurances by medical per- and caring for himself along with a family his- sonnel. Classically, people with factitious dis- tory of learning disabilities, you suspect the order are more likely to be female and to have diagnosis of mental retardation. A diagnosis of experience in health care. Often, these patients mental retardation includes subnormal func- will have numerous admissions to many dif- tioning with an IQ of less than 70. Either the ferent hospitals with a variety of symptoms. WISC or the Stanford-Binet Intelligence Scale is The psychological factors underlying this dis- an adequate instrument for estimating IQ. The order are not well understood. No effective MMPI-2 is frequently used to assess personality 188 7: Differential Diagnosis and Management

structure, and the Goodenough Draw-a-Person intoxication. Amphetamines are commonly used Test and Kohs Block Test assess visual-motor to treat ADHD and may be drugs of abuse in this coordination. The Peabody-Vocabulary Test is age group. Intoxication with amphetamines may used when patients suffer from a language present as seizures and can be easily ruled out by barrier. a urine toxicology screen.

56. (B) Mental retardation is classified according to 60. (E) Although syphilitic infections have been on severity based on IQ scores: mild mental retar- the decline since World War I, they must still be dation, 50–70; moderate, 35–49; severe, 20–34; considered in the differential for a wide variety and profound, less than 20. Borderline intellec- of psychiatric disorders, especially when com- tual functioning is a term no longer in use, but bined with neurologic abnormalities. In this referred to IQ scores in the 71–84 range. case, two forms of neurosyphilis (tabes dor- salis and general paresis) are combined in one 57. (A) Tourette disorder is a syndrome of multiple patient. The taboparetic form occurs in about motor and vocal tics that occur daily for at least 20% of patients with neurosyphilis. Patients 1 year, with the onset before age 18. Tourette demonstrate sensory ataxia with a wide-based occurs more commonly in men than women gait, a positive Romberg sign, and a loss of and is associated with attention deficits and vibratory and proprioceptive senses initially obsessive and compulsive behaviors. Currently, in the lower extremities. Deep tendon reflexes it is unclear whether obsessive-compulsive are decreased and pupil abnormalities are behaviors and ADD behaviors are separate enti- common, although the full Argyll-Robertson ties from Tourette or if these behaviors are pupil is infrequent. General paresis is usually always comorbid. This patient demonstrates associated with a dementing process along with none of the psychoses that characterize child- neurologic symptoms such as pupil abnormal- hood schizophrenia. ities, tremors, dyscoordination, and spasticity in the lower extremities. Testing for neurosyphilis 58. (E) Prior to induction of therapy with pimozide, should include a blood test for VDRL and fluo- and while in active treatment, ECGs should be rescent treponemal antibody absorption test performed to evaluate the QT interval. Pimozide (FTA-ABS). Consideration should also be given has been associated with QT prolongation and to performing a lumbar puncture to test cere- an increased risk of ventricular arrhythmias. brospinal fluid for evidence of treponemal infection. Although this patient has some 59. (D) Epilepsy is a complex set of disorders that symptoms of Wernicke encephalopathy and may initially come to the attention of a psychi- Korsakoff psychosis, neither diagnosis alone atrist because of unusual behaviors. Complex would account for this patient’s constellation of partial seizures, in particular, may be associated neuropsychiatric symptoms. Finally, a diagno- with a wide range of symptoms, including sen- sis of NMS should not be entertained because sory auras and automatic behaviors. Prior to he is currently not taking an antipsychotic the seizure, the patient may notice a bad smell, medication. an unusual taste, or gastrointestinal symptoms. During the event, the patient may look dazed or 61. (C) Pheochromocytoma is a tumor of the chro- frightened and may exhibit a number of automa- maffin cells, usually occurring in the adrenal tisms such as lip smacking, eye blinking, and medulla. Symptoms may differ according to fumbling with his or her clothes. Additionally, the predominant catecholamine released. As there is impairment of consciousness. Complex norepinephrine-secreting tumors are the most partial seizures are usually associated with tem- common, baseline hypertension with paroxys- poral lobe epilepsy but can also be derived from mal exacerbations are frequent. Anxiety, fear, lesions elsewhere in the brain. In this case, and palpitations may accompany exacerba- another diagnosis to rule out is amphetamine tions of hypertension. Because this may mimic Answers: 56–67 189

symptoms of a panic attack or GAD, it is essen- Hyperthyroidism may cause mild cognitive tial to consider this in the differential diagnosis deficits in calculation and recent memory. when patients present with anxiety. Neurologically, patients may show a fine 8- to 12-Hz tremor, lid lag, brisk deep tendon 62. (C) Hepatic encephalopathy can present in reflexes, proximal myopathy with muscle wast- many ways and may have a fluctuating course ing, and myalgias. Patients may report hot making it difficult to diagnose without periph- flashes and/or increased sensitivity to heat. eral stigmata. Neurologically, the patient may exhibit asterixis, which, although characteristic 65. (F) Somatization disorder is characterized by a for hepatic encephalopathy, is not specific for variety of somatic complaints that cannot be the disorder. Patients may suffer 6- to 9-Hz verified by laboratory and physical findings. tremors, mildly increased deep tendon reflexes, The disorder has its onset before age 30 and is and an altered sensorium. Typical psychiatric more common in women than in men. Diagnosis symptoms may include changes in personality, is based on the occurrence of four pain symp- abrupt mood swings, and changes in cognitive toms, two gastrointestinal symptoms, one sexual ability. In an acute decompensation, depres- symptom, and one pseudoneurologic symptom. sion, catatonia, psychosis, and delirium may It is important to determine that these symp- develop. Physical stigmata reflecting changes toms are not intentionally produced or feigned in liver function can include ecchymosis, for other purposes. Similarly, it is important to peripheral edema, ascites formation, palmar ensure that the patient has received the appro- erythema, and spider angiomas. priate medical workup because a number of dis- ease processes may present with a constellation 63. (B) HIV may produce a spectrum of neuropsy- of symptoms. chiatric manifestations as a result of oppor- tunistic infections, neoplasms, or direct invasion 66. (A) Neuropsychiatric complications occur in of the CNS by the virus. HIV-associated demen- up to 60% of patients with SLE. Although neu- tia is one of the most common manifestations of ropsychiatric conditions may arise at any time primary CNS invasion. It may present in a during the course of the disease, they are most variety of ways, including motor abnormali- frequent during acute exacerbations or late in ties, cognitive decline, or behavioral changes. the disease process. Of the psychiatric mani- This case presentation describes typical behav- festations, organic mental syndromes, includ- ioral change, apathy, and social withdrawal. ing “lupus psychosis,” schizophrenia-like HIV dementia may also have neurologic abnor- syndromes, and affective disorders are the malities such as weakness, imbalance, or ataxia. most common. Organic mental syndromes Occasionally, patients present with irritability include psychotic symptoms, delirium, per- and apathy. Cognitively, this patient has been sonality changes, anxiety, and emotional described as increasingly forgetful, and the lability. Patients may also exhibit cognitive results of the MMSE support the diagnosis of impairment on neuropsychological testing. dementia. Criteria for dementia include the Most of the reactions are short, lasting hours to presence of multiple cognitive deficits, includ- days, and are associated with flares of the ill- ing aphasia, apraxia, agnosia, or disturbance in ness. Affective illnesses and schizophrenia-like executive function. syndromes are less common. Neurologically, patients can present with seizures, stroke, cra- 64. (H) Hyperthyroidism may present with symp- nial nerve abnormalities, and headache. toms consistent with a variety of psychiatric disorders, including depression, anxiety, psy- 67. (G) This patient is exhibiting signs and symp- chosis, mania, and, at an extreme, delirium. toms consistent with a delirium secondary to This case illustrates a more moderate pres- hypoglycemia. In this case, a psychiatric con- entation of hyperthyroidism with some sultation was likely requested to evaluate the depressive, hypomanic, and cognitive features. acute confusional state and to evaluate the 190 7: Differential Diagnosis and Management

refusal to provide blood. Signs of hypo- and psychomotor agitation or retardation. The glycemia manifested by this patient include time frame for withdrawal can last hours to tachycardia, tremor, hypertension, and, finally, weeks depending on the pattern of use and the seizure. Delirious states should receive imme- amount used. diate workup for an organic etiology because delirium carries a high rate of morbidity and 71. (C) Although it is concerning that this patient mortality. The standard workup consists of a has a family history of schizophrenia, the more CBC with differential, electrolytes, BUN, crea- likely cause of his failure at school is cannabis

tinine, VDRL, vitamin B12, folate, urinalysis, abuse. The findings of conjunctival injection, thyroid-stimulating hormone (TSH), calcium, increased appetite, and dry mouth are con- magnesium, phosphorus, glucose, urine toxi- sistent with this diagnosis. Mild tachycardia cology screen, liver function tests, peripheral can also be associated with cannabis abuse. oxygen saturation, chest x-ray, ECG, and mental Behavioral abnormalities such as impaired status and physical examination. coordination, poor judgment, social withdrawal, euphoria, or anxiety may also be present. At this 68. (H) Alcohol intoxication and dependence are point, there is no evidence of psychosis, but the most common substance abuse-related with a family history one would want to mon- problems. This case exhibits some of the itor closely for any prodromal symptoms of common signs of alcohol intoxication: impaired schizophrenia. judgment, slurred speech, lack of coordination, unsteady gait, and impaired attention. Other 72. (G) This patient is exhibiting the classic signs of signs can include nystagmus, stupor, and coma. opiate withdrawal: piloerection, diaphoresis, This patient’s vital signs are stable, and there is abdominal cramps, yawning, lacrimation, and no mention in the case of diaphoresis or tremor. muscle aches. Other signs that may be sugges- The presence of any of these would arouse sus- tive of withdrawal include dysphoric mood, picion for alcohol withdrawal. insomnia, fever, and nausea, vomiting, or both. Shorter-acting opiates tend to produce with- 69. (B) PCP intoxication can be quite striking in its drawal symptoms more quickly and more similarities to schizophrenia, especially with intensely than longer-acting agents. regard to psychosis. In any case in which a substance-induced state may mimic psychi- 73. (E) MDMA, or ecstasy, has stimulant proper- atric disorders, a toxicology screen should be ties and its biologic effects include enhanced obtained. This case highlights aspects of PCP perception/sensation, increased heart rate, intoxication, including nystagmus, hyperten- hypertension, dilated pupils, trismus (tight- sion, tachycardia, a decreased responsiveness ness of jaw muscles), (grinding of to pain, and muscular rigidity. Violence or teeth), hyperthermia, and diaphoresis. Drug aggressive tendencies may be apparent with paraphernalia associated with MDMA use or without psychotic symptoms. No clear psy- includes pacifiers, Popsicle sticks, and candy chotic symptoms are occurring in this patient. necklaces to help alleviate MDMA-induced Most patients recover in a few days from PCP- bruxism. induced psychosis, but others may remain psy- chotic for periods up to 1 month. 74. (D) Because the defense mechanism of denial is active in alcoholism, the best approach in eval- 70. (A) This case is typical for cocaine withdrawal uating a patient is to explore how alcohol or “crashing.” After cessation or reduction of affects his life, rather than direct questions cocaine use, patients frequently report an about drinking behavior. Hearing about his intensely dysphoric mood, which may be asso- drinking from his friends and family may pro- ciated with suicidal ideation. Additionally, vide a more accurate description of a patient’s patients are fatigued, with increased appetite, alcoholism than the alcoholic would provide insomnia or hypersomnia, unpleasant dreams, of himself. Although laboratory work might Answers: 68–82 191

provide clues to the presence of alcoholism, it is not directly related to severity of withdrawal. cannot be relied on to make the diagnosis. Route can have some effect on withdrawal severity, because IV drugs can be shorter 75. (B) GAD is defined by excessive anxiety or acting, but is not necessarily directly related; worry occurring for at least 6 months. Associated some IV drugs can still have long half-lives. symptoms include feeling restless, fatigued, or irritable; muscle tension; difficulty with concen- 79. (C) The shorter the onset of action, the quicker tration; and sleep disturbance. Patients with the “high” and the quicker the addiction panic disorder are usually more disabled by their potential; hence, users simply like, and get symptoms and seek help much earlier than addicted to, such drugs more easily. patients with GAD. Although depression and dysthymia share some symptoms with GAD, 80. (A) This is a classic presentation of Wernicke the depressive symptoms are more severe in the encephalitis: Ataxia, Confusion, and Eye move- mood disorders than with the anxiety disorders. ment findings (mnemonic: ACE). The other The onset of anxiety with panic disorder is more responses describe similar but distinct syn- sudden and more intense, with the fear of chok- dromes. Alcoholic cerebellar degeneration is a ing or dying more common. Separation anxiety disorder of the cerebellar vermis in which requires the onset of symptoms before the age of patients have difficulty with gait and stance but 18, and dependent personality disorder repre- characteristically do not have arm ataxia or nys- sents a pervasive need to be taken care of, lead- tagmus. Subacute combined degeneration results

ing to submissive, clinging behavior that begins from B12 deficiency, which leads to demyelina- in early childhood. tion and axonal degeneration of the peripheral nerves, posterior and lateral columns of the 76. (A) These symptoms are known as Beck’s triad. spinal cord, and the cerebrum. Patients have cog- Beck, who developed the Beck Depression nitive impairment, diminished position and Inventory, is a seminal thinker with respect to vibration sensation, and abnormal gait. Alcoholic cognitive therapy. His triad of symptoms has peripheral neuropathy is a sensory peripheral been very helpful in the diagnosis and treat- neuropathy in which alcohol may directly cause ment of MDD. This particular triad is not char- axonal damage, although malnutrition may also acteristically seen in other disorders. play a role. Ménière disease is a vestibular dis- order in which distention of the endolymphatic 77. (E) Cognitive therapy can help to alter these sac causes intermittent vertigo, tinnitus, and negative ways of perceiving the self and the hearing loss. environment. In paradoxical therapy, devel- oped by Bateson, the therapist suggests that 81. (C) Pathologic lesions in Korsakoff are found in the patient engage in the behavior with negative the mammillary bodies and thalamus. Con- connotations (e.g., a phobia or compulsion). fabulation is a common finding. Treatment is

Behavioral therapy is focused on behaviors, not with thiamine, not vitamin B12. The neurologic on possible underlying factors that cause behav- findings are ataxia and nystagmus. Lewy body iors such as cognitive errors. Couples therapy disease, which presents as a dementia similar would be less efficacious than cognitive therapy to Alzheimer disease, involves deposition of given this patient’s symptoms. The major focus inclusion bodies in the cerebral cortex. of interpersonal psychotherapy is communica- tion analysis and one’s role in relationships. 82. (D) Mood-congruent delusions are compatible and consistent with the state of mind of the 78. (A) This is a general rule that almost always patient. In this case, the patient feels guilty and applies, at least in the cases in which a drug has presumably believes he deserves punishment. a withdrawal syndrome. Lipophilia generally is His delusions express these thoughts. If he related to half-life in the sense that body stores described delusions of grandiosity, that would of the drug are available for a “self-taper,” but be an example of a mood-incongruent delusion. 192 7: Differential Diagnosis and Management

Bizarre delusions describe a circumstance that is 86. (A) Schizophrenia is commonly classified accord- virtually impossible to be true, for example, a ing to types, which try to capture descriptively schizophrenic patient believing a computer the symptoms that predominate the clinical pic- chip is implanted in his brain. The delusions in ture. In the paranoid type, the patient is preoccu- this question, however unlikely, are in some pied with delusions, typically persecutory in sense conceivable. Somatic delusions focus on nature, and often has auditory hallucinations. bodily functions and integrity. Ego-syntonic Speech and behavior appear relatively organ- delusions are experienced by the sufferer as ized. The disorganized type is characterized by acceptable; for example, the manic patient disorganized speech and behavior as well as believing he is the greatest actor in the world. flat or inappropriate affect. In catatonic type, This patient’s delusions are ego-dystonic, that is, motor activity is retarded to the point of immo- experienced as unacceptable and unpleasant. bility, or excessive, yet purposeless and unre- Often, the nature of the delusion and its rela- lated to external events. A diagnosis of tion to mood state provide clues to a diagnosis. undifferentiated type means that the patient Delusions arising out of a severe depression, as meets criteria for schizophrenia, but does not in this case, are frequently mood congruent. meet criteria for paranoid, disorganized, or catatonic types. The residual type describes an 83. (C) The delusions of schizophrenic patients can absence of positive symptoms and a prepon- be bizarre and are often unrelated to mood derance of negative symptoms. state. Mania produces delusions that are grandiose, usually related to a sense of inflated 87. (D) The most important aspect of voices heard self-esteem. The presence of delusions rules out by a patient with schizophrenia is the nature of dysthymia and adjustment disorder because what the voices are saying. Command audi- these affective illnesses do not reach an intensity tory hallucinations telling a patient to harm or that includes delusional thinking. kill him or herself or someone else is almost always a psychiatric emergency, and the 84. (B) Patients with schizoid personality disorder patient requires hospitalization to be safe or to are typically socially isolated and be distinctly keep someone else safe. The only possible inept at forming and carrying on interpersonal exception is the case in which an experienced relationships. They often come to the attention clinician knows the patient well and believes of physicians as a result of some other issue, as the situation to be safe; even then, hospitaliza- seen in this case. Their apparent apathy and tion is probably necessary. All of the other indifference may resemble the negative symp- choices in this question speak to important toms of schizophrenia, and indeed, that is in characteristics of auditory hallucinations and the differential here; but it would be difficult to should be evaluated but the presence of com- make a diagnosis of schizophrenia without evi- mand auditory hallucinations requires imme- dence of psychosis. This patient lacks the diate attention. actively odd and peculiar mannerisms or eccentric way of thinking seen in schizotypal 88. (D) Narcolepsy is characterized by persistent personality disorder. He exhibits none of the daytime hypersomnolence that is temporarily mistrust and suspiciousness seen in paranoid relieved by brief naps. Narcolepsy is often personality disorder. Although he may be accompanied by phenomena commonly asso- depressed, we are given no direct evidence of ciated with REM sleep, including hypnagogic this. hallucinations (vivid hallucinations upon waking up), cataplexy (a sudden dramatic loss 85. (E) Bipolar disorder has one of the largest of muscle tone, usually following an intense genetic components of all the mental illnesses. emotional reaction), and sleep paralysis (a loss of Having a monozygotic twin with bipolar dis- voluntary muscle tone at the beginning or end order increases the risk of developing bipolar of sleep, seen in this case). Idiopathic hypersom- disorder to 80–90%. nolence is also characterized by persistent Answers: 83–101 193

sleepiness but can be distinguished from nar- 90–101. [90 (D), 91 (C), 92 (B), 93 (D), 94 (B), 95 (A), colepsy by the fact that the naps accompanying 96 (A), 97 (C), 98 (A), 99 (B), 100 (D), 101 (A)] it are often long and unrefreshing. Although this set of questions is unlikely to show up on the United States Medical Licensing 89. (D) Delirium tremens, an acute confusional state Examination (USMLE) Step 2, it is useful to induced by severe alcoholic withdrawal, occurs be able to distinguish the various axes of the in 3–5% of patients withdrawing from alcohol. Diagnostic and Statistical Manual of Mental Although it may be seen in patients with Disorders (DSM’s) multiaxial classification system dementia, as well as in those without, any delir- from one another, particularly on the wards ium, including delirium tremens, differs from when it is necessary to write notes. The system dementia in that one finds a relatively acute is used to break out and describe the diagnoses onset. The patient with dementia, by history, of a given patient and what contributes to each. has a slow (months to years) decline, whereas Axis I includes all mental illness diagnoses, the patient in an alcohol withdrawal delirium including substance abuse and developmental is expected to manifest symptoms in a matter of disorders, except personality disorders and days, if not hours. Although auditory halluci- mental retardation, which are coded on Axis nations are possible, visual hallucinations (e.g., II. Axis III includes general medical conditions, little scary creatures lurking about the room) and Axis IV includes psychosocial and envi- are far more common. Delirium tremens usu- ronmental factors that may be contributing to ally occurs 3–7 days after the patient’s last the mental illness. The Global Assessment of drink. Autonomic instability, particularly fever Functioning, a scale of 0–100 that the assessor and elevated pulse and blood pressure, are uses to rate the level of daily functioning, is common signs of alcohol withdrawal and often coded on Axis V. provide a key clue to the fact that the patient is in withdrawal and in need of emergent treatment, namely, administration of benzodiazepines. This page intentionally left blank CHAPTER 8 Practice Test 1 Questions

DIRECTIONS (Questions 1 through 89): Each of the Questions 3 and 4 numbered items or incomplete statements in this section is followed by answer choices. Select the A 44-year-old woman presents to her primary care ONE lettered answer or completion that is BEST in doctor with multiple complaints, including weak- each case. ness in her lower extremities, bloating, headaches, intermittent loss of appetite, and back pain. A careful review of symptoms reveals many other vague Questions 1 and 2 symptoms. Her complaints date back to adolescence A 29-year-old man with a history of bipolar disorder and she has seen many doctors. Thorough workups, presents to the psychiatric emergency department including an exploratory laparotomy, have failed to saying that he is the king of “Pumbar” and needs uncover any clear, organic cause. everyone’s allegiance for the upcoming war with the Martians. In the past few days, he has slept a total of 3. What is the best approach to this patient? 3 hours but says that he is not tired. He has spent all (A) Tell her any physical workup is of his money soliciting phone sex. Now, he is agi- unnecessary. tated, demanding, and threatening. (B) Tell her to come back in 1 month and, if the symptoms are still present, you will 1. What is the best treatment for this patient in the initiate a physical workup. acute setting? (C) Tactfully ask her why she is inventing (A) hydroxyzine (Atarax) symptoms. (B) lithium (D) Assess her for other psychiatric illnesses. (C) divalproex sodium (Depakote) (E) Initiate a physical workup and arrange (D) haloperidol (Haldol) for follow-up in a year’s time. (E) carbamazepine (Tegretol) 4. Which statement regarding this patient’s 2. After treating the patient acutely, a medication diagnosis is correct? is needed to control his bipolar disorder. You (A) It has a good prognosis. find that he has a history of agranulocytosis. Which of the following is the best choice for a (B) It is most common in high socioeconomic mood stabilizer? groups. (C) Women are overwhelmingly more likely (A) lithium to receive this diagnosis than are men. (B) carbamazepine (D) Conversion symptoms are uncommon. (C) divalproex sodium (E) Regularly scheduled, frequent visits with (D) antipsychotic medication a primary care doctor will exacerbate (E) lorazepam (Ativan) symptoms.

195

Copyright © 2007 by The McGraw-Hill Companies, Inc. Click here for terms of use. 196 8: Practice Test 1

Questions 5 and 6 7. Which of the following symptoms is consid- ered a “negative symptom” with regard to her A 25-year-old man is a concern to his neighbors. He illness? dresses in odd, outdated clothes; seems to utter his own language; and although he tends to keep to him- (A) auditory hallucinations self, he has told neighborhood children that witches (B) delusions who live down the road have it in for him. (C) paranoia (D) flat affect 5. Which of the following statements about the (E) loose associations personality disorder from which this patient is most likely suffering is true? 8. The patient is started on medication and many (A) Auditory hallucinations are common. of her symptoms improve. She begins a new job (B) Visual hallucinations are common. and does well. One year later, she is brought to her doctor floridly psychotic, actively hearing (C) This diagnosis may be confused with voices, and extremely paranoid. She thinks her schizophrenia in remission. boss is trying to kill her. She has an upper res- (D) This diagnosis is found overwhelmingly piratory viral illness that she believes to be the in men. work of a foreign government. She discontinued (E) Onset of symptoms is usually in the her medication 4 weeks ago because she felt too second or third decade of life. sedated. In the past year, her cigarette smoking habit has decreased to one pack per day. What 6. The patient’s brother brings him to a doctor is the most likely cause of her exacerbation? after the death of their mother. Since then, the patient’s paranoia has caused him to question (A) stress from work his neighbors’ activities. He moved into a hotel (B) a reaction to the viral illness he could not afford to get away from the (C) medication noncompliance “spies” living next to him. What is an appro- (D) medication side effects priate intervention? (E) decreased cigarette smoking (A) no treatment (B) psychoanalysis Questions 9 and 10 (C) benzodiazepines A 44-year-old man complains to his doctor that he is (D) a neuroleptic always tired and is having difficulty getting out of (E) a selective serotonin reuptake inhibitor bed in the morning. Upon questioning, he reveals he (SSRI) has three or four drinks each night and perhaps more on the weekends, but denies he has any problem Questions 7 and 8 with alcohol.

A 25-year-old female college graduate is brought to 9. A diagnosis of alcohol dependence is made and her doctor by her mother. Described as “odd” since the patient comes to your office in acute alcohol she lost her job a year ago, the patient has complained withdrawal. He has a withdrawal seizure. What of hearing voices and believes that her body is a might be found on laboratory investigation? receiving antenna for a foreign spy operation. Her mother notes she has been isolating herself in her (A) thrombocytosis room. She is alert and oriented but suspicious and (B) elevated or depressed liver enzymes guarded on examination. Her affect is flat and her (C) decreased prothrombin time speech reveals loose associations. A complete medical (D) hypermagnesemia workup is negative. (E) a high blood alcohol level Questions: 5–14 197

10. Which of the following statements regarding 12. A 67-year-old woman with a history of depres- this patient is true? sion presents to your office for evaluation. Her symptoms of poor appetite, insomnia, and feel- (A) Cerebellar degeneration is uncommon. ings of hopelessness have worsened recently. (B) He is at risk for developing a peripheral She has been on SSRIs, which you learn have neuropathy. failed in several previous trials. She has not tried (C) Alcoholic “fatty liver” is irreversible. a tricyclic antidepressant (TCA). The pharma- (D) He is at decreased or normal risk for cology of TCAs involves which of the following? heart disease. (A) 5-hydroxytryptamine-2a (5-HT2a) (E) Immune function should remain antagonism relatively intact. (B) 5-HT2a agonism Questions 11 and 12 (C) dopamine blockade (D) inhibition of norepinephrine and A 28-year-old woman is brought to the emergency serotonin reuptake department for active suicidal ideation with a plan to (E) inhibition of dopamine reuptake overdose on acetaminophen. She has no history of a psychiatric illness but endorses many criteria for Questions 13 and 14 major depressive disorder (MDD), including poor sleep for the past 2 weeks. She recently lost her job A 29-year-old woman presents to the emergency and fears that she may not be able to pay her rent. You department with her 3-year-old child reporting that speak to a friend, who reports that the patient has the child suffered a seizure while at home. Hospital been abusing alcohol. Attempts to obtain additional records verify that this is the third emergency depart- collateral information have been unsuccessful. You ment visit in as many weeks for the same presentation. believe the patient requires inpatient evaluation but Neurologic workup for seizure disorder was negative. her health maintenance organization (HMO) denies Initiation of an anticonvulsant has been ineffective. authorization for inpatient care, alternatively author- izing eight outpatient visits with a psychiatrist. You 13. Which of the following statements is likely to speak to the weekend on-call physician-reviewer and be true? report that the patient remains unsafe and wishes to (A) The child suffers from a conversion be discharged. Upon learning that the patient does not disorder. have a history of psychiatric illness, the reviewer fails to authorize inpatient care, despite your assessment. (B) The child is experiencing separation anxiety. 11. Which of the following is the most appropriate (C) The child’s history is falsified. intervention? (D) The child does not have a therapeutic level of anticonvulsant. (A) Begin antidepressant therapy and arrange for outpatient follow-up the next day. (E) The child suffers from a seizure disorder. (B) Admit the patient on an emergency 14. The mother is most likely to suffer from which certificate to an inpatient facility on the of the following? basis of danger to self. (C) Administer an antipsychotic medication (A) schizophrenia and reevaluate the patient in 1 hour. (B) bipolar disorder (D) Explain to the patient that her HMO (C) depression did not authorize hospitalization and (D) epilepsy discharge her with follow-up care. (E) posttraumatic stress disorder (PTSD) (E) Prescribe a medication to help her sleep, discharge her, and arrange for follow-up care. 198 8: Practice Test 1

Questions 15 and 16 18. Which of the following adverse effects is asso- ciated with this medication? A 26-year-old man is being evaluated in the emergency department for sudden onset of chest pressure and (A) bradycardia dyspnea. This is his third emergency department visit (B) hypertension for similar symptoms for which he reports “I feel like (C) weight loss I’m going to die.” An electrocardiogram (ECG) and (D) galactorrhea stress test were normal. The patient denies risk factors (E) seizures for heart disease and does not have a family history of heart disease. Urine toxicology was negative. 19. Which of the following hematologic disorders associated with this medication can be life- 15. Which of the following is the most likely threatening? diagnosis? (A) a decreased granulocyte count (A) delirium (B) leukocytosis (B) panic disorder (C) thrombocytopenia (C) acute stress reaction (D) pancytopenia (D) acute myocardial infarction (E) microcytic anemia (E) hypochondriasis 20. A 35-year-old patient exhibits odd beliefs and 16. Which of the following medications is most thinking, paranoid behavior, and has no close useful in the initial treatment of this disorder? friends. Which of the following diagnoses is (A) sertraline (Zoloft) the most appropriate to consider? (B) propranolol (A) schizoid personality disorder (C) clonidine (Catapres) (B) avoidant personality disorder (D) haloperidol (C) paranoid personality disorder (E) lithium (D) narcissistic personality disorder (E) schizotypal personality disorder 17. A woman being treated for major depression is brought to the emergency department 21. A 28-year-old woman demonstrates a perva- after being found unconscious by a neighbor. sive pattern of unstable relationships, poor self- The neighbor said that over the past few days image, impulsiveness, and irritability. Which the woman had been complaining of severe of the following is the most appropriate diag- headaches. She also said the woman enjoys nosis to consider? red wine. The woman’s blood pressure is recorded as 220/110 mm Hg. This emergency (A) histrionic personality disorder is best managed by intravenous (IV) adminis- (B) borderline personality disorder tration of which of the following? (C) antisocial personality disorder (A) an alpha-blocking agent (D) dependent personality disorder (B) a beta-blocker (E) schizoid personality disorder (C) dantrolene sodium (Dantrium) (D) bromocriptine (Parlodel) Questions 22 and 23 (E) a calcium channel blocker A 30-year-old woman is brought to the emergency department by the police after being arrested for Questions 18 and 19 breach of the peace. The woman was observed acting irrationally at a local business where she demanded A 28-year-old man recently began taking clozapine to speak with the president of the company claiming (Clozaril) to treat symptoms of schizophrenia. He that she had new ideas for product development. The does not suffer from any medical illness. Questions: 15–27 199 patient reports that she has not slept for days and generalized seizure. Laboratory investigation that her mood is “fabulous.” Urine human chorionic reveals that the phenytoin level is 6.5 mg/dL gonadotropin is positive. Illicit substances were not (normal, 10–20 mg/dL). Which of the following detected. is the most appropriate intervention at this time? (A) Increase the phenytoin dose to achieve a 22. Which of the following is most likely to be pres- therapeutic level. ent on Mental Status Examination (MSE)? (B) Discontinue phenytoin and begin (A) racing thoughts divalproex sodium. (B) depressed mood (C) Add a benzodiazepine as an adjunct (C) auditory hallucinations medication. (D) daytime sleepiness (D) Ask the patient about compliance with (E) weight loss medications. (E) Add phenobarbital. 23. Which of the following medical disorders can present with similar symptoms? 26. A 24-year-old woman with a history of schizo- phrenia tells you that she would like to become (A) hyperglycemia pregnant. What is the chance for familial trans- (B) thyroid disorder mission of schizophrenia? (C) rheumatoid arthritis (A) 0.1% (D) diabetes mellitus (B) 1% (E) cirrhosis (C) 2% Questions 24 and 25 (D) 5% (E) 10% A 24-year-old man with a history of seizure disorder and polysubstance abuse has been incarcerated for 27. A 70-year-old man with a history of major assaultive behavior. The patient is evaluated by a depression is brought to the emergency depart- neurologist, who prescribes phenytoin (Dilantin). ment by his family, who is concerned because he was found wandering in the streets near 24. Which of the following conditions is associated his home. During the MSE, attention is best with this medication? assessed by asking the patient to do which of (A) leukocytosis the following? (B) hypertension (A) perform serial-sevens subtraction (C) gingival hyperplasia (B) recite his Social Security number (D) hepatic failure (C) spell a five-letter word forward and (E) Ebstein anomaly backward 25. The patient returns 1 month later for a follow-up (D) perform digit recall examination and reports that he experienced a (E) repeat the examiner’s first name 200 8: Practice Test 1

28. An 18-year-old patient experiences a sudden onset (A) haloperidol of euphoria, grandiose delusions, a decreased (B) clozapine need for sleep, and paranoia. Which of the fol- (C) loxapine (Loxitane) lowing conditions is most associated with these (D) pimozide (Orap) symptoms? (E) perphenazine (Trilafon) (A) schizophrenia, paranoid type (B) cannabis intoxication Questions 32 and 33 (C) MDD with psychotic features A 30-year-old woman with a history of unstable inter- (D) schizoaffective disorder personal relationships, suicidal gestures, and marked (E) cocaine intoxication impulsivity is referred to you for dialectical behav- ioral therapy (DBT). 29. A Malayan man experiences a dissociative episode characterized by an outburst of homi- 32. Which of the following disorders is this patient cidal behavior toward his friend, whom he likely to be suffering from? believed was insulting his appearance. Which of the following conditions is most associated (A) avoidant personality disorder with this presentation? (B) bipolar disorder (C) passive-aggressive personality disorder (A) substance intoxication (D) borderline personality disorder (B) dissociative identity disorder (E) schizoid personality disorder (C) Ganser syndrome (D) factitious disorder 33. The foundations of DBT include which of the (E) culture-bound syndrome following? (A) The patient is doing the best that she can. Questions 30 and 31 (B) The patient may fail in therapy. A 26-year-old woman is diagnosed with schizophre- (C) At times the patient does not want to nia. The psychiatrist decides to treat her symptoms improve. with a high-potency antipsychotic medication. The (D) The patient should not learn new patient experiences abnormal involuntary move- behaviors. ments associated with the use of the antipsychotic (E) The patient is responsible for causing all drug she has been prescribed. of her own problems.

30. Realizing that side effects may affect further 34. A 21-year-old man presents with a 4-week his- medication compliance, the psychiatrist should tory of paranoid delusions and auditory hallu- prescribe which of the following? cinations that comment on his appearance. After (A) clozapine thorough evaluation, you diagnose him with schizophrenia, paranoid type, and prescribe (B) risperidone (Risperdal) haloperidol 5 mg bid. One week later, the patient (C) benztropine (Cogentin) returns for a follow-up examination and reports (D) methylphenidate (Ritalin) that, although his symptoms have improved, (E) a cholinergic agonist he now experiences muscle stiffness in his arms and neck. You prescribe benztropine 1 mg bid 31. Despite the intervention, the patient abruptly and schedule a follow-up appointment in stops taking her antipsychotic medication. The 2 weeks. One week later, the patient’s mother psychiatrist decides to change the patient’s calls you and reports that her son is more agi- medication. Which of the following may be tated and confused. Physical examination reveals considered? tachycardia, dilated pupils, and flushed skin. Questions: 28–38 201

The next appropriate measure would be which going to die.” Upon returning to the hospital, of the following? she experiences intense fear about revisiting the same hospital from which she was recently (A) discontinue benztropine and prescribe discharged. The most appropriate diagnosis to amantadine (Symmetrel) consider is which of the following? (B) increase haloperidol to 5 mg tid (C) increase benztropine to 2 mg bid (A) PTSD (D) discontinue haloperidol and prescribe (B) acute stress reaction risperidone (C) generalized anxiety disorder (GAD) (E) prescribe lorazepam 1 mg bid (D) adjustment disorder (E) depressive disorder 35. A 28-year-old male medical student is found to have an enlarged testicle during a routine phys- 37. A 40-year-old woman without a past psychi- ical examination. The student reports that it has atric history is admitted to the hospital for treat- been gradually enlarging for several months. ment of depression. During morning rounds, The physician asks why he did not report these the patient appears unresponsive and does not findings earlier. “I’m sure it’s nothing,” the stu- respond to verbal stimuli. There are no signs of dent replies. This response is an example of trauma or overdose. A review of her chart which of the following? reveals that the patient was well the night before and went to sleep without incident. You (A) a mature defense determine that the patient’s unresponsiveness is (B) a neurotic defense psychogenic. Which of the following findings is (C) a narcissistic defense most likely to be apparent on examination? (D) an immature defense (A) an abnormal electroencephalogram (EEG) (E) confidence (B) nonsaccadic eye movements 36. A 45-year-old Asian-American woman is (C) elevated temperature brought to the emergency department by her (D) decreased respirations husband, who reports that for the past 3 days (E) cold-caloric-induced nystagmus his wife has not been sleeping well, has been experiencing bad dreams, and appears “in a 38. A 24-year-old woman experiences fatigue, daze” with a sense of feeling “numb.” The weight gain, and hyperphagia during the winter patient endorses feeling anxious but does not months. She reports a sad mood and “can’t know why. She has been unable to perform her wait” until her vacation to Florida. The most usual activities of daily living. One week ago, likely diagnosis is which of the following? the patient was discharged from the hospital (A) dysthymia after experiencing an anaphylactic reaction to IV contrast dye while undergoing an imaging (B) sundowning syndrome procedure for sinusitis. Although she cannot (C) anxiety disorder recall specifics, her husband verifies the his- (D) stress disorder tory, adding that the doctors “thought she was (E) seasonal affective disorder 202 8: Practice Test 1

39. A 27-year-old woman is brought to the emer- (A) malingering gency department by her parents, who report (B) Ganser syndrome that their daughter is unable to recall her (C) Capgras syndrome name. The emergency department physician (D) Munchausen syndrome reports that a complete neurologic workup is within normal limits. Collateral information (E) fugue state reveals that the patient had episodes in which she would take unplanned trips, sometimes for 42. A 75-year-old man is admitted to the hospital days, without notice, and would return unable following a serious suicide attempt. The patient to recall the episode. A review of her medical exhibits clinical features of depression and has a chart notes a past history of possible sexual past history of suicide attempts. The medical abuse as a child. Urine toxicology is negative chart reveals that he is prescribed levodopa/ and she does not take any medications. Which carbidopa, digoxin, aspirin, and a medication of the following is the most likely diagnosis? for cardiac arrhythmia. The most appropriate treatment at this time is which of the following? (A) partial complex seizure disorder (A) prescribe risperidone (B) delirium (B) administer supportive psychotherapy (C) dissociative fugue (C) prescribe nortriptyline (Pamelor) (D) PTSD (D) prescribe diazepam (Valium) (E) depressive disorder (E) administer electroconvulsive therapy 40. A 26-year-old female graduate student reports (ECT) to you a 4-week history of a depressed mood that has caused her significant difficulty in 43. A 38-year-old woman presents to your office attending her classes. The patient reports diffi- with a 2-week history of symptoms of depression culty falling asleep at night, weight loss, and characterized by hypersomnolence and reaction passive suicidal ideation. A careful review of sensitivity. She reports to you that she recently her history reveals that for the past 2 years she enrolled in graduate school and is having trou- also experienced brief and distinct periods of ble with many of her classes. Which of the fol- an elevated and expansive mood, a decreased lowing diagnoses should be considered? need for sleep, and an increase in goal-directed (A) adjustment disorder activities. Which of the following is the most (B) major depression with atypical features appropriate diagnosis to consider? (C) dysthymic disorder (A) MDD (D) major depression with melancholic (B) bipolar I disorder, current episode manic features (C) bipolar II disorder, current episode (E) sleep disorder depressed (D) cyclothymic disorder 44. A 33-year-old man changes his first name to (E) dysthymic disorder honor a musician whom he idolizes. He recently bought the same guitar as the musician 41. You are asked to evaluate a 30-year-old male and formed a rock band to play his music. prisoner who reports a depressed mood and During practice, the man dresses like his idol. suicidal ideation. During your examination, This behavior is an example of which of the you note that the prisoner responds to your following? questions with approximate answers. Which (A) identification of the following is the most appropriate diag- (B) regression nosis to consider? (C) fixation Questions: 39–50 203

(D) projection (A) It is self-limiting. (E) idealization (B) It is dose dependent. (C) It may require surgery. 45. A 60-year-old man with schizophrenia sits (D) It occurs after long-term therapy. motionless in his chair. The patient is mute and (E) It is related to dopamine blockade. reacts very little to his environment. His eyes appear fixated on a distant object. At times, he Questions 49 and 50 assumes bizarre postures and imitates the movements of others. What is the best descrip- A 60-year-old man reports a history of excessive worry tion of his behavior? about his daughter since she moved away to college 1 year ago. His wife of 40 years verifies his complaint, (A) partial complex seizure adding that “he worries about everything.” Recently, (B) absence seizure his wife made plans to travel abroad to visit friends. (C) catatonia The patient is unable to accompany his wife because (D) cataplexy of respiratory problems caused by years of heavy (E) catalepsy smoking and is anxious about her leaving. He reports difficulty falling asleep, excessive daytime fatigue, 46. During an examination to evaluate muscle and difficulty concentrating at work. rigidity, a male patient is somewhat resistant to movement of his arms but maintains the arm in 49. Which of the following should be considered to the position in which you place it. This is an treat his symptoms? example of which of the following? (A) buspirone (BuSpar) (A) catalepsy (B) fluoxetine (B) cataplexy (C) alprazolam (Xanax) (C) rigidity (D) diazepam (D) dystonia (E) lorazepam (E) malingering 50. Two weeks later, the patient reports that the Questions 47 and 48 medication you prescribed has helped every- thing but his difficulty falling asleep. He reports A 42-year-old healthy man is taking a medication to that during the past week he has slept only help him sleep at night. Unexpectedly, he experi- 4 hours per night. After careful consideration, ences painful, persistent penile erection. you decide to prescribe a 1-week supply of med- ication to help him sleep. Which of the following 47. Which of the following medications is most medications is most appropriate to consider? likely to cause this condition? (A) clorazepate (Tranxene) (A) fluoxetine (Prozac) (B) zolpidem (Ambien) (B) sertraline (C) oxazepam (Serax) (C) paroxetine (Paxil) (D) diphenhydramine (Benadryl) (D) trazodone (Desyrel) (E) temazepam (Restoril) (E) nortriptyline

48. Which of the following statements regarding this condition is correct? 204 8: Practice Test 1

Questions 51 and 52 (D) Administer parenteral thiamine. (E) administer IV dextrose. A 25-year-old woman is scheduled for a gynecologic procedure. Upon discharge from the hospital, she is 54. Which of the following language deficits is likely to receive pain medication. For 1 year, you characteristic of Broca aphasia but not of have been prescribing a monoamine oxidase inhibitor Wernicke aphasia? (MAOI) to treat a single episode of depression with excellent results. (A) nonfluency (B) impaired repetition 51. Which of the following medications may cause (C) impaired writing a fatal drug interaction in this patient? (D) impaired naming (A) codeine (E) impaired reading comprehension (B) acetaminophen (C) oxycodone 55. A 34-year-old executive reports drinking up to (D) meperidine (Demerol) six cups of caffeinated coffee per day. He boards a plane scheduled for an 18-hour flight (E) ibuprofen on which only decaffeinated beverages are served. Which of the following symptoms is 52. The patient tolerates the procedure well and the executive most likely to experience? 2 months later reports to you that she is preg- nant. Which of the following interventions is (A) depressed mood most appropriate? (B) muscle cramping (A) MAOIs are safe in pregnancy and (C) headache should be continued. (D) irritability (B) Antidepressant therapy should be (E) nausea discontinued. (C) Discontinue the MAOI and initiate treat- Questions 56 and 57 ment with fluoxetine. A 25-year-old man with a history of medication- (D) Discontinue the MAOI during the first responsive schizophrenia presents to the emergency trimester only. department with severe muscle stiffness and an ele- (E) Discontinue the MAOI and begin vated temperature. The patient appears confused, is maintenance ECT. diaphoretic, and vital signs are unstable. A urine tox- icology screen is provisionally negative. Questions 53 and 54 56. Which of the following is the most essential A 62-year-old homeless man presents to the emer- intervention? gency department with impaired gait and nystag- mus. His vital signs are stable and an ophthalmologic (A) Administer IV dantrolene sodium. examination is within normal limits. The patient is (B) Apply cooling . unable to recall the date and has difficulty sustaining (C) Administer naloxone (Narcan). attention without evidence of amnesia. (D) Administer bromocriptine. 53. Which of the following is the most appropriate (E) Discontinue medications. initial intervention? 57. Which of the following laboratory abnormali- (A) Perform a breath analysis. ties is most likely to be present? (B) Obtain a head computed tomographic (A) anemia scan. (B) leukopenia (C) Perform a lumbar puncture. Questions: 51–62 205

(C) elevated creatine phosphokinase (CPK) (D) Obtain a urine pregnancy test. (D) decreased transaminases (E) Obtain renal function studies. (E) elevated blood urea nitrogen (BUN) 61. A young healthy man is diagnosed with bipo- 58. An 18-year-old man presents to the emergency lar I disorder and is prescribed lithium carbon- department after being involved in a motor vehi- ate to treat his symptoms of mania. Which of the cle accident. The medical chart reports a history following laboratory tests should be obtained of substance use. Which of the following tests can before initiating treatment with lithium? confirm your diagnosis of substance abuse? (A) EEG (A) serum liver function studies (B) creatinine clearance (B) breath analysis (C) liver function studies (C) urine toxicology screen (D) 24-hour urine volume (D) naloxone injection (E) thyroid-stimulating hormone (E) elevated heart rate 62. A 27-year-old woman has been prescribed flu- 59. A 72-year-old man without a prior psychiatric oxetine for depression for the past year. She history presents to the outpatient clinic with a comes to you complaining of medication side recent history of cognitive deficits. The patient effects and wishes to discontinue her antide- has had a stable level of consciousness and pressant. As her physician, you discuss with her denies any current or past substance abuse. The the appropriate risks and benefits, and you dis- patient has not been prescribed any new med- continue her treatment. Two weeks later, she ications. The medical chart reveals a history of returns complaining of depressive symptoms megaloblastic anemia and a subtotal gastrectomy characterized by sleep and appetite changes, for severe peptic ulcer disease. Which of the fol- poor concentration, and a depressed mood for lowing initial interventions is most appropriate? most of the day. You decide that she is experi- encing a recurrent major depressive episode and (A) Order a magnetic resonance imaging decide to prescribe her a different class of med- scan of the brain to confirm dementia, ication to treat her symptoms. After discussing Alzheimer type. the appropriate precautions with her, you pre- (B) Order a reversible workup for dementia. scribe an MAOI. Within 1 week, she begins (C) Discuss long-term extended-care facility experiencing irritability, abdominal pain and admission. diarrhea, and autonomic instability character- (D) Obtain a forensic examination to evaluate ized by hypertension and tachycardia. Her tem- competency. perature is 103.5°F. The patient also reports (E) Obtain a neurologic consult. experiencing myoclonic-like jerking of her mus- cles and vivid visual hallucinations of colorful 60. A 37-year-old woman presents to the emer- flowers spinning toward the sky. She states that gency department complaining of severe diar- she has followed your directions carefully while rhea and a coarse tremor. The medical chart taking this medication. Which of the following notes a history of bipolar I disorder. The most can account for this presentation? appropriate intervention aimed at establishing (A) neuroleptic malignant syndrome (NMS) a diagnosis is which of the following? (B) tyramine-induced hypertensive crisis (A) Obtain a serum medication level. (C) malignant hyperthermia (B) Prescribe antidiarrheal medication. (D) serotonin syndrome (C) Obtain a urine toxicology screen. (E) hallucinogen abuse 206 8: Practice Test 1

Questions 63 and 64 (D) body weight 85% of expected normal (E) sexual promiscuity A 17-year-old girl is referred to the school nurse for frequent episodes of vomiting in the bathroom 66. The mortality rate associated with this disorder during lunch breaks. Her friends report that, despite may be as high as which of the following? a preoccupation with her body image, she eats “twice as much as anybody else.” The parents are called to (A) 5% the school and recall that a recent bill from their (B) 10% charge account at the local pharmacy indicated a (C) 20% large number of laxative purchases. The girl reports (D) 25% that her mood is fine, and her records at school indi- cate that she is an above-average student. (E) 30%

63. Medical complications of this disorder are most Questions 67 and 68 often due to which of the following? A 50-year-old woman reports a depressed mood, (A) chronic vomiting poor appetite, and weight loss 1 month following (B) weight loss the death of her husband. After his death, she began to feel that she would be “better off dead.” At times, (C) ritualistic fasting she believes that she can hear his voice calling to her. (D) suicide gestures She denies any feelings of worthlessness but feels (E) substance abuse guilty about not being able to do the right things for him before he died. When talking with others, she 64. Which of the following medications is the most believes that her feelings are normal. appropriate initial treatment for her symptoms? (A) antiemetics 67. Which of the following describes this condition? (B) antidepressants (A) uncomplicated bereavement (C) antianxiety agents (B) complicated bereavement (D) appetite stimulants (C) major depressive episode (E) hypnotic medications (D) adjustment disorder with depressed mood Questions 65 and 66 (E) dysthymic disorder A 16-year-old female high school student is referred 68. Which of the following is the most appropriate by her parents for an evaluation. One year prior to course of action? the evaluation, the girl began restricting her food intake and started a rigorous exercise program to (A) no treatment improve her appearance. Aspiring to be a model, the (B) prescribing antidepressant medication girl lost 25 lb but remained preoccupied with her (C) prescribing sedative-hypnotic appearance despite weighing only 85 lb. Her friends medication reported that she constantly referred to herself as (D) admission to a hospital being “fat” and did not seem interested in dating. The girl continued to lose weight and was reluctant to dis- (E) evaluating for a sleep disorder cuss her condition. 69. A 25-year-old man reports a 5-year history of 65. The girl is most likely to experience which of excessive hand washing and a preoccupation the following? with feeling clean. The thought of contracting an infectious disease persists throughout the day (A) normal breast development even though he makes attempts to ignore it. His (B) three consecutive missed periods condition has progressively worsened and has (C) the loss of feelings of hunger caused significant impairment while at work and Questions: 63–75 207

at home. Which of the following medications is (A) Administer IV atropine. the best initial choice to treat his symptoms? (B) Administer an anticholinesterase (A) antipsychotics medication. (B) antianxiety agents (C) Administer IV dantrolene sodium. (C) antidepressants (D) Administer haloperidol. (D) antiepileptic agents (E) Administer a benzodiazepine. (E) lithium 73. After the appropriate intervention, the patient experiences nausea and vomiting and has a Questions 70 and 71 seizure. Which of the following medications A 52-year-old obese man experiences excessive day- should be administered next? time sleepiness and a depressed mood. His wife (A) atropine reports that he snores loudly and is restless while (B) lorazepam sleeping. There is no evidence of substance abuse, and the patient does not have a prior psychiatric history. (C) physostigmine (D) epinephrine 70. Which of the following is the most likely cause (E) prochlorperazine (Compazine) of his symptoms? 74. A 40-year-old woman suffers from depression (A) depression and obsessive ritualistic behavior. The physi- (B) abnormal circadian rhythm cian decides to prescribe an SSRI. Which of the (C) periodic limb movements of sleep following medications might the physician (D) airway obstruction consider? (E) nocturnal myoclonus (A) buspirone (B) phenelzine (Nardil) 71. Which of the following is the treatment of choice for his symptoms? (C) clomipramine (Anafranil) (D) doxepin (Sinequan) (A) breathing air under positive pressure (E) fluvoxamine (Luvox) (B) nasal surgery (C) benzodiazepine medication Questions 75 and 76 (D) antidepressant medication A 56-year-old man with a long history of alcohol (E) uvulopalatoplasty abuse and liver disease is ordered by the court to enroll in an inpatient detoxification program for alco- Questions 72 and 73 hol dependence. He begins to experience clinical A patient with chronic paranoid schizophrenia who signs and symptoms of withdrawal. has been stable for years on a low-potency antipsy- chotic agent begins experiencing parkinsonian-like 75. Which of the following medications is the most side effects. His physician prescribes a drug to alle- appropriate to prescribe during his treatment? viate some of these side effects. One week later, the (A) chlordiazepoxide (Librium) patient is seen in the emergency department with (B) phenobarbital dilated pupils, dry mouth, warm skin, and tachycar- dia. He is experiencing visual hallucinations. (C) disulfiram (Antabuse) (D) lorazepam 72. The most appropriate intervention is which of (E) alprazolam the following? 208 8: Practice Test 1

76. Soon after his admission, the man begins to includes significant blockade of serotonin reup- report hallucinations and becomes agitated. take. Which of the following antidepressants Which of the following medications should be should be considered? prescribed to treat these symptoms? (A) maprotiline (Ludiomil) (A) sertraline (B) amitriptyline (Elavil) (B) clonazepam (Klonopin) (C) bupropion (Wellbutrin) (C) haloperidol (D) desipramine (Norpramin) (D) lithium (E) clonazepam (E) fluoxetine 80. A patient who is human immunodeficiency 77. A 48-year-old woman successfully completes an virus (HIV)-positive reports a depressed mood, inpatient program for alcohol detoxification for low energy, and hopelessness. Which of the fol- which she was prescribed chlordiazepoxide. lowing medications can the physician prescribe Upon discharge, the patient is prescribed a med- to achieve rapid relief of some of his symptoms? ication that disrupts alcohol metabolism. Soon (A) methylphenidate after discharge, the patient attends an office party where she reports having a few drinks. (B) fluoxetine She has been compliant with her prescribed (C) bupropion medication and does not have any active med- (D) buspirone ical problems. Which of the following symp- (E) nefazodone (Serzone) toms is the patient likely to experience? (A) high blood pressure Questions 81 and 82 (B) euphoria A college-aged man reports that he and his friends (C) blurred vision have been experimenting with the new drug craze on (D) vomiting campus called “huffing.” His roommate reports that (E) urinary retention he has been accumulating typewriter correction fluid, nail polish remover, and model airplane glue. 78. A 22-year-old man with a history of bipolar disorder is prescribed lithium carbonate to treat 81. Which of the following is experienced by a his symptoms. During a weekend rugby tour- person after huffing? nament, he hurts his knee and an orthopedic (A) staring into space physician prescribes a medication to reduce his (B) tolerance symptoms of pain and swelling. Although the (C) depressed reflexes patient reports relief from this medication, he (D) increased appetite begins to experience abdominal pain, diarrhea, and drowsiness. Which of the following med- (E) diminished response to pain ications may contribute to the production of these symptoms? 82. Which of the following is the current treatment for this type of drug abuse? (A) ibuprofen (A) antidepressant agents (B) acetaminophen (B) dialectic behavioral therapy (C) aspirin (C) abstinence (D) codeine (D) exposure and response prevention (E) meperidine (E) antipsychotic agents 79. A patient experiences clinical signs and symp- toms of depression. The physician decides to 83. A 30-year-old man with schizophrenia exhibits prescribe a medication whose mechanism symptoms that have not been stable over time. Questions: 76–88 209

He complains of hair loss that began after he evaluated for similar difficulties while in public school. started taking one of his medications. Which of The student seemed to improve when the family the predominant symptoms would support the moved out of the area and he attended private school. diagnosis of the paranoid subtype? 86. Which of the following is the most likely (A) immobility diagnosis? (B) inappropriate affect (C) disorganized speech and behavior (A) attention deficit hyperactivity disorder (ADHD) (D) posturing (B) bipolar I disorder (E) prominent hallucinations or delusions (C) bipolar II disorder 84. You are treating a 32-year-old woman for bipo- (D) conduct disorder lar I disorder with a combination of medica- (E) learning disability tions. She complains of hair loss. This side effect is a reversible side effect of which of the 87. The patient is prescribed methylphenidate for following medications? symptoms of inattention and hyperactivity. After an adequate trial and dose, the patient does not (A) divalproex sodium fully benefit from the medication. Which of the (B) ziprasidone (Geodon) following actions is the most appropriate? (C) carbamazepine (A) Add a low-dose amphetamine. (D) clozapine (B) Add an anxiolytic medication. (E) olanzapine (Zyprexa) (C) Add an antidepressant. 85. A young man experiences erectile dysfunction (D) Discontinue methylphenidate and pre- and is told that the medication he is prescribed scribe a mixed amphetamine compound. is likely to be the cause. Which of the following (E) Discontinue methylphenidate and medications is associated with erectile dys- prescribe an antidepressant. function? 88. A 19-year-old man is brought to the emergency (A) amantadine department by the police for an evaluation. The (B) methylphenidate written police report states that the patient has (C) SSRIs been calling 911 for the past 4 weeks reporting (D) antihistamines that he is being spied on by aliens from a distant (E) cholinergic agents planet. The patient reports that he is receiving messages from the aliens through his radio and Questions 86 and 87 that he hears voices in his head commenting on his appearance. He has not been sleeping well at A 21-year-old male college student without prior med- night because he has been “guarding his bed- ical or psychiatric history is evaluated for poor work room.” You note that his affect is flat, and he performance. The student reports to the dean that he appears tired during your examination. A urine finds it hard to follow through with assignments and toxicology screen is negative. Which of the fol- is easily distracted by environmental stimuli. The dean lowing is the most likely diagnosis? comments to the student about how frequently he interrupts others during conversations and observes (A) schizophrenia that it seems that the student is not paying attention to (B) bipolar I disorder with psychotic features what he is saying. Overall, his grades are poor except (C) schizoaffective disorder in the class he considers “the most interesting class I (D) brief psychotic disorder have ever had.” He often forgets required materials for (E) MDD with psychotic features classes and his assignments are frequently late. The student’s parents report that he was incompletely 210 8: Practice Test 1

89. An 18-year-old woman with recently diag- Questions 93 through 95 nosed schizophrenia is acutely psychotic and in labor with her first child. The obstetric service Match the clinical presentation with the appropriate requests a psychiatric consultation for a safe neuropsychological test. medication to control the patient’s psychosis (A) Minnesota Multiphasic Personality while she is in labor. Which of the following Inventory-2 (MMPI-2) would be relatively safe? (B) WISC (A) haloperidol (C) Rey-Osterrieth Test (B) chlorpromazine (Thorazine) (D) Beck Depression Inventory (C) thioridazine (Mellaril) (E) Blessed Rating Scale (D) droperidol (Inapsine) (F) WAIS-R (E) trihexyphenidyl (Artane) (G) Boston Diagnostic Aphasia Examination (H) Rorschach Test DIRECTIONS (Questions 90 through 100): Each (I) Mini-Mental Status Examination (MMSE) set of items in this section consists of a list of (J) Bender-Gestalt Test lettered headings followed by several numbered (K) Wisconsin Card Sorting Test (WCST) words or phrases. For each numbered word or phrase, select the ONE lettered option that is most (L) Wada Test closely associated with it. Each lettered option may be selected once, more than once, or not at all. 93. A 35-year-old woman with a protein S defi- ciency shows evidence of left side hemineglect. Questions 90 through 92 94. A 7-year-old boy with difficulty in school needs Match the appropriate test to assess for intelligence further evaluation of academic potential. in each age group. 95. You would like to address the deficits of a 69- (A) Wechsler Preschool and Primary Scale year-old man with nonfluent speech and for- of Intelligence (WPPSI) mulate a specific treatment plan. (B) Wechsler Adult Intelligence Scale— Revised (WAIS-R) Questions 96 through 100 (C) Wechsler Memory Scale (WMS) (D) Wechsler Intelligence Scale for Children Select the drug most likely to cause the associated (WISC) symptoms. (A) lysergic acid diethylamine (LSD) 90. A 4-year-old girl whose parents wish to have (B) phencyclidine (PCP) her evaluated for entrance into an “elite” (C) cocaine kindergarten. (D) marijuana 91. A 65-year-old female physician who is now (E) opiates concerned about memory difficulties. (F) nicotine

92. An 8-year-old boy whom the school system 96. An 18-year-old high school student with con- wants evaluated for possible special needs in junctival redness, increased appetite, dry mouth, the classroom. tachycardia, and a sensation of slowed time. Questions: 89–100 211

97. A 31-year-old man with miosis, bradycardia, 99. A 16-year-old girl with abdominal cramps, con- hypotension, hypothermia, and constipation. fusion, palpitations, and muscle twitching.

98. An assaultive 26-year-old man with vertical 100. A 21-year-old man with tachycardia, dilated nystagmus, echolalia, paranoid ideation, and pupils, hallucinations, and complaints of chest hallucinations. pain. Answers and Explanations

1. (D) Antipsychotics are indicated for acute treat- illnesses. In managing patients with somati- ment of agitation and violence sometimes seen in zation disorder, it is important to accept that manic patients. Haloperidol works quickly doses symptoms are not consciously produced and (20–30 minutes). Doses of 2–5 mg by mouth (PO) to let patients know that you realize their or intramuscularly (IM) are usual initial. It may symptoms are a source of great consternation. be given IV as well, especially in the intensive Regardless of history, any patient presenting to care unit (ICU). Hydroxyzine is an antihistamine; a physician with physical complaints deserves it is not effective in mania. Lithium, divalproex a reasonable physical investigation; patients sodium, and carbamazepine have all been shown with somatization disorder are as likely, if not to control mood fluctuations in manic patients. more likely, to develop identifiable medical However, these agents take days to work and are conditions. However, rather than repeat tests, it not effective in the acute management of this may be necessary to contact previous treaters. patient. Sending this patient away for a year would not be helpful; rather regular, frequent checkups and 2. (A) Lithium is not associated with significant fostering a therapeutic alliance and support are blood dyscrasias, although it can cause a modest in order. benign increase in the white blood cell (WBC) count. Carbamazepine is commonly associated 4. (C) Women are as much as 20 times more likely with reduced WBC count, but the incidence of to receive the diagnosis of somatization disor- agranulocytosis is approximately 1 in 10,000. der than are men. Rarely will any therapy or Agranulocytosis is a rare complication with intervention “cure” the patient of somatization divalproex sodium, a bit more common is disorder; at best, patients learn to minimize benign thrombocytopenia. Antipsychotic med- the impact somatization has on their lives. ication is occasionally associated with a decrease Somatization disorder is common in lower in leukopoiesis, but the WBC count usually socioeconomic groups. Conversion symptoms returns to normal with continued treatment. are common in somatization disorder and are Agranulocytosis can also occur in the setting of included in the diagnostic criteria. Patients antipsychotic medication use in approximately with this disorder benefit from regular, sched- 1 in 10,000 cases (clozapine has a higher inci- uled visits that allow them to forge a trusting dence). Lorazepam is not associated with WBC therapeutic alliance. abnormalities. 5. (C) This patient is most likely suffering from 3. (D) This patient is most likely suffering from schizotypal personal disorder. This personality somatization disorder, which often coexists disorder with its attendant social isolation and with other psychiatric illnesses such as anxiety, subtle distortions of reality may indeed resem- depressive, and personality disorders. The sus- ble schizophrenia in remission; the diagnosis pected presence of a somatization disorder can be sorted out by a thorough history. Frank should prompt a search for other treatable hallucinations of any type are uncommon; only

212 Answers: 1–11 213

subtle distortion of environmental cues is seen. alcoholism, the liver may be “burnt out” and Although men may be slightly more at risk for liver function tests may reveal low or normal being diagnosed with schizotypal personality levels of these enzymes. As a result of liver disorder, the disparity is not stark. As with damage, prothrombin time is typically increased. any personality disorder, schizotypal person- Hypomagnesemia, not hypermagnesemia, is ality disorder represents a lifelong maladap- more likely to be found in alcoholism, usually as tive approach to life and does not suddenly a result of dietary deficiency. In the patient express itself well into adulthood. having a withdrawal seizure, the problem is absence, not presence, of alcohol. His blood 6. (D) In schizotypal personality disorder, the alcohol is expected to be zero. The alcoholic is subtle disconnection from reality, which may more likely to have thrombocytopenia than be exacerbated in times of stress as in this case, thrombocytosis. can be treated with low doses of neuroleptics. These patients lack the capability, stable sense 10. (B) As a result of vitamin deficiencies and the of self, and trust to be able to engage in, or ben- direct insult alcohol exacts on the nerve func- efit from, psychoanalysis. Antidepressant med- tion, peripheral neuropathy can be seen in 10% ication of any type, including tricyclics and of heavy drinkers. Another important nervous SSRIs, are helpful in the schizotypal patient system effect of alcohol and its metabolites is who displays significant affective (mood) cerebellar degeneration, which can be sug- symptoms, which is not seen in this case. There gested by an unsteady gait and nystagmus. is no evidence of anxiety that would make ben- Alcoholic fatty liver—a swollen liver resulting zodiazepines useful. from the deposition of fats and proteins in the hepatocytes—reverses with abstinence from 7. (D) This patient is most likely suffering from alcohol. Although there is some evidence that schizophrenia. Symptoms of schizophrenia are a glass of wine each day may impart some pro- commonly divided into positive and negative tective cardiac effects, heavy drinking is cer- symptoms. Flat affect, a negative symptom, rep- tainly destructive to the cardiovascular system. resents an absence, in this case, of a normally It raises the blood pressure and levels of triglyc- reactive and variable affect. Auditory hallucina- erides, thereby increasing the risk of myocar- tions, delusions (including paranoid delusions), dial infarction. Heavy drinking also lowers and loose associations are all positive symptoms. WBC count and interferes with many specific aspects of the immune system; for example, it 8. (C) The exacerbation seen is most likely pre- compromises T-cell function. cipitated by medication noncompliance. Stress from work or a viral illness may indeed con- 11. (B) The patient does not have a history of tribute to a relapse, but they are less strongly mental illness, which gives you information predictive of a reemergence of psychotic symp- about past behaviors, especially suicidality. She toms than medication noncompliance. Nicotine is unable to contract for safety, and attempts to has been shown to lower neuroleptic levels, contact collaterals have been unsuccessful. which has been offered as a reason cigarette Collateral information is essential to validate smoking is rampant among patients with schiz- the present history as well as to provide data ophrenia. However, this patient’s smoking about baseline mental status. In the absence of decreased, which if anything would be expected collateral input, from either family or health- to increase neuroleptic levels. care professionals, and the important fact that she remains suicidal, the patient should not 9. (B) Classically, patients with hepatitis second- be discharged from the emergency depart- ary to alcohol abuse or dependence have ele- ment. The patient does not exhibit psychotic vated liver enzymes (such as gamma-glutamyl behavior and is not a management problem. transpeptidase, aspartate transaminase, or ala- The administration of an antipsychotic med- nine transaminase). However, in advanced ication can cause side effects and may affect 214 8: Practice Test 1

future compliance with psychotropic medica- with Munchausen syndrome by proxy and most tions. A thorough history, physical, and MSE, sufferers demonstrate a dissociation of affect. in conjunction with sound judgment based on Schizophrenia has been reported in only a few the available facts, supplants judgment based cases. Of the choices listed, depression is the on a telephone presentation or decisions by most likely; bipolar disorder and PTSD have managed care organizations. not been clearly described in the literature. Epilepsy is not associated with the disorder to 12. (D) TCAs inhibit reuptake of norepinephrine any significant extent. and serotonin to varying degrees. Structurally, they can be divided into tertiary amines 15. (B) Panic disorder is characterized by the sudden (amitriptyline, clomipramine, doxepin, imipr- unexpected occurrence of panic attacks and amine, and trimipramine) and secondary periods of intense anxiety or fear accompa- amines (desipramine, nortriptyline, and pro- nied by somatic symptoms, commonly causing triptyline). Tertiary amines tend to inhibit reup- the misdiagnosis of a medical illness such as take of serotonin to a greater degree than myocardial infarction. The frequency of panic norepinephrine, and secondary amines primarily attacks varies widely from many per day to a inhibit reuptake of norepinephrine. Bupropion is few per year. Panic disorder is often associated an antidepressant that inhibits both norepi- with agoraphobia, the fear of being alone in nephrine and dopamine reuptake. public places. The lifetime prevalence is up to 3%. Concerns of death from cardiac or res- 13. (C) This scenario represents a case of Munchausen piratory disorders occur frequently. Delirium is syndrome by proxy, in which one person feigns ill- characterized by the sudden onset of a distur- ness in another to vicariously gain medical atten- bance of consciousness with cognitive changes tion. Commonly, the victim is a child and the caused by a general medical condition. Acute perpetrator is the child’s mother. Apparent bleed- stress disorder is diagnosed in individuals who ing, seizures, and central nervous system (CNS) have experienced a traumatic event and sub- depression are typical presentations. The disorder sequently develop symptoms within 4 weeks is underdiagnosed and typically takes more than of the event that remit after 1 month. The trau- a year to diagnose due both to the elusive and matic event must be reexperienced by the crafty planning of the perpetrator and the unwill- patient, and this causes the patient to avoid ingness of health-care providers to accuse the stimuli that arouse recollections of the trauma. ostensibly caring parent. Conversion disorder is Other criteria necessary for the diagnosis are characterized by the presence of one or more the presence of dissociative symptoms and neurologic symptoms that are left unexplained marked anxiety or increased arousal not better by a known medical or neurologic disorder. It is accounted for by other medical or psychiatric associated with such comorbid psychiatric dis- illnesses. orders as MDD, anxiety disorder, or schizo- phrenia, and adolescents and young adults 16. (A) Tricyclic and tetracyclic medications, MAOIs, suffer from this disorder most commonly. SSRIs, and the benzodiazepines are effective in Paralysis, blindness, and mutism are the most the treatment of panic disorder. Beta-adrenergic common conversion disorder symptoms. The drugs like propranolol are not effective for the diagnosis requires the association of psycholog- treatment of panic disorder. Haloperidol, an ical factors to the initiation and exacerbation of antipsychotic agent; lithium, a mood stabilizer; conversion symptoms. In this case, there are no and clonidine, an alpha-2-adrenergic agonist, are data to support this diagnosis. ineffective. Of the choices listed, sertraline, an SSRI, is most effective. 14. (C) The mental status of the mother has been described as depressed, anxious, and suicidal. 17. (A) Hypertensive crisis is a potentially life- Borderline and histrionic personality disorders threatening complication that occurs when are the most common Axis II diagnoses associated patients taking an MAOI eat tyramine-containing Answers: 12–23 215

foods such as wine, beer, pickled foods, and a pervasive pattern of behavior characterized aged cheese. Clinical features include hyper- by social inhibition, feelings of inadequacy, and tension, severe occipital headache, stiff neck, hypersensitivity to negative evaluation. nausea, vomiting, and sweating. IV phento- Narcissistic personality disorder represents a per- lamine, an alpha-adrenergic receptor-blocking vasive pattern of grandiosity, lack of empathy, drug, is given to control hypertension. It has and a need for admiration. been shown to be more effective than beta- blockers or calcium channel blockers. Admission 21. (B) Borderline personality disorder is a pervasive to an ICU and supportive measures are indi- pattern of instability of interpersonal relation- cated. Although muscle rigidity can occur, the ships, self-image, affect, and marked impul- use of dantrolene, a muscle relaxant, is not sivity. Antisocial personality disorder describes indicated. individuals with long histories of disregard for the rights of others and often dishonesty used 18. (E) Clozapine is an effective antipsychotic med- in attempts to gain something for themselves. ication that has been associated with fewer Patients with histrionic personality disorder, like extrapyramidal side effects than the conven- those with borderline personality disorder, may tional antipsychotics (which primarily act by display excessive emotionality and attention- blocking dopamine type 2 receptors). About seeking behavior, but their core symptoms 5% of patients taking more than 600 mg/day center around superficial seductiveness and of clozapine experience clozapine-associated theatricality. Those with dependent personality seizures. Tachycardia, hypotension, sedation, disorder require an excessive amount of advice, fatigue, and weight gain have all been associ- reassurance, and approval from others. Schizoid ated with clozapine treatment. Clozapine, personality disorder represents a pervasive pat- unlike the conventional antipsychotic agents, tern of detachment from social relationships does not affect prolactin secretion and thus and a restricted range of expressed emotions. does not cause galactorrhea. 22. (A) This patient is most likely suffering from 19. (A) Agranulocytosis is a potentially life- bipolar I disorder. Racing thoughts, pressured threatening side effect of clozapine treatment. speech, expansive mood, a decreased need for It is defined as a decrease in the number of sleep, and an increase in goal-directed activity WBCs, with a specific decrease in the number are all common manifestations of mania. The of neutrophil granulocytes. It occurs in 1–2% of patient is usually energized during the day all patients treated with clozapine. The other even after only a few hours of sleep. Weight choices are bone marrow disorders that are not loss and depressed mood are characteristic of a associated with clozapine. depressive disorder. Auditory hallucinations may be present in severe cases of mania but are 20. (E) Schizotypal personality disorder is character- usually a symptom of a psychotic disorder such ized by a pervasive pattern of social and inter- as schizophrenia. personal deficits. Individuals demonstrate a reduced capacity to establish any close rela- 23. (B) Many disorders can mimic symptoms of tionships, and eccentric behavior is present. mania. A complete history, physical examina- The presence of odd beliefs or magical thinking tion, and routine laboratory tests can suffi- separates schizotypal personality disorder from ciently rule out most medical causes of mania. schizoid personality disorder. Paranoid ideation They include endocrine disorders such as thy- is common in both schizotypal and paranoid rotoxicosis and Cushing disease, hypoglycemia, personality disorders but not in the others. electrolyte disorders, substance abuse and with- Schizoid personality disorder represents a perva- drawal, medications such as steroids and anti- sive pattern of detachment from social rela- cholinergic agents, nutritional deficiencies, and tionships and a restricted range of expressed CNS insults. emotions. Avoidant personality disorder represents 216 8: Practice Test 1

24. (C) Gingival hyperplasia is associated with requires that during the period of illness there administration of phenytoin. Other, dose- are symptoms that suggest a diagnosis of both related symptoms include nystagmus, dizziness, schizophrenia and a mood disorder (major slurred speech, ataxia, mental confusion, and depressive episode, manic episode, or a mixed decreased coordination. Hepatic failure is asso- episode). Cannabis intoxication can present ciated with divalproex sodium, and Ebstein with euphoria, paranoia, and impaired judg- anomaly is associated with lithium therapy. ment in association with physical symptoms of conjunctival injection, increased appetite, dry 25. (D) Before any pharmacologic changes are con- mouth, and tachycardia. A decreased need for sidered, the physician must assess compliance sleep does not suggest cannabis intoxication. with medications. The subtherapeutic pheny- toin level may be due to patient noncompliance, 29. (E) Culture-bound syndromes denote recurrent, especially if side effects are being experienced. locality specific patterns of behavior. The syn- drome of amok is a culture-bound syndrome of 26. (E) The risk of schizophrenia among first- Malayan origin that refers to a violent or furious degree relatives of patients with the disease outburst with homicidal intent. Four defining may be as high as 10%, compared with a 1% characteristics are prodromal brooding, a homi- risk in the general population. cidal outburst, persistence in reckless killing without an apparent motive, and a claim of 27. (D) Attention is the ability to focus one’s percep- amnesia. The attack typically results in multiple tion on an outside or inside stimulus. A simple casualties and is most common in young men test of attention is digit recall. Most people can whose self-esteem has been injured. Ganser syn- recall a 7-digit number forward and 5–7 digits in drome is characterized by a patient who responds reverse. Concentration refers to sustained atten- to questions by giving approximate or outright tion to an internal thought process. The serial- ridiculous answers. Additional features include sevens examination, in which the examiner asks altered consciousness, hallucinations, conver- the patient to subtract 7 from 100 and continue sion phenomenon, and amnesia for the episode. the subtraction process for as many calculations Dissociative identity disorder is a chronic state in as possible, is a good test of concentration. which two or more separate ongoing identities Another test of concentration is to have the or personalities alternate in consciousness. It patient spell a five-letter word forward and usually occurs in patients who experienced backward. Asking the patient to recall the exam- severe and repeated abuse as young children. iner’s name is an example of recent memory, and asking him to recall his Social Security 30. (C) Neuroleptic-induced extrapyramidal side number is a test of remote memory. effects due to the blockade of dopamine are common in the treatment of psychosis. The higher- 28. (E) Cocaine intoxication can appear like a potency neuroleptic drugs are more likely to cause manic episode associated with bipolar I dis- extrapyramidal side effects than the lower-potency order with an increase in energy, euphoria, neuroleptics. Extrapyramidal side effects are grandiosity, and impaired judgment. The acute diminished by agents such as benztropine, anti- onset of symptoms makes the diagnosis of histamines, benzodiazepines, and dopamine ago- major depression unlikely. A diagnosis of schiz- nists. There are four types of extrapyramidal side ophrenia requires the presence of symptoms effects: acute dystonic reactions, akathisia (rest- during a significant portion of the previous lessness), pseudoparkinsonism, and tardive dysk- month, which is not evident. In this case, the inesia or tardive dystonia. In this case, the patient experiences a sudden onset of euphoria treatment of the involuntary movements involves that is not characteristic of schizophrenia. evaluation of dose and type of neuroleptic. Negative symptoms (e.g., flattened affect, avo- Reducing the dose, substituting with a higher- lition, alogia) are more common in schizophre- potency agent, or adding an antiparkinsonian nia. The diagnosis of schizoaffective disorder drug are all feasible therapeutic interventions. Answers: 24–36 217

31. (B) Atypical antipsychotic agents are thought to haloperidol but experienced extrapyramidal side act on various subpopulations of dopamine neu- effects, common to the high-potency antipsy- rons as well as on various dopamine receptor chotic medications. Benztropine, an anticholin- subtypes and other neurotransmitter systems. ergic medication, is prescribed to alleviate They are considered atypical agents because they extrapyramidal side effects, but in this patient it are associated with a lower risk of extrapyrami- caused anticholinergic side effects. Amantadine dal symptoms. Clozapine, introduced to the is an antiviral medication that is used to help United States in 1989, is the first antipsychotic to alleviate extrapyramidal side effects and should be labeled atypical. The other choices are all con- be instituted to replace benztropine. sidered typical antipsychotics. 35. (C) Denial, a narcissistic defense, is a common 32. (D) Borderline personality disorder is a pattern unconscious emotional defense mechanism of instability in interpersonal relationships, self- used to avoid becoming aware of a painful image, affect, and marked impulsivity. DBT, a aspect of reality. Denial can be used in both manualized treatment for chronically parasui- normal and pathologic states. As a defense cidal patients that incorporates elements of cog- mechanism it serves to keep internal or exter- nitive, behavioral, and supportive therapies, nal reality out of the conscious to avert stress is a form of cognitive-behavioral treatment and anxiety. Regression, an immature defense, for borderline personality disorder. Avoidant is an emotional and physical retreat from adult personality disorder represents a pervasive standards of behavior toward an infantile level pattern of behavior characterized by social inhi- of passivity and dependence. At times, regres- bition, feelings of inadequacy, and hypersensi- sion represents a retreat from the arduous task tivity to negative evaluation. Schizoid personality of recovery. Rationalization is a neurotic defense disorder represents a pervasive pattern of detach- mechanism in which unacceptable behavior, ment from social relationships and a restricted feelings, or thoughts are logically justified by range of expressed emotions. Bipolar disorder is elaborate and reassuring answers. Suppression, an affective illness characterized by alternating a mature defense, is a conscious act of control- periods of mania and major depression. Passive- ling and inhibiting unacceptable impulses, aggressive personality disorder describes a emotions, or ideas. pervasive pattern of negativistic attitudes and passive resistance to demands for ade- 36. (B) The essential feature of acute stress disorder quate performance. is the development of anxiety and dissociative symptoms within 1 month of exposure to an 33. (A) General principles of DBT include believing extremely traumatic stressor. Diagnosis should that the patient wants to improve and is doing be considered if the symptoms persist at least the best she or he can. The patient is encour- 2 days and cause significant distress or impair- aged to take responsibility and solve her or his ment. While experiencing the stressor, the indi- own problems. Patients may not have caused all vidual must experience three of these five of their problems, but they are encouraged to symptoms: a subjective sense of numbing, solve them anyway. Patients learn new behav- detachment, or absence of emotional respon- iors in a relative context and believe that they siveness; a reduction in awareness of sur- cannot fail in therapy. roundings; derealization; depersonalization; or dissociative amnesia. In addition, the person 34. (A) The patient is experiencing anticholinergic must reexperience the event in some way (e.g., toxicity as evidenced by dilated pupils, dry or bad dreams), must experience anxiety or flushed skin, agitation, confusion, and tachy- increased arousal, and must experience a cardia. Additional manifestations include dis- marked avoidance of the stimuli that arouse orientation and urinary retention. More severe recollections of the traumatic event. The diagno- toxicity may result in hyperthermia or coma. sis of PTSD requires more than 1 month of symp- In this case, the patient reported benefit from toms. Adjustment disorder can be considered for 218 8: Practice Test 1

those individuals who do not meet criteria for abuse. Partial complex seizures tend to be brief acute stress disorder but develop similar symp- and do not last as long as the clinical picture toms in excess of what is to be expected given presents. Delirium is characterized by a distur- the nature of the stressor. GAD is character- bance of consciousness and a change in cogni- ized by excessive anxiety and worry that occur tion that develops over a short period of time. for at least 6 months. MDD can exist in the con- text of an acute stress reaction. However, in 40. (C) This patient does not meet full criteria for this case, the symptoms are not suggestive of a major depression (e.g., 4/5 Diagnostic and depressive illness. Statistical Manual of Mental Disorders, Fourth Edition, Text Revision [DSM-IV-TR] criteria), so 37. (E) Psychogenic unresponsiveness can be delin- she cannot be diagnosed with bipolar II disor- eated from coma by obtaining an EEG, which der, which is characterized by the occurrence of is normal in a psychogenic state. On physical one or more major depressive episodes accom- examination, deep tendon reflexes may be sup- panied by at least one episode of hypomania (a pressed. In patients who are awake, cold water distinct period of a persistently elevated, expan- introduced into the ear produces nystagmus sive, or irritable mood that lasts at least 4 days). with the fast component away from the ear. In Bipolar II disorder differs from bipolar I disor- coma, the eyes either do not react or they may der in that, although the manic and hypomanic slowly and smoothly deviate toward the ear in episodes of these two disorders have identical which the cold water was introduced. Voluntary list criteria, bipolar II disorder is not severe eye movements called saccades are rapid and enough to cause marked impairment in social smooth. Saccadic eye movements are elicited in or occupational functioning or to cause hospi- patients by asking them to stare at an object at talization. Dysthymic disorder is also ruled out one side of the visual field and then ask them to by the presence of manic symptoms making shift their gaze to the opposite visual field. cyclothymic disorder the best choice. Typically, eye movements in coma or persistent vegetative states are spontaneous and random. 41. (B) The hallmark of Ganser syndrome is that the patient responds to questions by giving 38. (E) Seasonal affective disorder describes a sea- approximate or ridiculous answers. Often con- sonal pattern of symptoms associated with fused with malingering, Ganser syndrome is major depressive episodes in MDD, bipolar I primarily described in the prison population. It disorder, and bipolar II disorder. The essential involves the production of answers to ques- feature is the onset of depressive symptoms at tions that are relative but not quite correct (e.g., characteristic times of the year. More common in 4 × 5 = 19). It is classified in the DSM-IV-TR as women, most episodes begin in the fall or winter a dissociative disorder not otherwise specified. and remit in the spring. Major depressive Munchausen syndrome is a factitious disorder episodes that occur in a seasonal pattern are typ- with physical symptoms. Capgras syndrome ically characterized by anergy, hypersomnia, describes a content-specific delusion in which overeating, weight gain, and carbohydrate crav- the patient believes that a significant other, ing. Age is a strong predictor, with young per- usually a family member, has been replaced by sons at higher risk. an identical imposter.

39. (C) The essential features of dissociative fugue 42. (E) ECT provides safe and effective treatment of are sudden and unexpected travel away from MDD in the elderly, especially in a patient with home or one’s usual place of daily activities a high risk of suicide or medical contraindica- with a subsequent inability to recall the episode tions (e.g., heart disease) to the use of psy- accompanied by confusion about personal iden- chotropic medications. Nortriptyline, a TCA, is tity or the assumption of a new identity. The dis- lethal in overdose and should not be prescribed order is more common in women and is often in an acutely suicidal patient. Cardiac arrhyth- associated with a childhood history of sexual mia is also a contraindication to TCA use. Answers: 37–50 219

43. (B) The specifiers of melancholic features and Cataplexy is a sudden and brief loss of muscle atypical features can be applied to a current or tone involving either a few muscle groups or recent major depressive episode that occurs in most of the antigravity muscles of the body. the course of MDD and to a major depressive Dystonia is a stiffening of muscle groups some- episode in bipolar I or bipolar II disorder. The times seen after administration of typical neu- atypical features specifier can also be applied to roleptics. The patient does not seem to have a dysthymic disorder. The essential features of secondary gain for his symptoms making the atypical specifier are mood reactivity and malingering unlikely. two of the following four features: increased appetite or weight gain, hypersomnia, leaden 47. (D) Priapism can occur with a number of anti- paralysis, and rejection sensitivity. The essential depressant or antipsychotic agents but occurs feature of a major depressive episode with most often with the use of trazodone (1 in 1000 melancholic features is a loss of interest or to 1 in 10,000 incidence). The three SSRIs (flu- pleasure in all or almost all activities or a lack oxetine, sertraline, and paroxetine) listed can of reactivity to usually pleasurable stimuli. cause impotence, but are unlikely to cause pri- apism. Nortriptyline is a TCA not associated 44. (A) Identification is an unconscious defense with priapism. mechanism in which the person incorporates the characteristics and qualities of another 48. (C) The condition usually occurs within the person or object into his or her own ego system. first 4 weeks of treatment but can happen as The defense serves to strengthen the ego. late as 18 months into treatment. The condi- Regression, an immature defense, is an emo- tion is independent of dose. Priapism is a med- tional and physical retreat from adult standards ical emergency and may require surgery. The of behavior toward an infantile level of passiv- condition may respond to intracavernosal injec- ity and dependence. Projection is the false attri- tion of an alpha-adrenergic agonist to promote bution of one’s own unacceptable feelings to detumescence and constriction of veins. another. Fixation refers to an overactive attach- ment to a person or object; idealization is the 49. (A) The patient is most likely suffering from attribution of near-perfect, unrealistic attrib- GAD. Treatment of GAD includes antide- utes to that person or object. pressants, benzodiazepines, buspirone, and cognitive-behavioral therapies. However, ben- 45. (C) The essential feature of the catatonia observed zodiazepines should be avoided in patients with in patients with schizophrenia is a marked respiratory compromise, because these medica- psychomotor disturbance involving motor tions can exacerbate breathing-related disorders. immobility, excessive motor activity, mutism, Alprazolam, diazepam, and lorazepam are all negativism, peculiar voluntary movements, benzodiazepines. Buspirone, a nonbenzodi- echolalia (parrot-like senseless repetition of words azepine anxiolytic, is a logical alternative and or phrases), or echopraxia (imitation of move- has been demonstrated to be effective in the ments of another person). Catalepsy is just one treatment of GAD. Beta-blocking agents such potential symptom of catatonia, and is not a com- as propranolol are effective but should not be plete answer. Cataplexy is a sudden and brief first-line therapy in patients with chronic loss of muscle tone involving either a few muscle obstructive pulmonary disease or other respi- groups or most of the antigravity muscles of the ratory disorders. body. The fact that the patient occasionally mimics the movements of others makes seizure 50. (B) The long-term use of benzodiazepines activity unlikely. (temazepam, oxazepam, and clorazepate) for sleep is controversial especially in patients 46. (A) Catalepsy is a general term for the assump- with substance abuse or breathing-related dis- tion of an immobile position that is constantly orders. Zolpidem is not a member of the ben- maintained, especially after an examination. zodiazepine class but does produce its effects 220 8: Practice Test 1

at the gamma-aminobutyric acid (GABA)- the mammillary bodies and the thalamus. benzodiazepine receptor and as a result has A history of alcohol abuse is common but other a much lower risk of dependence. Zaleplon causes of thiamine deficiency such as starva- (Sonata) is a newer nonbenzodiazepine hypnotic tion, prolonged vomiting, and gastric carci- that interacts with the GABA-benzodiazepine noma can cause this syndrome. The use of IV receptor complex. It has an ultrashort half-life of dextrose prior to the administration of thiamine 1 hour. A nonbenzodiazepine medication is a may aggravate Wernicke encephalopathy. The logical alternative in cases in which respiratory other interventions may be appropriate if thi- function is compromised. Diphenhydramine amine does not resolve the symptoms. should be avoided in the elderly because of its anticholinergic effects. 54. (A) Aphasia is characterized by the loss of lan- guage abilities produced by brain dysfunction. 51. (D) Hypertensive crisis is a life-threatening Cardiovascular disorders, brain tumors, infec- emergency that may result from the combina- tion, trauma, and degenerative diseases are the tion of meperidine and an MAOI. The com- most common causes. Broca aphasia is charac- bined use of these medications is absolutely terized by short, nonfluent output with intact contraindicated. The treatment of hypertensive comprehension. Reading comprehension is crisis is with IV alpha-adrenergic blocking preserved. Wernicke aphasia presents as fluent agents and ICU admission and support. All of speech output with poor reading comprehen- the other medications are safe in combination sion. Repetition, naming, writing, and reading with MAOIs. aloud are impaired in both disorders. Wernicke aphasia results from lesions of the posterior 52. (C) The use of any medications in pregnancy superior left temporal gyrus (Wernicke area). requires examining the risks and benefits of treat- Broca aphasia results from a lesion of the ment. The use of MAOIs is contraindicated in middle third of the left inferior frontal gyrus pregnancy partly because they can exacerbate (Broca area). pregnancy-induced hypertension. Although ECT is safe in pregnancy, maintenance ECT is indi- 55. (C) Caffeine withdrawal occurs after the abrupt cated after an initial course is completed. The cessation or a marked reduction in the use of safety of newer antidepressants such as paroxe- caffeinated products. Studies estimate that up tine in pregnancy has not yet been completely to 75% of caffeine users will experience symp- established, but studies of fluoxetine suggest that toms of withdrawal at some point. Caffeine with- it is relatively safe. Specific recommendations drawal symptoms have their onset 12–24 hours regarding the course of treatment for a single after the last dose, and symptoms peak in episode of MDD do not exist. Many physicians 24–48 hours. Symptoms typically remit in find that the treatment of a single episode 1 week. Although all the possible choices are requires 9–12 months of pharmacotherapy. The associated with caffeine withdrawal, up to 50% return of symptoms suggests a recurrent MDD of sufferers experience headache, the most that requires longer-term pharmacotherapy. Of common symptom. course, patient preference plays a role as well. 56. (E) This patient is most likely suffering from 53. (D) The patient presents with clinical symp- NMS. Death, reported in up to 30% of cases, toms of Wernicke encephalopathy, a medical usually results from the failure to recognize emergency, characterized by ophthalmoplegia, the syndrome or from delayed treatment. Dis- ataxia, and delirium and caused by thiamine continuation of the offending agent and sup- deficiency. If the thiamine deficiency is not cor- portive treatment in an ICU setting is imperative. rected, Wernicke-Korsakoff syndrome emerges Cooling measures reduce hyperthermia. IV characterized by amnesia, ataxia, and nystag- dantrolene sodium and oral bromocriptine have mus. The syndrome results from chronic thi- been used but their benefits are unclear. The exact amine deficiency with resultant pathology in frequency remains unknown with some studies Answers: 51–62 221

suggesting up to 1.9% of all patients treated with 60. (A) Lithium, commonly used to treat bipolar antipsychotics experience this syndrome. NMS disorder, can be dangerous in intoxication, may be more frequent in men and younger which is a neurotoxic event and can lead to patients. Naloxone does not have a place in the death or permanent damage to the nervous treatment of NMS. system particularly the cerebellum. Cardio- vascular and renal manifestations may also be 57. (C)An elevated serum CPK demonstrates muscle present. Lithium has a low therapeutic index injury. Leukocytosis is also seen. Elevated BUN with a therapeutic range of 0.5–1.5 mEq/L. may be seen in dehydration resulting from NMS Plasma concentrations above the therapeutic but is not as common as CPK elevation. range, especially greater than 2.0 mEq/L, pre- cipitate severe CNS and renal impairment. 58. (D) Physical dependence emphasizes the pres- Early clinical signs of intoxication include ence of tolerance or withdrawal. Naloxone dysarthria, coarse tremor, and ataxia. Impaired injection can precipitate opiate withdrawal in consciousness, fasciculations or myoclonus, dependent persons. Positive urine toxicology seizures, and coma are ominous manifesta- or breath analysis can identify only substance tions. The goal of treatment is removal of abuse not dependence. Elevated liver function lithium from the body. Gastric lavage, rehydra- studies can be present in individuals who tion, and hemodialysis are possible interven- abuse alcohol but can also be present in other tions. Given the danger of lithium intoxication, non-substance-related disorders. Elevated heart a serum drug level should be obtained before rate can be present in many withdrawal syn- any of the other interventions. dromes or medical disorders and is not specific for any substance. 61. (E) Baseline medical tests should be obtained and documented before starting many psycho- 59. (B) Dementia is an organic and global decline of tropic medications including the mood stabiliz- functioning without impaired consciousness. ers lithium, carbamazepine, and divalproex (This differentiates it from delirium character- sodium. Assessment of thyroid function, com- ized by a waxing and waning consciousness.) plete blood count, electrolytes, ECG, and renal The most common type of dementia (50–60%) is function studies (BUN, creatinine, urinalysis) are the Alzheimer type. Vascular dementia is the important before the initiation of lithium. Older second most common type of dementia, occur- patients require additional tests, including EEG, ring in 10–20% of cases. Other degenerative thyrotropin-releasing hormone test, 24-hour processes that cause dementia are Parkinson urine volume determination, and creatinine disease, Huntington disease, and Pick disease. clearance. Lithium-induced polyuria results from Infectious, metabolic, endocrine, neoplastic, and the inhibition of the effect of antidiuretic hor- toxic processes can also cause dementia. In this mone. Reversible T-wave changes on the ECG case, the history of cognitive decline without are common. impaired consciousness suggests a diagnosis of dementia. The history reveals a megaloblastic 62. (D) The serotonin syndrome occurs when an

anemia, the cause of which may be a vitamin B12 SSRI is combined with another drug that can deficiency possibly secondary to a subtotal gas- potentiate serotonin, such as the monoamine trectomy. Folate deficiency can also cause mega- oxidase-inhibiting drugs, pentazocine, L- loblastic anemia. Vitamin deficiencies such as tryptophan, lithium, and carbamazepine.

B12 and folate can cause clinical signs of demen- Symptoms of the syndrome include abdominal tia but represent one of a few possible reversible pain, diarrhea, diaphoresis, hyperpyrexia, causes of this process. A complete reversible tachycardia, hypertension, myoclonus, irri- workup for dementia should also include thy- tability, agitation, seizures, and delirium. roid function studies and a rapid plasma reagin Coma, cardiovascular collapse, and death have to test for syphilis. Alzheimer disease is con- been reported. In this case, the serotonergic firmed only by autopsy. effects of fluoxetine are potentiated by the 222 8: Practice Test 1

MAOI. Fluoxetine has a prolonged half-life with females are diagnostic criteria. Underdeveloped active metabolites that require up to 6 weeks’ breasts, abnormal insulin secretion, endocrine clearance time. Treatment with the MAOI was disorders, and poor sexual development with a initiated before adequate washout of the fluoxe- marked decreased interest in sexual activity are tine and resulted in the serotonin syndrome. also associated with this disorder. For anorexia Management involves discontinuation of the nervosa to be diagnosed, a postmenarchal offending agent and supportive care. There is female must have missed at least three consec- no evidence that the patient is continuing to take utive periods. Patients do not lose their feel- her MAOI and eat a tyramine-heavy diet, that ings of hunger despite their refusal to eat she has been abusing hallucinogens, or that she properly. is taking antipsychotics that might induce any of the remaining choices. 66. (C) The mortality rate for anorexia nervosa ranges from 5% to 20%. 63. (A) Bulimia nervosa, more common than anorexia nervosa, consists of repeated episodes of binge 67. (A) Bereavement refers to the reaction to the eating of large amounts of food associated with death of a loved one. Some individuals develop the feeling of being out of control. The patient symptoms of a major depressive episode. For usually purges by self-induced vomiting or example, 50% of all widows and widowers repeated laxative or diuretic use and engages meet criteria for major depression within the in excessive amounts of exercise to prevent first year. The bereaved usually refers to feeling weight gain. Unlike individuals with anorexia depressed as being normal. If the symptoms nervosa, individuals with bulimia nervosa usu- of a major depressive episode begin within ally maintain a normal body weight. Medical 2 months of the loss and do not persist beyond complications of bulimia nervosa are usually the 2-month period, they are considered the secondary to either chronic vomiting or laxa- result of bereavement unless associated with tive abuse. Fluid and electrolyte imbalances marked impairment of any of the following: (alkalosis, hypokalemia, hypochloremia, dehy- thoughts of death other than feelings that they dration); dental caries and enamel loss; gas- would be better off dead or should have died trointestinal disturbances such as esophagitis, with the deceased person, expressing guilt Mallory-Weiss tears, and constipation; and sore other than about the actions taken or not taken throat are also common. by the survivor at the time of the death, a morbid preoccupation with worthlessness, 64. (B) Antidepressant medications reduce binge marked psychomotor retardation, or halluci- eating and purging independent of the presence nations other than that he or she hears the voice of a mood disorder. Imipramine, desipramine, of (or sees the image of) the deceased individ- MAOIs, and fluoxetine have been effective in ual. Complicated bereavement is chronic and controlled studies. The other choices treat symp- unremitting distress. Chronic grief may occur toms of the illness without treating the under- if the relationship between the bereaved and lying disorder. the deceased was close or ambivalent, if they were dependent on one another, or if few other 65. (B) Anorexia nervosa is clinically characterized support systems are available. Bereavement by a deliberate self-imposed starvation in the reactions are very intense after sudden or unex- pursuit of being thin, driven by a fear of being pected deaths. Prolonged denial, anger, or guilt fat. The patient refuses to maintain accepted complicates the course of bereavement. normal body weight appropriate for age and Adjustment disorder requires the development height. Failure to achieve 85% of expected body of symptoms within 3 months of an identifiable weight, the intense fear of being fat, a disturbed stressor, with marked distress in excess of what perception of the body weight or shape (or would be expected from experiencing the stres- denial of the seriousness of the current low body sor: In this case, the symptoms are considered weight), and amenorrhea in postmenarchal acceptable reactions to the death of a spouse. Answers: 63–74 223

Dysthymic disorder requires 2 years of symp- effective in normalizing sleep in these patients. toms marked by a depressed mood. The use of nasal CPAP appears to prevent long- term morbidity and mortality and provides 68. (A) Most bereaved do fine without treatment. restorative sleep. Surgical procedures are Support and reassurance usually come from reserved for CPAP treatment failures. family, friends, or clergy. When the bereaved feel unable to move beyond the grief, psy- 72. (B) Anticholinergic toxicity can be caused by chotherapeutic measures are appropriate. The many psychotropic medications. The low- treatment of anxiety or insomnia with medica- potency agents chlorpromazine and thioridazine tions remains controversial and no controlled and the TCAs amitriptyline and imipramine are studies have established clear efficacy. However, particularly anticholinergic. The syndrome often medications may be warranted if cognitive or results when an anticholinergic medication is other higher functions of daily living are sub- coadministered with an antipsychotic medica- stantially affected. tion to prevent or treat extrapyramidal side effects such as parkinsonism and dystonia. 69. (C) The treatment of patients with obsessive- Clinical features include decreased secretions, compulsive disorder (OCD) requires integrat- agitation, dry skin, flushing of the skin, hyper- ing multiple treatment modalities including thermia, tachycardia, dilated pupils, urinary medications, psychotherapy, and collateral retention, constipation, and hypotension. support systems. The best-studied medication Seizures, hallucinations, and coma may result to treat obsessions is clomipramine, a TCA. from severe intoxication. Drugs that inhibit the SSRIs have also been effective in treating symp- enzyme that breaks down acetylcholine (anti- toms of OCD, but at doses higher than those cholinesterase medications such as physostig- used to treat depression. MAOIs are helpful mine) reverse the syndrome and are usually for those who suffer from comorbid anxiety. administered IV with appropriate monitoring. Lithium has not been proven to be helpful in Atropine would further increase the heart rate; the treatment of OCD. Antipsychotic agents haloperidol would likely worsen anticholiner- are used in conjunction with antidepressants gic symptoms and benzodiazepines are but are not usually indicated as individual reserved for use in agitation and anxiety. agents. Anxiolytic agents are of little use in treating obsessions. Antiepileptics are not indi- 73. (A) Anticholinesterase toxicity, manifesting as cated for the treatment of OCD. nausea, vomiting, bradycardia, and seizures, result from excessive action of acetylcholine at 70. (D) is defined as the cessation of specific receptors. Atropine, an antimuscarinic respirations lasting at least 10 seconds. The agent, can reverse the symptoms. Physostigmine presence of more than five episodes of apnea would worsen the toxicity. Although prochlor- per hour of sleep is diagnostic. Sleep apnea is perazine would combat vomiting, it would not classified as obstructive, central, or a mixed treat the underlying condition and might picture of both types. Classified as a breathing- worsen the anticholinergic toxicity. Similarly, related sleep disorder, sleep apnea leads to epinephrine would treat the bradycardia with- excessive sleepiness or insomnia. Characterized out treating the underlying condition. by upper airway obstruction, is associated with , morning 74. (E) SSRI medications are used to treat a variety of headache, excessive movements at night, and medical and psychiatric disorders. Fluvoxamine mental dullness. The typical patient is a middle- is an SSRI particularly indicated for both depres- aged obese man. sion and OCD. Doxepin and clomipramine are TCAs and phenelzine is an MAOI. Buspirone is 71. (A) Nasal continuous positive airway pressure a nonbenzodiazepine medication used to treat (CPAP) is the treatment of choice for obstructive anxiety disorders. sleep apnea. No medications are consistently 224 8: Practice Test 1

75. (D) Because they act at the same GABA receptors severe cardiac disease, pregnancy, or peripheral as alcohol, the benzodiazepines are the preferred neuropathy. In this case, the patient was pre- medications used in detoxification from alcohol. scribed chlordiazepoxide and presumably had For uncomplicated detoxification, the long-acting preserved liver function. benzodiazepines such as chlordiazepoxide and diazepam are appropriate because they are essen- 78. (A) Many nonsteroidal anti-inflammatory drugs tially self-tapering. The shorter-acting benzodi- can decrease the clearance of lithium and pro- azepines such as lorazepam or oxazepam are duce significant increases in serum levels. used in patients with impaired liver function, Ibuprofen, indomethacin, ketoprofen, diclofenac, because they are not oxidized to long-acting phenylbutazone, naproxen, and piroxicam have metabolites and do not accumulate. Patients all been reported to produce such potential dan- with unstable medical problems, cognitive gerous interactions. The symptoms of abdominal impairment, or old age are also candidates for pain, diarrhea, and drowsiness indicate mild to the shorter-acting medications. Phenobarbital moderate (1.5–2.0 mEq/L) lithium toxicity. was formerly used for detoxification and has Acetaminophen, aspirin, and the opiates do not been replaced by benzodiazepines. Disulfiram, interfere with lithium clearance. Meperidine, which interferes with the metabolism of alcohol, however, can cause dangerous interactions is reserved for long-term behavioral therapy of with MAOIs. alcohol dependence. 79. (B) Clinically effective antidepressant medica- 76. (B) The patient is experiencing hallucinations tions potentiate the actions of serotonin, nor- due to alcohol withdrawal. Benzodiazepines epinephrine, and dopamine in the brain. The will treat the underlying withdrawal as well antidepressant action of amitriptyline, a TCA, as the hallucinations. An antipsychotic is is to significantly block the reuptake of sero- unnecessary and may also reduce the seizure tonin and to a lesser degree norepinephrine. threshold. The patient shows no evidence of a Bupropion is a weak inhibitor of dopamine mood disorder that would be best treated with reuptake with modest effects on norepineph- sertraline or fluoxtine (SSRIs), or lithium, a rine. It does not have any effect on serotonin. mood stabilizer. Lorazepam is an anxiolytic medication whose actions are mediated by GABA receptors. 77. (D) Ethanol is metabolized in the liver to Maprotiline is a TCA with modest effects on acetaldehyde, which is enzymatically converted norepinephrine and no effect on serotonin. to acetate by the enzyme aldehyde dehydroge- Desipramine, a TCA, has potent effects on nor- nase. Disulfiram is a medication that irreversibly epinephrine reuptake and has weak effects on inhibits the action of alcohol dehydrogenase. the serotonergic system. Acetaldehyde accumulates and accounts for the aversive effects associated with the disulfiram- 80. (A) Medically ill patients with depressive disor- ethanol reaction. Flushing, sweating, dyspnea, ders may respond to psychostimulants, an excel- hyperventilation, tachycardia, hypotension, lent choice if the patient is unable to tolerate nausea, and vomiting are common symptoms. TCAs. The rapid onset of action and rapid clear- Extreme reactions can result in respiratory ance are beneficial. Nefazodone, fluoxetine, and depression, cardiovascular collapse, myocar- bupropion are effective antidepressants but may dial infarction, seizures, and death. Blurred take 4–6 weeks to accomplish beneficial results. vision and urinary retention are anticholinergic Buspirone is a nonbenzodiazepine anxiolytic effects associated with antipsychotic, antide- medication not indicated in the treatment of pressant, and antiparkinsonian medications. depression. Hypertension is associated with but not char- acteristic of the disulfiram-ethanol reaction. 81. (C) Inhalant abuse is commonly a social activity Disulfiram is not recommended for patients with undertaken in groups. Volatile or toxic solvents moderate to severe liver disease, renal failure, such as toluene (found in glues and adhesives), Answers: 75–87 225

trichloroethane (found in correction fluid), or carbamazepine, clozapine, olanzapine, or hydrocarbons (gasoline) are placed in a bag and ziprasidone, although these too have side effects. the fumes are inhaled. The practice is known as huffing, sniffing, or bagging. Effects are almost 85. (C) Sexual dysfunction has become an increas- immediate because the substances are rapidly ingly common side effect associated with many absorbed. Signs and symptoms of intoxication antidepressant medications. It may be under- include visual disturbances, dyscoordination reported for many reasons. Decreased libido, (a drunken appearance), depressed reflexes, erectile dysfunction, ejaculatory dysfunction, euphoria, and nystagmus. An increased appetite and anorgasmia have been reported. None of suggests marijuana use, and staring into space the other choices are associated with sexual and euphoria are associated with PCP use. dysfunction to the same degree as the SSRIs, and some of the choices may be used to reverse 82. (C) Abstinence is the only treatment for inhalant the dysfunction. Various pharmacologic strate- abuse. Antipsychotic agents are of no use and gies aimed at treating these side effects include may aggravate organic presentations of inhalant the alpha-2-antagonist yohimbine; the dopamine abuse. Agitation can be managed by benzodi- agonists methylphenidate and amantadine; azepine medications such as lorazepam. bupropion; cyproheptadine (an antihistamine with serotonin antagonist properties); and 83. (E) The five subtypes of schizophrenia (para- bethanacol, a cholinergic agonist. noid, catatonic, residual, disorganized, and undifferentiated) are based on the presence or 86. (A) ADHD is a disorder of unclear etiology. As absence of predominant symptoms. The para- many as 9% of children suffer from this disor- noid type is marked by prominent hallucina- der, and up to 60% of those diagnosed carry tions or delusions; the disorganized type is the diagnosis into adulthood. The disorder is marked by disorganized speech and behavior characterized by symptoms of inattention, and inappropriate affect; and the catatonic type hyperactivity, and impulsivity that are inap- is marked by waxy flexibility or motoric immo- propriate for age. Although the DSM-IV-TR cri- bility, excess motor activity, negativism, bizarre teria suggest that symptoms must have been posturing, and echolalia or echopraxia. The present before the age of 7 years, many cases are residual type is marked by the absence of pre- not diagnosed until adolescence or adulthood. dominant hallucinations or delusions, disor- In these cases, a retrospective history from both ganized speech, or catatonic behavior. The the patient and the family often support the undifferentiated type is reserved for patients presence of early life symptoms. Although some who meet criteria for schizophrenia but do not symptoms are reminiscent of the manic com- meet criteria for the other four types. ponent of bipolar disorder, these disorders rep- resent two separate entities. A careful history by 84. (A) Divalproex sodium, indicated for the treat- astute physicians will secure the proper diag- ment of simple or complex absence seizures or as nosis. Depressive, anxiety, and substance-abuse an adjunct medication in the treatment of partial disorders are common comorbid conditions seizures, can cause sometimes significant hair that often make the diagnosis of ADHD more loss that is usually reversible. Common side difficult. The absence of prominent mood symp- effects associated with divalproex sodium are toms and the relative degree of impairment help nausea, vomiting, and indigestion. Sedation and distinguish ADHD from other disorders. mildly elevated serum transaminase levels are also seen. Hepatic failure resulting in fatalities 87. (D) The approach to the treatment of the core has occurred, usually during the first 6 months of symptoms of ADHD has been the initiation of treatment. Divalproex sodium is also used in a trial of a psychostimulant medication. Often, the treatment of migraine headache and as a if one stimulant fails to treat symptoms, another mood stabilizer. Hair loss is not associated with stimulant is prescribed. Many patients benefit 226 8: Practice Test 1

from one stimulant and not another. The by an agitated, highly changeable psychosis appropriate choice in this case would be to try that usually develops between the third and a second type of stimulant before alternative fourteenth day postpartum and can lead to the strategies are implemented. fatal complication of infanticide. The incidence of postpartum psychosis in primiparous 88. (A) The median age of onset for the first psy- women is about 1 in 500. For those affected chotic episode of schizophrenia is in the early to after the first pregnancy, subsequent pregnan- mid-twenties for men and the late twenties for cies carry a risk of about 1 in 3. women. The onset can be abrupt or insidious. Estimates of prevalence have ranged from 90–92. [90 (A), 91(C), 92 (D)] Wechsler developed a 0.2% to 2.0%. Schizophrenia is characterized by variety of scales each to be used depending on a 1-month period of two of the following symp- the age of the examinee. WPPSI is designed for toms: delusions, hallucinations, disorganized children ages 4–6.5, the WISC is for children speech, disorganized behavior, and negative ages 5–15, and the WAIS-R is for people age symptoms such as affect flattening, alogia, or 15 and older. The WMS was designed to eval- avolition. Continuous signs of the disturbance uate a variety of memory deficits in adults. should be present for 6 months, with a decline in functioning in at least two areas (work, school, 93. (C) The Rey-Osterrieth figure is sensitive to interpersonal, or self-care). A prodrome often deficits in copying and lack of attention to precedes the acute phase of the disorder. detail in people with right-sided parietal lobe Distinguishing schizophrenia from mood disor- lesions. It appears that this young woman may ders with psychotic symptoms (e.g., bipolar I have had a stroke in this area resulting from her disorder with psychotic features) and schizoaf- protein S deficiency, hypercoagulable state. fective disorder can often be difficult in that a mood disturbance is common during the pro- 94. (B) The appropriate test to evaluate intelligent dromal and active phases of schizophrenia. If quotient (IQ) is the WISC. psychotic symptoms occur exclusively without the presence of mood symptoms, then schizo- 95. (G) The Boston Diagnostic Aphasia Examination phrenia is more likely. Schizoaffective disorder is a series of tests given by an experienced clini- describes the presence of a major depressive cian to evaluate and make treatment recom- episode, a manic episode, or a mixed episode mendations for individuals with aphasia. The concurrent with symptoms that are characteris- MMPI-2 (A) is an objective test interpreted by tic of schizophrenia: delusions, hallucinations, skilled evaluators used in personality assess- disorganized speech or behavior, or negative ment. It is the most widely used and highly symptoms. A brief psychotic disorder is defined standardized test of personality structure. The by the presence of delusions, hallucinations, dis- Beck Depression Inventory (D) is a 21-item test, organized speech or behavior, or catatonic with three responses per item, that is an easily behavior lasting for at least 1 day but less than used screening tool to evaluate for depression. 1 month. The Blessed Rating Scale (E) is a tool that asks friends or families of the patient to assess the 89. (A) In the context of labor, high-potency typi- ability of the patient to function in his usual cal antipsychotics are considered relatively environment. The WAIS-R (F) is used to pro- safe to use to control psychosis. For a mother vide an assessment of IQ of people older than 15. suffering psychosis during delivery, the benefit The Rorschach Test (H) is a projective test used of using high-potency antipsychotics outweighs to assess personality structure. The MMSE (I) is the risk because strictly speaking during delivery, a quick, easily administered test that allows for it is unclear if there is any risk in using these immediate assessment of dementia. Scores of medications. Thorazine and droperidol could less than 24 are suggestive of a dementing lower blood pressure significantly. A related process. The Bender-Gestalt Test (J) involves issue is postpartum psychosis characterized copying figures, which helps to determine if Answers: 88–100 227

organic brain disease is present. The Wisconsin Symptoms include miosis, bradycardia, hypoten- Card Sorting Test (K) assesses executive func- sion, hypo-thermia, and euphoria. Intoxication tions of the brain such as mental flexibility and can lead to fatal respiratory depression. the ability to abstract and reason. These capac- ities are believed to be located in the frontal 98. (B) PCP is an hallucinogen that produces a dis- lobes. Damage to the frontal lobes can lead to sociative anesthesia. It can cause unpredictable abnormalities on this test. Wada Test (L) is used behavior, assaultiveness, and belligerence. to evaluate hemispheric language dominance Agitation, nystagmus (vertical or horizontal), prior to surgical amelioration of seizure focus. tachycardia, a numbed response to pain, muscle Whereas most right-handed individuals show rigidity, hyperacusis, hypertension, echolalia, and left hemispheric dominance for language, left- anticholinergic effects are associated symptoms. handed individuals may either be right or left dominant. The test consists of injecting sodium 99. (F) Nicotine found in tobacco smoke produces amytal into the carotid artery and observing the symptoms of excitement. Intoxication may pro- transient effects on speech. Injection into the left duce symptoms of confusion, muscle twitching, carotid artery will anesthetize the left side of the weakness, abdominal cramps, depression, pal- brain; those with left hemispheric language pitations, coma, and respiratory failure. Use of dominance will show interrupted speech. hallucinogens (e.g., LSD, MDMA [3,4-methyl- enedioxy-N-methylamphetamine, or “Ecstasy”]) 96. (D) Cannabis (marijuana) is the most com- causes symptoms that usually begin 1 hour after monly abused illicit drug in the United States. ingestion and generally last 8–12 hours. Most The onset of action after smoking is immediate, hallucinogenic drugs have stimulant-type effects and symptoms include conjunctival injection, and produce elevated vital signs and increased mild sedation, dose-dependent hypothermia, activity. Hallucinations and a heightened sense of dry mouth, increased appetite, tachycardia, perception of objects and colors are typical. and euphoria. A sensation of slowed time and paranoid ideation can also occur. 100. (C) As a sympathomimetic agent, cocaine increases the heart rate and dilates the pupils. 97. (E) The onset of action for opiates can be almost It can also cause constriction of the coronary immediate when smoked, about 5 minutes arteries. The hallmark of cocaine intoxication is when injected and longer if taken orally. hallucinations. This page intentionally left blank CHAPTER 9 Practice Test 2 Questions

DIRECTIONS (Questions 1 through 4): Each set of 5. An 18-year-old woman recently diagnosed with items in this section consists of a list of lettered a first episode of schizophrenia agrees to take a headings followed by several numbered words or neuroleptic medication to help decrease her hal- phrases. For each numbered word or phrase, select lucinations, but she is adamantly opposed to the ONE lettered option that is most closely asso- taking any medication that may cause her to ciated with it. Each lettered option may be selected gain excessive weight. Which of the following once, more than once, or not at all. atypical neuroleptic medications is not associ- ated with significant weight gain? (A) citalopram (A) quetiapine (Seroquel) (B) fluoxetine (B) olanzapine (Zyprexa) (C) mirtazapine (C) risperidone (Risperdal) (D) nefazodone (D) ziprasidone (Geodon) (E) paroxetine (E) clozapine (Clozaril) (F) sertraline (G) venlafaxine 6. A 72-year-old woman complains to her physi- cian that she has been seeing “little people” 1. Has a warning causing hepatotoxicity walking around her house over the past few days; moreover, she saw one while waiting in 2. May increase blood pressure in higher doses the doctor’s office. Examination proves her to be alert and oriented. What term best describes 3. Has the most specific serotonin reuptake this phenomenon?

4. Has a half-life of approximately 7 days (A) illusion (B) hallucination DIRECTIONS (Questions 5 through 94): Each of (C) flashback the numbered items in this section is followed by (D) formication answer choices. Select the ONE lettered answer or (E) hallucinosis completion that is BEST in each case.

229

Copyright © 2007 by The McGraw-Hill Companies, Inc. Click here for terms of use. 230 9: Practice Test 2

7. A 25-year-old patient presents to her primary (D) She has a comorbid diagnosis of alcohol care doctor complaining of a sudden onset of dependence. intense fear and a feeling that she was going to (E) Her presenting complaint was of poor die while stuck in traffic earlier in the week. She appetite, insomnia, fatigue, poor was short of breath, diaphoretic, and tremu- concentration, and a feeling of lous, and she could feel her heart pounding. hopelessness. Her initial impulse was to drive to an emer- gency department, but her distress subsided 10. A man brings his wife to the doctor because she on its own in about 20 minutes. An appropriate believes that her husband is actually an course of action for the primary care doctor is imposter who looks exactly like her husband. which of the following? The term for this delusion is which of the (A) Prescribe a short-acting benzodiazepine following? for the next such episode. (A) koro (B) Refer the patient to a psychiatrist. (B) amok (C) Reassure the patient that her symptoms (C) Capgras syndrome are most likely benign and not a cause (D) pseudocyesis for concern. (E) couvade syndrome (D) Prescribe a selective serotonin reuptake inhibitor (SSRI) and ask to see the 11. A 49-year-old woman with diabetes presents to patient back in 1 week. her primary care physician. She says that as a (E) Conduct a thorough medical screening result of divorcing her husband 1 month ago, for heart and lung disease. she has been sleepless, agitated, and depressed. In her twenties, she drank alcohol abusively but 8. You are seeing a 45-year-old man with an she has not had any alcoholic drinks in many unknown psychiatric history. On Mental Status years. In terms of the classification of psychi- Examination (MSE), he speaks coherently and atric disease within the Diagnostic and Statistical articulately, but makes no sense because many of Manual of Mental Disorders, Fourth Edition, Text the words he uses are of his own invention. This Revision (DSM-IV-TR) multiaxis system, which patient demonstrates which of the following? of the following statements is true? (A) word salad (A) Her Axis II diagnosis is adjustment disorder. (B) negative symptoms (B) Alcohol abuse, in remission, should be (C) dysarthria placed on Axis I. (D) neologisms (C) Diabetes should be included on Axis IV. (E) clang associations (D) Recent divorce should be included on Axis V. 9. A 34-year-old woman is diagnosed with dys- thymic disorder. Which of the following state- (E) Her Global Assessment of Functioning ments regarding this patient is likely true? (GAF) should be included on Axis III.

(A) She has had a feeling of depression, for Questions 12 and 13 most of the day, for at least the past 6 months. You are asked to evaluate a 12-year-old boy who has (B) At some point in her illness, she may a history of Tourette disorder and complex vocal and have experienced mood-congruent motor tics. On evaluation, you note the patient inter- delusions. mittently making obscene gestures and repeating your phrases. (C) Her presenting complaint was suicidality. Questions: 7–18 231

12. In your report, you should describe the motor 16. A 49-year-old man comes to you complaining tics as which of the following? of headaches, memory loss, disorientation, and occasional paralysis that affects his arms and (A) blepharospasm lasts several hours. During the MSE, you notice (B) bruxism that the patient is giving approximate answers (C) copropraxia to many questions (e.g., there are six toes on the (D) echopraxia foot and 2 + 2 = 5). You also notice that the (E) torticollis patient is talking past the point. Feeling that this is a dissociative-type disorder, which of 13. When noting the vocal tics that were observed, the following could you label it? which of the following is the term best used? (A) amok (A) coprolalia (B) Ganser syndrome (B) echolalia (C) piblokto (C) palilalia (D) malingering (D) parapraxis (E) fugue state (E) dysarthria Questions 17 and 18 Questions 14 and 15 A 19-year-old woman presents with complaints of A wealthy middle-aged woman presents to you after fear, apprehension, and trembling without predis- being arrested for shoplifting. The patient says she posing situations noted. On examination, you note has been stealing for years, although she would have brown skin, smoky brown rings on the outer cornea, no difficulty affording the objects stolen. She states and occasional rapid, seemingly purposeless move- that she steals “on the spur of the moment” and that ments. After completing your workup, your diagno- these impulses seem foreign and distressing. sis is Wilson disease.

14. What is this form of impulse control disorder is 17. In preparing your report, the ocular findings called? are best termed which of the following? (A) pyromania (A) Kayser-Fleischer rings (B) trichotillomania (B) arcus senilis (C) intermittent explosive disorder (C) xanthelasma (D) kleptomania (D) Brushfield spots (E) pathologic gambling (E) subconjunctival hemorrhage

15. The fact that these impulses are distressing to 18. Which of the following is the best term for her indicate that they are which of the following? rapid, jerky movements that worsen with vol- untary movement? (A) ego-syntonic (B) ego-dystonic (A) choreiform (C) mood congruent (B) athetoid (D) mood incongruent (C) hemiballismus (E) delusional (D) myoclonus (E) myotonia 232 9: Practice Test 2

Questions 19 and 20 (C) nymphomania (D) thought broadcasting A 21-year-old woman is brought to the psychiatric emergency department after calling the police to turn (E) alexia herself in. She claims that she was responsible for the loss of her neighbor’s pregnancy. She believes her 22. The patient’s incorrect perception of his wealth negative thoughts toward the woman caused her mis- and education is most likely a form of which of carriage. On further questioning, she tells you that she the following? felt threatened by her neighbor because she believed (A) micropsia her thoughts could be heard through the walls. She (B) macropsia feels that this is an invasion of her privacy. (C) palinopsia 19. The delusion that the patient’s thoughts toward (D) delusion of grandeur the neighbor were the responsible factors (E) delusion of infidelity for the lost pregnancy is best termed which of the following? Questions 23 and 24 (A) magical thinking A devout husband finds that his wife is having an (B) ideas of reference affair with his best friend. One week later, he finds that (C) displacement he cannot walk. A thorough neurologic workup fails to reveal a cause to his sudden paraplegia. His neuro- (D) projection logic examination is not consistent with upper or lower (E) reaction formation motor neuron findings. Despite this dramatic disabil- ity, he seems quite unaffected by it emotionally. 20. Which of the following is the best term for the patient’s fear that her thoughts could be over- 23. Which of the following is the best label for the heard? sudden neurologic symptom without evidence (A) thought broadcasting of etiology? (B) thought insertion (A) projection (C) thought control (B) conversion (D) transference (C) sublimation (E) echolalia (D) dissociation (E) rationalization Questions 21 and 22

A 25-year-old man who is hospitalized for bipolar 24. This patient’s disregard for the severity of his disorder tells you that he has his doctorates in six symptoms is called which of the following? fields and currently is the richest person in the world. (A) déjà entendu He states that he speaks 28 languages, one of which (B) jamais vu is an angelic language. He tells you that he is con- (C) déjà vu cerned, however, because a well-known supermodel is infatuated with him and she will not leave him (D) la belle indifférence alone. He wishes to leave the hospital to spend some (E) folie à deux time with her. 25. A 67-year-old woman with pulmonary carci- 21. Which of the following is the best term for the noma and secondary brain metastases recently patient’s belief that a famous supermodel is in drafted a will in the presence of her family love with him? attorney. She has a history of major depressive disorder (MDD) that is now in remission. (A) erotomania She decides that her children who are well- (B) satyriasis established in their careers do not need any Questions: 19–29 233

inheritance and that her estate would best serve his insurance company. Which of the following charity. To secure the validity of the will, the is the next appropriate course of action? patient asks her psychiatrist to submit a letter (A) Notify a collection agency to obtain to her attorney regarding her competency. reimbursement. Which of the following is most important in determining a person’s testamentary capacity? (B) Contact the patient’s insurance company and demand that they issue you another (A) knowledge of natural heirs to the estate payment. (B) the presence of a conservator of person (C) Inquire with family members whether (C) history of mental illness the patient has financial problems. (D) actus reus (D) Directly address this issue with the (E) presence of a judge to witness the sign- patient at the next scheduled ing of the will appointment. (E) Send the patient a letter of termination 26. Which of the following is an exception to and instruct him to no longer call your informed consent? office. (A) nonemergent situations Questions 29 and 30 (B) patient waiver of informed consent (C) inability to read or write You are asked to perform a “competency to stand (D) presence of a language barrier trial” evaluation of an 18-year-old man who was arrested for violently assaulting his girlfriend. The (E) a living will patient has no past psychiatric history, and friends report that he has never demonstrated any violent 27. A deputy sheriff serves you a subpoena for the tendencies. The patient reports that his relationship records of one of your patients who is the with his girlfriend “went downhill” shortly after they defendant in a civil liability lawsuit. Which of graduated from high school and his girlfriend stopped the following is the most appropriate course of calling him. On MSE, his affect is constricted and his action? mood is reported as “crazy.” Thought processes are (A) Release the patient’s records to the goal-directed, and he denies any hallucinations or plaintiff because the subpoena overrides delusions. The patient is unable to correctly perform patient consent. simple calculations. Each time he is asked to multiply, (B) Hand over only the information that is subtract, or add a pair of numbers, his answers are relevant to the case. wrong by one or two digits. For example, he responds (C) Contact the patient and ask if she or he “22” when asked to multiply 7 by 3. would like the information released. 29. Intentionally feigning symptoms to avoid pros- (D) Release the patient’s records directly to ecution is consistent with which of the follow- the court. ing disorders? (E) Refuse to tell the sheriff if you are the patient’s physician to maintain confi- (A) factitious disorder dentiality. (B) malingering (C) factitious disorder 28. A 23-year-old graduate student you have been (D) conversion disorder seeing in long-term psychotherapy has stopped (E) Munchausen by proxy paying his bills despite being reimbursed by 234 9: Practice Test 2

30. Which of the following statements best distin- (D) Notify the probate court to have the guishes malingering from conversion disorder? patient legally removed from the alleged perpetrator. (A) Malingerers seek primary gain incentives, whereas individuals with (E) Contact the state’s Child Protective conversion disorder seek secondary gain Services while keeping the patient safe. incentives. Questions 32 and 33 (B) Malingerers are usually friendly and cooperative, whereas individuals with A 48-year-old male is brought into the emergency conversion disorder are uncooperative. room (ER) via an ambulance. He smells of alcohol, is (C) Malingering and conversion disorder covered in vomit, and not responsive to questions. He can be distinguished based on the is unsteady and uncooperative with the physical absence of an underlying neurologic examination, but he is noted to have disconjugate problem. eye movements. (D) Malingering represents an unconscious process, whereas conversion disorder 32. Which of the following would be the most results from conscious feigning of appropriate next step in the management of symptoms. this patient? (E) Malingerers avoid evaluation and treat- (A) Administer thiamine intramuscularly ment, whereas patients with conversion (IM) before IV fluids and glucose. disorder are usually eager for evaluation (B) Administer thiamine intravenously (IV) and treatment. before IV fluids and glucose. (C) Administer thiamine IV after IV fluids 31. A 7-year-old girl is brought to the emergency and glucose. department for evaluation of a sore throat and fever. Her parents took her to the pediatrician’s (D) Administer naloxone IV before IV fluids office about 1 week ago and he recommended and glucose. fluids and bed rest. Within the last 2 days, the (E) Administer naloxone IV after IV fluids patient developed dysphagia and severe abdom- and glucose. inal pain. History from the parents indicates the patient has been anxious and impulsive over 33. The above patient is stabilized and admitted to the last month. There has also been a marked the ICU. After 3 days, he begins to become decline in school performance, and she has not confused again, with visual hallucinations, been interested in playing with her friends. tremors, diaphoresis, and elevated blood pres- Physical examination is remarkable for signif- sure and pulse. Which of the following is the icant erythema over the posterior pharynx with most appropriate treatment for this patient? gray exudate. There are abrasions in the region (A) Administer a barbiturate. of the patient’s labia. Complete blood count (B) Administer a benzodiazepine. shows a white blood cell (WBC) count of (C) Administer additional thiamine. 14,000/µL with a left shift. Which of the fol- lowing is the next appropriate step? (D) Administer an antipsychotic. (E) Administer hydralazine. (A) Detain the parents while you notify the police. 34. A young White male, age unknown, is brought (B) Arrange for a family meeting to into the emergency room unresponsive to ques- determine a safe disposition. tioning. His vitals demonstrate a normal tem- (C) Refer the patient to the family’s perature, a low pulse and blood pressure, and pediatrician. decreased respirations. He appears pale with Questions: 30–37 235

pupils that are constricted and minimally Questions 36 and 37 responsive. Administration of which of the fol- lowing would be most likely to improve his You are asked to see a 37-year-old woman with end- condition? stage ovarian cancer because she has told her oncol- ogist that she wants to die. When you approach the (A) antabuse patient’s bed, you see a cachectic but smiling woman (B) benzodiazepines surrounded by her husband and two young daugh- (C) flumazenil ters. She has a history of depression but is not under (D) naloxone any treatment for it now. “I am ready to go, Doctor,” she says to you. Her oncologist has told you that he (E) thiamine would like to try a new chemotherapy for which the patient is a good candidate but the patient has 35. A 77-year-old woman without prior psychi- refused. atric history is brought into her family physi- cian’s office with her husband. He is concerned 36. Which of the following statements is most that she is “depressed” and needs treatment. accurate regarding suicide statistics? He describes her withdrawing emotionally over the past year or two, with a gradual (A) Suicide is the second leading cause of decline in her ability to care for herself. They death among individuals aged 15–24. have been unable to take part in their usual (B) Suicide is common in children under the social activities, and the patient “just wanders age of 13. around the house during the day.” He has also (C) Women are more likely to commit sui- noticed that she is forgetful, often misplacing cide than men. items, and mixing up names of acquaintances. (D) Men are more likely to attempt suicide When asked, the patient denies any difficul- than women. ties, stating she feels “fine,” and that her hus- band worries too much. She adamantly refutes (E) Risk of suicide decreases with increas- problems with her memory, instead blaming ing age. him for “moving things around in our house.” She has no significant medical problems. Upon 37. According to the American Medical Association’s MSE, she is pleasant and cooperative with (AMA) Code of Medical Ethics, physician- questions, although defensive at times. Her assisted suicide is which of the following? affect is neutral but full. There is no suicidal or (A) justifiable if a patient’s suffering homicidal ideation, and she denies any psy- becomes intolerable despite adequate chotic symptoms. Her Mini-Mental State pain control Examination is 20/30. Her physical is essen- (B) unethical tially unremarkable. Which of the following (C) defined as euthanasia is the most appropriate primary treatment (D) a matter on which the AMA has no offi- for this patient? cial position (A) citalopram (E) not an ethical issue and must be left to (B) galantamine state legislatures (C) ginkgo biloba (D) memantine (E) ziprasidone 236 9: Practice Test 2

38. A 16-year-old girl is brought to your office by (D) a cognitive deficit her parents to get a pregnancy test. She con- (E) impaired judgment sents for the evaluation but requests that you keep the results confidential. Beta-human chori- Questions 41 and 42 onic gonadotropin is positive. The patient’s par- ents demand to know the results. Which of the 41. A 35-year-old woman presents to your office following should you do? with a referral from a psychologist for psychiatric (A) Notify the parents of the results because treatment. She has been suffering “the blues” for they pay her medical bills. several months including crying spells and insomnia. She is convinced that her fiancé is (B) Encourage the patient to discuss the about to break up with her because she has been results with her parents. eating more than usual. Sometimes, however, (C) Report the results to Child Protective she feels “just fine.” The treatment of choice for Services. this patient is which of the following? (D) Invoke testimonial privilege. (A) amitryptiline (Elavil) (E) Disclose the results in a meeting with the patient and her parents. (B) phenelzine (Nardil) (C) fluoxetine (Prozac) Questions 39 and 40 (D) divalproex sodium (Depakote) (E) fluoxetine and ziprasidone (Geodon) After you ask her how many siblings she has, a patient launches into a long discussion first on her 42. The patient responds well to treatment and sister Sally, then one of Sally’s old boyfriends, Mark, after several months returns to your office for who is now an accountant, she says, which in turn a follow-up visit. She tells you she has just been leads her to a monologue on accountants and married and would like to become pregnant. bankers, whom she says are all “in cahoots with the She and her husband have been having unpro- IRS and CIA—you can’t trust ’em!” You note that tected sex for a month. You proceed to do she never comes back to answering the question fully which of the following? until you prompt her to. She further tells you that she believes “the government has experimentally planted (A) Tell her to stop her medication alien eggs in my ovaries, you know. I know they’ve immediately. told you. They think I don’t know.” (B) Ask how important pregnancy is to her. (C) Refer her for electroconvulsive therapy 39. Which of the following is an example of the (ECT). above patient’s thought processes when asked (D) Discuss the risks and benefits of contin- about her siblings? uing her treatment during pregnancy. (A) trichotillomania (E) Add another antidepressant to prevent (B) flight of ideas postpartum depression. (C) tangentiality (D) circumstantiality 43. A 45-year-old man is brought to the psychi- atric emergency room by police after being (E) isolation of affect found screaming and cutting down a tree in the town square. He tells you that the of 40. Delusions as in the above patient’s case are an his wife, who died 3 months ago, is living in the example of tree. The man has no previous psychiatric (A) a perceptual disturbance history, but reports being “very depressed” (B) a disturbance in thought content since the death of his wife and having con- (C) disorganized thought processes sidered suicide. He is disheveled and appears to have ignored his personal hygiene for some Questions: 38–47 237

time. His vital signs are normal. Which of the you after his appointment and tells you that following do you decide to do? she is concerned about news reports saying that SSRIs increase the risk of suicide in children. She (A) Prescribe an antipsychotic. asks you to consider stopping the medication. (B) Prescribe an antidepressant. Which of the following is the best course of (C) Prescribe an antipsychotic and an anti- action? depressant. (A) Stop the fluoxetine immediately as the (D) Obtain a urine toxicology screen. patient is a minor and it is his mother’s (E) Ask him the CAGE questions to rule wish. out alcoholism. (B) Switch to paroxetine (Paxil) because it does not increase suicide risk. Questions 44 and 45 (C) Ask the mother, “What’s really bother- 44. You are seeing a 56-year-old man with a history ing you about your son being on antide- of alcoholism. He complains about not being pressants?” able to sleep and requests a sleeping aid. He (D) Switch to amitryptiline (Elavil). has no other signs of depression and reports (E) Discuss the risks and benefits of fluoxe- that “everything is going well.” You decide to tine and other SSRls in the pediatric give him which of the following? population. (A) trazodone (Desyrel) (B) alprazolam (Xanax) 47. A month later, the boy and his mother return to your office. She informs you that after her son’s (C) diazepam (Valium) last appointment, she has been giving him (D) zolpidem (Ambien) St. John’s wort because she believes he needs (E) zalpelon (Sonata) a “natural remedy.” Which of the following 45. The side effect of this medication that you warn should you do? him about is which of the following? (A) Tell the mother to stop the St. John’s (A) impotence wort immediately. (B) retrograde ejaculation (B) Tell the mother there is no evidence that St. John’s wort is an effective treatment (C) priapism for depression. (D) incontinence (C) Tell the mother to watch out for side (E) anorgasmia effects including photosensitivity and dizziness. Questions 46 and 47 (D) Refer the case to social services for child 46. You are seeing a 9-year-old boy with a history abuse. of depression and suicidal ideation. He has been (E) Increase the dose of his antidepressant taking fluoxetine (Prozac) for 6 months and has as St. John’s wort will lower blood con- had a good response. His mother asks to see centrations of the antidepressant. 238 9: Practice Test 2

Question 48 afternoon that she “wanted to end it all.” She reports thoughts of overdosing on her pills, which she has A 60-year-old patient has suffered from bipolar illness stockpiled. This woman has no prior history of sui- for decades, developing along the way severe reac- cide attempts but has had thoughts of suicide on tions to each of the mood stabilizers you have tried multiple prior occasions over the course of her near- him on. He is currently in the midst of a severe manic lifelong history of depression. She lost her husband episode. His family is “at wit’s end,” according to a to a sudden heart attack 1 year ago, but her daugh- daughter. You suggest ECT to which the patient ter says that she still has many very supportive agrees. His daughter, however, says she has heard of friends whom she spends significant amounts of time “depressed and psychotic people who had it done with. She still carries on her avid hobby of gardening, to them, with little or no benefit. And it seems so although “at a slower pace” because of her worsen- barbaric.” ing arthritis. She expresses worries over a recent “big drop” in the value of retirement stocks she has. 48. You respond by saying your advice is to still consider ECT. Which of the following describes 50. Which of the following is the most significant the factor you base this on? factor mitigating her risk for suicide? (A) Bipolar illness, especially in a manic (A) her social supports episode, has an extremely high rate of (B) her gardening hobby response to ECT, perhaps higher than either of the other two disorders the (C) her race daughter mentioned. (D) her slower movements secondary to (B) ECT is proven to be safer in bipolar ill- arthritis ness than in any other of the disorders (E) the fact that she has not attempted sui- for which ECT is used. cide before, despite her long history of (C) ECT is proven to have more lasting depression results in bipolar disorder than in the other two. 51. Important medical causes of depression to rule out in this and any depressed patient include (D) Bipolar illness is shown to respond best which of the following? in late life to ECT.

(E) ECT is shown to have more lasting (A) vitamin B12 and tryptophan deficiency effects than medications. (B) thyroid illness and malignancy (C) adrenal illness and basement membrane 49. Treating bipolar illness is believed to do which disorders of the following? (D) vascular disease and phospholipid (A) have no change on the likelihood of fur- metabolism disorders ther episodes (E) infectious illness and lipid disorders (B) have no effect on future episodes (C) decrease the likelihood of further Questions 52 and 53 episodes as well as the severity of them A 20-year-old college sophomore is brought to the (D) increase the patient’s response to med- emergency department after being found passed out ications or ECT in the future in her bathroom. She had been vomiting, which she (E) decrease the patient’s response to med- admits now was self-induced. History is significant ications or ECT in the future for a breakup 4 months ago with her boyfriend of 2 years. Her weight is in the 82nd percentile for her Questions 50 and 51 height. She is amenorrheic. There is no history of binge eating. A 60-year-old widow is brought to your clinic by her daughter, who reports that her mother said to her this Questions: 48–57 239

52. Which of the following do you most expect her the hospital. He continues the medications you examination to show? prescribed and on a follow-up visit you notice he moves slowly and has a festinating gait. (A) dental decay The receptor most likely to account for the (B) pitting fingernails above side effect is which of the following? (C) pectus excavatum (A) nigrostriatal D receptors (D) palpable spleen 2 (B) mesolimbic D receptors (E) café au lait spots 2 (C) basal ganglia acetylcholine receptors

53. Additional social history would be most likely (D) D4 receptors

to reveal that she is which of the following? (E) 5-hydroxytryptamine-2 (5-HT2) (A) a gymnast 56. A 16-year-old boy began having repetitive eye (B) a victim of rape blinking at the age of 6. By age 12, he began (C) an identical twin blurting stereotyped phrases such as “Balls!” (D) a genius and “Shitty!” as well as making kissing noises. (E) carefree Concomitant with this, he had increasing diffi- culty with paying attention in school and occa- 54. A 14-month-old girl presents with her parents sionally displayed wild, disinhibited behavior. who report that she has become less responsive Which of the following is the treatment of choice to them and less interactive with her 6-year-old for the primary condition evident in this case? brother. In addition, her gait has become clumsy, (A) pimozide (Orap) and most recently, she has been noted to habit- (B) methylphenidate (Ritalin) ually rub her knuckles together. The parents also bring the chart from their daughter’s pediatri- (C) fluoxetine (Prozac) cian with a note indicating that although her (D) topiramate (Topamax) head circumference was normally sized at birth, (E) clonidine (Catapres) it has now fallen into the 10th percentile for her age. The girl most closely fulfills criteria for 57. A 12-year-old boy has recently been arrested for which of the following? setting fire to a neighbor’s barn. He has been in trouble before for playing with fireworks in the (A) autistic disorder neighborhood, and when he was 10 years old (B) childhood disintegrative disorder he was suspended for lighting matches at his (C) Rett syndrome school. Which is the most frequently diagnosed (D) Asperger syndrome DSM-IV-TR diagnosis associated with fire set- (E) childhood-onset schizophrenia ting in children? (A) a pervasive developmental disorder 55. A 24-year-old man with a history of schizo- (B) mental retardation phrenia presents to the emergency department. He had previously taken haloperidol (Haldol), (C) conduct disorder but now refuses to take it. Because the patient (D) a tic disorder did not take the haloperidol, you decide to try (E) destructive personality disorder of fluphenazine (Prolixin), which seems to miti- childhood gate his symptoms. One week later, he leaves 240 9: Practice Test 2

Questions 58 and 59 61. Given the history of prenatal alcohol exposure in this patient, which of the following might A 9-year-old boy with a history of normal develop- you expect? mental milestones has been wetting his bed for the last 6 months. His parents report that the bed-wetting (A) microcephaly initially occurred sporadically, but for the last 2 months (B) cleft palate it has been happening about every other night. (C) chromosome 13 translocation (D) congenital blindness 58. Which of the following is the most accurate (E) prominent jaw description of this boy’s problem? (A) primary diurnal enuresis 62. A 5-year-old girl has come under the care of (B) primary nocturnal enuresis state Child Protection Services. Her mother was (C) secondary diurnal enuresis known to be using crack cocaine and IV heroin, and supporting her habit with prostitution. The (D) secondary nocturnal enuresis girl’s temporary foster parents reported to state (E) primary diurnal encopresis workers that she was indiscriminately social with adults and unusually clingy. In her social 59. Once medical causes have been ruled out, interactions with other 5-year-old children at which of the following is the pharmacologic her preschool, she was described as being inap- treatment of choice for this condition? propriately aggressive with them. This case (A) clonidine most closely illustrates a diagnosis of which of (B) haloperidol (Haldol) the following? (C) carbamazepine (Tegretol) (A) mixed receptive-expressive language (D) diphenhydramine (Benadryl) disorder (E) imipramine (Tofranil) (B) reactive attachment disorder of infancy and early childhood Questions 60 and 61 (C) childhood disintegrative disorder An 11-year-old boy was adopted at age 6 months. His (D) attention deficit hyperactivity disorder biological mother suffered from severe alcohol (ADHD) dependence. He has been in a special education (E) conduct disorder school due to his global learning difficulties and his intelligence quotient (IQ) has recently been meas- Questions 63 and 64 ured at 60. A 12-year-old girl has recently come under psychi- atric care for her depression and treatment of alcohol 60. According to his IQ measurement, the patient abuse. During the course of her evaluation, she would be diagnosed as having which of the revealed that she had a 3-year-long sexual relation- following? ship with her stepfather beginning when she was (A) borderline intellectual functioning 8 years old. (B) mild mental retardation (C) moderate mental retardation (D) severe mental retardation (E) mental retardation, severity unspecified Questions: 58–67 241

63. Which of the following generalizations related appropriate single pharmacotherapy strategy to this case is most accurate? for this patient would be which of the following? (A) Sexual activity between daughter and (A) lorazepam (Ativan) father (or stepfather) is the most (B) imipramine common form of incest. (C) fluoxetine (B) The patient is at decreased risk of future (D) divalproex sodium (Depakote) sexual abuse or rape due to her (E) citalopram (Celexa) heightened defense mechanisms around sexual issues. Questions 66 and 67 (C) The patient has increased risk of developing borderline personality A 36-year-old woman presents with depressed mood, disorder. a sense of hopelessness, and thoughts of suicide for (D) The patient has increased risk of the last month. She also reports increased appetite. developing schizophrenia after She also has been sleeping up to 16 hours a day. On adolescence. MSE, her mood is sad most of the time, but she (E) The patient’s history of alcohol abuse laughs in response to humorous statements. The makes her a less reliable historian and patient did say that last week her mood brightened her story about the sexual abuse less a bit when she won $150 from playing the lottery. likely to be true. 66. Additional history that would suggest a diag- 64. The patient’s stepfather has a decreased likeli- nosis of an atypical major depression versus a hood, compared to the general population, of melancholic depression would be which of the having which of the following? following? (A) other paraphilias (A) severe concentration difficulty and cognitive impairment (B) alcohol dependence (B) a normal dexamethasone suppression (C) a history of violence test (D) a sexually gratifying relationship with (C) rejection hypersensitivity spouse (D) the presence of psychotic symptoms (E) a personal history of childhood sexual abuse (E) a history of manic symptoms after anti- depressant treatment 65. A 27-year-old law student presents to you as a referral for treatment of depression diagnosed 67. Which of the following is a psychiatric drug by his primary care physician. On MSE, you class specifically associated with good treat- find that the patient’s speech is pressured and ment response for atypical major depression that he quickly jumps from one topic to another. as in this case? He says that his mood has improved greatly (A) anticonvulsants since he last saw his primary care doctor: “In (B) monoamine oxidase inhibitors (MAOIs) fact, I’m on top of the world.” The patient (C) SSRIs relates that now he does not get much sleep. He (D) tricyclic antidepressants (TCAs) reports 3–4 hours a night for the last week, “not that I need that much sleep anyway.” The most (E) typical neuroleptics 242 9: Practice Test 2

Questions 68 and 69 relationships. He typically idealizes his partner, then devalues her, before ending the relation- You have been treating a 34-year-old man for depres- ship suddenly. At times, he loses “a sense of sion with an SSRI for the last 2 months. He had been self” in relationships, not knowing who he is or on fluoxetine 20 mg/day for the first month, and what he wants. During this time he relies on then you raised the dose to 40 mg by mouth (PO) qd others to advise him on what he should do. for the last month. His depressive symptoms have However, if he takes this advice and his plans remained refractory despite the increase in dosage. fail, he uses this to devalue his advising friend. You have decided to change the medication to The patient makes it very clear that he sought phenelzine (Nardil), an MAOI, in hopes of eliciting a your treatment as a psychotherapist “because better antidepressant response. I’ll only accept the best in the city.” The patient also reports to you that he has some unique 68. In making the transition from the SSRI to the abilities of perception that allow him to detect MAOI, which of the following is the best what or who will provide the best possible out- strategy? come for him. For example, he picked you as a (A) Begin tapering down the dose of the therapist not only because of your reputation, SSRI and start tapering up on the dose but also because of your office phone number, of the MAOI simultaneously. which has digits in ascending value order. (B) Stop the SSRI abruptly and immediately Which of the following is the most appropriate begin tapering up on the MAOI. Axis II DSM-IV-TR diagnosis for this patient? (C) Taper off SSRI and then 5 weeks after (A) borderline personality disorder the last dose begin tapering up on the (B) narcissistic personality disorder MAOI. (C) obsessive personality disorder (D) Stop the SSRI abruptly and begin with (D) personality disorder not otherwise the MAOI at an equipotent dose the specified next day. (E) schizotypal personality disorder (E) Start the MAOI until therapeutic levels have been reached, then taper off the 71. A 43-year-old woman is hospitalized during a SSRI. psychotic episode in which she had the delu- sion that she was being shot with invisible laser 69. With regard to taking the MAOI, which of the beams from satellites in Earth orbit. According following is very important that the patient be to her family, she had stopped taking her regi- aware of? men of haloperidol and lithium 4 weeks prior (A) may experience stiffness and tremor to her admission to the hospital, and she had (B) should take the medications with bland been telling her family about her concerns food regarding the laser beams for the last 2 weeks. Her MSE on presentation to the hospital (C) should avoid foods rich in tryptophan revealed primarily thought disorder and delu- (D) should avoid foods rich in tyramine sional symptoms without a prominent mood (E) may feel overly sedated component. The patient’s past psychiatric his- tory is significant for three prior hospitaliza- 70. A 42-year-old accountant presents for long- tions over the last 20 years. At the time of her term psychotherapy treatment. In the first ses- first admission, she presented with a 3-week sion, you find that he has been divorced three history of manic symptoms and psychosis. times. The poor quality of his romantic and During her second hospitalization, she was sui- other relationships are the focus of his con- cidal and noted to be depressed with psychotic cerns. Over the course of months, you find out symptoms. During her third admission, she was that he is a perfectionist and inflexible within again depressed with prominently disorganized Questions: 68–76 243

behavior. Which of the following is the most 74. A clinical research psychiatrist at university A has appropriate diagnosis for this patient? been running a double-blind placebo-controlled trial of a new antidepressant. The psychiatrist (A) bipolar disorder concludes after a statistical analysis of the data (B) MDD that there is no better effect of the drug com- (C) psychotic disorder not otherwise pared to placebo. The psychiatrist’s colleagues at specified five separate institutions (universities B, C, D, E, (D) schizoaffective disorder and F) have run identical studies and determine (E) schizophrenia that there is a treatment effect of the drug com- pared to placebo. Given this information, one Questions 72 and 73 may conclude that the research trial at university A resulted in which of the following? The medical team on an inpatient service calls you to help them with a 72-year-old female patient. They (A) high variance report that at night she becomes confused, thinking (B) low predictive value that she is in a hotel. She makes demands of the nurs- (C) standard error ing staff that they feel are unreasonable, and she has (D) type I error become disruptive of their other work. She has the (E) type II error most trouble from 10 PM until about 2 AM, when she usually falls asleep. In the morning, she is much Questions 75 and 76 better oriented and cooperative. A review of her med- ications reveals that she is taking cimetidine 400 mg A 56-year-old man with a long history of paranoid PO bid, furosemide 40 mg PO qd, atenolol 50 mg PO schizophrenia has been taking his chlorpromazine qd, digoxin 0.125 mg PO qd, and diphenhydramine regularly for 27 years. About 5 years ago, he devel- 50 mg PO qhs and 25 mg PO qhs (second dose oped writhing movements of his wrists and fingers 3 hours post first dose for insomnia). that disappear when he goes to sleep.

72. Which of the following medications most likely 75. Which of the following extrapyramidal syn- to cause this patient’s nighttime confusion? dromes is most consistent with this case? (A) atenolol (A) akathisia (B) cimetidine (B) dystonia (C) digoxin (C) neuroleptic malignant syndrome (NMS) (D) diphenhydramine (D) parkinsonism (E) furosemide (E) tardive dyskinesia

73. The neurotransmitter system most directly 76. The anatomic structure in the brain most likely implicated in the etiology of this medication- implicated in the etiology of this movement induced confusion is which of the following? disorder is which of the following? (A) acetylcholine (A) basal ganglia (B) dopamine (B) cerebellum (C) gamma-aminobutyric acid (GABA) (C) frontal cortex (D) serotonin (D) midbrain (E) norepinephrine (E) motor cortex 244 9: Practice Test 2

77. You are seeing a 55-year-old Vietnam veteran at 79. You may inform the patient’s family that the a Veteran’s Administration hospital. He has prevalence of psychotic episodes in patients been hospitalized numerous times previously with complex partial epilepsy is closest to for drug abuse and psychotic symptoms. During which of the following percetages? this admission, he complains of not being able to (A) 1% sleep for more than an hour at a time because he feels he is under attack, and fears that someone (B) 5% is going to jump out of the bushes at his apart- (C) 10% ment complex and “ambush” him. He knows (D) 20% intellectually that this will not happen, but he (E) 50% cannot stop the thoughts. A poor prognostic indicator for a diagnosis of posttraumatic stress 80. An investigation into the history of this patient’s disorder (PTSD) is which of the following? personality traits may reveal classic signs of (A) existence of premorbid personality dis- complex partial epilepsy-associated personality order including increased or decreased sexuality, hypergraphia, heightened experience of emo- (B) exposure to trauma in middle adult- tions, and which of the following? hood (C) rapid onset of symptoms (A) depression (D) short duration of symptoms (B) hyperreligiosity (E) strong social supports (C) urinary incontinence (D) obsessive-compulsive behaviors 78. A 29-year-old man presents to your outpa- (E) sleep disorders tient psychiatric clinic complaining of anxiety and depression. His social history reveals that 81. After being called to the emergency depart- he is a junior faculty member of a local uni- ment to evaluate a young woman who over- versity, working about 90 hours per week. He dosed on barbiturates, you find her in bed with reports stress arising from his marital rela- her eyes closed. She opens her eyes briefly in tionship that is exacerbated by his occupa- response to pain and demonstrates flexion tional demands. You identify a central conflict from pain but makes no sounds. Which of the in this patient of issues of intimacy versus following numbers represents her Glasgow isolation borrowing from the theoretical work Coma Scale score? of which of the following? (A) 7 (A) Carl Jung (B) 6 (B) Karen Horney (C) 5 (C) Erik Erikson (D) 4 (D) Jean Piaget (E) 3 (E) Sigmund Freud 82. A 46-year-old man with a history of schizo- Questions 79 and 80 phrenia had been maintained on thioridazine (Mellaril) 700 mg qd since he was diagnosed at A 46-year-old man with a history of complex partial the age of 20. Over the last 8 months, he has epilepsy is referred to your psychiatric clinic by his been complaining of hearing derogatory voices, neurologist for onset of delusional thinking. and his dose has been increased to 800 mg qd. He has no signs of tardive dyskinesia. What is the most harmful side effect associated with 1000 mg or more of thioridazine? Questions: 77–89 245

(A) constipation 86. If the patient did not want detoxification but (B) dry eyes did want to stop using heroin, which of the fol- (C) urinary retention lowing medications would be most appropriate for maintenance? (D) pigmentary retinopathy (E) nephrogenic diabetes insipidus (A) lorazepam (B) alprazolam (Xanax) 83. A 24-year-old man is admitted to the psychi- (C) methadone atric ward with new-onset command suicidal (D) sertraline (Zoloft) auditory hallucinations. He is given haloperi- (E) lithium dol 10 mg IM. Fifteen hours later, he develops torticollis. What is the best treatment for this Questions 87 and 88 occurrence? (A) acetaminophen A 75-year-old man with Alzheimer disease presents to your clinic complaining of depression. He has a (B) labetalol (Normodyne) past medical history including diabetes mellitus, two (C) benztropine myocardial infarctions, atrial fibrillation, and one (D) a lower-potency neuroleptic coronary artery bypass graft. (E) penicillamine 87. Assuming this patient is suffering from depres- 84. A 28-year-old woman with a history of bipolar sion, which antidepressant class would you disorder is admitted to the medical service most want to avoid? because of weakness, mental status changes, (A) SSRIs and a serum sodium of 154 mmol/L. Over the last 2 weeks, she has had polyuria, excessive (B) TCAs thirst, and polydipsia. What drug is most likely (C) butyrophenones associated with this medical condition? (D) dibenzodiazepines (A) lithium carbonate (E) benzodiazepines (B) haloperidol 88. Blockade of which neurotransmitter system (C) diazepam (Valium) would most likely result in a cognitive decline (D) valproic acid in this patient? (E) buspirone (BuSpar) (A) histamine Questions 85 and 86 (B) serotonin (C) dopamine A 47-year-old man who is addicted to heroin has (D) acetylcholine been admitted to the inpatient psychiatric ward for (E) norepinephrine detoxification. 89. A 35-year-old woman presents to the emer- 85. What is the most effective treatment of auto- gency department after taking an overdose of nomic dysregulation during detoxification? lithium. Which of the following signs is most (A) haloperidol associated with lithium toxicity? (B) thioridazine (A) acute dystonia (C) amantadine (Symmetrel) (B) abdominal pain (D) clonidine (C) paresthesias (E) pimozide (D) paranoid delusions (E) leg pain 246 9: Practice Test 2

Questions 90 and 91 (A) vitamin B12 (B) IV haloperidol A 67-year-old woman presents to your office with a long history of depression. She has tried a number of (C) supportive care different antidepressant medications in the past but (D) vitamin E has not had a remission with any of them. Currently, (E) cheese she is on lithium, venlafaxine (Effexor), nortriptyline (Pamelor), lorazepam, risperidone, and benztropine. 94. A 45-year-old woman is admitted to the psychi- She is starting to believe that her next-door neighbors atric inpatient ward. She is severely depressed, are out to harm her. You want to offer her ECT. believes that her neighbors are conspiring to murder her, and occasionally can hear a man’s 90. Which of her medications would be the best to voice that says she should die. After talking with continue given the ECT? the neighbors, you find out that they are the ones who brought the patient to the hospital. They (A) lithium have been good friends for 20 years and have (B) venlafaxine noticed the patient has not been leaving the (C) haloperidol house much, has taken to looking out of her win- (D) lorazepam dows with suspicious glances, and has not (E) clozapine cleaned her house in the last 2 months. Which of the following medication combinations would be 91. Which of the following is a relative contraindi- proper treatment for this patient? cation to ECT? (A) sertraline and lithium (A) a space-occupying lesion of the brain (B) paroxetine (Paxil) and thyroid (B) a history of seizures augmentation (C) diabetes mellitus (C) fluoxetine and risperidone (D) a small-bowel obstruction (D) sertraline and lorazepam (E) catatonia (E) lorazepam and clozapine

Questions 92 and 93 DIRECTIONS (Questions 95 through 100): Each set of items in this section consists of a list of let- A 26-year-old man presents to the psychiatric emer- tered headings followed by several numbered gency department with paranoia, visual hallucina- words or phrases. For each numbered word or tions, feelings of unreality, depersonalization, and phrase, select the ONE lettered option that is most extreme agitation. A urine toxicology screen is posi- closely associated with it. Each lettered option tive for phencyclidine (PCP). may be selected once, more than once, or not at all.

92. What is the best treatment for extreme agitation Questions 95 through 98 in this patient? Match the clinical vignette with the most likely (A) a phenothiazine antipsychotic substance-induced disorder. (B) a butyrophenone antipsychotic (C) trihexphenidyl (Artane) (A) cannabis intoxication (D) trazodone (Desyrel) (B) cocaine intoxication (E) fluoxetine (C) cocaine withdrawal (D) alcohol intoxication 93. If this patient were not extremely agitated, (E) alcohol withdrawal what would be the best treatment? (F) heroin intoxication Questions: 90–100 247

(G) heroin withdrawal Questions 99 and 100 (H) 3,4-methylenedioxymethamphetamine Choose the medication most likely associated with (MDMA) (Ecstasy) intoxication side effects in the following scenarios. (I) PCP intoxication (J) methamphetamine intoxication (A) benztropine (K) inhalant intoxication (B) clozapine (L) nicotine intoxication (C) desipramine (Norpramin) (M) nitrous oxide intoxication (D) fluoxetine (N) psilocybin withdrawal (E) haloperidol (F) lorazepam 95. A 23-year-old man is brought to the emergency (G) lithium department by the police. He shows agitation, (H) nefazodone (Serzone) vertical nystagmus, and analgesia. (I) olanzapine (J) phenelzine 96. A 62-year-old homeless man admitted to a psy- (K) valproic acid chiatric unit begins having visual hallucina- tions, tremor, and seizures. (L) venlafaxine

97. A 43 year-old woman presents to the psychiatric 99. A 34-year-old man with schizophrenia pres- emergency department complaining of depres- ents with fever and chills. He is found to be µ sion and suicidality. She is observed to be bacteremic and has a WBC count of 900/ L. fatigued, irritable, and with a dysphoric effect. 100. A 41-year-old woman on medications for her 98. A 30-year-old musician presents in the outpa- affective disorder presents with fatigue, tient clinic. He complains of fever, diarrhea, and weight gain, cold intolerance, constipation, and myoclonus of extremities for the last 24 hours. decreased concentration. Answers and Explanations

1. (D) Nefazodone is an antidepressant, which has any corresponding sensory stimuli—her the advantage of not causing sexual dysfunc- experience is best characterized as hallucinosis. tion like most antidepressants. Unfortunately, it Hallucinosis is a hallucination occurring in a has a black-box warning for hepatitis and liver patient who is completely alert and oriented, failure, which is not predictable, so it requires and most commonly occurs in alcohol with- regular monitoring of liver function tests and drawal or chronic alcohol abuse. An illusion is therefore is not used as frequently. a misinterpretation of something that is already there (e.g., a floor lamp being seen as a stand- 2. (G) Venlafaxine is an antidepressant that inhibits ing person). A flashback is a sensory reexperi- the reuptake of serotonin as well as, in higher ence of a past, emotionally intense event (e.g., doses, norepinephrine. It therefore can cause a war veteran’s having a flashback of the sights increased blood pressure in doses above 300 mg and sounds of a battle). Formication is the tactile daily. experience of bugs crawling over, or within, one’s body. 3. (A) Although all of the listed antidepressants likely increase serotonin as a therapeutic mech- 7. (E) This patient has described a panic attack. anism, they all affect other neurotransmitters to Panic attacks are characterized by a sudden a certain extent. Citalopram tends to more onset of intense fear, dread, or anxiety, and specifically affect the serotonin system. accompanied by a variety of physical symp- toms. Although virtually any organ system can 4. (B) Fluoxetine stands out as having a signifi- seem to be the source of distress in panic dis- cantly longer half-life than the other listed order, common symptoms include diaphoresis, antidepressants. This translates into fewer palpitations, shortness of breath, tremulousness, “withdrawal” symptoms if suddenly stopped. flushing, chest pain, and dizziness. Regardless of It also may be an advantage in cases of occa- the clinical setting or psychiatric history, patients sional poor compliance. The long half-life does presenting with symptoms suggestive of panic not appear to affect the time to significant clin- disorder must be thoroughly worked up to rule ical improvement, which remains several weeks out physiologic pathology as the source of the for antidepressants. symptoms. Untreated, panic disorder tends to run a chronic course and can cause severe dis- 5. (D) Quetiapine, olanzapine, clozapine, and ability. risperidone have all been shown to cause weight gain as a side effect. The only atypical 8. (D) Neologistic speech, a positive symptom of antipsychotic to date that has not been shown schizophrenia, consists of words made up by to cause significant weight gain is ziprasidone. the patient, having meaning only to him. Word salad is an illogical, incoherent collection of 6. (E) Although this woman is having words or phrases. Clang associations are words hallucinations—perceptions created without or phrases strung together because of the

248 Answers: 1–14 249

sounds they make, not because of the meaning schizophrenia, generalized anxiety disorder, and they convey. Dysarthria is a problem with pro- drug-related illnesses, such as alcohol abuse. nunciation and articulation of words; it does Axis II represents long-standing pervasive prob- not imply an inherent inability to understand lems in mental life such as personality disorders and use language; for example, a dysarthric and mental retardation. Axis III is for all medical person could conceivably write a coherent and conditions, which may or may not relate directly meaningful sentence. to the psychiatric diagnosis. Axis IV is for psy- chosocial stressors. Axis V, the GAF, is a numerical 9. (E) Dysthymia is a low-grade, chronic, inter- value, from 1 (lowest) to 100 (highest), character- mittent depression. The symptoms must be izing the overall level of impairment resulting from present in adults for at least 2 years. Like depres- the conditions described on Axis I through Axis IV. sion, other than a subjective feeling of depressed The multiaxis representation of this patient would mood, or the blues, the key features of dys- appear as: thymia are alterations in appetite and sleep, feel- • Axis I: Adjustment disorder, alcohol abuse, ings of hopelessness, difficulty concentrating, in remission and low energy or fatigue. Although dysthymia and MDD share a number of features, it is gen- • Axis II: Deferred erally the severity and duration of symptoms • Axis III: Diabetes that distinguish the two disorders. In the pres- • Axis IV: Psychosocial stressor (recent ence of alcohol dependence, it would be difficult divorce) to convince any clinician that the mood distur- • Axis V: GAF: 70 bance is not due to the effects of the drug, and would exclude a diagnosis of dysthymia. 12. (C) Both copropraxia and echopraxia are terms used in relation to movements and are specifi- 10. (C) Patients with Capgras syndrome suffer cally used in reference to obscene gestures from the delusion that someone familiar is an and repeating another’s movements, respec- identical-appearing replacement. Koro is a trau- tively. Bruxism is the grinding of the teeth. matic fear that the penis is shrinking into the Blepharospasm is the spasm of the periorbital body cavity. Amok is a violent fit followed by muscles causing sustained or exaggerated amnesia. Pseudocyesis is the physiologic signs blinking. Torticollis is a spasm of the neck mus- and symptoms of pregnancy developing in cles such as the sternocleidomastoid on one or the absence of pregnancy. Couvade syndrome both sides. This usually brings the neck to one occurs when the husband of a pregnant woman side or another but can also pull forward or goes into a sort of labor. Koro, amok, and cou- backward. vade syndrome are culture bound, that is, they describe unusual, recurrent patterns of behav- 13. (B) Coprolalia and echolalia are terms used in ior that resist placement in any Western diag- relation to language or speech and are specifi- nostic category (the DSM). cally used in reference to obscene speech and repeating another’s words, respectively. Palilalia 11. (B) The multiaxis system of diagnosis in criteria is also a repetition of words but is the repetition uses the signs and symptoms of psychiatric ill- of one’s own words as the person appears to ness to assign diagnosis and to provide a snap- get stuck on the same word or phrase. Parapraxis shot picture of a particular patient. It was is a slip of the tongue, and dysarthria refers to a originally intended to lend objectivity to psychi- problem of word articulation, usually secondary atric diagnoses, principally in the service of to cerebellar or motor control abnormalities. research; today, it is widely used clinically. Axis I includes psychiatric illnesses that appear in a 14. (D) All of the terms listed are impulse control defined period or periods and result in some disorders of various forms. The content of each diminished function as compared to previous is listed below: functioning. Common Axis I diagnoses are 250 9: Practice Test 2

• Pyromania: Fire setting that is deliberate, variant. Subconjunctival hemorrhage is the find- more than once ing of blood in the areas of the eye surrounding • Trichotillomania: Hair pulling with apprecia- the iris. ble hair loss • Kleptomania: Stealing objects not out of 18. (C) Hemiballismus is an uncontrolled swinging need or monetary value of an extremity. It is usually sudden, and once initiated it cannot be controlled. Choreiform • Pathologic gambling: Maladaptive gambling movements are involuntary, irregular, and jerky behavior but lack the ballistic-like nature of hemiballis- • Intermittent explosive disorder: Aggressive mus. Athetoid movements, or athetosis, are slow, impulses resulting in assault or the snake-like movements of the fingers and hands. destruction of property Myoclonus is a sudden muscle spasm, and myotonia is prolonged muscle contraction. 15. (B) Ego-syntonic and ego-dystonic refer to whether thoughts or behaviors are distressing 19. (A) Magical thinking and ideas of reference are or not distressing to the patient. In this case, the two types of delusions that can be found in impulses are distressing and would therefore psychotic disorders. Magical thinking is the be classified as ego-dystonic. Mood congruency belief that one’s thoughts can control outside usually refers to delusions or hallucinations in events. Ideas of reference is the belief that other mood disorders and whether they are themes people or the media are referring to or talking consistent with the mood. Delusions are fixed, about the person experiencing this delusion. false beliefs that are not shared by others. Reaction formation, displacement, and projec- tion are all types of defense mechanisms. 16. (B) Ganser syndrome is a dissociative disorder Reaction formation is the formation of thoughts described by Ganser in three prisoners. It is that are opposite to anxiety-provoking feelings. characterized by approximate answers and Displacement is the transferring of a feeling talking past the point and is associated with toward an object that is less threatening, such amnesia, disorientation, fugue, and conversion as the family pet or one’s spouse or children. symptoms. Amok and piblokto are dissociative Projection is the false attribution of one’s own trance disorders that are described as distur- unacceptable feelings to another. bances in consciousness and are indigenous to particular cultures. Malingering is difficult to 20. (A) Thought broadcasting is the delusion that distinguish in these situations, but in this case one’s thoughts can be heard by others. It is a secondary gain is not identifiable. Logorrhea is often seen in schizophrenia. Thought insertion excessive speech. and thought control also can be seen in schizo- phrenia and are delusions that others can insert 17. (A) Kayser-Fleischer rings are golden brown or thoughts into a patient’s mind or that another gray-green rings of pigment at the corneal can control the patient’s thoughts or behaviors, limbus that can be present in a disorder of respectively. Transference, in strict terms, is the copper metabolism such as Wilson disease. reexperiencing of past experiences with the Arcus senilis is a light gray ring beginning supe- analyst in the setting of psychoanalytic psy- riorly and extending to the limbus. It can chotherapy. In general, this term has come to resemble a corneal arcus, which is associated mean the transferring of emotions and feelings with hypercholesterolemia, but in an elderly that one has from one’s past to the physician or patient it is often a normal finding. Also asso- care provider. Echolalia is the repetition of ciated with hypercholesterolemia is xanthel- another’s words or phrases. asma, raised yellow areas around the eyelids. Brushfield spots are lighter-colored areas in the 21. (A) Erotomania is the term used to describe a outer third of the iris that can be associated delusion in which individuals falsely believe with Down syndrome, but can also be a normal that another person is in love with them. Answers: 15–27 251

Nymphomania is insatiable sexual desire in a (2) knowledge of one’s assets; (3) knowledge woman, and satyriasis is its male counterpart. of natural heirs; (4) absence of acute psychosis Thought broadcasting is the delusion that one’s (i.e., delusions), which might compromise thoughts can be heard by others. Alexia is an rational decision making; and (5) freedom from inability to read. undue influence or coercion. The validity of the will may be undermined by demonstrating that 22. (D) This patient demonstrates delusions of the individual failed to meet any of the above grandeur, that is, an elevated sense of one’s criteria. The presence of a conservator of person position or wealth. A delusion of infidelity is an and history of mental illness are important only unfounded belief that one’s spouse or signifi- insofar as they bear upon any of the noted fac- cant other is being unfaithful. Micropsia and tors. A history of mental illness is not sufficient macropsia are misperceptions of visual stimuli. to invalidate one’s testamentary capacity. Actus Objects appear smaller (micropsia) or larger reus refers to the voluntary act of committing a (macropsia) than they are in reality. Palinopsia crime and is an element used in determining is the persistence of the visual image after the criminal responsibility. The presence of a judge stimulus has been removed. at the signing of a will is not required.

23. (B) A conversion disorder is often a dramatic set 26. (B) Informed consent is crucial in maintaining of physical symptoms, usually neurologic, that patient autonomy in decision making. Elements result from unconscious conflicts. Sublimation is involved in obtaining informed consent include a defense mechanism employed to deal with providing the patient with information regard- unacceptable feelings or desires that one may ing the risks and benefits of treatment, alterna- possess and to channel these motivations into tive treatment options (including no treatment), socially acceptable behaviors. Like sublimation, and establishing the competency of the patient rationalization is a defense against undesired to make a voluntary decision. Exceptions to motivations, but in this case these motivations informed consent include (1) emergency situa- are concealed by elaborate and reassuring tions in which obtaining informed consent explanations that avoid the actual underlying would jeopardize patient safety, (2) patient motives. Dissociation is a defense mechanism waiver of informed consent, and (3) situations that deals with stressors via a breakdown of in which disclosing information would directly the usual integration of memory, behavior, and harm the patient (also known as therapeutic priv- perception. ilege; it is seldom used). The other choices are not valid exceptions to informed consent. 24. (D) La belle indifférence is the indifference shown toward a deficit or loss of function classically 27. (C) Confidentiality is an integral part of the seen in a conversion disorder. Déjà entendu is physician-patient relationship and is especially the feeling that one is hearing what one has important in establishing the trust of a psy- heard before. It is usually associated with anx- chotherapeutic relationship. The physician- iety states or fatigue. Déjà vu is a similar expe- patient privilege should be breached only in rience but refers to the sensation that something specific instances such as the patient’s waiving has been seen before. Jamais vu is the opposite of of the privilege (i.e., patient initiates litigation, déjà vu in that it refers to something that should consent to release information is obtained, be familiar but seems quite unfamiliar. Folie à there is a duty to protect, emergency situation, deux is a shared delusion aroused in one person or in cases in which the court orders release). In by the influence of another. this case, the information should not be released unless the patient has waived his physician- 25. (A) Testamentary capacity (the level of compe- patient confidentiality privilege. Lying to or tence required to make a valid will) is based on misleading the court is unethical and may sub- the presence of all of the following: (1) an ject one to prosecution. The remaining choices understanding of the nature of the will; fail to maintain confidentiality. 252 9: Practice Test 2

28. (D) When a patient has stopped paying his or production of neurologic or other medical her bill, the physician should directly address symptoms resulting from intrapsychic needs or the issue with the patient. Failure to pay one’s conflicts. In contrast to factitious disorder and bills may reflect underlying issues for which malingering, symptoms associated with con- the patient has entered into psychotherapy or version disorder are not consciously produced. that are related to psychiatric symptoms. Contacting a collection agency before address- 30. (E) Individuals with malingering tend to be ing the issue with the patient is inappropriate. suspicious and uncooperative with diagnostic If a patient continues to refuse payment after evaluations. Furthermore, they do not typically discussing the issue, the patient should be noti- follow up with recommended treatment regi- fied in writing of the outstanding balance and mens and refuse opportunities to return to social a referral to a collection agency might be made. or work activities. Malingerers consciously feign To preserve confidentiality, care should be symptoms for financial incentives or to avoid taken to disclose the minimum information work/social obligations (secondary gain). In needed for collection (i.e., date of service and contrast, conversion disorder is the uncon- charge) without referring to diagnosis or treat- scious or involuntary production of symptoms ment rendered. Asking the insurance company resulting from intrapsychic needs or conflicts. to pay you after a check has been disbursed to Individuals with conversion disorder are more the patient is futile because the insurance com- likely to be friendly, cooperative, interested in pany’s obligation toward the insured has been evaluations that may “find the answer,” and fulfilled. Disclosing the existence of a profes- avidly follow up with prescribed treatments. sional relationship with the patient to his family The other choices are not accurate statements is a breach of confidentiality. Terminating the distinguishing between malingering and con- therapeutic relationship before exploring the version disorder. dynamics behind the patient’s payment delin- quency might harm any future therapeutic 31. (E) Sexual abuse is estimated to occur with an alliance. incidence of 0.25% of children per year. Known acquaintances (e.g., fathers, stepfathers, and 29. (B) Malingering is the intentional production of male relatives) are often the perpetrators. In symptoms for secondary gain (e.g., to avoid this vignette, the patient likely has acquired work, to evade criminal prosecution, or to gain Neisseria gonorrhoeae from a male perpetrator. financial rewards). A diagnosis of malingering Other physical findings may include injuries to should be suspected in medicolegal cases, in the genitalia (i.e., hymen, vagina) or the per- individuals with antisocial personality disor- ineum. Psychiatric manifestations of sexual der, when symptoms are out of proportion to abuse may include anxiety, agitation, aggres- objective findings, or when a patient in distress sive or impulsive behavior, and exhibitionism. does not cooperate with evaluation/treatment. When sexual abuse of a child is suspected, the Factitious disorder can be differentiated from first step is to ensure the safety of the child and malingering based on the absence of external immediately notify the state’s Child Protective incentives. In factitious disorder, symptoms Services. Reporting is mandatory in cases of are intentionally feigned to satisfy an intrapsy- physical and sexual child abuse. The other chic incentive to assume the sick role. choices fail to protect the child and do not sat- Munchausen syndrome is a factitious disorder isfy state-mandated reporting statutes. characterized by repeated acts of feigning ill- nesses to gain medical attention. Intrapsychic 32. (A) This case represents Wernicke encephalopa- gain obtained by the act of getting medical thy, a condition seen in chronic alcohol depend- attention through feigning medical illness in ency. The etiology is acute thiamine deficiency another individual (usually a child or elderly and it presents with the classic triad of confu- person) is sometimes referred to as Munchausen sion, ataxia, and opthalmoplegia. Although it is by proxy. Conversion disorder describes the usually reversible with thiamine replacement, Answers: 28–38 253

it is important to administer the thiamine prior but they are not primary treatments. Ginkgo to giving fluids and glucose. Administration biloba has not been shown to be effective for of glucose and fluids first may provoke a wors- the memory deficits in dementia. ening of symptoms resulting in permanent damage. 36. (A) Suicide is the second leading cause of death among individuals aged 15–24; only accidents 33. (B) The patient is experiencing symptoms con- account for more deaths in this population. Risk sistent with severe alcohol withdrawal, namely factors for completed suicide include (1) age delirium tremens (DT), characterized by delir- >45, (2) male gender, (3) separated/divorced ium, confusion, combativeness, and elevated >married, (4) White race >Black race, and (5) vitals signs. DT carries a significant mortality if Jews/Protestants >Catholics. The lifetime sui- not treated. The treatment of choice is IM or IV cide rate in individuals with a diagnosis of a benzodiazepines. Although barbiturates may major psychiatric disorder (i.e., schizophrenia, also treat withdrawal, they have a narrower bipolar I disorder, MDD) is estimated at therapeutic index, with a higher risk of sedation 10–15%. Suicide in children younger than age 13 and respiratory depression. Neither the antipsy- is rare. Women attempt suicide up to four times chotic or thiamine will treat the underlying alco- more frequently than men; however, men are hol withdrawal, and giving hydralazine may be three times more likely to succeed at suicide dangerous as it may mask the symptoms. than are women.

34. (D) The patient is presenting with evidence of 37. (B) Physician-assisted suicide involves the overdose with opiates, which is characterized by doctor’s facilitating a person’s death by pro- the triad of somnolence, respiratory depression, viding the information or equipment necessary and pinpoint pupils. Opiate intoxication can be to end his or her life. The AMA’s Code of rapidly reversed with IV administration of an Medical Ethics strongly condemns such action opiate antagonist, such as naloxone. Antabuse as “fundamentally incompatible with the blocks acetaldehyde dehydrogenase and is given physician’s role as healer.” Rather, the ethical to alcoholics as an aversive stimulus to avoid obligation of the physician is to adequately alcohol consumption. Benzodiazepines would respond to a patient’s end-of-life issues. The be used in cases of severe alcohol withdrawal, AMA’s Code of Medical Ethics emphasizes that or DT, which would demonstrate elevated participation in physician-assisted suicide would vital signs. Flumazenil is a benzodiazepine have detrimental implications to both society antagonist used in the emergency treatment of and the practice of medicine. Euthanasia occurs overdose with benzodiazepines, which does not when a physician carries out the killing of a cause pupilary constriction. Thiamine would be patient to relieve intolerable and refractory suf- given in cases of Wernicke encephalopathy, fering. Euthanasia is not the mere facilitation of caused most commonly by chronic, heavy alco- the terminal act, but involves the actual admin- hol consumption. It presents with a triad of con- istration of the fatal agent. Like physician-assisted fusion, ataxia, and opthalmoplegia. suicide, it is condemned by the AMA’s Code of Medical Ethics. Recently, a Michigan physician 35. (B) This elderly woman is showing signs of was found guilty of second-degree murder for dementia, likely Alzheimer disease, the most giving a lethal injection at the request of a patient common cause of dementia. The primary treat- suffering from Lou Gehrig disease. ment modality is to begin an anticholinesterase inhibitor, such as galantamine. Memantine, an 38. (B) Physician-patient confidentiality in the N-methyl d-aspartate (NMDA) receptor antag- treatment of minors must be maintained unless onist, is a newer medication that may be added otherwise mandated by law (e.g., cases of abor- to the regimen. Antidepressants and antipsy- tion in some states) or when parental involve- chotic medications may be used for associated ment is necessary in making complex and depression or agitation/psychosis, respectively, life-threatening medical decisions. In this case, 254 9: Practice Test 2

the physician must respect the patient’s confi- discussion of the risks and benefits of treat- dentiality but should remain cognizant of the ment is therefore most appropriate. It does not implications this will have on the family system. have the withdrawal syndrome known to occur Therefore, it is most appropriate to encourage with paroxetine, but still stopping it immedi- the minor to discuss the issue with her parents. ately should be avoided. Asking the patient At this point, there is no indication of child how important pregnancy is to her would be harm or neglect that would mandate reporting inappropriate, since it is a personal choice and, to Child Protective Services. Testimonial privi- according to the information provided in the lege refers to the privilege invoked to protect vignette, it is not something that puts her in physician-patient confidentiality when medical danger of any kind. ECT may be an option but information is subpoenaed without patient con- not before a discussion of the risks and benefits sent. The remaining two choices fail to maintain of her current treatment. Finally, there is no physician-patient confidentiality. indication that she is at particular risk for postpartum depression and there is no evi- 39. (C) Tangentiality is similar to circumstantiality dence in the literature that adding a second in its representing speech/thoughts that are antidepressant in pregnancy will prevent this logically connected but that drift off the point. disorder. Tangentiality differs from circumstantiality in that the speaker never comes back to the point. 43. (D) There are many reasons why this man could be exhibiting psychosis (cutting down a 40. (B) Delusions are disturbances in thought con- tree that his wife’s ghost lives in) and depres- tent and are fixed, false beliefs not held by sion. It’s more than possible that he is suffering others. They are included in none of the other from some kind of severe grief disorder, although choices, which are categories of the MSE. 3 months is too short to make this diagnosis. It is also possible that his wife’s death unmasked a 41. (C) Because of their side effect profile, the treat- psychotic depression, or that alcoholism has led ment of choice in major depression is the SSRIs to some kind of dementia. Regardless of the even- such as fluoxetine (Prozac). Although MAOIs tual diagnosis, it is important to first rule out such as phenelzine are thought to be somewhat other irreversible causes of his behavior such as more effective in atypical depression—for drug abuse. Certainly it is premature to prescribe example, eating more, rather than decreased any medications for him at this time. appetite—they carry the risk of hypertensive crisis when combined with foods containing 44. (A) Although there are a number of sleeping tryptophan. Amitryptiline and other tricylic aids available, many are benzodiazepines. antidepressants include the risk of arrhythmias Because these overlap with alcohol in binding and can be lethal in overdose, unlike the SSRIs. to GABA receptors—hence their use in alcohol And while the patient reports periods during withdrawal—these should be avoided in patients which she is “just fine,” there is nothing to indi- with a history of alcoholism. The same is true cate a cycling pattern that would require a of zolpidem, which has been shown to have mood stabilizer such as divalproex sodium. similar abuse potential to diazepam. Zalpelon is Similarly, being convinced that her fiancé is relatively contraindicated in those with severe about to leave her is not psychotic, but is liver dysfunction, which is a potential issue in a instead a mild mood-congruent delusion that patient with a history of alcoholism. Trazodone, does not require an adjunct antipsychotic such which was developed as an antidepressant, has as ziprasidone. been shown to be an effective sleeping aid and is therefore useful in this population. 42. (D) Fluoxetine is a class C drug, meaning that it is potentially harmful to the developing fetus, 45. (C) Trazodone carries a 1 in 1000 to 1 in 10,000 according to animal studies, although no risk of priapism. The other sexual side effects definitive studies in humans are available. A among the choices are known to occur with Answers: 39–54 255

the SSRIs, but not particularly with trazodone. likely more severe episode. Conversely, treat- Incontinence is not a significant side effect of ment of the decompensations is theorized to be any antidepressant. protective against the above. There is no known effect on the response to ECT or medications in 46. (E) All of the SSRIs have been in the news the future. recently, mostly because of controversy over data on increased suicidal gestures among 50. (E) Lack of suicide attempt history is her most children taking these medications. Paroxetine significant protective factor; a lesser protective has been the focus of many of the reports. factor is social support. Other factors do not However, fluoxetine is the only medication play much of a role. indicated for treatment of childhood depres- sion, which makes switching to paroxetine or 51. (B) There are many potential medical causes of amitryptiline incorrect in this setting. Fluoxetine depression; the only choice here that lists two

does not have the withdrawal syndrome known well-described such causes is B. Vitamin B12 to occur with paroxetine, but still stopping it deficiency and adrenal disease as well as vas- immediately should be avoided. Asking the cular disease have also been shown to also be child’s mother what is “really bothering her” is potential causes. unlikely to be productive, as it is somewhat accusatory and a leading question. 52. (A) This patient is most likely suffering from anorexia. Excessive dental decay is common in 47. (C) Although many people think of “natural patients with anorexia or bulimia who purge remedies” such as St. John’s wort as safe, these by self-induced vomiting. Although the other supplements can have risks of significant side choices may be present in severe starvation, effects. In particular, St. John’s wort side effects excessive dental decay is most common and include increased photosensitivity, stomach appears earliest. upset, rashes, fatigue, restlessness, headache, dry mouth, dizziness, and confusion. These 53. (A) Occupations such as modeling, ballet, and side effects are more likely when St. John’s wort gymnastics, which encourage thinness, have dis- is combined with SSRIs. St. John’s wort has proportionately high rates of anorexia. Patients been shown to have some benefits in mild with certain personality disorders, including depressions and is thought to act like an MAOI. borderline personality disorder, have a higher The precise pharmacokinetics of St. John’s wort likelihood of being victims of abuse or trauma. are unknown, but if anything, doses of SSRIs Education and “type A” personalities are not cor- should be decreased while patients are taking related with eating disorders, nor is being a twin. the supplement because of possible potentia- tion of side effects. As with any antidepressant, 54. (C) Rett syndrome has been described only in it is not a good idea to stop it abruptly. There is girls. Diagnostic criteria require symptom onset no reason to think that child abuse is involved, between ages 5 months and 4 years with appar- as the child’s mother is not putting her son in ently normal prior development. Criteria for danger. autism do not require deceleration of head growth. In childhood disintegrative disorder, 48. (A) Bipolar manic episodes have perhaps the as in Rett syndrome, development is initially highest rate of response to ECT. The safety of normal. However, in childhood disintegrative ECT depends on underlying medical conditions, disorder, development is normal for at least the not on the disorder being treated. Its effects are first 2 years after birth. In Asperger syndrome, not longer lasting than those of medications. there is no delay in language. Childhood schiz- ophrenia cannot be diagnosed in children meet- 49. (C) This question refers to kindling, the hypoth- ing criteria for a pervasive developmental esis that each decompensation in a mental illness disorder who do not have prominent delusions leads to increased risk to have another, and and hallucinations. 256 9: Practice Test 2

55. (A) Nigrostriatal D2 receptors that are blocked anticonvulsant sometimes used as a mood sta- by neuroleptic medications cause an imbalance bilizer. Diphenhydramine is used for insomnia. between dopamine and acetylcholine, which accounts for the parkinsonian symptoms. 60. (B) Borderline intellectual functioning is a term

Mesolimbic D2 receptor blockade is believed no longer used that referred to IQ in the range to account for the antipsychotic effect of neu- of 71–84. In mild mental retardation, IQ ranges roleptics and does not cause extrapyramidal from 50 or 55 to approximately 70. In severe symptoms. Typical neuroleptics block both mental retardation, IQ ranges from 20 or 25 to nigrostriatal and mesolimbic dopamine, whereas 35 or 40. Learning disorder not otherwise spec- atypical neuroleptics preferentially block ified is diagnosed with global learning impair- mesolimbic dopamine, accounting for decreased ments, but the disability in specific areas of extrapyramidal symptom liability. learning is mild and would not result in such low IQ results as in this case. 56. (A) This patient is most likely suffering from Tourette syndrome. Pimozide, a dopamine 61. (A) Microcephaly, short palpebral fissures, flat blocker, has been shown to most clearly sup- midface, and thin upper lip are all associated press tic activity compared to agents affecting with fetal alcohol syndrome (FAS). Cleft palate other neurotransmitter systems. Although is not specifically associated with FAS. There methylphenidate may help this patient’s are no genetic abnormalities associated with apparent ADHD symptoms, it may also exac- FAS. Congenital blindness is not associated erbate his tic disorder. Fluoxetine, topiramate, with FAS, although methyl alcohol consump- and clonidine may be effective for comorbid tion can cause blindness in adults. Prominent conditions found with Tourette disorder but jaw is associated with fragile X syndrome. have not been found to be specifically helpful for this primary condition. 62. (B) Mixed receptive-expressive language dis- order does not imply an abnormality in emo- 57. (C) Fire setting in a repeated pattern in children tional communication such as in displays of often accompanies other behaviors that are aggression toward other children found in reac- defiant of rules and authority, as in conduct tive attachment disorder. Patients with child- disorder. Although fire setting may occur in hood disintegrative disorder show histories of children diagnosed with pervasive develop- decline in language and social communication mental disorder, mental retardation, or tic dis- operating that are more profound than in reac- order, it is much more unusual. Destructive tive attachment disorder, and there are no crite- personality disorder of childhood is not a DSM- ria noting an association of symptoms of neglect IV-TR diagnosis and here serves as a distracter. or abuse. ADHD and conduct disorder are most often diagnosed in school-age children who 58. (D) Secondary enuresis means that the child have histories of reactive attachment disorder. achieved continence for over a year and then lost it, as the case suggests. Nocturnal means the 63. (C) The majority of patients diagnosed with incontinence happens at night. Primary enuresis borderline personality disorder report histo- means that the patient has never achieved com- ries of sexual abuse. Sexual activity among sib- plete continence. Diurnal enuresis is daytime lings is the most common form of incest, incontinence. Encopresis is the repeated passage although parent-to-child incest is the most of feces into inappropriate places. commonly reported form. A history of sexual abuse is in fact a risk factor for being a future 59. (E) More than 40 double-blind trials of imipramine victim of sexual abuse. There is no association have confirmed the drug’s efficacy for enuresis. of childhood sexual abuse and schizophrenia. Clonidine is an antihypertensive sometimes Abuse of alcohol or other illicit substances at used in opiate withdrawal. Haloperidol is a such a young age as this patient is commonly typical antipsychotic. Carbamazepine is an found in victims of childhood sexual abuse. Answers: 55–71 257

64. (D) Many incestuous fathers act in the face of is 5 weeks. This precaution avoids serotonin disappointing and/or conflicted sexual rela- syndrome, which can occur when significant tionships with their spouses. Incestuous fathers blood levels of both drugs are present. and stepfathers all have increased likelihoods Serotonin syndrome, which can be fatal, is of having alcoholism, impulsive or violent marked by symptoms of tremor, diaphoresis, behavior, paraphilias such as exhibitionism and rigidity, myoclonus, autonomic dysregulation, pedophilia, and a personal history of being a hyperthermia, rhabdomyolysis, renal failure, victim of sexual abuse. and coma.

65. (D) This patient has symptoms most consis- 69. (D) MAOIs prevent the peripheral breakdown tent with a bipolar mood disorder and there- of tyramine, which may be ingested with cer- fore would most benefit from lithium or tain foods such as beer, red wine, aged cheeses, another mood stabilizer such as valproic acid. liver, smoked fish, dry sausage, and fava beans. Lorazepam may be used to treat psychomotor If too much tyramine builds up in the blood, it agitation and insomnia of mania but does not may activate peripheral sympathetic nerve generally treat the underlying disorder and has receptors and cause a hypertensive crisis. a dependency liability in the long term. Stiffness and tremor are usually associated with Imipramine and fluoxetine, as TCAs and typical neuroleptics and are rare with MAOIs. SSRIs, respectively, can exacerbate mania. Bland food is a nonspecific request. Eating food Chlorpromazine, a typical antipsychotic, may rich in tryptophan, a serotonin precursor, is not help the patient with his mild thought disorder a problem. MAOIs are not usually particularly and his psychomotor agitation but may not oversedating compared to tricyclic or other treat the underlying mood disorder if given atypical antidepressants. alone. 70. (D) This case deliberately brings together many 66. (C) Atypical depression is characterized by character traits drawn from “pure” diagnoses mood reactivity, rejection hypersensitivity, of character disorders, including borderline, hypersomnolence, increased appetite, and lead narcissistic, dependent, obsessive-compulsive, pipe psychomotor retardation. Severe concen- and schizotypal personalities. In clinical prac- tration impairment is a feature of melancholic tice, a large proportion of patients diagnosable depression. Patients with either melancholic or in Axis II do not in fact have any clear-cut char- atypical symptoms may have either a normal acter syndrome. Such mixed cases are referred dexamethasone suppression test or psychotic to as personality disorder not otherwise specified. symptoms. A manic response to antidepressant Patients with pure borderline personality dis- treatment may be indicative of a bipolar syn- order are known for their intense personal rela- drome rather than a unipolar depression. tionships, self-destructive behaviors, anger management issues, and poor senses of self- 67. (B) Although not commonly used because of image. Narcissistic personality disorder is the risk of side effects, MAOIs are actually the marked by a grandiose sense of self-worth, most effective class of drugs for treating atyp- entitlement, lack of real empathy, and tendency ical depression. SSRIs and TCAs are also effec- to use others for one’s own ends. Patients with tive. Some anticonvulsants are used for the obsessive-compulsive personality disorder are treatment of bipolar disorder, but not unipolar perfectionists, inflexible, and overly demand- depression. Neuroleptics are used to treat psy- ing of themselves and others. Schizotypal chotic disorders. patients are known for their quirkiness and seemingly magical thinking styles. 68. (C) When changing from an SSRI to an MAOI, one must allow a washout period of about 71. (D) Remembering the DSM-IV-TR criteria for five half-lives for the SSRI to be more than schizoaffective disorder can be daunting. 90% eliminated; in the case of fluoxetine, this Generally speaking, patients who at one time 258 9: Practice Test 2

appear to have schizophrenia (i.e., diagnostic CNS neurons enough to produce delirium in criteria for it are met) and at other times appear some patients. to have a prominent mood disorder (based on an episode that fulfills either major depression 73. (A) Delirium related to blockade of acetyl- or bipolar disorder with or without psychosis) cholinergic receptors is the most common cause may be diagnosed with schizoaffective disor- of medication-induced delirium in the elderly. der. Technically, the diagnosis requires a period Acetylcholine is uniquely involved with memory of 2 weeks of psychotic symptoms without processes in the brain and its involvement in the affective symptoms. Psychotic disorder not oth- pathophysiology of Alzheimer disease has been erwise specified is usually reserved for situa- heavily researched. Drugs that alter dopamine tions in which the clinician does not have neurotransmission are most associated with enough historical data to know whether the movement disorder and psychosis-related side patient has schizophrenia, a mood disorder effects. Drugs affecting GABA receptors may with psychotic features, or a medical-related cause sedation and ataxia as well as memory or substance-induced psychosis. disturbances. Fortunately, the diversity of drugs that affect these receptors is much narrower than 72. (D) The elderly are particularly vulnerable to the broad classes of drugs that block acetyl- medications known to have psychoactive prop- choline receptors. Drugs affecting serotonin may erties or side effects. This is in part due to the affect sleep, appetite, and mood, but few nonpsy- decreased ability of geriatric patients to rapidly chiatric drugs affect this system. Drugs affecting metabolize medications and thus the tendency central norepinephrine transmission may rarely of the drugs to build up to higher steady-state be associated with depression. levels than for younger persons. Additionally, elderly persons are generally more vulnerable 74. (E) Studies that find no statistical difference to the effects of medications that act on the cen- between groups when there actually is a dif- tral nervous system (CNS) due to decreased ference result in a type II error. In this case, the reserve of neurons using particular neuro- positive findings at the other universities make transmitter systems that may be affected by the finding at university A less likely to be cor- the drugs. For example, elderly people may be rect. University A may have needed a larger more susceptible to the dopamine-blocking number of subjects in the study to find the effects of haloperidol that cause parkinsonism treatment effect. Type I errors occur more often because they have a decreased number of than type II errors and cause the researcher to dopaminergic neurons that arise from the sub- conclude that there is a difference between stantia nigra. The patient in this case is on a groups when in fact there is none. Standard number of medications that may have CNS error is the degree to which the means of several side effects at high doses, especially in the eld- different samples would vary if they were erly. However, the highly anticholinergic prop- taken repeatedly from the same population. erties of diphenhydramine put this patient at The variance is the sum of the differences of particular risk of memory impairment and con- each data point from the mean. Positive predic- fusion. Cimetidine and furosemide are con- tive value is the proportion of abnormal test ceivable culprits for her confusion due to their results that are true positive, while negative pre- minor anticholinergic activity, but are much dictive value is the proportion of normal test less likely. Beta-blockers like atenolol have been results that are true negative. associated with depression presumably associ- ated with blockade of central adrenergic recep- 75. (E) This patient has signs most consistent with tors. Additionally, in some vulnerable patients, tardive dyskinesia, an extrapyramidal syn- they may reduce cardiac output enough to drome that usually affects perioral or limb cause a functional hypoperfusion of the brain musculature and causes choreiform move- causing confusion and dizziness. Digoxin may ments. Its onset is usually several years after actually decrease sodium ion conduction in being on the medication and it is more likely to Answers: 72–81 259

affect older patients. Akathisia is best described psychological experience spanning the life span, as psychomotor restlessness that may have an centered on stage-appropriate developmental onset of hours to days after beginning the neu- conflicts: basic trust versus mistrust (birth to roleptic. Dystonia is an acute reaction to neu- 1 year); autonomy versus shame and doubt roleptics in which particular muscle groups (1–3 years); initiative versus guilt (3–5 years); (neck or ocular muscles commonly) contract industry versus inferiority (6–11 years); identity involuntarily. It can be painful and should be versus role diffusion (11 years to end of adoles- treated immediately with anticholinergics. cence); intimacy versus isolation (21–40 years); NMS is a potentially lethal medical emergency generativity versus stagnation (40–65 years); in which patients may have global rigidity, and integrity versus despair (65 and older). Carl mental status changes, fever, cardiovascular Jung developed concepts of the collective instability, elevated creatine phosphokinases, unconscious, archetypes, individuation, intro- and risk of rhabdomyolysis. Parkinsonism may version, and extroversion. Karen Horney’s look identical to Parkinson disease (tremor, work is notable for an emphasis on the idea bradykinesia) and may have onset within that a person’s personality traits are the result of weeks to months of beginning the medication. an interaction of the person with the environ- ment, and that “self-realization” is the thera- 76. (A) The basal ganglia, implicated in the yoking peutic process that removes distorting influences of thought to motor action, and in controlling that prevent character growth. Jean Piaget is the initiation and quality of motor action, is known for his work using observations of chil- theorized to be central to the pathophysiology dren and adolescents to build a framework of extrapyramidal syndromes, including dys- describing cognitive stages of development tonia, parkinsonism, akathisia, and tardive beginning with the sensorimotor stage at birth dyskinesia. The cerebellum is important in con- and ending with the stage of formal operations. trolling the coordination of motor movements Sigmund Freud is the founder of psychoanalysis, and posture as well as participating in proce- notable for such concepts as ego, id, and super- dural memory. The frontal cortex is generally ego and drive theory. considered to be important for decision making, impulse control, short-term memory, and affect 79. (D) About 20% of patients with complex partial regulation. The midbrain contains nuclei that epilepsy experience psychotic symptoms at help to ensure CNS homeostasis by regulating some time. neurovegetative, autonomic, and arousal func- tions. The motor cortex serves as the last stage of 80. (B) Hyperreligious thinking or preoccupation cerebral processing of motor information before with moral behavior, altered sexual behaviors, it descends into the spinal cord. An intact motor hypergraphia or overelaborative communica- cortex is required for initiation of movement. tion styles (also referred to as viscosity), and heightened experience of emotions form a clas- 77. (A) Premorbid personality disorders and comor- sic constellation of personality traits associated bid Axis I disorders have been shown to increase with complex partial epilepsy. Urinary incon- the risk of acquiring PTSD with exposure to a sig- tinence is more often associated with normal nificant, acute psychological trauma. Exposure to pressure hydrocephalus or severe dementia. psychological trauma in middle adulthood as Sleep disorders, depression, and obsessive- opposed to childhood or elderly years is consid- compulsive symptoms may be associated with ered the period of least vulnerability to PTSD. structural brain injury of frontal and subcorti- Rapid onset of symptoms, short duration of cal areas but have not been described as being symptoms, and strong social supports are good related particularly to complex partial epilepsy. prognostic indicators for PTSD. 81. (B) The Glasgow Coma Scale measures level of 78. (C) Erik Erikson is perhaps best known for arousal and ranges from a scale of 3 (deep his description of eight stages of human coma) to 14 (fully alert). The categories assessed 260 9: Practice Test 2

are eye opening, best motor response, and best stabilizer), and buspirone (an anxiolytic/anti- verbal response. Eye opening ranges from a depressant) may be used to treat various man- score of 4 (opening spontaneously) to 1 (not ifestations of bipolar disorder, none of them opening at all). In this case the eyes open to are associated with diabetes insipidus. pain, so a score of 2 is given. Best motor response is as follows: obeys commands, 5; 85. (D) Clonidine, a central alpha-2-autoreceptor localizes pain, 4; flexion, 3; extension, 2; and no agonist, has been proven useful in the treatment response, 1. In this case, a score of 3 is given for of autonomic hyperactivity associated with the response. Best verbal response is also based opioid withdrawal. Haloperidol, pimozide, and on a scale ranging from 5 to 1. An oriented patient thioridazine are antipsychotics. Amantadine is receives a score of 5 and no response receives a used to treat Parkinson disease and also as an score of 1, as in this case. The scores are then anti-influenza agent. added, and in this case are 2 + 3 + 1 = 6. 86. (C) Methadone has been proven to greatly 82. (D) At doses of 1000 mg/day or higher, thior- reduce the use of heroin when used as a main- idazine has been associated with pigmentary tenance medication. Patients maintained on retinopathy, also known as retinitis pig- doses lower than 40 mg/day of methadone are mentosa. Therefore, doses should not exceed far more likely to relapse than those on higher 800 mg/day. Pigmentary retinopathy can cause doses. Lorazepam and alprazolam are benzo- loss of retinal response to contraction of the diazepines, sertraline is an SSRI, and lithium is visual field. An early sign may be nocturnal con- first-line treatment for bipolar disorder. fusion. Constipation, dry eyes, and urinary retention are side effects from the anticholiner- 87. (B) TCAs are considered class 1A antiarrhyth- gic properties of thioridazine, but typically are mics because they possess quinidine-like effects not so severe as to compromise health perma- that decrease conduction time through the nently. Nephrogenic diabetes insipidus is not bundle of His. They have been shown to associated with thioridazine. increase mortality in cardiac patients. TCAs can also increase the heart rate anywhere from 83. (C) Benztropine 1–2 mg IM is useful in the treat- 3 to 15 beats/min, and a patient with compro- ment of acute dystonic reactions. Alternatively, mised cardiac function may suffer from diphenhydramine 50 mg IM or IV can be used. increased oxygen demand. Finally, TCAs are If the symptoms do not resolve within 20 min- associated with significant orthostatic hypoten- utes, larger doses can be given. Benzodiazepines sion, which is further exacerbated in patients can also be tried but are not first-line treatment. with cardiac disease. The SSRIs are safely used For acute laryngeal dystonia, 4 mg of ben- in patients with cardiac disease, although atten- ztropine should be given within 10 minutes, then tion should be paid to cardiac medicines 2 mg of lorazepam IV if needed. None of the metabolized through the P-450 cytochrome other choices are indicated for the treatment of system such as fluoxetine, which could alter med- acute dystonia, although switching to a lower- ication levels. The butyrophenones and diben- potency neuroleptic might be considered as pro- zodiazepines are antipsychotics. Benzodiazepines phylaxis against further dystonic reactions. are not antidepressants. Labetalol is an antihypertensive, acetaminophen is an analgesic and antipyretic, and penicillamine 88. (D) Acetylcholine is the neurotransmitter is a chelating agent used to treat Wilson disease. most implicated in cognitive functioning. Anticholinergic effects of medications are fre- 84. (A) The disorder described is diabetes insipidus. quently implicated in cognitive decline and Lithium inhibits the effect of antidiuretic hor- drug-induced delirium. All of the neurotrans- mone on the kidney. Although haloperidol (an mitters listed may be involved in cognition antipsychotic), diazepam (a benzodiazepine), but acetylcholine is by far the most impli- valproic acid (an antiepileptic used as a mood cated. Dopamine and histamine are targeted by Answers: 82–98 261

antipsychotic medications, and serotonin and extreme agitation or violence. Haloperidol, a norepinephrine are thought to be involved in typical antipsychotic, is useful for extreme agi- depression. tation. Despite urban myths, vitamins and cheese do not have particular therapeutic ben- 89. (B) Lithium toxicity is characterized early on by efits for PCP intoxication. dysarthria, ataxia, coarse tremor, and abdomi- nal pain. Later manifestations include seizures, 94. (C) This patient is suffering from major depres- neuromuscular irritability, and impaired con- sion with psychotic features. Psychosis cannot sciousness (delirium to coma). Acute dystonias be treated with antidepressant medication are associated with the use of typical antipsy- alone, because psychosis should be treated with chotics; paranoid delusions are part of the an antipsychotic such as risperidone. Depression symptom profile of schizophrenia and some- should be treated with an antidepressant such as times of bipolar disorder. Leg pain is not asso- sertraline. Lithium and synthroid can both effec- ciated with lithium. tively augment antidepressants such as paroxe- tine. Lorazepam, a benzodiazepine, may help 90. (C) In general, the typical antipsychotics can be with anxiety, but will not be effective against continued without significant side effects psychosis or depression. Similarly, a combina- during ECT, and in fact can lower the seizure tion of lorazepam and clozapine, an atypical threshold, which facilitates ECT. Lithium has antipsychotic, will not treat the depression. been associated with prolonged delirium post- ECT. There are no good data on venlafaxine 95. (I) PCP, AKA angel dust, intoxication produces during ECT. As a benzodiazepine, lorazepam these symptoms as well as hyperthermia, is an antiepileptic and would hinder ECT. depersonalization, and hallucinations in audi- Benztropine could cause cognitive difficulties tory, tactile, and visual modalities. immediately after ECT. Clozapine, an atypical antipsychotic, can cause tardive seizures when 96. (E) These symptoms may begin within a day administered with ECT. for severe alcoholics in withdrawal, and they can progress to DTs by 3 days. DTs are associ- 91. (A) There are no absolute contraindications to ated with a risk of lethal autonomic collapse. ECT. A space-occupying lesion is a relative con- traindication to ECT. Other relative contra- 97. (C) These symptoms tend to be mild and last indications include high intracranial pressure, 1–2 days post last use in a regular cocaine user. intracerebral bleeding, recent myocardial infarc- Occasionally, the depression can be great tion, retinal detachment, pheochromocytoma, enough to raise the risk of self-injurious behav- and high anesthesia risk. The other choices are ior or suicide attempts. not considered risks. 98. (G) Heroin withdrawal peaks at about 72 hours 92. (B) For extreme agitation, the butyrophenone post last use in dependent users. It is experi- antipsychotic haloperidol is useful. Benzo- enced with many flu-like symptoms and gas- diazepines are also useful for this symptom. trointestinal complaints, such as cramping, Phenothiazine antipsychotics can cause auto- nausea, and diarrhea. The appearance of pilo- nomic instability when given to a patient with erection in the syndrome gave rise to the slang PCP intoxication. Trihexyphenidyl is used to term cold turkey to describe the action of total combat extrapyramidal symptoms associated cessation of use of the drug. Cannabis intoxi- with antipsychotic use. Trazodone, used com- cation (A) heightens sensitivity to external monly for insomnia, and fluoxetine, an SSRI, stimuli and impairs motor skills. Cocaine intox- are antidepressants. ication (B) is marked by elated mood, halluci- nations, and agitation. Alcohol intoxication 93. (C) Supportive care is the best treatment for (D) involves characteristic behavioral changes, PCP intoxication that is not complicated by as well as slurred speech, ataxia, and other 262 9: Practice Test 2

neurologic findings. It can progress to coma if psilocybin (N), a hallucinogen derived from severe. Heroin intoxication (F) is marked by an mushrooms, is not well-described. altered mood, psychomotor retardation, and drowsiness. The syndromes of MDMA (H) and 99. (B) Clozapine causes agranulocytosis in about methamphetamine (J) intoxication are similar 1% of patients. For this reason, weekly blood to those of cocaine intoxication. MDMA users counts are measured for the first 6 months of typically report an increased “sense of close- treatment, then every 2 weeks thereafter. ness” with other people. Intoxication by inhalants (K) (volatile substances such as gaso- 100. (G) Lithium is associated with clinical hypothy- line fumes) is marked by disorientation and roidism in about 5% of patients, most commonly fear and is short-lived. Nicotine intoxication women. Thyroid supplementation may be (L) is not a category of the DSM-IV-TR. Nitrous added to counter this side effect. Up to 30% of oxide intoxication (M) produces euphoria and patients may show elevated thyroid-stimulating light-headedness and usually subsides within hormone levels. hours without treatment. Withdrawal from Bibliography

American Medical Association. Code of Medical Ethics: Kaplan HI, Sadock BJ, eds. Comprehensive Textbook of Current Opinions and Annotations 2004–2005. Psychiatry. 7th ed. Baltimore, MD: Williams & Chicago, IL: American Medical Association; 2004. Wilkins; 2000. American Psychiatric Association. Diagnostic and Katona CLE, Robertson MM. Psychiatry at a Glance. Statistical Manual of Mental Disorders. 4th ed. Text Cambridge, MA: Blackwell Science; 2000. Revision. Washington, DC: American Psychiatric Levenson JL. Neuroleptic malignant syndrome. Am Press; 2000. J Psychiatry. 1985;142:1137–1145. American Psychiatric Association. DSM-IV-TR Lewis M. Child & Adolescent Psychiatry: A Comprehensive Casebook. Washington, DC: American Psychiatric Textbook. 3rd ed. Baltimore, MD: Williams & Press; 2002. Wilkins; 2002. Arana GW, Rosenbaum JF. Handbook of Psychiatric Lezak MD. Neuropsychological Assessment. New Drug Therapy. 4th ed. Boston, MA: Little, Brown & York, NY: Oxford University Press; 1995. Company; 2000. Lindenmayer JP, Czobor P, Volavka J, et al. Changes Becker AE, Grinspoon SK, Klibanski A, et al. in glucose and cholesterol levels in patients with Current concepts: eating disorders. N Engl J Med. schizophrenia treated with typical or atypical 1999;340:1092–1098. antipsychotics. Am J Psychiatry. 2003;160:290–296. Benedetti F, Sforzini L, Colombo C, et al. Low dose Lishman WA. Organic Psychiatry: The Psychological clozapine in acute and continuation treatment of Consequences of Cerebral Disorder. Malden, MA: severe borderline personality disorder. J Clin Blackwell Science; 1998. Psychiatry. 1998;59:103–107. Lowinson J, Ruiz P, Millman R, Langrod J, eds. Check JR. Munchausen syndrome by proxy: an Substance Abuse. 4th ed. Baltimore, MD: Williams & atypical form of child abuse. J Practical Psychiatry Wilkins; 2004. Behavl Health. 1998;4:340–345. O’Malley S, Jaffe AJ, Chang G, et al. Six-month Conley RR. Optimizing treatment with clozapine. follow-up of naltrexone and psychotherapy for J Clin Psychiatry. 1998;59:S44–S48. alcohol dependence. Arch Gen Psychiatry. 1996;53: Fogel B, Schiffer R, Rao S, eds. Neuropsychiatry: A 217–224. Comprehensive Textbook. Baltimore, MD: Williams & Ollendick TH, Hersen M, eds. Handbook of Child Wilkins; 2003. Psychopathology. 3rd ed. New York, NY: Plenum Friedman M. Current and future drug treatment for Press; 1998. post-traumatic stress disorder. Psychiatr Ann. Preskorn SH. Clinically relevant pharmacology of 1998;28:461. selective serotonin reuptake inhibitors. Clin Hyman S, Nestler E. The Molecular Foundations of Pharmacokinet. 1997;32:S1–S21. Psychiatry. Washington, DC: American Psychiatric Price L, Heninger G. Lithium in the treatment of Press; 1993. mood disorders. N Engl J Med. 1994;331:591–598. Kane JM, Jeste DV, Barnes TRE, et al. Tardive Sadock BJ, Sadock VA. Kaplan and Sadock’s Synopsis Dyskinesia: A Task Force Report of the American of Psychiatry:Behavioral Sciences, Clinical Psychiatry. Psychiatric Association. Washington, DC: American 9th ed. Baltimore, MD: Williams & Wilkins; 2002. Psychiatric Association; 1992.

263

Copyright © 2007 by The McGraw-Hill Companies, Inc. Click here for terms of use. 264 Bibliography

Schatzberg A, Cole J, DeBattista C, eds. Manual of Sternbach H. The serotonin syndrome. Am J Clinical Psychopharmacology. 3rd ed. Washington, Psychiatry. 1991;148:705–713. DC: American Psychiatric Press; 1997. Tasman A, Kay J, Lieberman J, eds. Psychiatry. Schatzberg A, Nemeroff C. Textbook of Psycho- Hoboken, NJ: John Wiley & Sons; 2003. pharmacology. 2nd ed. Washington, DC: American Yudofsky SC, Hales RD, eds. Textbook of Psychiatric Press; 2003. Neuropsychiatry and Clinical Neurosciences. 4th ed. Stahl S. Essential Psychopharmacology. 2nd ed. Washington, DC: American Psychiatric Press; 2002. Cambridge, UK: Cambridge University Press; 2000. Index

A memory deficits and, 59 eating disorders and, 206, 222 Abdominal pain, 15, 45, 69 Wernicke-Korsakoff syndrome mania and, 44, 68 lithium and, 245, 261 and, 79 OCD and, 207, 223 Academic performance, 3, 5 Alcohol withdrawal, 106, 122, placebo-controlled trials and, 50 Acetaminophen (Tylenol), 43 247, 253, 261 Antipsychotic medications, 98, 117 Acetylcholine, 243, 245, 260 benzodiazepines and, 107 Antipsychotics, 29, 90, 212 Acetylcysteine (Mucomyst), 32, 75 Alexia, 26, 38, 80 borderline personality disorder Acting out, 36, 82, 125, 134 Alpha-blocking agents, 215 and, 110 Acute stress disorder, 49, 71, 181, Alprazolam (Xanax), 5, 65, 69, Antisocial personality disorder, 18, 214, 217 84, 89, 110 25, 33, 37, 63, 82, 149, 158, 215 Acute stress reaction, 201 Altered levels of consciousness, male prison population and, ADHD. See Attention deficit medications and, 114 148, 157 hyperactivity disorder Altruism, 67 Anxiety disorders, 4, 5, 181 Adjustment disorder, 33, 71 Alzheimer disease, 65, 69, 84 children and, 16, 32 with depressed mood, 169, 185 medications and, 114, 253 medications and, 94 Adolescents, 13–39 AMA Code of Ethics, 156, 157 Aphasia, 38, 79, 220 depression and, 13, 29 Amantadine (Symmetrel), 201, 217 Apraxia, 38, 80 suicide and, 235, 253 American Law Institute test, 142, 154 Aprosodic speech, 59, 79 suicide behavior and, 15 Amitriptyline (Elavil), 98, 208, 224 Arcus senilis, 250 suicide risk and, 13, 29 Amnesia, 50, 57, 59, 72, 77, 78 Aripiprazole, 101, 102, 119 Adult psychopathology, Amnestic disorder, 73 Arnold-Chiari syndrome, 69 41–84 Amok syndrome, 216, 249 Asperger disorder, 27, 32, 38, 39 Affect, 62, 81 Amotivational syndrome, 104, 121 Aspirin, 88, 109 flat, 196, 213 Amphetamine compound, ADHD Associations, 8 isolation of, 132 and, 209 free, 124, 133 Affective disorder research, 70 Amygdala, 45, 69, 81 loosening of, 84 Affective disorders, completed Amyloid plaques, 65, 84 Astereognosis, 79 suicides and, 147, 157 Anemia, 26 Asthma, 14, 24 Agnosia, 38 Anger, infants and, 21, 35 Athetoid movements, 250 Agoraphobia, 64, 70, 83, 214 Anhedonia, 31, 102, 120 Ativan (lorazepam), 85, 99, 107, panic disorder and, 108, Anomia, 38, 60, 80 108, 118, 165, 169, 182, 185, 170, 186 Anorexia nervosa, 25, 26, 37, 38, 207, 224 Agranulocytosis, 105, 122, 215 46, 69, 171, 187, 222 elderly patients and, 114 Airway obstruction, 207, 223 dental decay and, 255 Atropine, 207, 223 Akathisia, 11, 59, 79, 98, 102, 105, hypercholesterolemia and, 35 Attachment 108, 116, 120, 121, 259 medications and, 110 children and, 240, 256 Akinesia, 79 Anorgasmia, 86, 108 infants and, 22, 36 Alcohol abuse, 48, 94, 174, 190, 261 Anterograde amnesia, 59, 79 Attention, 199, 216 disulfiram and, 138 Anticholinesterase medication, Attention deficit hyperactivity medications and, 113 207, 223 disorder (ADHD), 34, 209 treatment and, 207 Antidepressant medications, behavioral therapy and, 128, 136 Alcohol dependence, 48, 71 91, 111 distinguishing from mania, 23, 36 Alcoholism, 71, 175, 190 Antidepressants, 197 learning disorders and, 20

265

Copyright © 2007 by The McGraw-Hill Companies, Inc. Click here for terms of use. 266 Index

Attention deficit hyperactivity Munchausen syndrome by proxy Child custody cases, forensic disorder (ADHD) (Cont.): and, 214 evaluation for, 152, 160 report cards and, 129, 136 Boston Diagnostic Aphasia Childhood disintegrative disorder, stimulant medications and, 21, Examination, 4, 10, 210, 226 32 23, 35, 36 Brain, executive functions of, 2 Childhood schizophrenia, 39 treatment and, 225 Breast cancer, 4 Child Protective Services, 144, 155, Attention deficit hyperactivity Breuer, Joseph, 139 234, 252 disorder(ADHD), 34 Brief psychotic disorder, 184 Children, 13–39 Auditory hallucinations, 177, 192 Broca aphasia, 204, 220 suicide and, 15, 253 Autistic disorders, 21, 32, 35, 70 Brown-Peterson Task, 1, 7 treatment decisions and, 146, 157 differentiating, 27, 39 Brushfield spots, 250 Chlordiazepoxide (Librium), 69, Automatic thoughts, 125, 134 Bruxism, 249 208, 224 Autonomic instability, 178, 193 Bulimia nervosa, 14, 26, 30, Choreiform movements, 250 Avoidant personality disorder, 38, 69, 222 Chronological age, 3, 9 70, 138, 215 fluoxetine and, 110 Circadian rhythm sleep disorder, 74 Axes of the Diagnostic and Bulimia Test-Revised, 7 Circle copying, 21, 35 Statistical Manual of Mental Bupropion, 96, 100, 104, 115, Circumlocution, 60, 80 Disorders (DSM), 178, 193, 230 118, 121 Circumstantiality, 58, 59, 78, 79, Axis II, 19, 34 Business ventures, patients and, 81, 84 Axis IV, 15, 31 149, 158 Citalopram (Celexa), 85, 96, 229, 248 Buspirone (BuSpar), 92, 111, Clanging (clang associations), 66, 79, B 203, 219 80, 84 Basal ganglia, 81, 243, 259 Butyrophenone antipsychotics, Clarity, gaining, 133 Beck’s triad, 191 246, 261 Clomipramine, 92, 111 Beck Depression Inventory, 1, 5, Clonazepam (Klonopin), 88, 93, 109, 7, 11, 191, 226 C 112, 208 Behavioral therapy, 87, 108, 126, Cacodemonomania, 78, 79 Clonidine (Catapres), 18, 95, 96, 114, 132, 135, 137 Caffeine, headaches and, 204, 220 245, 260 ADHD and, 128, 136 California Personality Inventory, 11 Tourette disorder and, 33 La belle indifférence, 10, 232, 251 Cannabis abuse, 175, 190, 210, Clorazepate (Tranxene), 203, 219 Bender-Gestalt Test, 8, 10, 226 227, 261 Clozapine (Clozaril), 89, 92, 100, Benzodiazepines, 84, 97, 99, 116, Capgras syndrome, 41, 58, 67, 78, 110, 111, 200, 217, 247, 262 117, 163, 181, 224, 234, 253 80, 230, 249 side effects and, 100, 118, 198, 215 alcohol withdrawal and, 107 Carbamazepine (Tegretol), 90, 110 Cocaine abuse, 26, 45, 69, 101, 200, Benztropine (Cogentin), 89, 200, Cardiovascular disease, 164 211, 216, 227, 261 201, 216, 217, 245, 260 Catalepsy, 203, 219 Cocaine withdrawal, 175, 190, 247 Bereavement, 206, 222 Cataplexy, 58, 62, 78, 82, 192 Code of ethics, 156, 157 Biofeedback, 131, 138 Catapres (clonidine), 18, 95, 96, Cogentin (benztropine), 200, 216, headaches and, 135 114, 245, 260 217, 245, 260 Bipolar disorder, 42, 43, 47, 54, 67, Catatonia, 80, 82, 203, 219 Cognitive-behavioral therapy, 53, 75, 68, 70, 75, 91, 93, 96, 192, 202, 218 Catatonic schizophrenia, 51, 73 123, 132, 170, 186 genetics and, 44, 68 Catharsis, 134 bulimia nervosa and, 14 manic episodes and, 238, 255 Caudate nucleus, 60, 80 Cognitive disorder, 73 medications and, 98, 115, 117 Celexa (citalopram), 85, 96, Cognitive psychology, 125 racing thoughts and, 215 229, 248 Cognitive therapy, 125, 126, 130, 132, Bizarre delusions, 192 Cerebellar atrophy, 81 134, 135, 137, 176, 191 Blepharospasm, 121, 249 Charcot, Jean-Martin, 139 Cold-caloric-induced nystagmus, Blessed Rating Scale, 5, 7, 11, 226 Child abuse, 17, 24, 32, 36, 144 201, 218 Blocking, 36 Munchausen syndrome by proxy Cold turkey, 261 Borderline intellectual and, 25 Command auditory hallucinations, functioning, 188 reporting requirements and, 24, 111, 180 Borderline personality disorder, 49, 37, 144, 234, 252 Communication analysis, 135 63, 72, 82, 110, 125, 134, 198, 200, Childbirth, medication and, Communication patterns, 127 215, 217, 241, 256 210, 226 Comorbid Axis I disorder, 259 Index 267

Competence to stand trial, 143, 155 Delirium, 45, 50, 69, 73, 167, 183, 214 Diagnosis Complex partial epilepsy, 244, 259 Delirium tremens, 108, 193 differential, 161–193 Compulsions, 89 Delusional disorder, 43, 64, 68, 184 multiaxis system of, 230, 249 Concentration, 4, 10, 216 Delusion of grandeur, 232, 251 Diagnostic and Statistical Manual of Conceptual domain, 8 Delusion of infidelity, 251 Mental Disorders, Fourth Conditioned avoidance, 131, 138 Delusion of reference, 78 Edition, Text Revision. See Conduct disorder, 26, 27, 32, 38, 39, Delusions, 61, 162, 180, 191, 236, 250, DSM-IV-TR diagnoses 239, 256 251, 254 Dialectical behavioral therapy Confabulation, 59, 79, 80 Creutzfeldt-Jakob disease and, 80 (DBT), 200, 217 Confidentiality, 251 culture-bound, 80 Dialysis, discontinuing, 145, 156 minors and, 145, 156 vs. hallucinations, 81 Differential diagnosis and physician-patient, 145, 146 idea of reference and, 57, 77 management, 161–193 Confusion, 5, 48, 62, 81 paranoid, patient’s family and, Dilated ventricles, 48, 62, 71 Congenital defects, rubella and, 142, 154 Diphenhydramine, 243 24, 37 shared, 84 Discharging patients from care, Constipation, 122 Dementia, 1, 6, 7, 73, 253 152, 160 Content validity, 8 mild, 5 Discriminative validity, 2, 8 Continuous amnesia, 78 reversible workup for, 205, 221 Displacement, 4, 10, 11, 37, 123, Continuous positive airway pressure Dementia praecox, 84 132, 250 (CPAP), 223 Demerol (meperidine), 204, 220 Disruptive behavior, 18 Conversion disorder, 63, 75, 82, 171, Denial, 36, 132, 217 Dissociation, 10, 37, 251 187, 214, 232, 251 Dental decay, 239, 255 Dissociative amnesia, 79 Conversion symptoms, 80 Depakote (divalproex sodium), 57, Dissociative fugue, 202, 218 Coprolalia, 31, 249 70, 77, 169, 195, 212, 241 Dissociative identity disorder, 216 Coprophagia, 31 side effects and, 100, 118, 209, 225 Disulfiram, 94, 113, 138 Copropraxia, 231, 249 Dependence, 119, 121 Diurnal enuresis, 256 Cortisol levels, 46, 69, 70 Dependent personality disorder, Divalproex sodium (Depakote), 57, Cortisol secretion, increased, 44, 68 3, 215 70, 77, 169, 195, 212, 241 Cotard syndrome, 41, 58, 67, 78 Depersonalization, 79 side effects and, 100, 118, Countertransference, 4, 10, 49, 71, Depression, 1, 7 209, 225 124, 133 adolescents and, 13, 29 Donepezil, 95, 114 Couples therapy, 108 double, 105, 122 Dopamine, 89, 109 Court-mandated treatment, 146, 156 Hamilton Depression Scale and, Doppelganger, 78 Court orders for medical records, 65, 84 Double depression, 105, 122 149, 158 left middle cerebral artery stroke Downregulation of postsynaptic

Couvade syndrome, 249 and, 54, 75 5-HT2 receptors, 86, 108 Creatine phosphokinase (CPK), multiple sclerosis and, 55, 76 Downward drift hypothesis, 42, 67 205, 221 research and, 70 Draw-a-Person Test, 7, 8 Creutzfeldt-Jakob disease, 80 severe melancholic, 43, 68 Dreams, 128, 136 Criminal responsibility, 142, 154 See also major depressive disorder Drooling, 14, 30 Culture-bound syndromes, 80, Depressive disorders, 175, 191 Drug abuse, 69, 101, 104, 119, 200, 216 Derailment, 66, 80 121, 210 Cushing syndrome, 70 Derealization, 59, 79 medications and, 95 Cyclothymia disorder, 44, 68, 72 Dereism, 79 Drug overdose, 53, 74 Desensitization, 138 Drugs, half-lives/onset of action D Desferal (deferoxamine), 32 and, 176, 191

D4 receptors, 92, 112 Desyrel (trazodone), 91, 95, 111, 114, DSM-IV-TR diagnoses, 32, 51, 73 Danger, reporting potential, 141, 153 219, 237, 254 schizophrenia and, 56, 77 DDAVP, 32 priapism and, 203 Duloxetine, 103, 120 Death, understanding of by side effects and, 100, 118, 203 Dynamic, 136 children, 23 Devaluation, 132 Dysarthria, 66, 84 Deferoxamine (Desferal), 32 Dexamethasone, 70 Dyscalculia, 84 Déjà entendu, 4, 10, 251 Diabetes insipidus, 260 Dysphagia, 79 Déjà vu, 10, 251 Diabetes mellitus Type I, 18 Dyssomnia, 74 268 Index

Dysthymia, 44, 68, 230, 249 Eye movement desensitization and Gambling, pathologic, 250 Dystonia, 79, 120, 259 reprocessing (EMDR), 75 Ganser syndrome, 202, 216, 218, Dystonic reaction, 11, 120 Eye tics, 19, 34 231, 250 Eysenck Personality Questionnaire, 11 Gender identity disorder, 151, 160 E Generalized amnesia, 78 Early alterations in embryonic F Generalized anxiety disorder (GAD), development, 19 Factitious disorder, 54, 75, 171, 71, 110, 164, 170, 175, 186, 191 Early morning awakening, 43, 68 187, 252 Geodon (ziprasidone), 161, 179, Eating Disorder Inventory-2 Munchausen syndrome, 64, 83 229, 248 (EDI-2), 7 Failure to thrive, 3, 6 Geriatric Rating Scale, 7 Eating disorders, 171, 187, 206, 222 Family therapy, 108, 128 Gingival hyperplasia, 199, 216 fluoxetine and, 90 Fargo Map Test, 9 Glasgow Coma Scale, 7, 244, 259 infants and, 14, 30 Feedback, 133 Glossolalia, 65, 84 Echolalia, 231, 249, 250 Fetishism, 73, 80 Glucocorticoids, 65, 84 Echopraxia, 249 Firearms, 13, 29, 32 Go-around technique, 124, 133 Edetate calcium disodium Fixation, 219 Granulocyte count, 198, 215 (CaEDTA), 17, 32 Flashback, 248 Gustation, 66, 84 Effexor (venlafaxine), 85, 93, 107, Flat affect, 196, 213 Gymnastics, 239, 255 112, 229, 248 Flight of ideas, 66, 79, 81, 84 Gynecomastia, 105, 121 Ego-dystonic thoughts/behaviors, Flooding, 108, 139 231, 250 Florid visual hallucinations, 31 H Ego-syntonic thoughts/behaviors, Flumazenil (Romazicon), 32 Habeas corpus, 150, 159 192, 250 Fluoxetine (Prozac), 16, 32, 46, 52, Halcion (triazolam), 166, 183 Elavil (amitriptyline), 98, 208, 224 69, 88, 90, 94, 109, 229, 236, 237, Haldol (haloperidol), 49, 87, 89, 94, Elderly patients, lorazepam and, 114 246, 248, 254, 255 109, 113, 163, 167, 181, 184, 246, Electroconvulsive therapy (ECT), 1, bulimia nervosa and, 110 247, 261 7, 50, 72, 99, 117, 202, 218 PTSD and, 114 Hallucinations, 31, 42, 61, 69, 78, 80 contraindications and, 102, 119, side effects and, 169, 185 children and, 15 246, 261 Fluphenazine (Prolixin), 87 command auditory, 111 rapid antidepressant action and, Fluvoxamine (Luvox), 50, 72, 98, 116, delirium and, 45 104, 121 207, 223 vs. delusions, 81 Embryonic development, early Folie à deux, 10, 65, 67, 78, 84, 251 Hallucinogens, 121 alterations in, 34 Folstein Mini-Mental Status Hallucinosis, 229, 248 Empathy, 71 Examination (MMSE), 1, 6, 7, Haloperidol (Haldol), 49, 87, 89, 94, Encopresis, 256 11, 226 109, 113, 163, 167, 181, 184, 246, Enuresis, 16, 20, 32, 35, 38, 240, 256 Formication, 58, 79, 81, 104, 247, 261 Epilepsy, 173, 188 121, 248 Halstead-Reitan Test, 7 complex partial, 244, 259 Four As (association, affect, autism, Hamilton, 65 Erectile dysfunction, 209 and ambivalence), 68 Hamilton Depression Scale, Erikson, Erik, 244, 259 Four Ds of establishing medical 65, 84 Erotomania, 232, 250 malpractice, 153 Haptic hallucinations, 79 Esophageal pH measurement, 15, 30 Free association, 124, 133 Headaches, 16, 32, 126, 135 Ethical issues, 141–160 Fregoli syndrome, 78 caffeine and, 204, 220 Executive functions, of brain, 2 Freud, 131, 139 Hemiballismus, 231, 250 Exercise-induced amenorrhea, 69 Frontal sulcal widening, 81 Hemispheric dominance, 6, 12 Exhibitionism, 80, 81 Frotteurism, 60, 73, 80 Hepatic encephalopathy, 173, 189 Exner Comprehensive System, Fund of knowledge, 10 Hermaphroditism, 81 5, 11 Heroin abuse, 47, 53, 70, 74 Expert medical witnesses, 142, 153 G medications and, 96, 114 Expressive language disorder, 14, 30 GABA-A receptor, 106, 122 Heroin withdrawal, 247, 261 Externalization, 82 Gabapentin, 96, 115 High serum blood urea nitrogen Extinction, 133, 138 Gait ataxia, 81 (BUN), 46, 69 Extrapyramidal side effects, 216 Galactorrhea, 97, 116, 121 Hippocampus, 61, 81 Extreme emotional disturbance, 154 Galantamine, 235, 253 schizophrenia and, 81 Index 269

Histrionic personality disorder, 72, 215 Informed consent, 143, 151, 154, 160, Legal issues, 141–160 Munchausen syndrome by proxy 233, 251 Leukemia, 18 and, 214 nontreatment and, 146, 156 Liability to third parties, 141, 153 HIV. See Human immunodeficiency Inhalant abuse, 104, 121, 208, Librium (chlordiazepoxide), 69, virus 224, 262 208, 224 Hospitalized psychiatric patients, Inkblot stimuli, 2, 8 Lithium, 91, 101, 111, 119, 195, 212, schizophrenia and, 65, 84 Insanity defense, 142, 144, 150, 245, 247, 260, 262 Huffing, 208, 225 155, 158 abdominal pain and, 245, 261 Human immunodeficiency virus Insight-oriented therapy, 75, 129 serum drug level and, 205, 221 (HIV), 18, 33, 47, 173, 189 Insomnia, 21, 35, 116 side effects and, 100, 118 testing, 145, 156 fluoxetine and, 169, 185 Living wills, 151, 159 Human psychological experience, 259 medications and, 97 Localized amnesia, 57, 77 Humor, 132 Intellectualization, 4, 10, 67, 124, 132 Locus ceruleus, 62, 81 Hydrochlorothiazide, 91 Intelligence quotient. See IQ Logorrhea, 79 Hypercholesterolemia, 22, 25, 35 Intermittent explosive disorder, 250 Loosening of associations, 66, 78, 84 Hypermnesia, 79, 121 Internal reliability, 8 Lorazepam (Ativan), 85, 99, 107, 108, Hyperreligious thinking, 244, 259 Interpersonal psychotherapy, 75, 118, 165, 169, 182, 185, 207, 224 Hypersomnia, 78 127, 135, 137 elderly patients and, 95, 114 Hypertension, 167, 184 Interpretation, 133 Lorazepam IV, 86 Hypertensive crisis, 214, 220 Intoxication, 121 Loss of ego boundaries, 61, 81 Hyperthyroidism, 174, 189 Involuntary commitment, 143, 144, Low self-esteem, 23 Hypervigilance, 121 155, 159 Low-stimulus environment, 166, 183 Hypnagogic hallucinations, 78, 79, 192 IQ (intelligence quotient), 3, 9 Lupus erythematosus, 62 Hypnopompic hallucinations, 78, 79 testing, 11, 14, 226 Luria-Nebraska Neuropsychological Hypnosis, 54, 76, 108, 128, 137, 139 Irresistible impulse rule, 154 Battery, 8 Franz Anton Mesmer and, 131 Irritability, 84 Luvox (fluvoxamine), 50, 72, 98, 116, psychotic disorders and, 136 Isolation of affect, 132 207, 223 Hypochondriasis, 39, 64, 75, 82, 84 Hypoglycemia, 174, 189 J M Hypointensities, 81 Jamais vu, 10, 251 Macropsia, 80, 251 Hypokalemia, 26 Jaundice, obstructive, 105, 122 Magical thinking, 232, 250 Hypomania, 44, 68, 70 Major depressive disorder (MDD), Hyponatremia, 16 K 13, 14, 16, 31, 70, 71, 164, 181 Hypotension, orthostatic, 122 Kayser-Fleischer rings, 231, 250 acute episodes of, 56, 77 Kindling, 105, 255 adjustment disorder and, 33 I Kleptomania, 231, 250 with atypical features, 202, 219 Ibuprofen, 88, 208, 224 children and, 24, 37 children and, 16 Idealization, 132, 219 Klonopin (clonazepam), 88, 112, 208 cognitive deficits and, 73 Ideas of reference, 57, 61, 77, 81, 250 Klüver-Bucy syndrome, 45, 69 delusions and, 176, 192 Identification, 24, 37, 71, 202, 219 Koro, 61, 79, 80 with psychotic features, 168, 184 Idiopathic hypersomnolence, 192 Korsakoff syndrome, 64, 65, 83 sertraline and, 29 Illusion, 59, 80, 248 lesions and, 176, 191 suicide risk and, 74 Imipramine (Tofranil), 20, 35, 47, 92, Kuru, 80 Maladaptive behaviors, 126 112, 240, 256 Kyofusho, 80 Malingering, 75, 151, 159, 233, 252 Impaired colleagues, reporting Malpractice cases, 142, 153 requirements and, 152, 160 L Management, 123–139 Impotence, 55, 76 Lability, 84 Mania, 81 medications and, 97 Laboratory tests, 88, 109 antidepressants and, 44, 68 Impulse control disorder, 70 Lamotrigine, side effects and, distinguishing from ADHD, 23, 36 Incest, 257 102, 120 Marijuana (cannabis), 175, 210, Incontinence, 48, 62, 71, 81 Laryngospasms, 120 227, 261 Infants Learning disorders, 20, 34 Marital dysfunction, 128, 136 anger and, 35 Left middle cerebral artery stroke, MDMA (ecstasy) intoxication, 175, treatment decisions and, 146, 157 depression and, 54, 75 190, 262 270 Index

Medical illnesses, psychiatric Modeling, 139 Nontreatment, implications of, illnesses and, 19, 33 Monoamine oxidase inhibitors 146, 156 Medical malpractice, 142, 153 (MAOIs), 107, 241, 257 Normal pressure hydrocephalus Medical records, releasing under tyramine and, 242, 257 (NPH), 48, 71 court order, 149, 158 Monozygotic twins, 177 Norpramin (desipramine), 247 Medical screening, 238, 255 bipolar disorder and, 192 Nymphomania, 251 Medication compliance/noncompli- schizophrenia and, 41, 56, 67, ance, 196, 199, 213, 216 76, 78 O Medication errors, patients and, Mood congruency, 176, 191, 250 Objective tests, 3, 9 146, 156 Mood disorders, 44, 57, 65, 69, Obsessive-compulsive disorder Medications 77, 83 (OCD), 26, 38, 50, 69, 72, 80, pregnancy and, 52, 74 medication side effects and, 170, 186 side effects and, 118 47, 70 antidepressants and, 206, 223 Medicine, legal/ethical issues and, Motivation for change, 129, 137 medications and, 111 141–160 Motor tics, 19, 34 streptococcus and, 26 Meige syndrome, 110 Mucomyst (acetylcysteine), 32 Obstructive jaundice, 105, 122 Mellaril (thioridazine), 87, 244, 260 Multiaxis system of diagnosis, Oculogyric crisis, 120, 164, 181 Memory deficits, 55, 76 230, 249 Olanzapine (Zyprexa), 29, 96, 115, alcoholism and, 59 Multiple sclerosis, depression and, 129, 162, 163, 180 medications and, 95 55, 76 side effects and, 100, 118 Memory impairment, 1, 5 Munchausen syndrome by proxy, 25, Open-ended questions/statements, Meningitis, 48 64, 83, 214, 252 29, 31, 132 Mens rea, 142, 154 Murray, Henry, 8 Operant conditioning, 138 Mental age, 3, 9 Myoclonus, 250 Opiates, 211, 227, 253 Mental retardation, 5, 19, 34, 87, 172, Myotonia, 250 withdrawal from, 166, 175, 188, 240, 256 183, 190 ruling out, 30 N Opisthotonos, 103, 120, 121 severity of, 27, 39 Naloxone, 53, 74, 205, 221, 235, 253 Oppositional defiant Mental Status Examination (MSE), Naltrexone (ReVia), 94, 113 disorder, 39 7, 10 Narcissistic defense, 201, 217 Orap (pimozide), 172, 239, 256 Meperidine (Demerol), 204, 220 Narcissistic personality disorder, 70, side effects and, 99, 118 Mesmer, Franz Anton, 131 125, 134, 215 Orthostatic hypotension, 105, 122 Mesmerism, 139 Narcolepsy, 52, 73, 78, 177, 192 Oxazepam (Serax), 86, 108 Methadone, 47, 70, 75, 245, 260 Nardil (phenelzine), 70, 85, 107 Methamphetamine, 262 Nausea, fluoxetine and, 17 P Methylene blue, 75 Nefazodone, 229, 248 Pain Methylphenidate (Ritalin), 23, 52, Negative predictive value, 258 medications and, 96 74, 128, 208 Negative reinforcement, 131, 138 TCAs, 115 ADHD and, 209 Negativism, 67 Palinopsia, 80 side effects and, 88 Neisseria gonorrhoeae, 252 Panic disorder, 26, 37, 38, 53, 75, 162, stimulant medications and, 109 Neologistic speech, 80, 230, 248 179, 198, 214, 248 tics and, 109 Neuroleptic malignant syndrome agoraphobia and, 88, 108, Microcephaly, 240, 256 (NMS), 105, 122, 163, 180, 170, 185 Micropsia, 80, 251 220, 259 sertraline for, 24 Minors Neuroleptics, 163, 180, 181, 196, 213 Paradoxical therapy, 137 confidentiality and, 145, 156 Neurosyphilis, 173, 188 Paralinguistic components of physician-patient confidentiality Neurovegetative signs, 60, 80 speech, 80 and, 236, 253 Nicotine, 211, 227, 262 Paranoia, 77 Mirtazapine, 93, 103, 113, 120 , 74 Paranoid delusions regarding

MMPI, 9 Nigrostriatal D2 receptors, 239, 256 patient’s family, 142, 154 MMPI-2, 6, 8, 9, 11, 226 Nihilism, 67 Paranoid schizophrenia, 177, 192 Mnemonic devices, SIGECAPS Nitrous oxide intoxication, 262 Paraphilia, 61, 81 and, 119 Nocturnal enuresis, 256 Paresthesia, 79 Möbius syndrome, 69 Nonfluency, 204, 220 Parkinsonism, 120 Index 271

Paroxetine (Paxil), 44, 63, 68, 82, 90, 91 Posthallucinogen perception disor- Psychiatric diagnoses, ethics and, side effects and, 100, 118 der, 121 147, 157 SSRI discontinuation syndrome Postpartum psychosis, 42, 47, 67, 70 Psychiatric illnesses, medical and, 110 Posttraumatic stress disorder illnesses and, 33 Partial hospital programs, 150, 159 (PTSD), 53, 71, 75, 164, 181 Psychic homeostasis, 124 Pathologic gambling, 250 fluoxetine and, 114 Psychodynamic, 127, 136 Patients Posturing, schizophrenia and, Psychodynamic psychotherapy, business ventures with, 158 208, 225 108, 129, 137 discharging from care, 152, 160 Practice tests, 195–262 Psychogenic gastritis, 69 relationships with, 148, 157 Predictive factors, suicide risk and, Psychogenic unresponsiveness, 218 Paxil (paroxetine), 44, 63, 68, 82, 65, 83 Psychomotor agitation, 16, 31 90, 91 Predictive validity, 2, 8 Psychopharmacology, 85–122 side effects and, 100, 118 Prednisone, 84 Psychotic disorders, 72 SSRI discontinuation syndrome Pregnancy brief, 168 and, 110 fluoxetine and, 236, 254 hypnosis and, 136 Payment problems with patients, MAOIs and, 204, 220 psychoanalysis and, 129, 137 149, 158, 233, 252 medications and, 52, 74 PTSD. See Posttraumatic stress PCP (phencyclidine), 174, 190, 211, Premenstrual dysphoric disorder disorder 227, 247 (PMDD), 52, 74 Punchdrunk syndrome, 69 supportive care and, 246, 261 Premorbid personality disorder, Punishment, 138 Pedophilia, 80 244, 259 Pyromania, 250 Peripheral neuropathy, 197, 213 Pressured speech, 62, 78, 81 Perphenazine (Trilafon), 86, 168, 184 Priapism, 91, 104, 111 Q Perseveration, 78 trazodone and, 203, 219, 237, 254 Quetiapine, 102, 120 Personality change, 125, 134 Primary atonia, 82 Personality disorder not otherwise Primary enuresis, 256 R specified, 242, 257 Primary hypersomnia, 74 Rabbit syndrome, 79, 122 Personality disorders, 125, 215 Prison populations, antisocial per- Racing thoughts, 199, 215 combining therapies and, 134 sonality disorder and, 148, 157 Random Letter Test, 5, 11 Personality structure, 11 Prodromal schizophrenia, 81 Raphe nucleus, 63 Personalization, 137 Prognostic factors, 52, 55, 74, 76 Rationalization, 10, 37, 217, 251 Phenelzine (Nardil), 70, 85, 107 Progressive social withdrawal, 42, 67 Reaction formation, 11, 31, 37, 81, 250 Pheochromocytoma, 161, 173, 179, 188 Projection, 11, 71, 130, 137, 219, 250 Rebound, 5, 11 Physical abuse Projective tests, 3, 9 Recent memory, 10 of children. See child abuse Prolactin levels, 98, 116 Reciprocal inhibition, 130, 138 reporting requirements and, 155 Prolixin (fluphenazine), 87 Rectal bleeding, 26 Physical examination, 167, 184 Propranolol, 87, 101, 108, 118 Recurrence, 11 Physician-assisted suicide, 148, 157, Propylthiouracil, 101, 118 Refractory depression, 112 235, 253 Prosody, 79 Regression, 23, 82, 217, 219 Physician-patient confidentiality, Prosopagnosia, 79, 80 Rejection hypersensitivity, 241, 257 145, 156 Protecting victims, 141, 153 Relationships with patients, past or minors and, 236, 253 Prothrombin time, 196, 213 present, 148, 157 Pica, 26, 38 Prozac (fluoxetine), 16, 32, 46, 52, 69, Reliability, testing and, 8 Pick disease, 69 88, 90, 94, 109, 229, 236, 237, 246, REM latency, 58, 78 Pigmented retinopathy, 105, 122, 248, 254, 255 Remote memory, 10 245, 260 bulimia nervosa and, 110 Reporting requirements, 155 Pimozide (Orap), 172, 188, 239, 256 PTSD and, 114 child abuse and, 24, 144, 234, 252 side effects and, 99, 118 side effects and, 169, 185 impaired colleagues and, 152, 160 Placebo-controlled trials, 50, 243, 258 Pseudocyesis, 249 Repression, 36 Play interviews, 19, 33 Pseudodementias, 73 Resistance, 133 Pleurothotonos, 120 Pseudoparkinsonism disorder, Retinal pigmentation, 87, 108 Porphyria, 69 105, 121 Retrocollis, 181 Positive predictive value, 258 Pseudoseizure, 58, 78 Retrograde amnesia, 78, 79 Positive reinforcement, 131, 138 Psilocybin, 262 Retrograde ejaculation, 105, 122 272 Index

Rett syndrome, 17, 23, 27, 32, 36, 38, Screening tests, 2 Spatial Orientation Memory Test, 9 39, 239, 255 Seasonal affective disorder, Speech, 79 ReVia (naltrexone), 94, 113 201, 218 disorders and, 14, 59 Reward, 138 Secondary enuresis, 256 pressured, 62, 78, 81 Rey-Osterrieth Test, 3, 9, 210 Sedation, 23 Splitting, 42, 62, 67, 81, 123, 132, 138 Rhabdomyolysis, 87, 108 Sedative hypnotics, 99, 117 SSRI discontinuation syndrome, 110 Risperdal Consta, 97, 115 Seizures, 165, 166, 182, 183 St. John’s wort, 237, 255 Risperidone, 93, 112, 115, 246, 261 clozapine and, 198, 215 Standard error, 258 side effects and, 97, 100, 118 Selective amnesia, 77 State-Trait Anxiety Inventory, 7 Ritalin (methylphenidate), 23, 52, 74, Selective mutism, 25, 27, 37, 39 Stereotypic movement disorder, 128, 208 Self-disclosure, 134 26, 38 ADHD and, 209 Self-observation, 10 Stimulant medications, 20, 34 side effects and, 88 Sentence Completion Test, 8, 10 ADHD and, 21, 23, 35, 36 stimulant medications and, 109 Separation anxiety disorder, 15, tics and, 109 tics and, 109 30, 38 Stranger anxiety, 35 Romazicon (flumazenil), 32 Serax (oxazepam), 108 Streptococcus, OCD and, 26, 38 Rorschach, Hermann, 8 Serotonin, 55, 76 Stress disorders, 49, 71 Rorschach Test, 2, 6, 7, 8, 11, 226 Serotonin-specific reuptake Stress reduction, headaches and, Rubella, 24, 37 inhibitors (SSRIs), 53, 75, 86, 90, 126, 135 Rumination disorder, 14, 30 108, 163, 164, 181 Stroke, 3, 4 borderline personality disorder Stroop Test, 9, 10 S and, 110 Stuttering, 79 Satyriasis, 103, 121, 251 sexual dysfunction and, 182, Subconjunctival hemorrhage, 250 Scanning speech, 79 209, 225 Sublimation, 10, 67, 82, 251 Schizoaffective disorder, 161, 179, SSRI discontinuation syndrome Substance-induced psychotic 243, 257 and, 110 disorder, 15, 31 Schizoid personality disorder, 47, 70, Serotonin syndrome, 82, 85, 107, Substance ingestion, 16, 32 82, 177, 192, 215 205, 221 Substance-related disorders, 17, 32 Schizophrenia, 2, 204, 209, 226 Sertraline (Zoloft), 14, 24, 29, 37, Suicidal behavior, 15, 31 catatonic type, 43, 51, 73 100, 103, 161, 168, 170, 179, 186, Suicidal ideation, 197, 213 childhood, 39 198, 214 Suicidal patients, 130, 137 DSM-IV-TR diagnoses and, 56, 77 Serum growth hormone, 69 Suicide early term for, 65, 84 Severe melancholic depression, 68 adolescents and, 235, 253 genetics and, 56, 57, 67, 76, 78, Sexual abuse, 252 children and, 253 199, 216 Sexual dysfunction, 121 physician-assisted, 148, 157, hippocampus and, 81 SSRIs and, 209, 225 235, 253 hospitalized psychiatric patients Sexual masochism, 73, 80, 121 schizophrenia and, 49, 71 and, 65, 84 Sexual sadism, 60, 73, 80, 121 Suicide attempts, 157 medications and, 89, 96, 98, 100, Shared delusions, 65, 84 children and, 16, 32 102, 120 Sialorrhea, clozapine and, 118 Suicide risk, 2, 13, 53, 101, 119, monozygotic twins and, 41, 56, 67, SIGECAPS mnemonic device, 119 150, 158 76, 78 Sigmund Freud, 131, 139 adolescents and, 29 paranoid type, 22, 36, 177, 192 SLE, 174, 189 completed suicides and, 147, 157 posturing and, 208, 225 Sleep apnea, 223 MDD and, 74 prevalence of, 67 Sleep jerks, 82 past attempts and, 147, 238, 255 prodromal, 81 Sleep latency, 58, 78 predictive factors for, 65, 83 prolactin levels and, 98 Sleep paralysis, 78, 192 race and, 57, 77 psychopathology of, 7 Sleep terror, 13, 29 Supportive therapy, 75, 124, 130, suicide and, 49, 71 Social phobia, 38 133, 138 treatment options and, 129, 137 Somatic delusion, 77, 192 Suppression, 133, 217 Schizophreniform disorder, 48, 67, Somatic treatment, 85–122 Symmetreal (amantadine), 201, 217 71, 165, 182 Somatization disorder, 15, 31, 64, Synesthesia, 79 Schizotypal personality disorder, 70, 65, 82, 83, 174, 189, 212 Syphilitic infections, 173, 188 82, 198, 212, 215 Somatoform disorder, 75 Systematic desensitization, 130, 138 Index 273

T Toxocara, 26, 38 W Taijin, 80 Transference, 10, 71, 127, 131, 133, Wada Test, 6, 227 Tangentiality, 66, 78, 79, 80, 84, 135, 136, 138, 250 Wechsler Adult Intelligence Scale, 9 236, 254 Transference neurosis, 131, 138 Wechsler Adult Intelligence Scale- Tarasoff v. Regents of University of Transsexualism, 81 Revised (WAIS-R), 3, 5, 9, 11, 226 California, 141, 153 Transvestic fetishism, 80, 81 Wechsler Intelligence Scale for Tardive dyskinesia, 79, 243, 258 Tranxene (clorazepate), 203, 219 Children (WISC), 172, 187, TAT, 7, 8 Trazodone (Desyrel), 91, 95, 111, 114, 210, 226 TD, 89, 105, 109, 121 219, 237, 254 Wechsler Memory Scale (WMS), 9, Tegretol (carbamazepine), 90, 110 side effects and, 100, 118, 203 210, 226 Temporal Orientation Test, 3, 8 Treatment decisions, 123–139, 146, 157 Wechsler Preschool and Primary Tension headaches, 126, 135 Triazolam (Halcion), 166, 183 Scale of Intelligence (WPPSI), Testamentary capacity, 142, 154, Trichotillomania, 26, 38, 81, 250 210, 226 233, 251 Tricyclic antidepressants (TCAs), 93, Wernicke aphasia, 204, 220 Testing, 1–12 113, 197, 214, 245, 260 Wernicke encephalopathy, 162, 176, Test-retest reliability, 8 cardiac patients and, 116 179, 191, 220 Thalamus, 81 pain and, 115 Wernicke-Korsakoff syndrome, 71, 79 Thiamine, 99, 117, 162, 180 Trilafon (perphenazine), 86, White matter plaques, 55, 76 Thiamine deficiency, 204, 220, 234, 252 168, 184 Wills Thioridazine (Mellaril), 87, 244, 260 Trismus, 181 legal validity and, 142, 154, 233, 251 Third parties, liability to, 141, 153 Tylenol (acetaminophen), 43 living wills and, 151, 159 Thought broadcasting, 77, 232, 250 Type II errors, 243, 258 Wisconsin Card Sorting Test Thought control, 250 Tyramine, MAOIs and, 242, 257 (WCST), 2, 5, 7, 9, 11, 227 Thought insertion, 77, 250 Withdrawal, 11, 119, 121 Thumb sucking, 23 U Word Association Test, 8 Thyroid disorder, 199, 215 Undoing, 67, 138 Word salad, 80 Thyroid function, 165, 182 Unresponsiveness, 201 assessing, 205, 221 Uric acid levels, 48, 71 X Thyrotropin-releasing hormone Urinary porphobilinogen, 45 Xanax (alprazolam), 5, 69 (TRH), 69, 70 Xanthelasma, 250 Tics V methylphenidate and, 89 Variable-ratio reinforcement, 4, 10 Y stimulant medications and, 20, Variance, 258 Yohimbine, 97, 116 34, 109 Venlafaxine (Effexor), 85, 93, 107, Tofranil (imipramine), 20, 35, 47, 92, 112, 229, 248 Z 112, 240, 256 Ventilation, 134 Zar, 80 Tolerance, 119, 121 Ventricular-brain ratio, 22, 35 Ziprasidone (Geodon), 161, 179, Torticollis, 120, 121, 181, 249 Verbigeration, 66, 84 229, 248 Tourette disorder, 18, 19, 33, 34, 39, Victims, protecting, 141 Zoloft (sertraline), 14, 24, 29, 37, 172, 188 Visceral larva migrans, 26, 38 100, 103, 161, 168, 170, 179, clonidine and, 33 Vision, testing, 27, 39 186, 198, 214 medications and, 109 Visual nonverbal memory, 4, 9 Zyprexa (olanzapine), 115, 129, Toxicology screen, 165, 166, 168, 182, Vocal tics, 34 162, 163, 180 183, 185, 237, 254 Voyeurism, 61, 80, 81 side effects and, 96, 100, 118